230
MC LC STT Ni dung Trang LI NÓI ðẦU 2 1 MT SBÀI TOÁN VDÃY SNGUYÊN Lê Xuân ðại, Trn Ngc Thng - Trường THPT Chuyên Vĩnh Phúc 3 2 TÌM SHNG TNG QUÁT CA DÃY TRUY HI TUYN TÍNH CP 2 ðỂ GII QUYT MT SBÀI TOÁN VDÃY STrường THPT chuyên Hưng Yên 39 3 DÃY SSHC Trường THPT Chuyên Biên Hoà – Hà Nam 48 4 NG DNG ðẠO HÀM TRONG MT SBÀI TOÁN VDÃY SNguyn Thành ðô - Trường THPT Chuyên Bc Ninh 55 5 DÃY SVÀ GII HN Vũ ThVân - Trung hc phthông Chuyên Bc Giang 61 6 MT PHƯƠNG PHÁP DY THC KHÁI NIM GII HN DÃY SCHO HC SINH CHUYÊN TOÁN Nguyn Thế Sinh - Trường chuyên Nguyn Trãi - Hi Dương 78 7 CHUYÊN ðỀ VCP S, CĂN NGUYÊN THY, THNG DƯ BC HAI Phm Văn Ninh -Trường THPT Chuyên HLong - Qung Ninh 102 8 SDNG :HTHNG DƯ ðẦY ðỦ, HTHNG DƯ THU GN ðỂ GII TOÁN Nguyn Duy Liên - Trường THPT Chuyên Vĩnh Phúc 126 9 THNG DƯ TOÀN PHƯƠNG Trường THPT chuyên Trn Phú - Hi Phòng 143 10 ðỊNH LÝ PHECMA VÀ MT VÀI NG DNG Trường THPT Chuyên Lê Hng Phong - Nam ðịnh 155 11 MT SNG DNG CA ðỊNH LÝ FERMAT NHðỊNH LÝ CƠ BN VSNGUYÊN TNguyn Văn Tho - Trường THPT Chuyên Bc Giang 160 12 SDNG PHÉP ðỒNG DƯ TRONG BÀI TOÁN CHIA HT Trường THPT chuyên Hưng Yên 173 13 MT VÀI BIN ðỔI VÀ NG DNG ðƠN GIN TRONG SHC Dương Châu Dinh - Trường THPT chuyên Lê Quý ðôn - Qung Tr181 14 PHƯƠNG PHÁP GEN Phm Quang Thăng - Trường THPT Chuyên Thái Bình 190 15 p Nguyn Trung Tuân - Trường THPT Chuyên HLong - Qung Ninh 198 16 SDNG TÍNH CHT NGHIM CA PHƯƠNG TRÌNH BC BA TÌM HTHC LƯỢNG MI TRONG TAM GIÁC Trường THPT Chuyên Hưng Yên 202 17 HÀNG ðIM ðIU HÒA - CHÙM ðIU HÒA Trường THPT chuyên Lê Hng Phong Nam ðịnh 211 18 BÀI TOÁN VðƯỜNG THNG SIMSON VÀ NG DNG Phm ðức Tùng - TrườngTHPT chuyên Lương Văn Ty - Ninh Bình 219 19 MT SVN ðỀ VPHƯƠNG TRÌNH CAUCHY ðào ThLê Dung - Trường THPT chuyên Thái Bình 229 20 PHƯƠNG TRÌNH HÀM CAUCHY Trường THPT Nguyn Trãi - Hi Dương 236

Chuyên đề hội thảo một số trường chuyên về dãy số

Embed Size (px)

DESCRIPTION

Hay

Citation preview

Page 1: Chuyên đề hội thảo một số trường chuyên về dãy số

MỤC LỤC STT Nội dung Trang

LỜI NÓI ðẦU 2

1 MỘT SỐ BÀI TOÁN VỀ DÃY SỐ NGUYÊN

Lê Xuân ðại, Trần Ngọc Thắng - Trường THPT Chuyên Vĩnh Phúc 3

2 TÌM SỐ HẠNG TỔNG QUÁT CỦA DÃY TRUY HỒI TUYẾN TÍNH CẤP 2 ðỂ GIẢI QUYẾT MỘT SỐ BÀI TOÁN VỀ DÃY SỐ

Trường THPT chuyên Hưng Yên

39

3 DÃY SỐ SỐ HỌC Trường THPT Chuyên Biên Hoà – Hà Nam

48

4 ỨNG DỤNG ðẠO HÀM TRONG MỘT SỐ BÀI TOÁN VỀ DÃY SỐ Nguyễn Thành ðô - Trường THPT Chuyên Bắc Ninh

55

5 DÃY SỐ VÀ GIỚI HẠN Vũ Thị Vân - Trung học phổ thông Chuyên Bắc Giang

61

6 MỘT PHƯƠNG PHÁP DẠY TỰ HỌC KHÁI NIỆM GIỚI HẠN DÃY SỐ CHO HỌC SINH CHUYÊN TOÁN

Nguyễn Thế Sinh - Trường chuyên Nguyễn Trãi - Hải Dương

78

7 CHUYÊN ðỀ VỀ CẤP SỐ, CĂN NGUYÊN THỦY, THẶNG DƯ BẬC HAI

Phạm Văn Ninh -Trường THPT Chuyên Hạ Long - Quảng Ninh 102

8 SỬ DỤNG :HỆ THẶNG DƯ ðẦY ðỦ, HỆ THẶNG DƯ THU GỌN ðỂ GIẢI TOÁN

Nguyễn Duy Liên - Trường THPT Chuyên Vĩnh Phúc

126

9 THẶNG DƯ TOÀN PHƯƠNG Trường THPT chuyên Trần Phú - Hải Phòng

143

10 ðỊNH LÝ PHECMA VÀ MỘT VÀI ỨNG DỤNG Trường THPT Chuyên Lê Hồng Phong - Nam ðịnh

155

11 MỘT SỐ ỨNG DỤNG CỦA ðỊNH LÝ FERMAT NHỎ VÀ ðỊNH LÝ CƠ BẢN VỀ SỐ NGUYÊN TỐ

Nguyễn Văn Thảo - Trường THPT Chuyên Bắc Giang

160

12 SỬ DỤNG PHÉP ðỒNG DƯ TRONG BÀI TOÁN CHIA HẾT Trường THPT chuyên Hưng Yên

173

13 MỘT VÀI BIẾN ðỔI VÀ ỨNG DỤNG ðƠN GIẢN TRONG SỐ HỌC

Dương Châu Dinh - Trường THPT chuyên Lê Quý ðôn - Quảng Trị 181

14 PHƯƠNG PHÁP GEN Phạm Quang Thăng - Trường THPT Chuyên Thái Bình

190

15 p� �� �� �� �

Nguyễn Trung Tuân - Trường THPT Chuyên Hạ Long - Quảng Ninh

198

16 SỬ DỤNG TÍNH CHẤT NGHIỆM CỦA PHƯƠNG TRÌNH BẬC BA TÌM HỆ THỨC LƯỢNG MỚI TRONG TAM GIÁC

Trường THPT Chuyên Hưng Yên

202

17 HÀNG ðIỂM ðIỀU HÒA - CHÙM ðIỀU HÒA Trường THPT chuyên Lê Hồng Phong Nam ðịnh

211

18 BÀI TOÁN VỀ ðƯỜNG THẲNG SIMSON VÀ ỨNG DỤNG

Phạm ðức Tùng - TrườngTHPT chuyên Lương Văn Tụy - Ninh Bình

219

19 MỘT SỐ VẤN ðỀ VỀ PHƯƠNG TRÌNH CAUCHY ðào Thị Lê Dung - Trường THPT chuyên Thái Bình

229

20 PHƯƠNG TRÌNH HÀM CAUCHY Trường THPT Nguyễn Trãi - Hải Dương

236

Page 2: Chuyên đề hội thảo một số trường chuyên về dãy số
Page 3: Chuyên đề hội thảo một số trường chuyên về dãy số

HỘ

HỘI CÁC TRƯỜNG THPT CHUYÊN KHU VỰC DUYÊN HẢI VÀ ðỒNG BẰNG BẮC BỘ HỘI THẢO KHOA HỌC LẦN THỨ IV

3

MỘT SỐ BÀI TOÁN VỀ DÃY SỐ NGUYÊN

Lê Xuân ðại, Trần Ngọc Thắng Trường THPT Chuyên Vĩnh Phúc

Dãy số là một phần quan trọng của ñại số và giải tích toán học. Dãy số có

một vị trí ñặc biệt quan trọng trong toán học, không chỉ như là một ñối tượng ñể

nghiên cứu mà còn ñóng một vai trò như một công cụ ñắc lực của các mô hình

rời rạc của giải tích trong lý thuyết phương trình, lý thuyết xấp xỉ, lý thuyết biểu

diễn… Các vấn ñề liên quan ñến dãy số rất phong phú. Hiện nay có nhiều tài

liệu ñề cập ñến các bài toán về dãy số. Tuy nhiên, chủ yếu quan tâm ñến các tính

chất của dãy số như Giới hạn dãy, số hạng tổng quát, sự ñơn ñiệu của dãy, tính

bị chặn…

Các bài toán về dãy số nguyên là những bài toán hay và khó. Trong bài

viết này chúng tôi muốn trình bày một số vấn ñề cơ bản và các phương pháp

thường sử dụng về dãy số nguyên. Chuyên ñề này ñược chia thành 2 phần như

sau:

Phần 1: Giới thiệu một số phương pháp cơ bản giải các bài toán về dãy số

nguyên.

Phần 2: Khai thác một số bài toán ñiển hình qua các kì thi Olimpic toán học.

I- MỘT SỐ VẤN ðỀ VỀ LÝ THUYẾT CƠ BẢN

1. Dãy Fibonacci và dãy Lucas.

1.1. Dãy Fibonacci ( )nF mang tên chính nhà toán hoc Pisano Fibonacci. Dãy

cho bởi hệ thức truy hồi ñơn giản 1 2

2 1

1

1n n n

F F

F F F n+ +

= =

= + ∀ ≥

Dễ dàng thấy công thức tổng quát của dãy ( )nF là:

1 1 5 1 5

2 25

n n

nF + − = −

(Công thức Binet)

Từ sau, ñể thuận tiện cho việc tính toán, ta quy ước 0 0F = .

Page 4: Chuyên đề hội thảo một số trường chuyên về dãy số

HỘ

HỘI CÁC TRƯỜNG THPT CHUYÊN KHU VỰC DUYÊN HẢI VÀ ðỒNG BẰNG BẮC BỘ HỘI THẢO KHOA HỌC LẦN THỨ IV

4

1.2. Một vài tính chất số học của dãy Fibonacci:

1. 1( , ) 1n nF F + = với mọi n.

2. Nếu n chia hết cho m thì nF chia hết cho mF .

3. Nếu nF chia hết cho mF thì n chia hết cho m với m>2.

4. ( , )n m dF F F= với ( , )d m n= .

5. Nếu 5n ≥ và nF là số nguyên tố thì n cũng là số nguyên tố.

6. Dãy ( )nF chứa một tập vô hạn những số ñôi một nguyên tố cùng nhau.

7. 5 5 .n n nF F q= với nq không chia hết cho 5.

8. 5knF n k⇔M M .

9. nF có tận cùng là 0 khi và chỉ khi 15nM .

10. nF có tận cùng là hai chữ số 0 khi và chỉ khi 150nM .

1.3. Một vài hệ thức cơ bản của dãy Fibonacci:

1. 1 2 2... 1n nF F F F ++ + + = −

2. 1 3 2 1 2... n nF F F F−+ + + =

3. 2 4 2 2 1... 1n nF F F F ++ + + = −

4. 21 1. ( 1)n

n n nF F F− + − = −

5. 2 2 21 2 1... .n n nF F F F F ++ + + =

5. 0 1 2 3 2 1 2 2 1... 1n n nF F F F F F F− −− + − − + = −

6. 2 21 1 2.n n n nF F F F+ − +− = .

7. 21 2 2 3 2 1 2 2... n n nF F F F F F F−+ + + =

8. 1 2 3. . ( 1)nn n n nF F F F+ + +− = −

9. 42 1 1 21n n n n nF F F F F− − + +− =

1.4. Dãy Lucas ( )nL ñược xác ñịnh như sau: 0 1

2 1

2; 1

0n n n

L L

L L L n+ +

= =

= + ∀ ≥

Page 5: Chuyên đề hội thảo một số trường chuyên về dãy số

HỘ

HỘI CÁC TRƯỜNG THPT CHUYÊN KHU VỰC DUYÊN HẢI VÀ ðỒNG BẰNG BẮC BỘ HỘI THẢO KHOA HỌC LẦN THỨ IV

5

Những số hạng của dãy Lucas có thể coi như giống với dãy Fibonacci bởi hai

dãy này ñều có cùng hệ thức xác ñịnh dãy.

Tương tự công thức Binet cho dãy Fibonacci, ta có công thức tổng quát của dãy

Lucas:

1 5 1 5, 0

2 2

n n

nL n + −

= + ≥

2

2. Thặng dư bậc hai

2.1. ðịnh nghĩa. Ta gọi là một thặng dư bậc hai modulo (hay là một số

chính phương ) nếu tồn tại số nguyên sao cho , trong

ñó là một số nguyên dương.

2.2. ðịnh lí. Cho là một số nguyên tố.

(i) Nếu thì mọi số lẻ ñều là số chính phương .

(ii) Nếu . Khi ñó

là số chính phương khi và chỉ khi ;

không là số chính phương khi và chỉ khi .

2.3. Kí hiệu Legendre. Giả sử là số nguyên tố lẻ, là số nguyên không chia

hết cho . Khi ñó ta có các kết quả sau:

(1).

(2). Nếu thì

(3).

(4).

(5).

(6). Luật tương hỗ Gauss: Nếu là hai số nguyên tố lẻ thì:

Page 6: Chuyên đề hội thảo một số trường chuyên về dãy số

HỘ

HỘI CÁC TRƯỜNG THPT CHUYÊN KHU VỰC DUYÊN HẢI VÀ ðỒNG BẰNG BẮC BỘ HỘI THẢO KHOA HỌC LẦN THỨ IV

6

3. Phương trình sai phân tuyến tính cấp 2.

3.1. ðịnh nghĩa. Phương trình sai phân tuyến tính cấp hai ẩn là phương

trình sai phân dạng: (1)

Phương trình sai phân tuyến tính thuần nhất tương ứng với phương trình (1) có

dạng: (2)

Nghiệm tổng quát của (1) có dạng , trong ñó là nghiệm tổng

quát của (2), còn là một nghiệm riêng nào ñó của (1).

ðể tìm nghiệm của (2) ñầu tiên ta lập phương trình ñặc trưng của (2) là:

(3)

TH1. Nếu phương trình ñặc trưng (3) có hai nghiệm thực phân biệt thì:

TH2. Nếu phương trình ñặc trưng (3) có nghiệm kép thì:

TH3. Nếu phương trình ñặc trưng (3) có nghiệm phức

với .

Khi ñó:

Ở ñây là các hằng số thực ñược xác ñịnh dựa vào các ñiều kiện ban ñầu.

II- MỘT SỐ PHƯƠNG PHÁP GIẢI BÀI TOÁN VỀ DÃY SỐ NGUYÊN

1. Phương pháp quy nạp toán học

Bài toán 1. Cho dãy số ( )n

a xác ñịnh bởi 0 10; 1a a= = và

1 13( 1)

2nn n n

aa a+ −

− += −

với mọi n nguyên dương. Chứng minh rằng n

a là số chính phương với mọi

0n ≥ .

Lời giải. Chú ý rằng 2 3 4 51; 4; 9; 25a aa a= = = = .

Do ñó 2 2 2 2 2 2

0 0 1 1 2 2 3 3 4 4 5 5; ; ; ; ;F F F F Fa a a a a Fa= = = = = = , ở ñó ( )n

F là dãy

Fibonacci.

Page 7: Chuyên đề hội thảo một số trường chuyên về dãy số

HỘ

HỘI CÁC TRƯỜNG THPT CHUYÊN KHU VỰC DUYÊN HẢI VÀ ðỒNG BẰNG BẮC BỘ HỘI THẢO KHOA HỌC LẦN THỨ IV

7

Từ ñó ta có ñịnh hướng chứng minh 2

n na F= bằng quy nạp theo n.

Thậy vậy, giả sử 2

k ka F= với mọi k n≤ . Như vậy

2 2 2

1 1 2 2; ;n n n n n n

a F a aF F− − − −

= = = (1)

Từ giả thiết ta có 1 13 2( 1)n

n n naa a

+ −− + = − và 1

1 23 2( 1)n

n n naa a

− −− + = −

Cộng hai ñẳng thức trên ta ñược: 1 1 22 2 0, 2n n n n

a a a a n+ − −

− − + = ≥ (2)

Từ (1) và (2) suy ra

( ) ( )2 22 2 2 2

1 1 2 1 1 2

2 2 2 2

1 2 2 1

2 2

n n n n n n n n n

n n n n

F F F F F F F F

F F

a

F F

+ − − − − −

+ − − +

= + − = + + − −

= + − =

Vậy 2 , 0n n

F na = ∀ ≥ và ta có ñiều phải chứng minh.

Bài toán 2. Cho dãy số ( )n

a xác ñịnh bởi 1 2 31; 4aa a= = = và

3 2 12 2n n n n

a a a a+ + +

= + −

với mọi n nguyên dương. Chứng minh rằng n

a là số chính phương với mọi

1n ≥ .

Bài toán 3. Cho dãy số ( )n

a xác ñịnh bởi 0 1 2 30; 1; 2; 6a a a a= = = = và

4 3 2 12 2 , 0n n n n n

a a a a a n+ + + +

= + − − ∀ ≥ .

Chứng minh rằng n

a chia hết cho n với mọi 1n ≥ .

Lời giải. Từ giả thiết ta có 4 5 612; 25; 48a a a= = = .

Ta có 1 2 3 4 5 61; 2; 3; 5; 81 2 3 4 5 6

a a a a aa= = = = = = , như vậy n

n

aF

n= với mọi n = 1, 2,

3, 4, 5, 6, ở ñó ( )n

F là dãy Fibonacci.

Từ ñó ta có ñịnh hướng chứng minh n n

a nF= với mọi 1n ≥ bằng quy nạp theo

n.

Bài toán 4. Cho k nguyên dương lớn hơn 1. Xét dãy số ( )n

a xác ñịnh bởi:

( )

2 2

0 1 2

1 1 1 2

4; ( 2)

2 8, 2n n n n n n

a a a k

a a a a a a n+ − − −

= = = −

= − + − + ∀ ≥

Chứng minh rằng 2n

a+ là số chính phương với mọi 0n ≥ .

Page 8: Chuyên đề hội thảo một số trường chuyên về dãy số

HỘ

HỘI CÁC TRƯỜNG THPT CHUYÊN KHU VỰC DUYÊN HẢI VÀ ðỒNG BẰNG BẮC BỘ HỘI THẢO KHOA HỌC LẦN THỨ IV

8

Lời giải. Gọi ,α β là hai nghiệm của phương trình 2 1 01

kt kt

α β

αβ

+ =− + = ⇒

=

+ Ta chứng minh bằng quy nạp theo n: ( )22 2 , 0n nF F

na nα β= + ∀ ≥ (1)

Dễ thấy (1) ñúng với n = 0, 1, 2.

Giả sử (1) ñúng ñến n. Ta có:

( )( ) ( )

( ) ( ) ( )1 1 2 2

1 1

1 1 2

2 2 24 4 4 4 4 4

4 4

2 2 2 2

.n n n n n n

n n

n n n n

F F F F F F

F F

a a a a

α β α β α β

α β

− − − −

+ +

+ − −− = − − − −

= + + − +

= +

Suy ra ( )1 1 1 124 2 2 22 n n n nF F F F

na α β α β+ + + += + + = + . Do ñó (1) ñược chứng minh.

+ Từ (1), ta có ( )22 22 2 n n n nF F F F

nn aa α β α β= + + = ++ = là số chính phương

(ñpcm).

Bài toán 4 (IMO 1981). Tìm giá trị lớn nhất của 2 2P m n= + , trong ñó m, n là

các số nguyên thoả mãn 1 ; 1981m n≤ ≤ và 2 2 2( ) 1n mn m− − = .

Lời giải. Ta xét các nghiệm nguyên dương ( , )x y của phương trình:

2 2 2( ) 1x xy y− − = (1) với x y> .

Gọi ( , )n m là một nghiệm như thế (n m> )

+ Xét bộ ( ; )m n n+ ta có: ( ) ( )2 22 2 2 2( ) ( ) ( ) 1m n m n n n n mn m+ − + − = − − − =

Suy ra ( ; )m n n+ cũng là một nghiệm của (1)

Rõ ràng (2; 1) là một nghiệm của (1), nên ta có các bộ sau cũng là nghiệm của

(1): (3,2);(5,3);(8,5);(13,8);(21,13);(34,21);.....

+ Xét bộ ( ; )m n m− ta có: ( ) ( )2 22 2 2 2( ) ( ) ( ) 1m m n m n m n mn m− − − − = − − − =

Suy ra ( ; )m n m− cũng là một nghiệm của (1).

- Nếu 2 2 22 ( ) 2 1 ( 1)m n m n m n n m m n mn m m≤ − ⇒ ≥ ⇒ − ≥ ⇒ − − > > (vô lí)

- Nếu m n m> − thì bộ ( ; )m n m− là một nghiệm của (1) nhỏ hơn nghiệm

( , )n m .

Page 9: Chuyên đề hội thảo một số trường chuyên về dãy số

HỘ

HỘI CÁC TRƯỜNG THPT CHUYÊN KHU VỰC DUYÊN HẢI VÀ ðỒNG BẰNG BẮC BỘ HỘI THẢO KHOA HỌC LẦN THỨ IV

9

Quá trình phải dừng lại và kết thúc ở nghiệm ( ,1) ( 1)n n > . Chú ý thêm rằng

(2, 1) là bộ duy nhât thoả mãn (1) mà 1n = .

Tóm lại tất cả các nghiệm nguyên dương của (1) sẽ là: ( )1;n n

F F−

với 2n ≥ .

Như vậy, giá trị lớn nhất của P bằng giá trị lớn nhất của 2 2

1n nF F

−+ với 1981.

nF ≤

Dãy các số hạng của dãy Fibonacci thoả mãn là: 1, 1, 2, 3, 5, 8, 13, 21, 34, 55,

89, 144, 233, 377, 610, 987, 1597.

Vậy giá trị nhỏ nhất của P bằng 2 21597 987+ .

Bài toán 6. Chứng minh rằng với mọi số nguyên dương 4n ≥ thì 1n

F + không

là số nguyến tố.

Lời giải. Ta có ñẳng thức 42 1 1 21n n n n nF F F F F− − + +− = (1)

Giả sử tồn tại 4n ≥ sao cho 1n

F + là số nguyên tố. Khi ñó từ (1) thì 1n

F + chia

hết ít nhất một trong các số 2 1 1 2; ; ;n n n nF F F F− − + + .

Nhưng 2 11 ; 1n n n nF F F F− −+ > + > nên hoặc 11|n nF F ++ hoặc 21|n nF F ++ .

Trong trường hợp ñầu tiên thì

( ) ( )1 1 11| 1| ( 1) ( 1) 1| 1n n n n n n n nF F F F F F F F− − −+ + ⇒ + + + − ⇒ + − (vô lí)

Trong trường hợp thứ hai thì

( )1 1 11| 1| 2( 1) 2 1| 2n n n n n n n nF F F F F F F F+ − −+ + ⇒ + + + − ⇒ + − (vô lí)

Vậy 1n

F + là hợp số với mọi 4n ≥ .

Bài toán 7. Cho dãy số nguyên ( )n

x : 0 13; 11x x= = và 2 12 7 0n n n

x x x n+ +

= + ∀ ≥ .

Tìm tất cả các số nguyên dương lẻ a sao cho với mọi m, n nguyên dương tồn tại

k nguyên dương mà k

nx a− chia hết cho 2m .

Lời giải. Bằng quy nạp ta chứng minh ñược 3 (mod8)n

x ≡

* Chọn 3; 1m n= = thì theo giả thiết tồn tại k+∈� sao cho 11 (mod8)k

a≡

Suy ra 3 (mod8) 1;3 (mod8)ka a≡ ⇒ ≡ .

* Ta sẽ chứng minh tất cả các số 1(mod8)a ≡ hoặc 3(mod8)a ≡ ñều thoả mãn

ñề bài.

Page 10: Chuyên đề hội thảo một số trường chuyên về dãy số

HỘ

HỘI CÁC TRƯỜNG THPT CHUYÊN KHU VỰC DUYÊN HẢI VÀ ðỒNG BẰNG BẮC BỘ HỘI THẢO KHOA HỌC LẦN THỨ IV

10

- Với 1m = thì ta chọn 1k = thoả mãn

- Với 2,3m = thì ta chọn k chẵn nếu 1(mod8)a ≡ và chọn k lẻ nếu

3(mod8)a ≡ .

- Xét 3m > : Ta chứng minh bài toán bằng quy nạp theo m.

Giả sử với 3m > tồn tại số m

k sao cho 2 2 .m mk km m

n nx a x a b− ⇒ − =M

- Nếu b chẵn thì 10 (mod2 )mk m

nx a

+− ≡ , khi ñó ta chọn 1m mk k+ = .

- Nếu b lẻ, chọn 21 2m

m mk k−

+ = + .

Khi ñó ( ) ( ) ( ) ( )2 32 2 21 1 . 1 . 1 ... 1 2 .m m

m

n n n n n mx x x x x q

− −

− = − + + + = ; m

q lẻ.

Vì 2 1 ( 1)( 1) 8n n nx x x q− = − + = với q lẻ.

Suy ra ( ) ( ) ( )2 2 2

1 2 2 2 11 2 . . 2m m m

m mk k m m

n n n n n mx a x x a a x x b a q

− − −+ +− = − + − = + M .

Vậy số 1mk + thoả mãn.

Kết luận: Tất cả các số a cần tìm là 1(mod8)a ≡ hoặc 3(mod8)a ≡ .

2. Sử dụng các tính chất của phương trình sai phân tuyến tính.

Một tính chất cơ bản có rất nhiều ứng dụng của dãy tuyến tính cấp hai là tính

chất sau ñây:

Cho dãy tuyến tính cấp hai ( )n

u : 2 1 , 1n n n

u au bu n+ +

= + ∀ ≥ . Khi ñó

( )2 1 2

2 1 3 1 2( ) , 1n

n n nu u u b u u u n

+ +− = − − ∀ ≥ .

Bài toán 1. Cho dãy ( )n

a xác ñịnh bởi:

1 2

2 1

20; 30

3 , 1n n n

a a

a a a n+ +

= =

= − ∀ ≥

Tìm tất cả các số nguyên dương n sao cho 11 5n n

a a ++ là số chính phương.

Lời giải.

* Áp dụng kết quả trên với dãy ( )n

a ta ñược:

2 2 2

1 1 1 3 2 1 1500 500n n n n n n

a a a a a a a a a+ − + −

− = − = ⇒ + =

Page 11: Chuyên đề hội thảo một số trường chuyên về dãy số

HỘ

HỘI CÁC TRƯỜNG THPT CHUYÊN KHU VỰC DUYÊN HẢI VÀ ðỒNG BẰNG BẮC BỘ HỘI THẢO KHOA HỌC LẦN THỨ IV

11

* Xét với 4n ≥ , ta có: ( )2 2 2

1 1 12n n n n n n

a a a a a a+ + +

+ = + + , nhưng

2 2 2

1 1 19 6n n n n n

a a a a a+ − −

= − +

Suy ra

( )

( )

2 2 2 2

1 1 1 1

2 2

1 1 1 1

2

1 1 2

2 2

1 1 1

1

2 9 6

2 3 (3 ) 3

5

5 500

5 500

n n n n n n n n n

n n n n n n n n n

n n n n n

n n n n

n n

a a a a a a a a a

a a a a a a a a a

a a a a a

a a a a

a a

+ + − −

+ − − −

+ − −

+ − −

+

+ = + + − +

= + − + + −

= + −

= + − +

= −

Do ñó ( )2

1 1 15 500 5 1n n n n n n

a a a a a a+ + +

+ = − < +

Từ dãy ( )n

a tăng và 4n ≥ ta có 1 180 470 650n n

a a+

+ ≥ + =

Suy ra ( ) ( )2 2

1 1 11 2( ) 1+ + +

+ + = + + + +n n n n n n

a a a a a a

( )2

1 1501 5 1+ +

> + + = +n n n n

a a a a

Vậy ( ) ( )2 2

1 1 15 1 1n n n n n n

a a a a a a+ + +

+ < + < + + nên 11 5n n

a a ++ không chính

phương.

Bằng phép thử trực tiếp với n = 1, 2, 3 ta ñược n = 3 là giá trị duy nhất cần tìm.

Bài toán 2. Cho dãy số nguyên ( )n

a : 1 22; 7a a= = và

2

1

1

1 1, 2

2 2n

n

n

aa n

a+

− < − ≤ ∀ ≥ .

Chứng minh rằng n

a là số lẻ với mọi 1n > .

Lời giải. Ta có 1 2 3 4 52; 7; 25; 89; 317a a a a a= = = = = .

Ta thấy 2 2

1

1 1

1 1

2 2n n

n

n n

a aa

a a+

− −

− < ≤ + . Do ñoạn 2 2

1 1

1 1;

2 2n n

n n

a a

a a− −

− +

có ñộ dài bằng 1

nên 1na + tồn tại một cách duy nhất, như vậy dãy ( )

na xác ñịnh duy nhất.

Ta có ngay 1 23 2 (*)n n n

a a a− −

= + với 3,4,5n = . Ta hy vọng (*) cũng chính là

công thức truy hồi cho dãy ( )n

a . Tuy nhiên việc chứng minh khẳng ñịnh này

Page 12: Chuyên đề hội thảo một số trường chuyên về dãy số

HỘ

HỘI CÁC TRƯỜNG THPT CHUYÊN KHU VỰC DUYÊN HẢI VÀ ðỒNG BẰNG BẮC BỘ HỘI THẢO KHOA HỌC LẦN THỨ IV

12

không hề ñơn giản. Một kĩ thuật hay dùng ở ñây là ñi xét một dãy ( )n

b có tính

chất như dãy ( )n

a rồi chứng minh n n

a b= .

Ta xét dãy số ( )n

b xác ñịnh như sau: 1 2

1 2

2; 7

3 2n n n

b b

b b b− −

= =

= +

Khi ñó với mọi 2n ≥ thì 2 2

1 1. ( 2)n

n n nb b b

+ −− = − .

Ta cũng dễ dàng có 2n

nb > bằng quy nạp

Từ ñó 2 2

2 2

1 1 1

1 1

2 1. 2

2

n

n n

n n n n

n n

bb b b b

b b

+ − +

− −

− = ⇒ − = < .

Do dãy ( )n

a xác ñịnh duy nhất nên n n

a b= với mọi 1n ≥ .

Khi ñó 1 23 2 , 3n n n

a a a n− −

= + ∀ ≥ và ta có ngay n

a là số lẻ với mọi 1n > (ñpcm).

Bài toán 3. Cho trước a,b nguyên dương và dãy ( )n

x xác ñịnh bởi:

0

1

1

, 0n n

x

x ax b n+

=

= + ∀ ≥

Chứng minh rằng với mọi cách chọn a,b thì trong dãy ( )n

x tồn tại vô hạn hợp số.

Lời giải. Giả sử n

x là hợp số với hữu hạn n. Gọi N là số nguyên dương lớn hơn

tất cả các giá trị n thoả mãn. Khi ñó m

x là số nguyên tố với mọi m N> .

Chọn số nguyên tố m

x p= không chia hết 1a − .

Gọi t là số thoả mãn (1 ) (mod )t a b p− ≡ , khi ñó 1 ( ) (mod )n n

x t a x t p+ − ≡ −

Tiếp tục quá trình và ñặt biệt với m=n ta ñược

11 ( ) (mod ) ( ) (mod )p

m p m mx t a x t p x t p−

+ − − ≡ − ≡ −

Hay 1 0 (mod )m p

x p+ − ≡ , ñiều này vô lí vì 1m px + − là số nguyên tố lớn hơn p.

Bài toán 4. Cho dãy số ( )n

a xác ñịnh bởi:

0

2

1

1

7 45 36, 0

2n n

n

a

a aa n

+

= + −

= ∀ ≥

.

Chứng minh rằng:

Page 13: Chuyên đề hội thảo một số trường chuyên về dãy số

HỘ

HỘI CÁC TRƯỜNG THPT CHUYÊN KHU VỰC DUYÊN HẢI VÀ ðỒNG BẰNG BẮC BỘ HỘI THẢO KHOA HỌC LẦN THỨ IV

13

a) n

a là số nguyên dương với mọi 0n ≥ .

b) 1. 1n n

a a + − là số chính phương với mọi 0n ≥ .

Lời giải.

a) Ta có 1 5a = và dễ thấy ngay dãy ( )n

a tăng ngặt.

Từ giả thiết ta có 2

12 7 45 36n n n

a a a+

− = − . Bình phương hai vế ta ñược:

2 2

1 17 9 0n n n n

a a a a+ +

− + + = (1)

Từ (1) ta cũng có 2 2

1 17 9 0n n n n

a a a a− −

− + + = (2)

Từ (1) và (2) suy ra ( ) ( )2 2

1 1 1 17 0+ − + −

− − − =n n n n n

a a a a a

1 17 (3) 1+ −

⇔ = − ∀ ≥n n n

a a a n

Từ (3) suy ra n

a là số nguyên dương với mọi 0n ≥ .

b) Từ (1) ta có ( ) ( )2

2 11 1 19 1 1

3n n

n n n n n n

a aa a a a a a +

+ + +

+ + = − ⇒ − =

.

Do ñó 1. 1n n

a a + − là số chính phương với mọi 0n ≥ (ñpcm).

Bài toán 5. Cho dãy số ( )n

a xác ñịnh bởi:

0

2

1

2

4 15 60, 0n n n

a

a a a n+

=

= + − ∀ ≥.

Chứng minh rằng số ( )2

18

5 nb a= + có thể biểu diễn thành tổng của ba só

nguyên duơng liên tiếp với mọi 1n ≥ .

Lời giải. Tương tự bài toán trên ta ñược: 1 18 0, 1n n n

a a a n+ −

− + = ∀ ≥

Từ ñó tìm ra ( ) ( )4 15 4 15n n

na = + + − .

Nhận xét : Với mỗi 1n ≥ ñều tồn tại *k ∈� sao cho:

( ) ( )4 15 4 15 15.n n

na k= + + − =

Khi ñó ( ) ( ) ( ) ( )2 2 2

2 24 15 4 15 15 4 15 4 15 15 2n n n n

k k + + − = ⇒ + + − = −

Page 14: Chuyên đề hội thảo một số trường chuyên về dãy số

HỘ

HỘI CÁC TRƯỜNG THPT CHUYÊN KHU VỰC DUYÊN HẢI VÀ ðỒNG BẰNG BẮC BỘ HỘI THẢO KHOA HỌC LẦN THỨ IV

14

Do ñó ( ) 2 2 2 22

18 3 2 ( 1) ( 1)

5 nb a k k k k= + = + = − + + + và ta có ñpcm.

Bài toán 6. Cho hai số thực a và b khác 0. Xét dãy số ( )n

u xác ñịnh bởi:

0 1

2 1

0; 1

, 0n n n

u u

u au bu n+ +

= =

= − ∀ ≥.

Chứng minh rằng nếu có bốn số hạng liên tiếp của dãy ( )n

u là số nguyên thì mọi

số hạng của dãy là số nguyên.

Lời giải.

Ta có ngay 2

1 2

n

n n nu u u b

+ +− = (1)

+ Giả sử tồn tại bốn số hạng 1 2 3; ; ;m m m m

u u u u+ + + là số nguyên, từ (1) suy ra mb và

1mb

+ là số nguyên. Từ ñó suy ra b là số hữu tỷ. Nhưng mb là số nguyên nên b

nguyên.

+ Ta chỉ cần phải chứng minh a ∈� : Ta có 2 1

3 2 1

m m m

m m m

u au bu

u au bu

+ +

+ + +

= −

= −

Nếu a là số vô tỉ thì từ 2 hệ thức trên suy ra 1 2 0m m

u u+ += = , suy ra b=0, mẫu

thuẫn. Vậy a là số hữu tỷ.

Xét dãy ña thức hệ số nguyên 0 1

2 1

( ) 0; ( ) 1( ) :

( ) ( ) ( ) 0n

n n n

Q x Q xQ x

Q x xQ x bQ x+ +

= =

= − =

Khi ñó [ ]( )n

Q x x∈� , môníc và deg ( ) 1nQ x n= − .

Ta có ( )n nQ a u= , ñặc biệt 1 1( )m mQ a u+ += . Vậy a là nghiệm hữu tỷ của ña thức

1 1( ) ( )m mP x Q x u+ += −

Vì ña thức [ ]( )P x x∈� và môníc nên a ∈� . Vậy mọi số hạng của dãy ( )nu là số

nguyên.

Bài toán 7. Cho m là số nguyên dương và dãy số ( )nx xác ñịnh bởi:

0 1

2

1 1

0;

, 1n n n

x x m

x m x x n+ −

= =

= − ∀ ≥

Page 15: Chuyên đề hội thảo một số trường chuyên về dãy số

HỘ

HỘI CÁC TRƯỜNG THPT CHUYÊN KHU VỰC DUYÊN HẢI VÀ ðỒNG BẰNG BẮC BỘ HỘI THẢO KHOA HỌC LẦN THỨ IV

15

Chứng minh rằng với một cặp 2( ; )a b ∈� , với a b≤ là một nghiệm của phương

trình 2 2

2

1

a bm

ab

+=

+ khi và chỉ khi tồn tại n∈� ñể 1( ; ) ( ; )n na b x x += .

Bài toán 8. Cho a,b là các số nguyên lớn hơn 1. Dãy ( )nx xác ñịnh bởi:

0 1

2 2 1 2 2

2 1 2 2 1

0; 1

( 1)n n n

n n n

x x

x ax x n

x bx x

− −

+ −

= =

= − ≥ = −

Chứng minh rằng với mọi số tự nhiên m,n thì 1 1. ...n m n m nx x x+ + − + chia hết cho

1.m mx x − .

3. Phương pháp sử dụng giới hạn của dãy số

Ta có một tính chất rất thú vị về giới hạn của các dãy số nguyên

“Nếu dãy số nguyên ( )na hội tụ về số a thì tồn tại 0n sao cho với mọi 0n n≥ thì

na a= ”

Bài toán 1. Cho dãy số nguyên 0( )n na ≥ thoả mãn ñiều kiện sau:

1 20 7 10 9, 0n n n

a a a n+ +

≤ + + ≤ ∀ ≥ .

Chứng minh rằng tồn tại số tự nhiên 0n sao cho với mọi 0n n≥ thì 0n

a = .

Lời giải.

ðây quả thực là một bài toán rất khó. Từ giả thiết dãy ( )na nguyên và kết luận

của bài toán giúp ta ñịnh hướng việc chứng minh lim 0na = .

ðặt { } { }1 1min , ,... ; max , ,...k k k k k k

x a a y a a+ += = thì ( )kx là dãy tăng; ( )ky là dãy

giảm và k kx y≤ với mọi k.

Dãy ( )na bị chặn nên hai dãy ( )kx , ( )ky bị chặn. Do ñó cả hai dãy ñều hội tụ.

Giả sử lim ;limn nx x y y= = .

Do ;k kx y ∈� nên tồn tại 0n sao cho với mọi 0n n≥ thì ;n n

x x y y= = .

Tồn tại 0n n≥ sao cho 2 1; , 8 10 9n n n

a y a a x x y+ +

= ≥ ⇒ + ≤ (1).

Cũng vậy, Tồn tại 0m n≥ sao cho 2 1; , 10 8 0m m m

a x a a y x y+ +

= ≤ ⇒ + ≥ (2).

Page 16: Chuyên đề hội thảo một số trường chuyên về dãy số

HỘ

HỘI CÁC TRƯỜNG THPT CHUYÊN KHU VỰC DUYÊN HẢI VÀ ðỒNG BẰNG BẮC BỘ HỘI THẢO KHOA HỌC LẦN THỨ IV

16

Từ (1), (2) và x,y là số nguyên suy ra 0x y= = .

Do ñó lim 0na = , suy ra ñpcm.

Bài 2. Cho số tự nhiên 3c ≥ . Xét dãy số ( )na xác ñịnh bởi:

1

11 1 2

2n

n n

a c

aa a n−

=

= − + ∀ ≥

Chứng minh rằng tồn tại số tự nhiên 0n sao cho với mọi 0n n≥ thì 3n

a = .

Lời giải. Bài toán ñược giải quyết nếu ta chứng minh ñuợc lim 3na = .

+ Dễ chứng minh bằng quy nạp 3 1na n≥ ∀ ≥ .

+ Từ ñó suy ra ngay dãy ( )na giảm.

Vậy dãy ( )na hội tụ. Chuyển qua giới hạn ta ñược lim 3na = (ñpcm).

4. Phương pháp sử dụng tính tuần hoàn của dãy số dư

ðịnh lý. Cho dãy số nguyên ( )na thoả mãn

trong ñó là các số nguyên và m là số

nguyên dương lớn hơn 1. Gọi là số dư trong phép chia cho m. Khi ñó dãy

( )nr tuần hoàn.

Bài toán 1. Chứng minh rằng tồn tại vô hạn số hạng của dãy Fibonacci chia hết

cho 2012.

Lời giải.

Ta sẽ chứng minh bài toán tổng quát với mọi số tự nhiên n, tồn tại vô hạn số

hạng của dãy Fibonacci chia hết cho n.

Xét các cặp số dư khi chia hai số hạng liên tiếp trong dãy Fibonacci theo modulo

n. 0 1 1 2 2 3( , );( , );( , )....F F F F F F

Vì dãy Fibonacci là vô hạn mà chỉ có 2n khả năng cho mỗi cặp số dư theo

modulo n nên tồn tại 1( , )i i

F F+

thoả mãn i i m

F F+

≡ và 1 1i i mF F

+ + +≡ (mod n) với

m+∈� .

Xét i > 1, ta có: 1 1 1 1 (mod )i i i i m i m i m

F F F F F F n− + + + + + −

= − ≡ − =

Page 17: Chuyên đề hội thảo một số trường chuyên về dãy số

HỘ

HỘI CÁC TRƯỜNG THPT CHUYÊN KHU VỰC DUYÊN HẢI VÀ ðỒNG BẰNG BẮC BỘ HỘI THẢO KHOA HỌC LẦN THỨ IV

17

Quá trình cứ tiếp tục dẫn ñến (mod ) 0j j m

F F n j+

≡ ∀ ≥

Suy ra 0 20 ... (mod )m m

F F F n≡ ≡ ≡ ≡ , tức là có vô hạn các số km

F thoả mãn yêu

cầu bài toán. Vậy bài toán ñược chứng minh.

Bài toán 2. Cho dãy số ( )na xác ñịnh bởi:

0 1 2

3 2 1

29; 105; 381

3 2 (1) 0n n n n

a a a

a u u a n+ + +

= = =

= + + ∀ ≥.

Chứng minh rằng với mỗi số nguyên dương m luôn tồn tại số tự nhiên n sao cho

các số 1 2, 1, 2n n na a a+ +− − ñều chia hết cho m.

Lời giải.

Ta bổ sung thêm bốn số hạng của dãy là 1 2 3 48, 2, 1, 0a a a a− − − −= = = = .

Giả sử (mod ); 0 1n n na r m r m≡ ≤ ≤ − .

Xét các bộ ba 1 2( , , )n n nr r r+ + . Khi ñó tồn tại hai số nguyên p q< sao cho:

1 1 1 1

2 2 2 2

(mod )

(mod )

(mod )

p q p q

p q p q

p q p q

r r a a m

r r a a m

r r a a m

+ + + +

+ + + +

= ≡

= ⇒ ≡

= ≡

Kết hợp với (1) ta ñược: (mod ) q k p k

a a m k+ +≡ ∀ .

Do ñó (mod ) k q p ka a m k− +≡ ∀ . ðặt *t q p= − ∈� thì (mod ) k k ta a m k+≡ ∀

Suy ra * (mod ) ; k k hta a m k h+≡ ∀ ∈ ∀ ∈� � .

Nói riêng ta ñược 4 4

3 3

2 2

0 (mod )

1 (mod )

2 (mod )

ht

ht

ht

a a m

a a m

a a m

− −

− −

− −

≡ ≡

≡ ≡ − ≡ ≡

Với h ñủ lớn thì 4ht − ∈� . Khi ñó ñặt 4n ht= − ta ñược:

1 20(mod ), 1(mod ), 2(mod )n n na m a m a m+ +≡ ≡ ≡

Do ñó các số 1 2, 1, 2n n na a a+ +− − ñều chia hết cho m (ñpcm).

Bài toán 3. Cho dãy ( )nx , xác ñịnh bởi:

Page 18: Chuyên đề hội thảo một số trường chuyên về dãy số

HỘ

HỘI CÁC TRƯỜNG THPT CHUYÊN KHU VỰC DUYÊN HẢI VÀ ðỒNG BẰNG BẮC BỘ HỘI THẢO KHOA HỌC LẦN THỨ IV

18

Chứng minh rằng:

a) Mọi số hạng của dãy ñều là số nguyên dương.

b) Có vô số nguyên dương n sao cho có 4 chữ số tận cùng là 2003.

c) Không tồn tại số nguyên dương n sao cho có 4 chữ số tận cùng là

2004.

Bài toán 4. Cho dãy số ( )nx xác ñịnh bởi:

0 1

2

1 1

;

5 3 1n n n n

x a x b

x x x a n+ −

= =

= − + ∀ ≥.

Chứng minh rằng với mọi cách chọn các số nguyên a,b thì dãy trên hoặc không

có số nào chia hết cho 2011 hoặc có vô số số chia hết cho 2011.

Bài toán 5 Cho dãy số ( )nx xác ñịnh bởi:

0 1

2 1

22; 9

. 1 0n n n

x x

x x x n+ +

= =

= + ∀ ≥.

a) Cho p là số nguyên tố. Chứng minh rằng nếu tồn tại kx pM thì tồn tại

m k> sao cho mx pM

b) Giả sử kx pM và 1k ≥ . Chứng minh rằng dãy ( )nx tuần hoàn kể từ chỉ số

n k≥ .

III- KHAI THÁC MỘT SỐ BÀI TOÁN VỀ DÃY SỐ NGUYÊN QUA CÁC

KÌ THI OLIMPIC.

1. Bài toán 1 (TST VN 2011). Cho dãy số nguyên dương ( )na xác ñịnh bởi:

0 11, 3a a= = và 2

12 1 n

n

n

aa

a

++

= +

với mọi 0n ≥ .

Chứng minh rằng 22 1 2n

n n na a a+ +− = với mọi số tự nhiên n . Trong ñó [ ]x kí hiệu số nguyên lớn nhất không vượt quá x .

Lời giải.

Page 19: Chuyên đề hội thảo một số trường chuyên về dãy số

HỘ

HỘI CÁC TRƯỜNG THPT CHUYÊN KHU VỰC DUYÊN HẢI VÀ ðỒNG BẰNG BẮC BỘ HỘI THẢO KHOA HỌC LẦN THỨ IV

19

Hướng thứ nhất. Ta thử dự ñoán dãy số ( )na là dãy tuyến tính dạng

2 1n n na pa qa r+ += + + , với mọi 0n ≥ . Theo công thức truy hồi ta tính ñược

2 3 410, 34, 116a a a= = = . Khi ñó từ 2 1n n na pa qa r+ += + + , với mọi 0n ≥ ta ñược hệ: 3 10 4

10 3 34 2

34 10 116 0

p q r p

p q r q

p q r r

+ + = =

+ + = ⇔ = − + + = =

Do ñó 2 14 2n n na a a+ += − 0n∀ ≥ . (1) Ta sẽ chứng minh dãy ( )n

a thỏa mãn công thức truy hồi (1) bằng hai cách sau

ñây: Cách 1. Ta chứng minh bằng quy nạp công thức truy hồi (1). Thật vậy, trước

hết từ ñẳng thức 2

12 1 n

n

n

aa

a

++

= +

, bằng quy nạp ta suy ra 1 2 , 0n na a n+ > ∀ ≥ nên

1 02 ... 2 2 , 1n n

n na a a n−> > > = ∀ ≥ . (2) Ta dễ thấy (1) ñúng với 0n = , ta giả sử (1) ñúng ñến 0n k= ≥ tức là:

( )2 22 1 12 1 2 1 1 1

1

2 24 2 2 ... 2 .kk k k k

k k k k k k k k k

k k

a a a aa a a a a a a a a

a a

+ + −+ + + + + −

+

+ += − ⇒ = ⇒ − = − = =

(3)

Ta có ( )2

2 2 22 1 1 1 1 1

1 1

22 2k k k

k k k k k k k k

k k

a a aa a a a a a a a

a a

++ + + − + −

+ +

− = − ⇒ − = −

( )2 22 1 22

1 1 1 11 1 1 1

42 4 42 4 2 4k k kk k k k

k k k k

k k k k

a a aa a a aa a a a

a a a a

+ + +++ − + −

+ + + +

−⇒ − = − ⇒ − = −

( )22 2 2 2 21 11 12 2 2 2

2 1 11 1 1 1 1 1 1 1

44 4.2 24 2 4

k kk k kk k k k k

k k k

k k k k k k k k

a a aa a a a aa a a

a a a a a a a a

− ++ −+ + + +

+ + −

+ + + + + + + +

−⇒ − = + − = + = + = +

(do (3)). Kết hợp với (2) ta ñược: 2 2 2 2

2 2 2 22 1 2 1 3

1 1 1 1

4 2 1 4 2 1 1k k k k

k k k k k

k k k k

a a a aa a a a a

a a a a

+ + + ++ + + + +

+ + + +

< − < + ⇒ − = + = + =

. Do ñó ñẳng

thức (1) ñúng với 1n k= + . Vậy ñẳng thức (1) ñúng với mọi 0n ≥ . Từ ñẳng thức (1) ta suy ra ñược:

( )2 22 1 12 1 2 1 1 1

1

2 24 2 2 ... 2 .nn n n n

n n n n n n n n n

n n

a a a aa a a a a a a a a

a a

+ + −+ + + + + −

+

+ += − ⇒ = ⇒ − = − = =

Cách 2. Bây giờ ta xây dựng dãy ( )n

b thỏa mãn ñiều kiện: 0 11, 3,b b= = và 2 14 2n n nb b b+ += −

0n∀ ≥ . Từ cách xác ñịnh dãy ( )nb ta ñược:

2 22 1 12 1 1 1

1

2 22 2n n n n

n n n n n n

n n

b b b bb b b b a a

b b

+ + −+ + + −

+

+ += ⇒ + = +

( ) ( )2 2 22 1 1 1 2 0 12 ... 2 2n n

n n n n n nb b b b b b b b b+ + + −⇒ − = − = = − =

Page 20: Chuyên đề hội thảo một số trường chuyên về dãy số

HỘ

HỘI CÁC TRƯỜNG THPT CHUYÊN KHU VỰC DUYÊN HẢI VÀ ðỒNG BẰNG BẮC BỘ HỘI THẢO KHOA HỌC LẦN THỨ IV

20

21

2

2n

n

n

n n

bb

b b

++⇒ = + (4)

Bằng quy nạp dễ thấy dãy ( )nb là một dãy tăng và do ñó:

21 2 1 2 14 2 2 2 ... 2 2 2n n n

n n n n n nb b b b b b b− − − −= − > > > > = ⇒ > (5) Từ (4) và (5) ta suy ra

2 2 2 2 21 1 1 1 1

2 2

21 1 1 , 0

n

n n n n n

n n

n n n n n n

b b b b bb b n

b b b b b b

+ + + + ++ +

< = + < + ⇒ = + = + ∀ ≥

(6)

Từ (6) suy ra dãy ( )nb cũng thỏa mãn:

0 11, 3b b= = và 2

12 1 , 0n

n

n

bb n

b

++

= + ∀ ≥

.

Do ñó ta ñược , 0n na b n= ∀ ≥ . Vì vậy 22 1 2n

n n na a a+ +− = với mọi số tự nhiên n . Hướng thứ hai. Ta sẽ dự ñoán ñẳng thức 2 14 2n n na a a+ += − 0n∀ ≥ như sau. Giả sử ta chứng minh ñược ñẳng thức 2

2 1 2n

n n na a a+ +− = với mọi số tự nhiên n . Khi ñó ta có:

( )2 2 2 1 1 2 02 1 1 1

1 1

2 2 22 ... 4n n n n

n n n n n n

n n

a a a a a aa a a a a a

a a a

+ + −+ + + −

+

+ + +− = − ⇒ = = = =

⇒ 2 14 2n n na a a+ += − . ðể chứng minh công thức truy hồi trên ta thực hiện giống như cách 1, cách 2 trong chứng minh theo hướng thứ nhất.

Nhận xét 1. Từ cách xác ñịnh của dãy 0 11, 3a a= = và 2

12 1 n

n

n

aa

a

++

= +

với mọi

0n ≥ , bằng phương pháp quy nạp ta chỉ ra 2 , 0n

na n≥ ∀ ≥ . Khi ñó ta có: 2 2

1 12

21

n

n n

n

n n

a aa

a a

+ ++

+= + =

. Từ ñó ta ñề xuất bài toán sau:

Bài 1.2 Cho dãy số nguyên dương ( )na xác ñịnh bởi:

0 11, 3a a= = và 2

12

2n

n

n

n

aa

a

++

+=

với mọi 0n ≥ .

Chứng minh rằng 22 1 2n

n n na a a+ +− = với mọi số tự nhiên n . Trong ñó [ ]x kí hiệu số nguyên lớn nhất không vượt quá x .

Bài tập tương tự. Bài 1.3(IMO Shortlist 1988) Cho dãy số ( )n

a thỏa mãn ñiều kiện:

0 12, 7a a= = và 2

12

1

2n

n

n

aa

a

++

= +

với mọi 0n ≥ .

Chứng minh rằng na là số lẻ và ( )22 1 2

n

n n na a a+ +− = − với mọi 0n ≥ .

Bài 1.4 (VMO 1997, bảng B) Cho dãy số nguyên ( ) ,na n ∈ � ñược xác ñịnh như

sau:

0 1 2 11, 45, 45 7n n na a a a a+ += = = − với mọi 0n ≥ .

Page 21: Chuyên đề hội thảo một số trường chuyên về dãy số

HỘ

HỘI CÁC TRƯỜNG THPT CHUYÊN KHU VỰC DUYÊN HẢI VÀ ðỒNG BẰNG BẮC BỘ HỘI THẢO KHOA HỌC LẦN THỨ IV

21

a) Tính số các ước nguyên dương của 21 2n n n

a a a+ +− theo n . b) Chứng minh rằng 2 11997 4.7n

na++ là số chính phương với mọi n .

Bài 1.5 Dãy số ( ) ,n

a n ∈� ñược xác ñịnh như sau:

0 1

21

2

1

2, 0,1,2,...n

n

n

a a

aa n

a

++

= =

+= =

Chứng minh rằng mọi số hạng của dãy số ñều là số nguyên dương. Bài 1.6 Dãy số ( ) ,

na n ∈� ñược xác ñịnh như sau:

0 1

21

2

1

2, 0,1,2,...n

n

n

a a

aa n

a

++

= =

+= =

Chứng minh rằng 22 1 2, 0,1, 2,..n n na a a n+ +− = ∀ =

Bài 1.7

Gọi a là nghiệm dương của phương trình 2 2012 1 0x x− − = . Xét dãy số ( )nx : [ ]10 , 01;

n nax nx x

+= ∀ ≥= . Tìm phần dư khi chia 2012x cho

2012.

2. Bài toán 2 (China South East Mathematical Olimpiad 2011). Cho dãy thỏa mãn ñiều kiện: và

với mọi Chứng minh rằng với mọi số nguyên dương số là một số chính phương.

Lời giải. Hướng thứ nhất. Ta tính ñược

nên ta dự ñoán , trong ñó

Ta sẽ tìm tính chất của dãy . ðầu tiên ta thử dự ñoán dãy là tuyến tính tức là với mọi Do ñó ta có hệ sau:

⇔ ⇔ .

Từ ñó ta có hướng giải như sau: Ta lập dãy ñược xác ñịnh như sau: và với mọi Sau ñó ta sẽ chứng

minh với mọi Cách 1. Từ dãy truy hồi của và ta ñược:

.

Page 22: Chuyên đề hội thảo một số trường chuyên về dãy số

HỘ

HỘI CÁC TRƯỜNG THPT CHUYÊN KHU VỰC DUYÊN HẢI VÀ ðỒNG BẰNG BẮC BỘ HỘI THẢO KHOA HỌC LẦN THỨ IV

22

Khi ñó ta kiểm tra ñược ngay ñẳng thức với mọi

Cách 2. Ta chứng minh bằng quy nạp ñẳng thức trên. Thật vậy, từ cách xác ñịnh của dãy ta chỉ ra ñược:

(1) Theo công thức truy hồi của dãy và (1) ta có:

hay . Do ñó với mọi Hướng thứ hai. Từ công thức truy hồi của dãy ta tìm ñược công thức tổng

quát: . Khi ñó ta chứng minh ñược:

Bài tập tương tự. Bài 2.1 (Problem M1174, Kvant). Cho dãy số nguyên ñược xác ñịnh bởi:

và với mọi Chứng minh rằng với mọi số nguyên dương , số là một số chính phương. Bài 2.2. Cho dãy số ( )

na xác ñịnh bởi 0 10; 1a a= = và

1 13( 1)

2nn n n

aa a+ −

− += −

với mọi n nguyên dương. Chứng minh rằng n

a là số chính phương với mọi

0n ≥ .

3. Bài toán 3 (VMO 2011). Cho dãy số nguyên ( )n

a xác ñịnh bởi:

0 11, 1a a= = − và 2 16 5n n na a a+ += + với mọi 0n ≥ . Chứng minh rằng 2012 2010a − chia hết cho 2011 . Lời giải. Hướng thứ nhất. Xét phương trình ñặc trưng của dãy số 2 16 5n n na a a+ += + là:

Page 23: Chuyên đề hội thảo một số trường chuyên về dãy số

HỘ

HỘI CÁC TRƯỜNG THPT CHUYÊN KHU VỰC DUYÊN HẢI VÀ ðỒNG BẰNG BẮC BỘ HỘI THẢO KHOA HỌC LẦN THỨ IV

23

2 6 5 0 3 14x x x− − = ⇔ = ± , ta thấy nghiệm này lẻ nên công thức của na sẽ phức

tạp. Do bài toán chỉ yêu cầu chứng minh 2012 2010a − chia hết cho 2011 nên ta có thể thay dãy ( )n

a bởi dãy ( )nb sao cho ( )mod 2011 , n=0,1,2,...

n na b≡ ∀

Bây giờ ta sẽ chọn dãy ( )nb thỏa mãn: 0 11, 1b b= = − và ( )2 16 5

n n nb b k b+ += + + với

mọi 0n ≥ và k là số ta sẽ chọn sau. Khi ñó phương trình ñặc trưng sẽ là: 2 6 5 0 ' 14x x k k− − − = ⇒ ∆ = + , ñể '∆ là số chính phương ta sẽ chọn 2011k = . Như

vậy ta xây dựng dãy ( )nb ñược xác ñịnh như sau: 0 11, 1b b= = − và

2 16 2016n n nb b b+ += + với mọi 0n ≥ . Phương trình ñặc trưng 2 6 2016 0 48; 42x x x x− − = ⇔ = = − , khi ñó

( )1 248 42nn

nb c c= + − và kết hợp với 0 11, 1b b= = − suy ra

( ) ( )41 49

48 42 90 41.48 49. 4290 90

n nn n

n nb b= + − ⇔ = + − (1)

suy ra 2012 2012201290 41.48 49.42b = + .

Do 2011 là số nguyên tố nên theo ñịnh lí Fecma nhỏ ta có: ( )201148 48 mod 2011 ,≡ ( )201142 42 mod 2011≡ do vậy ta thu ñược:

( ) ( ) ( ) ( )2 22012 201290 1 41.48 49.42 90 mod 2011 0 mod 2011 1 0 mod 2011b b+ ≡ + + ≡ ⇒ + ≡

hay 2012 2010b − chi hết cho 2011. Từ cách xác ñịnh của dãy ( )n

a và ( )nb ta có: ( )mod 2011 , n=0,1,2,...

n na b≡ ∀ Do ñó

2012 2010a − chi hết cho 2011. Hướng thứ hai. Từ dãy truy hồi 2 16 5n n na a a+ += + ta sẽ tìm công thức tổng quát cho na .

+) Phương trình ñặc trưng của dãy trên là: .

Khi ñó , sử dụng giả thiết ta ñược:

(2)

+) ðặt ta ñược:

Chú ý: ,

trong ñó: (3)

Và (4) Dễ dàng chứng minh ñược: (5)

Ta có suy ra với mọi . Do ñó theo (3) và (4) ta ñược:

Page 24: Chuyên đề hội thảo một số trường chuyên về dãy số

HỘ

HỘI CÁC TRƯỜNG THPT CHUYÊN KHU VỰC DUYÊN HẢI VÀ ðỒNG BẰNG BẮC BỘ HỘI THẢO KHOA HỌC LẦN THỨ IV

24

và , từ ñây

kết hợp với (5) ta thu ñược: (6)

Ta có nên theo ñịnh lí Fecma nhỏ và (6) ta ñược: hay ta ñược

chia hết cho . Nhận xét 2. Trong (1) nếu ta thay 2011n = ta ñược:

( ) ( ) ( )20112011

201190 41.48 49. 42 41.48 49.42 mod 2011 90 mod 2011b = + − ≡ − ≡ − ,

suy ra 2011 20111 2011 1 2011b a+ ⇒ +M M . Từ ñó ta có bài toán sau:

Bài 3.1 Cho dãy số nguyên ( )na xác ñịnh bởi:

0 11, 1a a= = − và 2 16 5n n na a a+ += + với mọi 0n ≥ . Chứng minh rằng 2011 2010a − chia hết cho 2011 . Nhận xét 3. Nếu trong (1) thay n bởi số nguyên tố 5p > ta ñược:

( ) ( ) ( )90 41.48 49. 42 41.48 49.42 mod 90 mod 1pp

p pb p p b p= + − ≡ − ≡ − ⇒ + M . Từ ñó ta

có bài toán sau: Bài 3.2 Cho dãy số nguyên ( )n

a xác ñịnh bởi:

0 11, 1a a= = − và 2 16 2016n n na a a+ += + với mọi 0n ≥ . Chứng minh 1pa + chia hết cho p , trong ñó p là một số nguyên tố lớn hơn 5.

Nhận xét 4. Nếu trong (1) thay n bởi số 1p + , trong ñó p là số nguyên tố lớn hơn 5 ta ñược:

( ) ( ) ( )

( )

11 2 21

1 1

90 41.48 49. 42 41.48 49.42 mod 180900 mod

90 2010 2010

pp

p

p p

b p p

b p b p

+++

+ +

= + − ≡ + ≡

⇒ − ⇒ −M M

Bài 3.3 Cho dãy số nguyên ( )na xác ñịnh bởi:

0 11, 1a a= = − và 2 16 2016n n na a a+ += + với mọi 0n ≥ . Chứng minh rằng 1 2010pa + − chia hết cho p , trong ñó p là một số nguyên tố lớn

hơn 5. Nhận xét 5. Bây giờ ta sẽ ñưa ra bài toán tổng quát cho bài toán 3. Trong cách chứng minh theo hướng thứ nhất bài toán 3 ta thấy số nguyên tố thỏa mãn

hay là số chính phương . Do vậy trong bài toán 3 ta có thể thay số nguyên tố bằng số nguyên tố thỏa mãn là số chính phương . Khi ñó ta có bài toán sau: Bài 3.4 Cho dãy số nguyên ( )n

a xác ñịnh bởi:

0 11, 1a a= = − và 2 16 5n n na a a+ += + với mọi 0n ≥ . Tìm tất cả các số nguyên tố sao cho là số chính phương và

chia hết cho . Lời giải. Trước hết ta tìm tất cả các số nguyên tố sao cho là số chính phương .

Page 25: Chuyên đề hội thảo một số trường chuyên về dãy số

HỘ

HỘI CÁC TRƯỜNG THPT CHUYÊN KHU VỰC DUYÊN HẢI VÀ ðỒNG BẰNG BẮC BỘ HỘI THẢO KHOA HỌC LẦN THỨ IV

25

là số chính phương khi và chỉ khi hoặc

.

TH1. . Khi ñó ta có: từ

ñó xảy ra hai khả năng sau: +) Nếu ( ) thì

hay ta ñược

. +) Nếu ( ) thì

hay ta ñược

.

TH2. . Khi ñó ta có: từ

ñó xảy ra hai khả năng sau: +) Nếu ( ) thì

hay ta ñược

. +) Nếu ( ) thì

hay ta ñược

, do nguyên tố nên ta loại trường hợp .

Vậy tất cả các số nguyên tố cần tìm có dạng: .

Chú ý các số nguyên tố có dạng trên là tồn tại vì theo ñịnh lí Dirichlet với hai số nguyên dương nguyên tố cùng nhau thì tồn tại vô hạn các số nguyên tố dạng . Ta trở lại chứng minh bài 3.4. Ta sẽ dựa theo hướng giải thứ nhất của bài toán 3. Do là số chính phương

nên tồn tại số nguyên dương sao cho . Xét dãy số ( )nb ñược xác ñịnh như sau: 0 11, 1b b= = − và ( )2

2 16 9n n nb b m b+ += + − với mọi 0n ≥ ,

dễ thấy là số thỏa mãn . Khi ñó phương trình ñặc trưng của dãy là: 2 26 9 0 3 ; 3x x m x m x m− − + = ⇔ = − = + suy ra ( ) ( )

2 2

1 23 3nb c m c m= + + −

và kết hợp với 0 11, 1b b= = − ta ñược:

( )( ) ( )( )

( ) ( ) ( )( ) ( ) ( ) ( ) ( ) ( )1 1 2 2

1

2 4 3 4 3

2 4 3 4 3 4 3 4 3 mod

n n

n

p p

p

mb m m m m

mb m m m m m m m m p+ +

+

= + + + − −

⇒ = + + + − − ≡ + + + − −

Page 26: Chuyên đề hội thảo một số trường chuyên về dãy số

HỘ

HỘI CÁC TRƯỜNG THPT CHUYÊN KHU VỰC DUYÊN HẢI VÀ ðỒNG BẰNG BẮC BỘ HỘI THẢO KHOA HỌC LẦN THỨ IV

26

⇒ (1) Bằng quy nạp ta dễ dàng chứng minh ñược:

với mọi (2) Mặt khác ta có: suy ra

Từ (1) và (2) ta ñược: . Vậy tất cả các số nguyên tố thỏa mãn yêu cầu bài toán là:

. Nhận xét 6. Theo hướng chứng minh thứ hai của bài toán 3 ta có:

nên theo ñịnh lí nhỏ Fecma ta có:

Do ñó số nguyên tố thỏa mãn chia hết cho khi và chỉ khi

chia hết cho hay là số chính phương . Do ñó ta thu ñược bài toán sau: Bài 3.5 Cho dãy số nguyên ( )n

a xác ñịnh bởi:

0 11, 1a a= = − và 2 16 5n n na a a+ += + với mọi 0n ≥ .

Tìm tất cả các số nguyên tố sao cho chia hết cho . Lời giải. Theo nhận xét 6 thì tất cả số nguyên tố sao cho chia hết cho là số nguyên tố thỏa mãn là số chính phương .

Ta có là số chính phương khi và chỉ khi hoặc

.

TH1. . Khi ñó ta có:

từ ñó xảy ra hai khả năng sau:

+) Nếu ( ) thì

hay ta ñược

. +) Nếu ( ) thì

hay ta ñược

.

TH2. . Khi ñó ta có:

từ ñó xảy ra hai khả năng sau:

Page 27: Chuyên đề hội thảo một số trường chuyên về dãy số

HỘ

HỘI CÁC TRƯỜNG THPT CHUYÊN KHU VỰC DUYÊN HẢI VÀ ðỒNG BẰNG BẮC BỘ HỘI THẢO KHOA HỌC LẦN THỨ IV

27

+) Nếu ( ) thì

hay ta ñược

. +) Nếu ( ) thì

hay ta ñược

, do nguyên tố nên ta loại trường hợp .

Vậy tất cả các số nguyên tố cần tìm có dạng:

Nhận xét 7. Theo hướng chứng minh thứ hai của bài toán 3 ta có

(1)

Và (2) Ta có với mọi ta có nên từ (1) và (2) ta

ñược: và . Mặt khác ta có

suy ra

Do ñó số nguyên tố thỏa mãn chia hết cho khi và chỉ khi

chia hết cho hay là số chính phương . Do ñó ta thu ñược bài toán sau: Bài 3.6 Cho dãy số nguyên ( )na xác ñịnh bởi:

0 11, 1a a= = − và 2 16 5n n na a a+ += + với mọi 0n ≥ .

Tìm tất cả các số nguyên tố 5p > sao cho chia hết cho . Từ cách giải bài 3.5 ta suy ra lời giải của các bài toán sau: Bài 3.7 Cho dãy số nguyên ( )n

a xác ñịnh bởi:

0 11, 1a a= = − và 2 16 5n n na a a+ += + với mọi 0n ≥ .

Tìm tất cả các số nguyên tố 5p > sao cho chia hết cho . Bài 3.8 Cho dãy số nguyên ( )n

a xác ñịnh bởi:

0 11, 1a a= = − và 2 16 5n n na a a+ += + với mọi 0n ≥ .

Tìm tất cả các số nguyên tố 5p > sao cho chia hết cho . Bây giờ ta tiếp tục suy nghĩ bài toán 3 xem nó phục thuộc vào giá trị ban ñầu như thế nào? Tại sao người ta lại lấy và ta có thể tìm ñược ñiều kiện của ñể kết quả bài toán không thay ñổi không? Sau khi nghiên cứu vấn ñề này tôi ñã thu ñược bài toán sau: Bài 3.9 Cho là các số nguyên cho trước. Dãy số nguyên ñược xác ñịnh như sau:

Page 28: Chuyên đề hội thảo một số trường chuyên về dãy số

HỘ

HỘI CÁC TRƯỜNG THPT CHUYÊN KHU VỰC DUYÊN HẢI VÀ ðỒNG BẰNG BẮC BỘ HỘI THẢO KHOA HỌC LẦN THỨ IV

28

và với mọi số tự nhiên . Tìm tất cả các số nguyên sao cho chia hết cho . Lời giải.

Ta xây dựng dãy ( )nb ñược xác ñịnh như sau: 0 11, 1b b= = − và

2 16 2016n n nb b b+ += + với mọi 0n ≥ . Phương trình ñặc trưng 2 6 2016 0 48; 42x x x x− − = ⇔ = = − , khi ñó

( )1 248 42nn

nb c c= + − và kết hợp với 0 1,b a b b= = suy ra

( ) ( ) ( ) ( )42 48

48 42 90 42 .48 48 . 4290 90

n nn n

n n

a b a bb b a b a b

+ −= + − ⇔ = + + − − (1)

suy ra ( ) ( )2012 2012201290 42 .48 48 .42b a b a b= + + − .

Do 2011 là số nguyên tố nên theo ñịnh lí Fecma nhỏ ta có: ( )201148 48 mod 2011 ,≡ ( )201142 42 mod 2011≡ do vậy ta thu ñược:

( ) ( ) ( ) ( ) ( )

( ) ( )( )

2 22012

2012

90 1 42 .48 48 .42 90 mod 2011 0 mod 2011

90 1 90 5 6 1 mod 2011

b a b a b

b a b

+ ≡ + + − + ≡

⇒ + ≡ + +

hay 2012 2010b − chi hết cho 2011 khi và chỉ khi .

Từ cách xác ñịnh của dãy ( )na và ( )n

b ta có: ( )mod 2011 , n=0,1,2,...n n

a b≡ ∀ Do ñó

2012 2010a − chi hết cho 2011 khi và chỉ khi . Từ phương trình ñồng dư này ta tìm ñược , trong ñó là các số nguyên tùy ý. Tương tự lời giải của các bài toán trên ta có thể ñưa ra các bài tập sau: Bài 3.10 Cho là các số nguyên cho trước. Dãy số nguyên ñược xác ñịnh như sau:

và với mọi số tự nhiên . Tìm tất cả các số nguyên sao cho chia hết cho . Bài 3.11 Cho là các số nguyên cho trước. Dãy số nguyên ñược xác ñịnh như sau:

và với mọi số tự nhiên . Tìm tất cả các số nguyên sao cho chia hết cho , trong ñó là một số nguyên tố sao cho là số chính phương . Bài 3.12 Cho là các số nguyên cho trước. Dãy số nguyên ñược xác ñịnh như sau:

và với mọi số tự nhiên . Tìm tất cả các số nguyên sao cho chia hết cho , trong ñó là một số nguyên tố sao cho là số chính phương . Bây giờ ta nghiên cứu bài toán theo hướng sau: thay giả thiết

với mọi số tự nhiên bởi với mọi

Page 29: Chuyên đề hội thảo một số trường chuyên về dãy số

HỘ

HỘI CÁC TRƯỜNG THPT CHUYÊN KHU VỰC DUYÊN HẢI VÀ ðỒNG BẰNG BẮC BỘ HỘI THẢO KHOA HỌC LẦN THỨ IV

29

số tự nhiên và , trong ñó là các số nguyên. Khi ñó ta sẽ tìm ñiều kiện cho các số sao cho chia hết cho hay tổng quát hơn ta tìm ñiều kiện cho , trong ñó là số nguyên tố lớn hơn 5 sao cho chia hết cho . Ta sẽ suy nghĩ theo hướng giải thứ nhất của bài toán 3 . Khi ñó ta có bài toán sau: Bài 3.13 Cho dãy số nguyên ñược xác ñịnh như sau:

và với mọi số tự nhiên ; trong ñó là các số nguyên cho trước.

Tìm tất cả các số nguyên sao cho chia hết cho ; trong ñó là số nguyên tố lớn hơn 5, thỏa mãn là số chính phương và không chia hết cho . Lời giải.

Do là số chính phương nên tồn tại số nguyên dương sao cho . Xét dãy số ( )n

b ñược xác ñịnh như sau:

và với mọi , dễ thấy là

số thỏa mãn . Khi ñó phương trình ñặc trưng của dãy là: , nên ta ñược:

, kết hợp với ta thu ñược:

.

Do nên chia hết cho khi và chỉ khi . Mặt khác bằng quy nạp dễ thấy ,

với mọi số tự nhiên . Vậy chia hết cho khi và chỉ khi .

Bài 3.14 Cho dãy số ñược xác ñịnh như sau: và với mọi

Chứng minh rằng chia hết cho . Lời giải. Ta thấy phương trình ñặc trưng vô nghiệm nên việc thực hiện theo hướng giải thứ 2 của bài toán 3 là rất phức tạp. Bây ta viết công thức truy hồi của dãy dưới dạng nên ta xét dãy xác ñịnh bởi . Từ ñó dễ thấy

nên . Tương tự như cách suy luận của các bài toán trên ta hoàn toàn có thể ñưa ra các bài toán tổng quát cho bài 3.14

Page 30: Chuyên đề hội thảo một số trường chuyên về dãy số

HỘ

HỘI CÁC TRƯỜNG THPT CHUYÊN KHU VỰC DUYÊN HẢI VÀ ðỒNG BẰNG BẮC BỘ HỘI THẢO KHOA HỌC LẦN THỨ IV

30

ðể kết thúc việc nghiên cứu dạng bài toán 3, bạn ñọc có thể dựa vào hướng giải thứ nhất của bài toán 3 ñể giải các bài tập sau: Bài 3.15 Dãy số nguyên ñược xác ñịnh như sau:

và với mọi số tự nhiên . Chứng minh rằng chia hết cho . Bài 3.16 Dãy số nguyên ñược xác ñịnh như sau:

và với mọi số tự nhiên . Chứng minh rằng chia hết cho . 4. BÀI TOÁN 4 (IMO 2010). Tìm tất cả các dãy số nguyên dương thỏa mãn tính chất là số chính phương với mọi số nguyên dương

. Lời giải. Trước hết ta chứng minh bổ ñề sau: Bổ ñề. Nếu là một số nguyên tố thỏa mãn thì . Chứng minh. Ta xét hai trường hợp sau: TH1. suy ra , trong ñó là một số nguyên. Chọn số ñủ lớn sao cho và không chia hết cho . Lấy ta ñược

và không chia hết cho

Mặt khác và là các số chính phương nên:

suy ra .

TH2. và không chia hết cho . Chọn sao cho và không chia hết cho . Khi ñó ta ñược:

và không chia hết cho .

Mặt khác và là các số chính phương nên:

suy ra .

Vậy bổ ñề ñược chứng minh. Trở lại bài toán ta thấy nếu thì với số nguyên tố ñủ lớn theo bổ ñề ta ñược , ñiều này chỉ xảy ra khi . Cũng theo bổ ñề trên dễ thấy với mọi số nguyên dương . Từ ñây ta ñược hoặc . +) Nếu hay . Do

(ta thấy không xảy ra trường hợp vô lí). Do ñó bằng quy nạp ta suy ra với mọi số nguyên dương . Thử lại ta thấy thỏa mãn.

Page 31: Chuyên đề hội thảo một số trường chuyên về dãy số

HỘ

HỘI CÁC TRƯỜNG THPT CHUYÊN KHU VỰC DUYÊN HẢI VÀ ðỒNG BẰNG BẮC BỘ HỘI THẢO KHOA HỌC LẦN THỨ IV

31

+) Nếu hay . Do (ta thấy

không xảy ra trường hợp vô lí). Do ñó bằng quy nạp ta suy ra với mọi số nguyên dương . ðiều này không xảy ra vì với ñủ lớn thì vô lí. Vậy trường hợp này không xảy ra.

Vậy , với mọi nguyên dương và là một hằng số nguyên không âm.

Dựa theo bài toán 4 ta thu ñược bài toán sau: Bài 4.1 Tìm tất cả các dãy số nguyên dương thỏa mãn tính chất

là lập phương của một số nguyên dương với mọi số nguyên dương . Lời giải. Trước hết ta chứng minh bổ ñề sau: Bổ ñề. Nếu là một số nguyên tố thỏa mãn thì . Chứng minh. Ta xét hai trường hợp sau: TH1. suy ra , trong ñó là một số nguyên. Chọn các số ñủ lớn sao cho và không chia hết cho . Lấy ta ñược

và không chia hết cho và không chia hết cho .

Do là lập phương của một số nguyên dương nên . (1)

Mặt khác và không chia hết cho . Theo giả thiết ta có là lập phương của một số nguyên dương nên . (2) Từ (1) và (2) ta ñược . TH2. và không chia hết cho . ðặt , trong ñó

là số nguyên không chia hết cho . Chọn các số ñủ lớn sao cho và không chia hết cho . Khi ñó lấy và . Khi ñó ta ñược:

và không chia hết cho . và không chia hết cho .

Do là lập phương của một số nguyên dương nên . (3)

Mặt khác và không chia hết cho . Theo giả thiết ta có là lập phương của một số nguyên dương nên . (4) Từ (3) và (4) ta ñược . Vậy bổ ñề ñược chứng minh. Trở lại bài toán ta thấy nếu thì với số nguyên tố ñủ lớn theo bổ ñề ta ñược , ñiều này chỉ xảy ra khi .

Page 32: Chuyên đề hội thảo một số trường chuyên về dãy số

HỘ

HỘI CÁC TRƯỜNG THPT CHUYÊN KHU VỰC DUYÊN HẢI VÀ ðỒNG BẰNG BẮC BỘ HỘI THẢO KHOA HỌC LẦN THỨ IV

32

Cũng theo bổ ñề trên dễ thấy với mọi số nguyên dương . Từ ñây ta ñược hoặc . +) Nếu hay . Do

(ta thấy không xảy ra trường hợp (vô lí). Do ñó bằng quy nạp ta suy ra với mọi số nguyên dương

. Thử lại ta thấy thỏa mãn. +) Nếu hay . Do

(ta thấy không xảy ra trường hợp vô lí). Do ñó bằng quy nạp ta suy ra với mọi số nguyên dương . ðiều này không xảy ra vì với ñủ lớn thì vô lí. Vậy trường hợp này không xảy ra. Vậy , với mọi nguyên dương và là một hằng số nguyên không âm. Bài 4.2 (tổng quát của bài toán 4.1) . Tìm tất cả các dãy số nguyên dương thỏa mãn tính chất là lũy thừa bậc của một số nguyên dương với mọi số nguyên dương , trong ñó là số nguyên dương lớn hơn cho trước.

Bây giờ ta sẽ tổng quát bài toán 4 (IMO 2010) theo hướng khác. Trong bài toán 4 ta thay giả thiết bằng là số chính phương với mọi số nguyên dương ; trong ñó là một số nguyên dương cho trước. Khi ñó ta phát biểu bài toán như sau: Bài toán A. Cho là một số nguyên dương cho trước. Tìm tất cả các dãy số nguyên dương thỏa mãn tính chất là số chính phương với mọi số nguyên dương .

Trong quá trình tìm lời giải bài toán A, tôi nhận thấy là bài toán rất khó và có thể không giải ñược trong trường hợp là số nguyên dương bất kì. Tuy nhiên khi ta xét trong một số trường hợp ñặt biệt ta sẽ giải ñược bài toán. Các bài toán trong trường hợp là các số ñặc biệt sẽ lần lượt xét ở dưới ñây và chúng cũng là những bài toán rất khó.

Trước hết ta chứng minh các bổ ñề sau: Bổ ñề 1. Cho là hai số nguyên tố khác nhau và là một số nguyên dương cho trước. Khi ñó luôn tồn tại số nguyên dương sao cho chia hết cho nhưng không chia hết cho . Chứng minh. Chọn suy ra tồn tại số nguyên dương sao cho (1) Chọn sao cho không chia hết cho . Lấy và kết hợp với (1) ta ñược:

Page 33: Chuyên đề hội thảo một số trường chuyên về dãy số

HỘ

HỘI CÁC TRƯỜNG THPT CHUYÊN KHU VỰC DUYÊN HẢI VÀ ðỒNG BẰNG BẮC BỘ HỘI THẢO KHOA HỌC LẦN THỨ IV

33

. Do ñó chia hết cho nhưng không chia hết cho . Từ ñó ta chọn ta ñược kết luận của bổ ñề.

Bổ ñề 2. Cho là hai số nguyên tố khác nhau và là một số nguyên dương cho trước. Khi ñó luôn tồn tại số nguyên dương sao cho chia hết cho

nhưng không chia hết cho . Chứng minh. Theo ñịnh lí Euler ta có Chọn suy ra tồn tại số nguyên dương sao cho (2) Chọn sao cho không chia hết cho . Lấy và kết hợp với (2) ta ñược:

. Do ñó chia hết cho nhưng không chia hết cho . Từ ñó ta chọn ta ñược kết luận của bổ ñề.

Bổ ñề 3. Cho là hai số nguyên tố khác nhau và là các số nguyên dương cho trước. Khi ñó luôn tồn tại số nguyên dương sao cho chia hết cho

nhưng không chia hết cho . Chứng minh. Theo ñịnh lí Euler ta có

Chọn suy ra tồn tại số nguyên dương sao cho (3) Chọn sao cho không chia hết cho . Lấy và kết hợp với (3) ta ñược:

. Do ñó chia hết cho nhưng không chia hết cho . Từ ñó ta chọn ta ñược kết luận của bổ ñề. Bổ ñề 4. Cho và là một số nguyên tố. Chứng minh rằng tồn tại hai số nguyên dương sao cho không là số chính phương. Chứng minh. Chọn sao cho không chia hết cho , trong ñó . ðặt

và dễ thấy không chia hết cho . Từ ñó suy ra . ðặt ta

ñược: . Bây giờ ta chọn . Khi ñó

Page 34: Chuyên đề hội thảo một số trường chuyên về dãy số

HỘ

HỘI CÁC TRƯỜNG THPT CHUYÊN KHU VỰC DUYÊN HẢI VÀ ðỒNG BẰNG BẮC BỘ HỘI THẢO KHOA HỌC LẦN THỨ IV

34

không là số chính phương vì không chia hết cho . Vậy bổ ñề ñược chứng minh. Bổ ñề 5. Cho số nguyên dương , là một số nguyên tố và là một số nguyên dương. Chứng minh rằng tồn tại các số nguyên dương sao cho số

không là số chính phương. Chứng minh.

Do nên có ước nguyên tố khác . Giả sử là số nguyên dương sao cho chia hết cho nhưng không chia hết cho . Khi ñó tồn tại không chia hết cho sao cho . Gọi là số tự nhiên sao cho , trong ñó không chia hết cho . Chọn ta ñược:

(4) Theo bổ ñề 3 tồn tại số nguyên dương sao cho , trong ñó không chia hết cho . Do ñó với , ñược chọn như trên và kết hợp với (4) ta thu ñược

Không là số chính phương vì không chia hết cho . Vậy bổ ñề ñược chứng minh. Bổ ñề 6. Nếu là một số nguyên tố thỏa mãn thì . Ta xét hai trường hợp sau: TH1. suy ra , trong ñó là một số nguyên. Theo bổ ñề 1 ta chọn ñược sao cho chia hết cho và không chia hết cho . Từ ñó ta ñược:

và không chia hết cho nhưng không chia hết cho .

Mặt khác và là các số chính phương nên:

suy ra .

TH2. và không chia hết cho . Theo bổ ñề 2 thì tồn tại số nguyên dương sao cho: chia hết cho nhưng không chia hết cho . Khi ñó ta ñược:

chia hết cho nhưng không chia hết cho và không chia hết cho .

Mặt khác và là các số chính phương nên:

Page 35: Chuyên đề hội thảo một số trường chuyên về dãy số

HỘ

HỘI CÁC TRƯỜNG THPT CHUYÊN KHU VỰC DUYÊN HẢI VÀ ðỒNG BẰNG BẮC BỘ HỘI THẢO KHOA HỌC LẦN THỨ IV

35

suy ra .

Vậy bổ ñề ñược chứng minh. Bổ ñề 7. Nếu thì . Chứng minh.

Giả sử là số nguyên tố ñủ lớn sao cho . Khi ñó từ giả thiết suy suy ra vô lí. Vậy bổ ñề ñược chứng minh. Bổ ñề 8. Cho là một số nguyên tố lớn hơn 3. Chứng minh rằng không tồn tại các số tự nhiên và thỏa mãn ñẳng thức . Chứng minh. Ta xét hai trường hợp sau: TH1. Nếu là số lẻ khi ñó có ước nguyên tố lẻ, vô lí. TH2. Nếu thì . Từ ñây suy ra tồn tại hai số tự nhiên sao cho

. Do ñó ta ñược vô lí.

Vậy bổ ñề ñược chứng minh. Bổ ñề 9. Cho là một số nguyên tố. Chứng minh rằng không tồn tại các số tự nhiên và thỏa mãn ñẳng thức . Chứng minh. Ta xét các trường hợp sau: TH1. Nếu là số lẻ thì có ước nguyên tố lẻ, vô lí. TH2. Nếu là một số chẵn, . (i) Nếu là một số chẵn, ta có nên chia hết cho hay . Tiếp theo ta có: . Từ ñây suy ra tồn tại hai số tự nhiên sao cho . Do ñó ta ñược vô lí. (ii) Nếu là một số lẻ thì thay vào phương trình ta ñược:

suy ra là nghiệm nguyên dương của phương trình Pell loại 2: (1) Xét phương trình Pell loại 1 liên kết với phương trình trên: (2) Phương trình (2) có nghiệm nguyên dương nhỏ nhất là . Khi ñó ta xét hệ

phương trình: Hệ phương trình có nghiệm duy nhất là . Do ñó là tất cả các nghiệm của phương trình (1) và ñược xác ñịnh như sau:

Từ ñó suy ra , kết hợp với là số lẻ nên ta có các trường hợp sau: +) suy ra trường hợp này không xảy ra.

Page 36: Chuyên đề hội thảo một số trường chuyên về dãy số

HỘ

HỘI CÁC TRƯỜNG THPT CHUYÊN KHU VỰC DUYÊN HẢI VÀ ðỒNG BẰNG BẮC BỘ HỘI THẢO KHOA HỌC LẦN THỨ IV

36

+) suy ra trường hợp này không xảy ra. +) ( ) ( )n h m sn 6h 3 3u 2 64 .8 0 mod16 ;7 1 49 1 2 mod16= + = ⇒ = ≡ + = + ≡

suy ra trường hợp này không xảy ra. Vậy bổ ñề ñược chứng minh. Bài 4.3 Cho là một số nguyên tố. Xét tất cả các dãy số nguyên dương thỏa mãn tính chất là một số chính phương với mọi số nguyên dương .

a) Chứng minh rằng với mỗi số nguyên dương tồn tại số tự nhiên sao cho .

b) Chứng minh rằng không thể xảy ra ñẳng thức với mọi số nguyên dương .

Lời giải a) Nếu số có một ước nguyên tố , với là một số nguyên dương

nào ñó. Khi ñó theo bổ ñề 6 ta có hay . Do ñó

chỉ có hai ước là hoặc . Do ñó tồn tại số tự nhiên sao cho .

b) Giả sử ñẳng thức xảy ra với mọi số nguyên dương . Khi ñó ta có hoặc .

+) Nếu hay . Do (ta thấy

không xảy ra trường hợp vô lí, do bổ ñề 7). Do ñó bằng quy nạp ta suy ra với mọi số

nguyên dương .Theo bổ ñề 3 dãy xác ñịnh như vậy không thỏa mãn. +) Nếu hay . Do

(ta thấy không xảy ra trường hợp vô lí, do bổ ñề 7).

Do ñó bằng quy nạp ta suy ra với mọi số nguyên dương . ðiều này không xảy ra vì với ñủ lớn thì vô lí. Vậy trường hợp này

không xảy ra. Vậy ta có kết luận của phần b Bài 4.4 Cho là một số nguyên tố thỏa mãn tính chất với mọi số nguyên dương thì và không có dạng , trong ñó là một số nguyên dương.

Xét tất cả các dãy số nguyên dương thỏa mãn tính chất là một số chính phương với mọi số nguyên dương .

a) Chứng minh rằng tồn tại số nguyên dương sao cho , với mọi số nguyên dương .

b) Chứng minh rằng với mọi Lời giải.

Page 37: Chuyên đề hội thảo một số trường chuyên về dãy số

HỘ

HỘI CÁC TRƯỜNG THPT CHUYÊN KHU VỰC DUYÊN HẢI VÀ ðỒNG BẰNG BẮC BỘ HỘI THẢO KHOA HỌC LẦN THỨ IV

37

a) Theo bài 4.3 ta có với mỗi số nguyên dương thì tồn tại số tự nhiên sao cho , từ ñây suy ra và và giả sử . Khi ñó sẽ xẩy ra hai trường hợp sau: TH1. Nếu và ⇒ , theo giả thiết nếu thì

sẽ có ước nguyên tố khác . Mặt khác theo bổ ñề 6 của thì các ước nguyên tố của chỉ là hoặc vô lí nên . TH2. Nếu và ⇒ , theo giả thiết nếu thì

sẽ có ước nguyên tố khác . Mặt khác theo bổ ñề 6 của thì các ước nguyên tố của chỉ là hoặc vô lí nên . Do vậy ta chứng minh ñược . Theo bài 4.3 b thì không thỏa mãn nên là một số nguyên dương. b)Theo phân a ta có với mọi nên xảy ra hai trường hợp: TH1. Nếu tồn tại số nguyên dương sao cho suy ra theo bổ ñề 7 ta thu ñược vô lí. TH2. Nếu không tồn tại số nguyên dương sao cho

thì hay với mọi

Bài 4.5 Cho là một số nguyên tố lớn hơn , thỏa mãn không có dạng và , trong ñó là một số nguyên dương. Tìm tất cả các dãy số nguyên dương thỏa mãn tính chất là số chính phương với mọi số nguyên dương . Lời giải.

Nếu số có một ước nguyên tố , với là một số nguyên dương

nào ñó. Khi ñó theo bổ ñề 6 ta có hay . Do ñó

chỉ có hai ước là hoặc . Do ñó với mỗi số nguyên dương thì tồn tại số tự nhiên sao cho , từ ñây suy ra và

và giả sử . Khi ñó sẽ xẩy ra hai trường hợp sau: TH1. Nếu và ⇒ . +) Nếu thì theo bổ ñề 9 ta có không có dạng suy ra số sẽ có ước nguyên tố khác . Mặt khác theo bổ ñề 6 của thì các ước nguyên tố của chỉ là hoặc vô lí . Nên trường hợp không xảy ra. +) Nếu thì theo giả thiết không có dạng nên nó sẽ có một ước nguyên tố khác . Mặt khác theo bổ ñề 6 của thì các ước nguyên tố của chỉ là hoặc vô lí . Nên trường hợp không xảy ra. Vậy ta có . TH2. Nếu và ⇒ .

Page 38: Chuyên đề hội thảo một số trường chuyên về dãy số

HỘ

HỘI CÁC TRƯỜNG THPT CHUYÊN KHU VỰC DUYÊN HẢI VÀ ðỒNG BẰNG BẮC BỘ HỘI THẢO KHOA HỌC LẦN THỨ IV

38

+) Nếu thì theo bổ ñề 8 số sẽ không có dạng suy ra số nó sẽ có một ước nguyên tố khác . Mặt khác theo bổ ñề 6 của thì các ước nguyên tố của chỉ là hoặc vô lí nên trường hợp . +) Nếu thì theo giả thiết không có dạng nên nó sẽ có một ước nguyên tố khác . Mặt khác theo bổ ñề 6 của thì các ước nguyên tố của chỉ là hoặc vô lí . Nên trường hợp không xảy ra. Vậy trong mọi trường hợp ta có . Do vậy ta có . Nếu thì ñẳng thức xảy ra với mọi số nguyên dương . Khi ñó ta có hoặc . +) Nếu hay . Do

(ta thấy không xảy ra trường hợp vô lí, do bổ ñề 7). Do ñó bằng quy nạp ta suy ra với mọi số nguyên dương . Theo bổ ñề 3 dãy xác ñịnh như vậy không thỏa mãn ñiều kiện

là số chính phương với mọi số nguyên dương . +) Nếu hay . Do

(ta thấy không xảy ra trường hợp vô lí, do bổ ñề 7).

Do ñó bằng quy nạp ta suy ra với mọi số nguyên dương . ðiều này không xảy ra vì với ñủ lớn thì vô lí. Vậy trường hợp này

không xảy ra. Vậy nên ta có ñẳng thức thức xảy ra với mọi số nguyên dương . Khi ñó ta có hoặc . +) Nếu hay . Do

(ta thấy không xảy ra trường hợp vô lí, do bổ ñề 7). Bằng quy nạp ta suy ra với mọi số nguyên dương . Theo bổ ñề 5 nếu dãy xác ñịnh như vậy không thỏa mãn ñiều kiện là số chính phương với mọi số nguyên dương . Do ñó hay với mọi số nguyên dương , thử lại thấy thỏa mãn. +) Nếu hay . Do

(ta thấy không xảy ra trường hợp vô lí, do bổ ñề 7). Bằng quy nạp ta suy ra với mọi số nguyên dương . ðiều này không xảy ra vì với ñủ lớn thì vô lí. Vậy trường hợp này không xảy ra.

Kết luận. Vậy dãy số ñược xác ñịnh như sau: với mọi số nguyên dương , trong ñó là một số nguyên dương tùy ý.

Page 39: Chuyên đề hội thảo một số trường chuyên về dãy số

HỘ

HỘI CÁC TRƯỜNG THPT CHUYÊN KHU VỰC DUYÊN HẢI VÀ ðỒNG BẰNG BẮC BỘ HỘI THẢO KHOA HỌC LẦN THỨ IV

39

TÌM SỐ HẠNG TỔNG QUÁT CỦA DÃY TRUY HỒI TUYẾN TÍNH CẤP 2 ðỂ GIẢI QUYẾT MỘT SỐ BÀI TOÁN VỀ DÃY SỐ

Trường THPT chuyên Hưng Yên

Phần I: Tìm số hạng tổng quát của dãy truy hồi tuyến tính cấp 2. I. LÝ THUYẾT:

ðó là các dãy số thực có dạng n 2 n 1 nu au bu+ += + (*) với mọi n 0≥ , trong

ñó a và b là các hằng số thực. Cách xác ñịnh số hạng tổng quát của dãy như sau: Xét phương trình ẩn t sau ñây: 2t at b 0− − = (**) ñược gọi là phương trình ñặc trưng của (*). Phương trình có biệt thức 2a 4b∆ = + .

Trường hợp 1: 2a 4b 0∆ = + > khi ñó (**) có hai nghiệm thực phân biệt

1 2t ; t . Số hạng tổng quát của (*) có dạng n nn 1 2u x.t y.t= + , với mọi n 0≥ và x, y

là hai số thực tuỳ ý; x và y sẽ hoàn toàn xác ñịnh khi cho trước 0u và 1u .

Trường hợp 2: 2a 4b 0∆ = + = khi ñó (**) có một nghiệm kép thực t. Số hạng tổng quát của (*) có dạng n n 1

nu x.t y.nt −= + , với mọi n 0≥ ( ở ñây ta qui

ước 10 0− = ) và x, y là hai số thực tuỳ ý; x và y sẽ hoàn toàn xác ñịnh khi cho trước 0u và 1u .

Trường hợp 3: 2a 4b 0∆ = + < , ( **) có hai nghiệm phức. Thuật toán làm trong trường hợp này như sau:

Bước 1: Giải phương trình 2t at b 0− − = và nhận ñược nghịêm phức

a iz .

2

+ −∆=

Bước 2: ðặt r = | z | là module của z, còn Argzϕ = , ta nhận ñược n

nu r (pcosn qsin n )= ϕ − ϕ với mọi p, q là các số thực. Bước 3: Xác ñịnh p, q theo các giá trị cho trước 0 1u ;u .

Về cơ sở lí thuyết của cách làm trên ñược chứng minh bằng kiến thức của ñại số tuyến tính. Ở ñây, tôi xin trình bày chứng minh trường hợp 1 và trường hợp 2 bằng kiến thức trung học phổ thông.

Trường hợp 1: 0∆ > (**) có hai nghiệm phân biệt 1 2t , t khi ñó theo ñịnh

lí Vi-et ta có: 1 2

1 2

t t a

t t b

+ =

= −. Khi ñó

n 1 1 2 n 1 2 n 1u (t t )u t t u+ −= + − 2 n

n 1 1 n 2 n 1 n 1 2 n 1 1 n 2 2 1 1 0u t u t (u t u ) t (u t u ) ... t (u t u )+ − − −⇔ − = − = − = = − .

Như vậy nn 1 1 n 2 1 1 0u t u t (u t u )+ − = − (1);

Tương tự nn 1 2 n 1 1 2 0u t u t (u t u )+ − = − (2). Trừ từng vế (2) cho (1) ta có:

Page 40: Chuyên đề hội thảo một số trường chuyên về dãy số

HỘ

HỘI CÁC TRƯỜNG THPT CHUYÊN KHU VỰC DUYÊN HẢI VÀ ðỒNG BẰNG BẮC BỘ HỘI THẢO KHOA HỌC LẦN THỨ IV

40

n n1 2 n 1 2 0 1 1 1 0 2(t t )u (u t u )t (u t u )t− = − − − . Do 1 2t t≠ nên

n n1 2 0 1 1 0n 1 2

1 2 1 2

(u t u ) (u t u )u t t

t t t t

− −= −

− −.

Vậy nu có dạng n nn 1 2u x.t y.t= + với x,y là hai số thực.

Trường hợp 2: 0∆ = khi ñó 2a

b4

−= , (**) có nghiệm kép

at

2= . Ta có

2 nn 1 n n 1 n 1 n n n 1 1 0u 2t.u t u u tu t(u tu ) ... t (u tu )+ − + −= − ⇔ − = − = = −

Như vậy nn 1 n 1 0u tu t (u tu )+ − = − (3);

Tương tự n 1n n 1 1 0u tu t (u tu )−

−− = − (4);

n 2n 1 n 2 1 0u tu t (u tu )−

− −− = − (5);

……………………………. 1 0 1 0u tu u tu− = − (n+3).

Nhân hai vế của (4) với t, hai vế của (5) với 2t , …, hai vế của (n+3) với nt và cộng lại ta ñược: n 1 n

n 1 0 1 0u t .u n.t .(u tu )++ = + − . Do ñó nu có dạng

n n 1xt yn.t −+ với x, y là hai số thực.

II. CÁC VÍ DỤ: Ví dụ 1: X¸c ®Þnh sè h¹ng tæng qu¸t cña d·y sè tho¶ m·n:

0 1

n 2 n 1 n

u 1,u 2.1 2

u u u , n 03 3+ +

= =

= − + ∀ ≥

Giải:

Phương trình ñặc trưng 2 1 2t t 0

3 3+ − = của dãy có hai nghiệm thực phân

biệt là 1 2

2t , t 1

3= = − . Do ñó

nn

n

2u x. y.( 1)

3

= + −

với x, y∈� . Ta lại có:

0

1

x y 1 9xu 1 52

u 2 4x y 2 y 53

+ = == ⇔ ⇔

= − = = −

. Vậy n nn

9 2 4u ( ) ( 1) , n 0.

5 3 5= − − ∀ ≥

Trong công thức tổng quát (*), khi chọn những giá trị a và b thích hợp ta có thể ñưa ra ñề toán thuộc vào trường hợp 2 và 3 ñược nói ñến ở trên. Hoặc là bằng cách biến ñổi nu ta cũng có thể ñưa ra ñược những ñề toán khá hay.

Chẳng hạn trong ñề bài trên:

*) ðặt nn

1u

v= , 0 0 1 1

1u 1 v 1;u 2 v

2= ⇒ = = ⇒ = .

Page 41: Chuyên đề hội thảo một số trường chuyên về dãy số

HỘ

HỘI CÁC TRƯỜNG THPT CHUYÊN KHU VỰC DUYÊN HẢI VÀ ðỒNG BẰNG BẮC BỘ HỘI THẢO KHOA HỌC LẦN THỨ IV

41

n n 1n 2 n 1 n n 2

n 2 n 1 n n n 1

1 2 1 1 2 3v vu u u v , n 0

3 3 v 3v 3v v 2v+

+ + +

+ + +

−= − + ⇔ = − + ⇔ = ∀ ≥

−.

Như vậy ta có ñề toán mới như sau:

Xác ñịnh số hạng tổng quát của dãy số thoả mãn: 0 1

n n 1n 2

n n 1

1v 1,v

23v v

v , n 0.v 2v

++

+

= =

− = ∀ ≥−

*) ðặt n nu ln v= , 20 0 1 1u 1 v e;u 2 v e .= ⇒ = = ⇒ =

2n3

n 2 n 1 n n 2 n 1 n n 2n 1

1 2 1 2 vu u u ln v ln v ln v v , n 0

3 3 3 3 v+ + + + +

+

= − + ⇔ = − + ⇔ = ∀ ≥ .

Như vậy ta có ñề toán mới như sau:

Xác ñịnh số hạng tổng quát của dãy số thoả mãn: 2

0 1

2n3

n 2n 1

v e,v e

vv , n 0.

v+

+

= =

= ∀ ≥

Ví dụ 2: Tìm nu biết 1

2n 1 n n

u

u a.u b.u c+

= α

= + +. Trong ñó: 2a b 1, 0,a 1− = α > > .

Giải:

Từ (*) 2 2n 1 n n n 1 n nu a.u b.u c u a.u b.u c 0+ += + + ⇔ − = + >

2 2 2 2 2 2n 1 n n n 1 n n 1 n n(u a.u ) b.u c u 2.a.u .u a u b.u c+ + +⇔ − = + ⇔ − + = +

2 2 2 2 2n 1 n n 1 n n 1 n n 1 nu 2.a.u .u u (a b) c u 2.a.u .u u c+ + + +⇔ − + − = ⇔ − + = n 1∀ ≥ 2 2 2 2n 1 n n 1 n n n n 1 n 1u 2.a.u .u u u 2.a.u .u u+ + − −⇒ − + = − + 2 2n 1 n 1 n n 1 n 1 n 1 n 1 n 1 n n 1u u 2.a.u (u u ) (u u ).(u 2.a.u u ) 0+ − + − + − + −⇒ − = − ⇔ − − + = (**)

Bằng quy nạp ta CM ñược: 1 2 n n 1 n 1u u ... u ... u u 0+ −< < < < ⇒ − ≠

n 1 n n 1u 2.a.u u 0+ −⇒ − + =

Từ ñó: 2

1 2

n 1 n n 1

u ;u a. b. c

u 2.a.u u 0+ −

= α = α + α +

− + =

Ta tính ñược nu theo dạng (1).

Phần II: Áp dụng việc tìm số hạng tổng quát của dãy truy hồi tuyến tính cấp 2 trong một số bài toán về dãy số.

Ví dụ 1: Cho dãy n(u ) thoả mãn 0 1

n 2 n 1 n

u 0;u 1.1

u u u ,n 02+ +

= =

= − ≥

Tìm nlimu .

Page 42: Chuyên đề hội thảo một số trường chuyên về dãy số

HỘ

HỘI CÁC TRƯỜNG THPT CHUYÊN KHU VỰC DUYÊN HẢI VÀ ðỒNG BẰNG BẮC BỘ HỘI THẢO KHOA HỌC LẦN THỨ IV

42

Giải:

Phương trình ñặc trưng của dãy là 2 1t t 0

2− + = có một nghiệm phức là

1 it

2

+= ; |t|=

1

2, Argt=

4

π. Do ñó số hạng tổng quát của dãy có dạng

n

n

2 n nu (x.cos ysin ),n 0;x, y .

2 4 4

π π= + ≥ ∈

Từ giả thiết 0 1u 0;u 1= = ta suy ra x= 0; y= 2. Vậy số hạng tổng quát của

dãy là

nn nn

n 2 2 nu 2sin ( ) sin ,limu 0

4 2 4( 2)

π π= = = vì n n

2| u | 0.

( 2)≤ →

Ví dụ 2: Cho dãy n(x ) thoả mãn 0 1

n 1 n n 1

x 1;x 5.

x 6x x , n 1+ −

= =

= − ∀ ≥

Hãy tìm n nlim x { 2x }.

Giải: Bằng phương pháp xác ñịnh số hạng tổng quát ở trên, ta xác ñinh ñược số

hạng tổng quát của dãy là n nn

2 2 2 2x ( )(3 2 2) (3 2 2)

4 4

+ −= + + −

. Hay

2n 1 2n 1n

1 1x ( 2 1) ( 2 1)

2 2 2 2+ += + + −

2n 1 2n 1n

2n 1 2n 1 2n 1n

2n 1 2n 12n 1 k 2n 1 k k 2n 1 k k

n 2n 1 2n 1k 0 k 0

1 12x ( 2 1) ( 2 1)

2 2 21

2x ( 2 1) ( 2 1) ( 2 1)2

12x ( 2 1) C ( 2) C ( 2) ( 1)

2

+ +

+ + +

+ ++ + − + −

+ += =

⇔ = + + −

⇔ − − = + − −

⇔ − − = − −

∑ ∑

2n 1 n

2t 1 2n 1 2k 1 2t 1 (n t)2n 1 2n 1

2t 1 1 t 0

C ( 2) C 2 .+

+ + − − + −

+ ++ = =

= = ∈∑ ∑ �

n2n 1 2t 1 (n t)

n 2n 1t 0

2x ( 2 1) C 2+ + −+

=

⇒ = − +∑

{ }

n2n 1 2t 1 n t

n 2n 1t 0

2n 1 2n 1n n n

0 2 1 1 0 ( 2 1) 1 2x C .2

2x 2x ( 2 1) 2x ( 2 1)

+ + −+

=

+ +

< − < ⇒ < − < ⇒ =

⇒ − = − ⇒ = −

Page 43: Chuyên đề hội thảo một số trường chuyên về dãy số

HỘ

HỘI CÁC TRƯỜNG THPT CHUYÊN KHU VỰC DUYÊN HẢI VÀ ðỒNG BẰNG BẮC BỘ HỘI THẢO KHOA HỌC LẦN THỨ IV

43

{ }

{ }

2(2n 1) 4n 2n n

4n 2n n

1 1x 2x (1 ( 2 1) ) (1 ( 2 1) )

2 2 2 21 1 1

lim x 2x lim( 2 1) .2 2 2 2 2 2

+ +

+

⇒ = + − = + −

= + − =

Ví dụ 3: Cho dãy số nu ñược xác ñịnh như sau:

n n

n

(2 3) (2 3)u ;n 0,1,2....

2 3

+ − −= =

a. Chứng minh rằng nu là số nguyên với mọi n = 0, 1, 2, ….

b. Tìm tất cả số hạng của dãy chia hết cho 3. Giải: a) Với 0 1n 0 u 0;n 1 u 1.= ⇒ = = ⇒ =

ðặt 2 3; 2 3α = + β = − , ta có 4

. 1

α + β =

α β =

Dễ thấy nu là số hạng tổng quát của dãy số cho bởi công thức:

0 1

n 2 n 1 n

u 0;u 1

u 4u u 0+ +

= =

− + =

Do 0 1u 0;u 1= = ∈� ; n 2 n 1 nu 4u u+ += − nên nu ∈� , n 0,1,...∀ = . b) Ta có n 2 n 1 n 1 nu 3u (u u )+ + += + − . Do n 1u + ∈� nên n 2 n 1 nu u u+ +≡ − (mod 3).

Bằng phép tính trực tiếp ta thấy 8 số hạng ñầu tiên của dãy 0 1 7u ,u ,...,u khi chia cho 3 có các số dư tương ứng là 0,1,1,0,2,2,0,1. Suy ra n 6 nu u+ ≡ (mod 3).

Từ ñó ta thấy trong dãy số nói trên mọi số hạng có dạng 3ku , k=0,1,2…

chia hết cho 3 và chỉ những số hạng ấy mà thôi. Ví dụ 4: (Thi HSG QG năm 2011): Cho dãy số nguyên n(a ) xác ñịnh bởi: a0 = 1,

1 n n 1 n 2a 1,a 6a 5a− −= − = + với mọi n 2≥ . Chứng minh rằng 2012a 2010− chia hết

cho 2011. Giải: Cách 1: Xét dãy ( nb ) ñược xác ñịnh như sau:

0 1 n n 1 n 2b 1;b 1;b 6b 2016b− −= = − = + , với mọi n 2≥ .

Dãy này có phương trình ñặc trưng là 2x 6x 2016 0− − = có hai nghiệm là x = -42, y = 48. Từ ñây suy ra số hạng tổng quát của dãy là

n n

n

41.48 49.( 42)b , n .

90

+ −= ∀ ∈�

Ngoài ra, ta cũng dễ dàng chứng minh bằng qui nạp rằng

n na b (mod 2011), n .≡ ∀ ∈�

Page 44: Chuyên đề hội thảo một số trường chuyên về dãy số

HỘ

HỘI CÁC TRƯỜNG THPT CHUYÊN KHU VỰC DUYÊN HẢI VÀ ðỒNG BẰNG BẮC BỘ HỘI THẢO KHOA HỌC LẦN THỨ IV

44

Do ñó ta chỉ cần chứng minh 2012b 1 0(mod 2011)+ ≡ . Ta có: 2012 2012

2012

41.48 49.( 42) 90b 1 .

90

+ − ++ =

Do 2011 là số nguyên tố và 2011, 90 là hai số nguyên tố cùng nhau nên ta chỉ cần chứng minh: 2012 201241.48 49.( 42) 90 0(mod 2011).+ − + ≡ (1)

Mà theo ñịnh lí Fermat nhỏ , ta có 2012 2012 2 241.48 49.( 42) 90 41.48 49.42 90(mod 2011)+ − + ≡ + +

= 90. 2b +90 = 90 [6(-1)+2016.1]+90

= 90.2010 + 90 = 90.2011 ≡0 (mod 2011). Vậy (1) ñúng, bài toán ñược chứng minh.

Cách 2: Phương trình ñặc trưng của dãy ñã cho là 2x 6x 5 0− − = có hai nghiệm

là 3 14− và 3 14+ , do ñó ta tìm ñược số hạng tổng quát của dãy là

na = n n(7 2 14)(3 14) (7 2 14)(3 14)

14

− + + + −

n 1 n 1

n n

( 7 14)(3 14) (7 14)(3 14)

14u 2v .

− −− + + − + −=

= − +

Trong ñó n 1 n 1 n 1 n 1

n n

(3 14) (3 14) (3 14) (3 14)u ,v

2 2 14

− − − −+ + − + − −= =

Sử dụng công thức khai triển nhị thức Niu-Tơn, ta có 1005 1005

2k 2011 2k k 2011 2k 2011 2k k2012 2011 2011

k 0 k 1

u C 3 14 3 C 3 14 .− −

= =

= = +∑ ∑

Do 1<2k<2011 với 1 k 1005≤ ≤ và 2011 là số nguyên tố nên 2k 1

2k 20102011

CC 2011( ) 2011

2k

= M

Mặt khác, theo ñịnh lí Fermat nhỏ thì 20113 3≡ (mod 2011).

Do vậy, kết hợp các lập luận lại với nhau ta ñược

2012u 3≡ (mod 2011) (2) Tương tự với nv , ta cũng sử dụng khai triển Niu-Tơn và thu ñược

1006 10052k 1 2012 2k k 1 2k 1 2012 2k k 1

2012 2011 2011k 1 k 1

v C 3 14 C 3 14 .− − − − − −

= =

= +∑ ∑

ðến ñây, cũng bằng cách sử dụng tính nguyên tố của 2011, ta thấy

Page 45: Chuyên đề hội thảo một số trường chuyên về dãy số

HỘ

HỘI CÁC TRƯỜNG THPT CHUYÊN KHU VỰC DUYÊN HẢI VÀ ðỒNG BẰNG BẮC BỘ HỘI THẢO KHOA HỌC LẦN THỨ IV

45

2k 22k 1 20102011

CC 2011 2011

2k 1

−−

= −

M

Với { }k 1,2,...,1005∈ . Vì vậy 1005

2012v 14≡ (mod 2011).

Do 2 214 2025 2011 45 2011 45= − = − ≡ (mod 2011) nên áp dụng ñịnh lí Fermat nhỏ, ta có

1005 201014 45 1≡ ≡ (mod 2011). Suy ra

2012v 1≡ (mod 2011). (3)

Từ (2) và (3), ta có

2012a 2010 3 2.1 2010 0− ≡ − + − ≡ (mod 2011)

Bài toán ñược chứng minh xong.

Ví dụ 5: Cho ( nu ) xác ñịnh:

( )

1

2

n 1 n n 1

u 7

u 50

u 4.u 5.u 1975 1+ −

=

= = + −

CMR: 1996u chia hết 1997

Giải: Tìm công thức xác ñịnh số hạng TQ:

Xét dãy n n n 1 n n 1

1975x u x 4.x 5.x

8 + −= − ⇒ = +

Giải PT ñặc trưng: 2 X 1X 4.X 5 0

X 5

=− − = ⇔ =

n nn n

n n

1747 2005 1747.( 1) 2005.( 1) 1975x .5 ( 1) . u

30 3 120 8

− − − + −⇒ = + − ⇒ = +

1996

1996

1747.5 49675u

120

− +⇒ =

19961996u .120 1747.(5 1) 1997.24⇒ = − − +

Suy ra 1996u chia hết 1997 vì 19965 1≡ (mod 1997)

Ví dụ 6: Cho dãy số ( )na :0

2n 1 n n

a 2

a 4a 15a 60+

=

= + −

Chứng minh rằng số 2n

1A (a 8)

5= + có thể biểu diễn thành tổng bình

phương của ba số nguyên liên tiếp với mọi n 1≥ . Giải:

Page 46: Chuyên đề hội thảo một số trường chuyên về dãy số

HỘ

HỘI CÁC TRƯỜNG THPT CHUYÊN KHU VỰC DUYÊN HẢI VÀ ðỒNG BẰNG BẮC BỘ HỘI THẢO KHOA HỌC LẦN THỨ IV

46

2n 1 n n

2 2n 1 n n

a 4a 15a 60

(a 4a ) 15a 60

+

+

− = −

⇔ − = −

2 2n 1 n 1 n na 8a a a 60 0+ +− + + = (1)

Áp dụng biểu thức trên với n ta có 2 2n n 1 n n 1a 8a a a 60 0− −− + + = (2)

Trừ từng vế (1) và (2) ta có 2 2

n 1 n 1 n 1 n 1 n

n 1 n 1 n 1 n 1 n

a a 8(a a )a 0

(a a )(a a 8a ) 0(*)+ − + −

+ − + −

− − − =

⇔ − + − =

Từ giả thiết suy ra n n 1 n na 0, n a 4a a+> ∀ ⇒ ≥ > suy ra n(a ) là dãy tăng. Suy ra n 1 n 1a a 0+ −− ≠ . Từ (*) suy ra n 1 n 1 na a 8a 0+ −+ − = .

Giải phương trình ñặc trưng 21,2t 8t 1 0 t 4 15− + = ⇒ = ±

n nn

2n 2n2n

a (4 15) (4 15)

a (4 15) (4 15)

⇒ = + + −

⇒ = + + −

Với mỗi n 1≥ tồn tại k ∈� ñể n n(4 15) (4 15) 15k+ + − =

2n n 2

2n 2n 2

22n

(4 15) (4 15) 15k

(4 15) (4 15) 2 15k

a 15k 2

⇔ + + − =

⇔ + + − − =

⇒ = +

2 2 2 2 22n

1 1A (a 8) (15k 10) 3k 2 (k 1) k (k 1)

5 5= + = + = + = − + + + (ñpcm).

Ví dụ 7: Cho dãy ( nu ) xác ñịnh: 1 2

n 2 n n 1

u 0,u 1

u u u 1+ +

= =

= + +

CMR: p là số nguyên tố p>5: thì p p 1u .(u 1)+ + chia hết cho p.

Giải:

Xét dãy: 2 11n n n n n

x u x x x+ += + ⇒ = +

Suy ra số hạng tổng quát:

1 1

1 5 1 1 5

2 25

n n

nx

+ + + − = −

.

1 1

1 5 1 1 51

2 25

n n

nu

+ + + − ⇒ = − −

( ) ( )11 .2 ( 5 1) ( 5 1) ( 5 1) ( 5 1)

2 5p p p

pu⇒ + = + + − − − .

( ) ( )1 1( 5 1) ( 5 1) ( 5 1) ( 5 1)

2 2 5p p p p= + − − + + + −

Page 47: Chuyên đề hội thảo một số trường chuyên về dãy số

HỘ

HỘI CÁC TRƯỜNG THPT CHUYÊN KHU VỰC DUYÊN HẢI VÀ ðỒNG BẰNG BẮC BỘ HỘI THẢO KHOA HỌC LẦN THỨ IV

47

0 0 0 0

1 1( ( 5) ( 5) ( 1) ) ( ( 5) ( 5) ( 1) )

2 2 5

p p p pk k k k p k k k k k p k

p p p p

k k k k

C C C C− −

= = = =

= − − + − −∑ ∑ ∑ ∑

( )1 1

2 22 2 2 1 2 1

0 0

1 11 .2 ( 2( 5) ( 2( 5)

2 2 5

p p

p k k k k

p p p

k k

u C C

− −

+ +

= =

+ = +∑ ∑

( )1

22 2 1

0

1 .2 ( )5

p

p k k k

p p p

k

u C C

+

=

+ = +∑ ; !

0( )! !

k

p

pC

p k k= ≡

− (mod ) 1 1p k p∀ ≤ ≤ −

( )1 1

0 0 2 22 . 1 .5 .5 5 1p p

p p

p p pu C C

− −

+ ≡ + ≡ + (mod )p (2)

Ta có 15 1p− ≡ (mod )p1 1

2 2(5 1)(5 1) 0p p− −

⇒ − + ≡ (mod )p .

Nếu 1

25 1 0p−

+ ≡ (mod )p Từ (2) ( )2 . 1 0p

pu⇒ + ≡ (mod )p mà (2;p)=1.

1 0p

u⇒ + ≡ (mod )p ( 1)p p

u u p⇒ + M (ñpcm)

Nếu 1

25 1p−

≡ (mod )p1

25 1 2p−

⇒ + ≡ (mod )p

Từ (2): ( )2 . 1 2p

pu + ≡ (mod )p ; (2,P)=1 2 2(mod ) 2 . 0(mod )p p

pp u p⇒ = ⇒ ≡ ,

(2,P)=1 0(mod ) ( 1)p p p

u p u u p⇒ ≡ ⇒ + M (ðpcm).

Phần III. Một số bài tập tự luyện: Bài 1: Tìm số hạng tổng quát của dãy n(u ),n 0≥ xác ñịnh bởi:

0 1

n 2 n 1 n

u u 1.

u 6u 9u ,n 0+ +

= =

= − ≥

Bài 2: Cho dãy số n(u ),n 0≥ xác ñịnh như sau:

n nn

3 5 3 5u ( ) ( ) 2,n 1,2,....

2 2

+ −= + − =

a. Chứng minh rằng nu là số tự nhiên n 1,2,....∀ = b. Chứng minh rằng 2011u là số chính phương.

Bài 3: Cho dãy số n(u ),n 0≥ xác ñịnh như sau: 0 1

n 2 n 1 n

u 0,u 1

u 2u 4u , n 0.+ +

= =

= − − ∀ ≥

Tìm n nn n

u ulim ;lim .

2 2

Page 48: Chuyên đề hội thảo một số trường chuyên về dãy số

HỘ

HỘI CÁC TRƯỜNG THPT CHUYÊN KHU VỰC DUYÊN HẢI VÀ ðỒNG BẰNG BẮC BỘ HỘI THẢO KHOA HỌC LẦN THỨ IV

48

DÃY SỐ SỐ HỌC

Trường THPT Chuyên Biên Hoà – Hà Nam

Hầu hết các bài toán dãy số có tính chất số học ñều sử dụng kết quả của : + Phương trình ñặc trưng dãy hoặc dùng các phép biến ñổi tuyến tính của dãy. + Dùng các ñịnh lý cơ bản của số học Fermat; Euler hoặc dùng các tính chất của số học…. Trong bài viết này xin ñược ñề cập ñến một vấn ñề nhỏ của dãy số số học, ñó là bài toán cơ bản mà bất kỳ học sinh chuyên Toán nào khi học về dãy số có tính chất số học cũng phải biết ñến và sử dụng nó. Dạng toán 1. Dãy số có sử dụng phương trình ñặc trưng hoặc sử dụng phép biến ñổi tuyến tính. Xét dãy tuyến tính xn = a1 xn-1 + a2 xn-2, n ≥ 2. Trong ñó a1, a2 ñã cho (có thể thực hoặc phức) và x0 = α0; x1 = α1 cho trước. Ta xét phương trình: t2 – a1t – a2 = 0 giả sử có 2 nghiệm phân biệt t1, t2 .Khi ñó ⇒ xn = c1 t1

n + c2t2n , n ≥ 0

Trong ñó c1, c2 là nghiệm của hệ : c1 + c2 = α0 c1t1 + c2t2 = α1 Nếu phương trình (1) có nghiệm kép t1 ≠0 thì: xn = (c1 + c2n) t1

n với

1 0

1 0 12

1

c

x tc

t

= α

− α=

Cụ thể ta lấy một ví dụ minh hoạ. Ví dụ: Tìm SHTQ của dãy xác ñịnh bởi. P0 = 0, P1 = 1; Pn = 2Pn-1 + Pn-2 n ≥2 Giải : Xét phương trình ñặc trưng t2 – 2t – 1 = 0 có t1 + 2 ; t2 = 1 – 2

⇒Pn = c1t1n + c2t2

n n ≥ 0 trong ñó: 1 2

1 2

c c 0

c (1 + 2) + c (1 - 2) = 1

+ =

Khi ñó Pn = 1

2 2 [ (1+ 2 )n –(1- 2 )n ] n ≥0

Nhận xét: Dãy trong ví dụ trên gọi là dãy Pell’s và nó là một trong các kết quả quan trọng của phương trình Diophantine . Trong tình huống tổng quát ta có thể xét dãy cho bởi dạng tổng quát. x1 = α1 ; x2 = α2 , …, xk-1 ═ α k-1 x1 = a1xn-1 + a2xn-2 + … + ak xn-k + b, n ≥k Với ai, b là số thực hoặc phức, αi ñó cho. Khi ñó bằng phép ñặt

Page 49: Chuyên đề hội thảo một số trường chuyên về dãy số

HỘ

HỘI CÁC TRƯỜNG THPT CHUYÊN KHU VỰC DUYÊN HẢI VÀ ðỒNG BẰNG BẮC BỘ HỘI THẢO KHOA HỌC LẦN THỨ IV

49

xn = yn + β sao cho (1 – a1 – a2 - … - ak) β = b và a1 + a2 +… + ak ≠ 1 Ta sẽ ñược dãy truy hồi tuyến tính nêu trong phần ñầu. Dưới ñây là một vài bài tập áp dụng các tính chất trên. Áp dụng Bài 1: ( Bulgarian 2001) Cho dãy {an} xác ñịnh bởi: a0 = 4; a1 = 22; an – 6an-1 + an-2 = 0 ∀n ≥2

Chứng minh: tồn tại dãy (xn) và (yn) nguyên dương sao cho: an = y2

n + 7xn - yn

, n ≥ 0

Lời giải: Xét dãy số nguyên dương (cn) với c0 = 2; c1 = 1; cn = 2cn-1 + cn-2 , n≥2 Nhận xét: Ta dễ dàng tìm ñược SHTQ của dãy (cn) và (an) từ ñó tìm ñược mối liên hệ giữa (cn) và (an) nhờ phương trình ñặc trưng. Tuy nhiên bằng phương pháp quy nạp ta có thể chứng minh ñược an = c2n+2 ∀n ≥ 0 Thật vậy, với n = 0 ⇒ a0 = 4 = c2 n = 1 ⇒ a1 = 22 = c4 Giả sử ñúng với n = k – 2 và n = k – 1 Khi ñó: c2k+2 = 2c2k+1 + c2k = 2(2c2k + c2k-1 ) + ak-1

= 4c2k + (c2k – c2k-2) + ak-1 = 6ak-1 – ak-2 = ak .Vậy an = c2n+2 ∀ n ≥ 0 Ta lại có a2

n+1 – an . an+2 = a2n+1

– a1(6an+1 – an)= a2n+1 – 6an+1 an + a2

n = an+1 ( 6am – an-1) – 6an+1an + a2

n = - an+1 an-1 + a2n

c2n+1 – cncn+2 = (-1)n(c1

2 – c0c2) = (-1)n(12 – 2 . 4) = (-1)n+1 . 7 Trong trường hợp ñặc biệt, ta có

c22n+1 – c2n c2n+2 = 7 ⇒ c2

2n+1 – c2nan = 7⇒ an = c2

2n+1+7c2n

∀ n ≥ 0

Vậy tồn tại xn = c2n + c2n+1 và yn = c2n+1 là các dãy số nguyên dương thoả mãn bài ra. Bài 2: Cho dãy {x1} xác ñịnh bởi. x0 = 0; x1 = 1 xn+2 = 3xn+1 – xn + 2 n ≥ 1 Xác ñịnh n ñể xn là số chính phương. Lời giải: Xét dãy yn = xn + 2⇒ y0 = 2; y1 = 3;yn+2 = 3yn+1 – yn Có phương trình ñặc trưng t2 – 3t + 1 = 0

⇒ yn = n

+

2

53 +

n

2

53

ðể xn là số chính phương thì yn- 2 là số chính phương.

⇔ xn =n

+

2

53 +

n

2

53 – 2 là số chính phương.

Page 50: Chuyên đề hội thảo một số trường chuyên về dãy số

HỘ

HỘI CÁC TRƯỜNG THPT CHUYÊN KHU VỰC DUYÊN HẢI VÀ ðỒNG BẰNG BẮC BỘ HỘI THẢO KHOA HỌC LẦN THỨ IV

50

xn = Cần tìm n ñể an = ∈Z ⇒ an+1 = 5 an – an-1 n ≥ 1 Bằng quy nạp theo n ta thấy n = 2k + 1 ⇒ an ∈Z n = 2k ⇒ a2k = k 5 ∉Z ( k ∈ z) Vậy n = 0; n = 2k + 1 ( k ∈ Z ) thì xn là số chính phương. Bài 3. (2002 Balkan ) Cho {an} thoả mãn ñiều kiện a1 = 20;a2 = 30; an+2 = 3an+1+ an ∀n ≥1 Tìm tất cả các số nguyên dương n sao cho 1 + 5an. an+1 là một số chính phương. Lời giải:

+ Dễ dàng kiểm tra ñược n = 3 thoả mãn; n = 1; 2 không thoả mãn. + Ta cần chứng minh với n ≥ 4 thì 1 + 5anan+1 không chính phương. Thật vậy: Trước hết cần chứng minh ∀n ≥ 2 ta có: a2

n + 500 = an-1 . an+1 Bằng quy nạp: Ta có n = 2 ⇒ ñúng. Giả sử ñúng tới n Suy ra a2

n + 500 = an+1 – an+1 an an+2 = (3an+1 – an) (an) = 3an+1 . an – an2

= 500 + an+1 (3an – an-1) = 500 + a2n+1

Vậy a2n + 500 = an-1 . an+1 ñúng ∀ n ≥ 2

+ Xét bài toán ñã cho với n ≥ 4 (an + a2

n+1) = a2n + a2

n+1 + 2anan+1 Mặt khác: a2

n+1 = 9a2n + a2

n-1 – 6an-1 an Nên (an + an+1)

2 = 2anan-1 + 3an(3an – an-1) + a2n-2 + a2

n – 3anan-1 = 5anan+1 + a2

n-1 – an.an-2 = 5an+1an + a2

n-1 – (a2n-1 + 800) = 5anan+1 – 500

Do ñó (an + an+1)2 = 5anan+1 – 500 < 5anan+1 + 1

⇒{an} t¨ng với n ≥ 4 ⇒an + an+1 ≥ 650 ⇒ (an + an+1 + 1)2 = (an + an+1)

2 + 2(an+an+1) + 1> (an + an+1)2 + 501 = 5anan+1 + 1

Do dãy số nguyên nên ⇒ 1 + 5an an+1 không chính phương ∀n ≥ 4. Vậy n = 3 là giá trị thoả mãn bài ra. Bài 4. Cho dãy {an} có SHTQ an = 2 + 2 (1+2n)2 n ≥ 0 Chứng minh rằng nếu an là số nguyên thì an là số chính phương Lời giải: Ta có an ∈ Z ⇔ 1 + 12 n2 = m2 ⇔ m2 – 12n2 = 1 Xét phương trình pell loại 1 có nghiệm nhỏ nhất (m,n) = (7,2) ⇒ Công thức nghiệm m0 = 1 , m1 = 7 , mk+1 = 14 mk – mk-1 no = 0; n1 = 2, nk+1 = 14 nk – nk-1

2

2

15

2

15

−−

+nn

−−

+nn

2

15

2

15

Page 51: Chuyên đề hội thảo một số trường chuyên về dãy số

HỘ

HỘI CÁC TRƯỜNG THPT CHUYÊN KHU VỰC DUYÊN HẢI VÀ ðỒNG BẰNG BẮC BỘ HỘI THẢO KHOA HỌC LẦN THỨ IV

51

Xét dãy {mk} có phương trình ñặc trưng t2 – 14 t + 1 = 0

⇒ mk = (7+ 48)k + (7- 48)k

2

Khi ñó ak = 2 + (7+ 48 )k + (7- 48 )k = 2 + (2 + 3 )2k + ( 2- 3 )2k = [(2+ 3 )k + (2- 3 )k ]2 Ta cần chứng minh ( 2+ 3 )k + (2- 3 )k là số nguyên . Thật vậy: Với dãy xn xác ñịnh bởi: xo = 2 x1 = 4 xn+1 = 4xn – xn-1 là dãy số nguyên có phương trình ñặc trưng x2 – 4x + 1 = 0⇒ x1,2 = 2 ± 3 ⇒xn = (2+ 3 )n + (2- 3 )n ∈ N Vậy an ∈ Z ⇒ an là số chính phương ( ñpcm) Bài tập tương tự 1. Cho (an) n ≥ 0 là một dãy không tăng các số nguyên dương, thoả mãn: a0 = a1 = 47 a2

n-1 + a2n + a2

n+1 - an-1 anan+1 = 4 ∀ n ≥1

Chứng minh rằng: 2 + an và 2 + 2+an là chính phương ∀ n ≥0

Gợi ý: Xét an = xn + 1xn

xn>1

Dẫn ñến xn+1 = xn. xn-1 (xn > 1) biến ñổi dãy dẫn ñến kết quả dãy xn = x0Fn

trong ñó F0 = F1 = 1;Fn+1 = Fn + Fn-1. 2. Cho dãy {an} n ≥ 0 xác ñịnh bởi. a0 = a1 = 97

an+1 = an-1 an + ( )( )11 22 −+ nn aa ∀ n ≥ 1

Chứng minh rằng 2 + 2+2an là một số chính phương ∀ n ≥ 0. 3.(Petersbug 2001) Cho a, b là hai số nguyên lớn hơn 1. Xác ñịnh {{xn} cho bởi x0 = 0; x1 = 1 x2n = ax2n-1 – x2n-2 ∀n≥ 1 x2n+1 = bx2n – x2n-1

Chứng minh rằng với m,n tự nhiên thì xm+n xm+n-1… xn+1 chia hế cho xm xm-1 4.(BMO) Cho m là một số nguyên dương. Xét dãy {an} ,n ≥ 0 xác ñịnh bởi a0 = 0, a1 = 1;an+1= m2 an – an-1 ∀n ≥ 1 Chứng minh rằng cặp số nguyên không âm (a,b) ( a ≤ b) là nghiệm của phương trình.

a2 + b2

ab+1 = m2 ⇔ (a,b) = (an, an+1) n ≥ 0.

Page 52: Chuyên đề hội thảo một số trường chuyên về dãy số

HỘ

HỘI CÁC TRƯỜNG THPT CHUYÊN KHU VỰC DUYÊN HẢI VÀ ðỒNG BẰNG BẮC BỘ HỘI THẢO KHOA HỌC LẦN THỨ IV

52

Dạng toán 2. Dạng toán dùng ñịnh lý Fermat và các tính chất số học. ðây là dạng toán mà câu hỏi tương ñối ña dạng, do ñó ñể làm ñược loại bài này yêu cầu người làm cần phải xử lý các kiến thức về số học linh hoạt. Bài toán về tính chia hết hay lý thuyết ñồng dư sẽ ñược sử dụng trong dạng toán này. Tuy nhiên ta cũng không thể không nhắc ñến ưu thế của ñịnh lý Fermat, ñiều ñó ñã ñược minh chứng trong ñề thi VMO 2011với bài toán về dãy số số học. Xét một số ví dụ tiêu biểu sau Bài 1. Xét dãy u1 = 1, u2 = 2, u3 = 24

un = 6un-1un-3 - 8un-1u

2n-2

un-2 - un-3 n ≥ 4

Chứng minh rằng với mọi n thì un: n

Lời giải: Xét un

un+1 =

6un-1

un-2 -

8un-1

un-3

ðặt vn = un

un-1 , ∀n ≥ 2 ⇒ vn = 6vn-1 – 8 vn-2 vµ v2 = 2; v3 = 12.

Suy ra vn+1 = 4n – 2n ⇒ vn = un

un-1 .

un-1

un-2 ….

u3

u2 .

u2

u1 . 1

= vn . vn-1 …. v2 = (4n – 2n) (4n-2 - 2n-2) …( 41 – 21)

Theo ñịnh lý Ferrmat nhỏ, với mọi số nguyên tố p ta có. 2p-1 ≡ 1 ( modp) , 4p-1 ≡ 1(mod p)

Suy ra 4p-1 – 2p-1 p. Nếu s là bội của p-1 thì 4s – 2s p Giả sử n = pα1

1pα2

2 … pαkk với p1 < p2 < … < pk ∈ P

Từ n pk1 ⇒ n p; n p2

1 …. n pα1

1⇒ d1 > d2 > …. > dα1 ⇒ n – d1 , …, n - dα1 là α1 bội khác nhau của p1 – 1

⇒ un pα1

1. Tương tự chứng minh un pαii ⇒ un n, ∀n. ( ñpcm)

Bài 2. (Bulgarian 2002) Cho b,c nguyên dương, xét dãy {an} xác ñịnh bởi a1 = b ;a2 = c;an+2 = | 3an+1 – 2an| ∀ n ≥ 1 Tìm tất cả các cặp (b,c) sao cho dãy a1, a2 … có duy nhất một số hữu hạn là tích của các số hạng trong dãy. Lời giải. Ta cần chứng minh các cặp (p, p) và (2p, p) với p không là hợp số và (7, 4) ñều thoả mãn ñiêù kiện bài ra. Ta thấy a1 , an … không giảm. Với mỗi an là một số nguyên dương tồn tại số nhỏ nhất k ≥ 1 sao cho ak+1 ≥ ak . Xét d·y mới b1, b2 … với b2 ≥ 1, bn = an+k -1 và bn+2 = |3bn-1 – 2bn|, n ≥ 1 Suy ra trong dãy b1, b2 … có duy nhất1 số hữu hạn là tích của các số hạng trong dãy. Nếu bn+1 ≥bn

Page 53: Chuyên đề hội thảo một số trường chuyên về dãy số

HỘ

HỘI CÁC TRƯỜNG THPT CHUYÊN KHU VỰC DUYÊN HẢI VÀ ðỒNG BẰNG BẮC BỘ HỘI THẢO KHOA HỌC LẦN THỨ IV

53

bn+2 = |3bn+1 – 2bn| = 3bn+1 – 2bn = bn+1 + 2 (bn+1 – bn) ≥ bn+1 Từ bn+2 = 3bn+1 – 2bn ⇒ bn = A.2n-1 + B Với n ≥ 1 và A = b2 – b1 ; B = 2b1 – b2 Giả sử A≠ 0. Khi ñó bn là 1 dãy tăng ⇒ có hữu hạn các số hạng là hợp số, bn = p với p nguyên tố > 2 và n ≥ 1. Hơn nữa ta có bn+1(p-1) chia hết cho P. Ta có bn+l(p-1) = A. 2n-1 . 2l(p-1)

+ B ≡ A.2n-1 + b ≡ 0 ( modp) Theo ñịnh lý Fermat. ðiều này vô lý ⇒ A = 0 và bn = b1 n ≥ 1 ⇒ b1 không là hợp số. ðặt b1 = p trong ñó p= 1 hoặc p nguyên tố. Ta xét a1, a2 … có các giá trị phân biệt k, a1 = b1 = b2 = a2 = p nên b = c = p với p không là hợp số ,vËy (p, p) thoả mãn. * Nếu k > 1, ak-1 > ak nhưng ak+1 = |3ak – 2ak-1| Và ak+1 = b2 = b1 = ak nên ak+1 = 2 ak-1 – 3ak ⇒ ak-1 = 2P Với k = 2 có b = 2p; c = p , p không là hợp số, vậy (2p,p) thoả mãn bài ra.

* Nếu k > 2 ⇒ ak-2 = 72 p nên p = 2. Với k= 3 ⇒ b = 7, c = 4 ;với k > 3; ak-2 ∉ Z

Vậy các cặp (p, p) và (2p, p) với p không là hợp số và (7, 4) thoả mãn ñiêù kiện bài ra. Nhận xét bài 2: Cách chứng minh trên có thể cho ta chứng minh bài toán tổng quát. Cho f ∈ Z (x1 ,…xk) là 1 ña thức và F(n) = f(n,2n, 3n,…, (k-1)n) , n ≥ 1. Nếu limn→∞ F(n) = ∞ thì tập các số nguyên tố chia hết các số hạng của dãy (Fn) là hữu hạn. Bài 3. (Peterbug 2002)

Cho dãy xác ñịnh bởi

nn

n 1n

nn

a 1

2a

2a

1 a

+

−≥

=

nÕu a 1 (1)

nÕu a < 1

Cho ao là 1 số nguyên dương, an ≠ 2 với mỗi n = 1,2,…2001 và a2002 = 2. Tính a0 . Lời giải: Ta chứng minh a0 = 3.22002 = 1 là giá trị cần tìm Trước hết ta chứng minh rằng a2002 = 2 và ai ≠ 2, i < 2002

Từ (1) có an+1 = an-1

a2002 với a2002 = 2 và ai > 2 , ∀ i < 2002

Xét bổ ñề sau: Với n ≤ 2000, nếu an không là số nguyên thì an = pn/qn trong ñó

pn, qn là các số nguyên dương lẻ và (p,q) = 1 ; q> 1.

Page 54: Chuyên đề hội thảo một số trường chuyên về dãy số

HỘ

HỘI CÁC TRƯỜNG THPT CHUYÊN KHU VỰC DUYÊN HẢI VÀ ðỒNG BẰNG BẮC BỘ HỘI THẢO KHOA HỌC LẦN THỨ IV

54

Chứng minh : Ta chứng minh bằng quy nạp, n = 2000; a2000 bằng 57 hoặc

15

Giả sử an+1 = p/q. Khi ñó ta xét hai trường hợp sau

TH1: an = 2p

q+1 → an = 2p + q

q . Giả sử d\ 2p+q và d\q

⇒ d\(2p + q – q) hoặc d\2p. Do q lẻ ⇒ d lẻ ⇒ (d, 2) = 1

⇒ d\p. Do d\q; d \ p và (p, q) = 1 ⇒ d= 1

Do ñó ( 2p + q, q) = 1; 2 p + q và q cùng lẻ ⇒ an thoả mãn bổ ñể .

TH 2: an = pq / ( 2+

pq ) =

p2q+p . Có d = (p; 2p + q)

⇒ d\ ( 2q + p – p) →d \ 2q ⇒ d = 1 ⇒ p, 2q + p lẻ và nguyên tố cùng nhau ⇒ an thoả mãn bổ ñề ⇒ bổ ñề ñược chứng minh.

Theo bài ta có a2001 = 1/2, khi ñó a2000 = 2 hoặc 57

Theo bổ ñề ta có a2000 = 57 ⇒ không có số nguyên thoả mãn bài ra.

Vậy a0 = 3.22002 = 1.

Bài tập luyện tập 1. Cho dãy x1 = x2 = x3 = 1 và xn+3 = xn + x n+1 xn+2 ∀ n ≥ 1. Chứng minh rằng với số nguyên dương m có số nguyên k > 0 sao cho m chia hết xk. 2. Cho {xn}, n ≥ 0 và x0 = 0

xn = xn-1 + 3r+1-1

2 nếu n = 3r (3k+1)

xn= xn-1 + 3r+1+1

2 nếu n = 3r (3k+2)

Với k, r nguyên không âm, chứng minh rằng mọi số nguyên xuất hiện ñúng một lần trong dãy. 3. ( Russian ) Cho dãy a0, a1, a2 ,… thoả mãn.

an+m + am-n = 12 ( a2m + a2n)

∀ m, n ≥ 0, m ≥ n và a1 = 1. Tìm an.

Page 55: Chuyên đề hội thảo một số trường chuyên về dãy số

HỘ

HỘI CÁC TRƯỜNG THPT CHUYÊN KHU VỰC DUYÊN HẢI VÀ ðỒNG BẰNG BẮC BỘ HỘI THẢO KHOA HỌC LẦN THỨ IV

55

Page 56: Chuyên đề hội thảo một số trường chuyên về dãy số

HỘ

HỘI CÁC TRƯỜNG THPT CHUYÊN KHU VỰC DUYÊN HẢI VÀ ðỒNG BẰNG BẮC BỘ HỘI THẢO KHOA HỌC LẦN THỨ IV

56

ỨNG DỤNG ðẠO HÀM TRONG MỘT SỐ BÀI TOÁN VỀ DÃY SỐ

Nguyễn Thành ðô Trường THPT Chuyên Bắc Ninh

A. MỞ ðẦU: Xét dãy số có dạng 1 ( )n nx f x+ = Cho hàm số f và giá trị ban ñầu 0x ta có thể xác ñịnh ñược các số hạng của dãy số. Xét sự hội tụ của dãy số này chỉ phụ thuộc vào hàm số f và giá trị ban ñầu 0x . Trong nhiều bài toán về dãy số, nếu chỉ sử dụng công cụ ñại số ñể ñánh

giá, ước lượng các số hạng cũng như các tính chất của dãy số ta gặp phải rất nhiều khó khăn. Việc sử dụng các tính chất của hàm số như: Tính chất hàm số co, tính ñồng biến, nghịch biến…cho ta thêm một hướng tiếp cận ñể ñánh giá dãy số ñã cho. B. CƠ SỞ VÀ ỨNG DỤNG I. DÃY CÔ SI VÀ HÀM SỐ CO: 1. Dãy { }nx ñược gọi là dãy Côsi nếu: 0 00; ; , m nN N m n N x xε ε∀ > ∃ ∈ ∀ > ⇒ − <

2. Dãy số { }nx có giới hạn hữu hạn khi và chỉ khi dãy số ñó là dãy Côsi

3. Hàm số :f D D→ ñược gọi là hàm số co trên D nếu tồn tại số thực q(0 1)q< < sao cho ( ) ( )f x f y q x y− ≤ − với mọi ;x y thuộc D .

4. Nếu hàm số ( )f x là một hàm co trên D thì dãy số { }nx xác ñịnh bởi

1 0( ),n n

x f x x a D+ = = ∈ hội tụ. Giới hạn của dãy số là nghiệm duy nhất trên D của phương trình ( )x f x= .

II. ỨNG DỤNG SỰ ðƠN ðIỆU CỦA HÀM SỐ: 1. Dãy { }nx bị chặn. Nếu f là hàm số tăng trên ñoạn [ ];a b thì dãy { }nx ñơn

ñiệu và hội tụ ñến L là nghiệm của phương trình ( )f x x= .

Chứng minh:

1 1( ) ( )n n n nx x f x f x+ −− = − cùng dấu với 1n nx x −− tiếp tục như vậy ta ñược 1n nx x+ − cùng dấu với 2 1x x− Nếu 2 1x x− dương thì dãy { }nx tăng

Nếu 2 1x x− âm thì dãy { }nx giảm

Dãy { }nx ñơn ñiệu và bị chặn nên dãy { }nx hội tụ.

Mặt khác { }nx hội tụ ñến L và hàm f liên tục nên ( ) ( )nf x f L→ hay ( )f L L= .

Vậy L là nghiệm của phương trình ( )f x x= . 2. Nếu f là hàm số nghịch biến thì các dãy con { }2nx và { }2n+1x của dãy { }nx

ngược chiều biến thiên.

Page 57: Chuyên đề hội thảo một số trường chuyên về dãy số

HỘ

HỘI CÁC TRƯỜNG THPT CHUYÊN KHU VỰC DUYÊN HẢI VÀ ðỒNG BẰNG BẮC BỘ HỘI THẢO KHOA HỌC LẦN THỨ IV

57

Nhận xét : Nếu dãy { }2nx hội tụ ñến L ; dãy { }2n+1x hội tụ ñến K

Với L K≠ thì dãy { }nx không có giới hạn.

Với L K= thì dãy { }nx có giới hạn L .

Ví dụ 1: Cho dãy số { }nx xác ñịnh:

0 10, n nx x c c x+= = − + Tìm tất cả các giá trị của c ñể mọi giá trị 0 (0; )x c∈ ñể dãy

số { }nx ñược xác ñịnh với mọi n và tồn tại giới hạn hữu hạn lim nx

x→∞

.

Giải: ðể 1x tồn tại thì 0

nc c x− − ≥ với mọi 0 (0; )x c∈ hay ( 1) 0c c − ≥ mọi 0 (0; )x c∈ suy

ra 2c ≥ . Với 2c ≥ thì 10 x c< < . Nếu 0 nx c< < thì 2nc c x c c− − > − suy ra

1nx + tồn tại và cũng có 10 nx c+< <

ðặt ' 1( ) ( )

4f x c c x f x x c c c x= − + → = − + − +

Với mọi (0; )x c∈ thì 1

( )( ) ( ) 2(2 2 2 )4

c x c c x c c c c+ − + > − + ≥ − + >

Từ ñó '( ) 1f x q≤ < với mọi (0; ),x c∈ tức là ( )f x là hàm co trên (0; ),c do vậy

dãy số hội tụ. Vậy tất cả các giá trị của c cần tìm là 2c ≥ .

Ví dụ 2:

Cho dãy số: 1 11

1 21; ( ); 2

2n n

n

u u u nu

= = + ≥ . Tìm giới hạn của dãy số.

Giải Ta có 0nu > với mọi n nguyên dương. Theo bất ñẳng thức Côsi ñược

2; 2nu n≥ ∀ ≥

Vậy trừ 1u còn lại các số hạng của dãy thuộc ñoạn )2; +∞

Xét hàm số: 1 2( ) ( )

2f x x

x= + ta có [ )

2,

2

2( ) 0; 2;

2

xf x x

x

−= ≥ ∀ ∈ +∞

Nên f(x) tăng trên ñoạn )2; +∞ ; mà 2 3 3 2

3 17;

2 12u u u u= = ⇒ <

Nên nếu loại bỏ 1u thì dãy { }nu là dãy giảm, bị chặn dưới nên tồn tại giới hạn L

là nghiệm của phương trình ( )f x x= hay 2x = . Vậy lim 2n

nu

→∞=

Ví dụ 3:

Cho dãy số: 11

11; 1 ; 2n

n

u u nu −

= = + ≥ . Tìm giới hạn của dãy số.

Page 58: Chuyên đề hội thảo một số trường chuyên về dãy số

HỘ

HỘI CÁC TRƯỜNG THPT CHUYÊN KHU VỰC DUYÊN HẢI VÀ ðỒNG BẰNG BẮC BỘ HỘI THẢO KHOA HỌC LẦN THỨ IV

58

Giải

Ta có 1 2; 1nu n≤ ≤ ∀ ≥ . Xét hàm số 1( ) 1f x

x= + ; có [ ]' 1

( ) 0; 1;2f x xx

= − < ∀ ∈

Nên ( )f x là hàm số nghịch biến trên ñoạn [ ]1;2 nên dãy số ñược chia ra làm hai

dãy ngược chiều biến thiên { }2nu ;{ }2 1nu +

Ta có 4 2

1

3u u− = − âm nên { }2nu là dãy giảm

3 1

1

2u u− = dương nên { }2 1nu + là dãy tăng. Các dãy con ñơn ñiệu và bị chặn nên

các dãy này hội tụ 2limn

nu a

→∞= ; 2 1lim n

nu b+

→∞= ; a b= nên 1 5

2a

+=

Ví dụ 4:

Xác ñịnh a ñể dãy số { }nu : 31 1 1

1; (2 2 1); 2

3n n nu a u u u n− −= = + − ≥ có giới hạn hữu

hạn. Giải:

Xét hàm số ( )f x 31( ) (2 2 1)

3y f x x x= = + −

Có ñạo hàm ' 2 2( ) 2 0;

3f x x x R= + > ∀ ∈ nên ( )f x tăng nên{ }nu ñược xác ñịnh.

Nếu dãy số này hội tụ ñến L thì L là nghiệm của phương trình ( )f x x=

hay 2( 1)(2 2 1) 0 1x x x x L− + + = ↔ = = .

Mặt khác 22 1 1 1 1

1( 1)(2 2 1)

3u u u u u− = − + + ; do 2

1 1(2 2 1) 0u u+ + > với mọi 1u

Nên chỉ phụ thuộc vào 1 1u −

Trường hợp 1: Nếu 1 1u = : ta ñược 2 1u = theo qui nạp ñược ta ñược 1lim =nu Trường hợp 2: Nếu 11 <u ta ñược 1212 0 uuuu <→<− dãy giảm do ñó nó tiến xa

dần số 1L = nên không hội tụ.

Trường hợp 3: nếu 11 >u ta ñược 1212 0 uuuu >→>−

Dãy tăng và xa dần số 1L = nên dãy không hội tụ. Nhận xét: Phương trình ( )f x x= có nghiệm nhưng dãy số chưa chắc ñã hội tụ.

Ví dụ 5: Tìm giới hạn của dãy số:

1;.... ≥+++= naaaun với n dấu căn; a là số dương cho trước.

Giải Từ biểu thức trên ta có thể viết

au =1 ; 1−+= nn uau với n = 2,3,........

Hiển nhiên aun ≥ với n = 1,2,3,....

Page 59: Chuyên đề hội thảo một số trường chuyên về dãy số

HỘ

HỘI CÁC TRƯỜNG THPT CHUYÊN KHU VỰC DUYÊN HẢI VÀ ðỒNG BẰNG BẮC BỘ HỘI THẢO KHOA HỌC LẦN THỨ IV

59

Xét hàm số xaxf +=)( với [ )+∞∈ ;ax ta có ' 1( ) 0

2f x

a x= >

+

nên ( )f x ñồng biến trên [ )+∞;a

với aau +=2 thì 012 >− uu nên { }nu là dãy số tăng

Xét phương trình f(x) = x ta ñược nghiệm:2

411 ax

++=

Ta chứng minh rằng 2

411 aun

++≤ với mọi n = 1,2,3,.....bằng phương pháp

qui nạp toán học.

Do vậy 2

411lim

aun

++=

Ví dụ 6:

Cho dãy số 21 1 << x ; 2

12

1n

nn

xxx −+=+ với n = 1,2,3,.......

Chứng minh rằng dãy { }nx có giới hạn và tìm giới hạn ñó.

Giải:

Ta có: 2

)1(3 2

1

−−=+

n

n

xx

Chứng minh bằng phương pháp qui nạp toán học ta ñược 21 << nx

Xét hàm số 2

1)(2x

xxf −+=

Có ' ( ) 1 0 (1;2)f x x x= − + < ∀ ∈ Nên ( )f x nghịch biến trên khoảng (1;2)

1/ Nếu 31 xx ≥ thì 4231 )()( xxxfxf ≤⇒≤

Cho nên .......531 ≥≥≥ xxx

....642 ≤≤≤ xxx Do ñó ta ñược hai dãy con { }12 +nx và { }nx2 hội tụ

giả sử bxax nn ==+ 212 lim;lim

Ta có hệ

−+=

−+=

21

21

2

2

aab

bba

từ ñó ta thu ñược: ( )( 4) 0a b a b− + − =

Vì ;a b thuộc khoảng (1;2) và 2

3

2

)1(3 2

1 <−−

=+n

n

xx

Page 60: Chuyên đề hội thảo một số trường chuyên về dãy số

HỘ

HỘI CÁC TRƯỜNG THPT CHUYÊN KHU VỰC DUYÊN HẢI VÀ ðỒNG BẰNG BẮC BỘ HỘI THẢO KHOA HỌC LẦN THỨ IV

60

Nên 32

3;

2

3≤+⇒≤≤ baba do vậy 04 ≠−+ ba

Suy ra a b= nên tồn tại giới hạn của dãy số. 2/ Tương tự với 31 xx ≤ cũng tồn tại giới hạn của dãy số

Và 2lim =nx .

III. MỘT SỐ BÀI TẬP LUYỆN TẬP Bài 1. Cho dãy số thực { }n

x ñược xác ñịnh như sau:

21 1

1, ln(1 ) 2011, 1.

2n nx a x x n+= ∈ = + − ≥�

Chứng minh rằng { }nx là một dãy hội tụ.

Bài 2. Cho dãy số thực { }nx ñược xác ñịnh như sau:

1 1

2 ( ln 2 1) 1; , 1

2 ln 2 1

n

n

x

n

n x

xx a x n+

− += = ∀ ≥

Tìm a ñể dãy số có giới hạn khác 0. Tìm gới hạn ñó. Bài 3. Cho dãy số thực { }n

x ñược xác ñịnh như sau:

1 1

os2,

2 4n

n

c xx x

ππ+

+= =

Tìm lim nx .

Bài 4. Cho dãy số thực { }nu xác ñịnh bởi 3

1

15

64n nu u+ = +

Tìm 1u ñể dãy số { }nu có gới hạn hữu hạn.

Bài 5. Cho dãy số thực { }nx ñược xác ñịnh như sau:

2

1 11, 1 ln( )1 ln

n

n

n

xx a x

x+= > = +

+

Tìm lim nx .

Bài 6. Cho dãy số thực { }nx ñược xác ñịnh như sau:

21 2 2 1

2 21; . sin

5 5n n nx x x x x

π

π+ += = = +

Chứng minh dãy số { }nx có giới hạn và tìm giới hạn ñó.

Bài 7. Chứng minh rằng với mỗi *n∈� thì phương trình 2 1 1n

x x+ = + có ñúng

một nghiệm thực. Gọi nghiệm ñó là nx . Tìm lim nx .

Page 61: Chuyên đề hội thảo một số trường chuyên về dãy số

HỘ

HỘI CÁC TRƯỜNG THPT CHUYÊN KHU VỰC DUYÊN HẢI VÀ ðỒNG BẰNG BẮC BỘ HỘI THẢO KHOA HỌC LẦN THỨ IV

61

TÀI LIỆU THAM KHẢO 1. Dãy số và các bài toán về dãy số Tác giả: Trần Nam Dũng- ðại học khoa học tự nhiên thành phố Hồ Chí Minh 2. Tạp chí toán học và tuổi trẻ Hàng tháng- từ năm 2007 ñến tháng 5 năm 2010 3. Dãy số - cấp số Tác giả: Lê Quan Ánh- Nhà xuất bản ðồng Nai- 1995. 4. Các ñề thi Olympic Toán sinh viên toàn quốc Chủ biên: Nguyễn Văn Mậu – Nhà xuất bản Giáo dục - 2004 5. Các ñề thi HSG Quốc gia môn Toán.

Page 62: Chuyên đề hội thảo một số trường chuyên về dãy số

HỘ

HỘI CÁC TRƯỜNG THPT CHUYÊN KHU VỰC DUYÊN HẢI VÀ ðỒNG BẰNG BẮC BỘ HỘI THẢO KHOA HỌC LẦN THỨ IV

62

DÃY SỐ VÀ GIỚI HẠN Vũ Thị Vân

Trung học phổ thông Chuyên Bắc Giang

Dãy số là một phần quan trọng của ñại số và giải tích toán học. ðối với toán học phổ thông, ñặc biệt với chương trình toán học dành cho học sinhchuyên, dãy số chiếm một mảng không nhỏ và thường rất khó. Trong các kì thi học sinh giỏi quốc gia, khu vực và quốc tế, các bài toán liên quan ñến dãy số ñược ñề cập thường xuyên, ñặc biệt là các bài toán liên quan ñến giới hạn của dãy số. Bài viết này xin trình bày một chút về giới hạn của dãy số. ðó là các phương pháp tìm giới hạn của dãy số và các bài toán tìm ñiều kiện hội tụ của dãy số. Bài viết trình bày theo hai chương. Chương I trình bày một số kiến thức mở ñầu. Nội dung chính ñược trình bày trong chương II với 3 phần nhỏ. Phần một trình bày các phương pháp tìm giới hạn của dãy số. Phần hai trình bày các bài toán tìm ñiều kiện ñể dãy số hội tụ. Phần ba ñưa ra một số bài tập củng cố.

Do trình ñộ và thời gian hạn chế, chuyên ñề không tránh khỏi những thiếu sót cả về mặt nội dung cũng như hình thức, rất mong nhận ñược sự góp ý chân thành của quý bạn ñọc. CHƯƠNG I. MỘT SỐ KIẾN THỨC MỞ ðẦU

Phần này trình bày một số ñịnh nghĩa, ñịnh lý, hệ quả và một số kết quả cần thiết áp dụng cho các bài toán về dãy số liên quan ñến giới hạn. 1.1 Các ñịnh nghĩa ðịnh nghĩa 1.1.1. Ta nói rằng dãy số (un) có giới hạn là hằng số a hữu hạn nếu

với mọi số dương ε (có thể bé tùy ý), luôn tồn tại chỉ số n0 ∈N sao cho với mọi

chỉ số 0,n N n n∈ ≥ , ta luôn có |un-a| < ε. Khi ñó kí hiệu lim un = a hay un → a,

và ta nói rằng dãy số (un) hội tụ về (ñến) a. Vậy

(un) → a(hữu hạn) ⇔ ∀ε, 0 0: ,| | .n

n N n n u a ε∃ ∈ ∀ > − <

ðịnh nghĩa 1.1.2. Dãy số (un) không hội tụ gọi là dãy phân kì.

ðịnh nghĩa 1.1.3. Cho dãy số (un) phân kì. Khi ñó, nếu

0 00, : , ,| |n

M n N n N n n u M∀ > ∃ ∈ ∀ ∈ > >

thì ta nói rằng dãy số (un) có giới hạn vô cực.

Kí hiệu limn

u = ∞ hoặc n

u → ∞ .

ðịnh nghĩa 1.1.4. Dãy số (un) ñược gọi là dãy Cauchy nếu nó thỏa mãn ñiều

kiện: 0 00, : , , , ,| | .m n

n N m n N m n n u uε ε∀ > ∃ ∈ ∀ ∈ > − <

Page 63: Chuyên đề hội thảo một số trường chuyên về dãy số

HỘ

HỘI CÁC TRƯỜNG THPT CHUYÊN KHU VỰC DUYÊN HẢI VÀ ðỒNG BẰNG BẮC BỘ HỘI THẢO KHOA HỌC LẦN THỨ IV

63

ðịnh nghĩa 1.1.5. Hàm số :f D D→ gọi là hàm số co trên D nếu tồn tại số

thực q với 0< q <1 sao cho | ( ) ( ) | | |, , .f x f y q x y x y D− ≤ − ∀ ∈

ðặc biệt, nếu hàm số : : [a;b]f D D= → , có ñạo hàm trên (a;b) và f’(x)<q,

với 0<q<1 thì f là hàm số co trên D.

ðịnh nghĩa 1.1.6. Dãy số ñược xác ñịnh bởi 0 1, ( ), ,n n

x a x f x n N+

= = ∀ ∈ trong

ñó a là hằng số cho trước ñược gọi là dãy số cho dưới dạng lặp.

1.2 Các ñịnh lý và hệ quả. ðịnh lí 1.2.1. Nếu một dãy số hội tụ thì giới hạn của nó là duy nhất.

ðịnh lí 1.2.2. (Tiêu chuẩn hội tụ Weierstrass)

1. Một dãy số ñơn ñiệu và bị chặn thì hội tụ.

2. Một dãy số tăng và bị chặn trên thì hội tụ.

3. Một dãy số giảm và bị chặn dưới thì hội tụ.

ðịnh lí 1.2.3. ðiều kiện cần ñể dãy số hội tụ là dãy số bị chặn.

ðịnh lí 1.2.4. Nếu (un) → a và (vn) ⊂ (un), (vn) là dãy vô hạn và không

là dãy hằng thì (vn) → a.

Hệ quả 1.2.1. Cho dãy (un) → a và k ∈ N là số cố ñịnh, cho trước. Khi

ñó dãy số (un+k ) → a.

Hệ quả 1.2.2. Cho dãy số ((xn ) xác ñịnh bởi công thức truy hồi xn+1 = f

(xn ), trong ñó f (x) là hàm số liên tục. Khi ñó nếu (xn ) → a thì a là nghiệm

của phương trình f (x) = x.

Hệ quả 1.2.3. Nếu có một dãy con của dãy (xn ) phân kì thì dãy ñó phân

kì.

ðịnh lí 1.2.5. Cho f là hàm số co trên D thì dãy số ñược xác ñịnh

xn+1= f (xn ) với số hạng ban ñầu là x0 = a∈ D, hội tụ và giới hạn ñó là

nghiệm duy nhất của phương trình f (x) = x.

ðịnh lí 1.2.6. Cho dãy số (xn) xác ñịnh bởi công thức truy hồi

xn+1 = f (xn) với hàm số f : D → D. Khi ñó

1) Nếu f tăng thì (xn ) sẽ là dãy (xn ) ñơn ñiệu tăng nếu x0< x1 , ñơn

ñiệu giảm nếu x0 > x1 .

2) Nếu f giảm thì các dãy con (x2p ), (x2p+1) là các dãy ñơn ñiệu và

ngược chiều nhau.

Hệ quả 1.2.4. Nếu có hai dãy con của dãy (un) mà tiến ñến hai giới hạn

khác nhau thì dãy (un) phân kì.

ðịnh lí 1.2.7. (Các phép toán về giới hạn)

Nếu lim un = a, lim vn = b thì

1. (un+ vn ) → a + b

2. (un − vn ) → a − b

3. (un. vn) → a.b

Page 64: Chuyên đề hội thảo một số trường chuyên về dãy số

HỘ

HỘI CÁC TRƯỜNG THPT CHUYÊN KHU VỰC DUYÊN HẢI VÀ ðỒNG BẰNG BẮC BỘ HỘI THẢO KHOA HỌC LẦN THỨ IV

64

4. n

n

u a

v b

với ñiều kiện 0b ≠ .

ðịnh lí 1.2.8. (ðịnh lí kẹp về giới hạn) Nếu với 0n n∀ ≥ ta luôn có và lim lim thì lim .

n n n n n nu x v u v a x a≤ ≤ = = =

ðịnh lí 1.2.9. (ðịnh lí Bolzano- Weierstrass)

Từ một dãy bị chặn luôn có thể trích ra một dãy con hội tụ.

ðịnh lí 1.2.10. (Tiêu chuẩn hội tụ Cauchy)

Dãy (un) hội tụ khi và chỉ khi (un) là dãy Cauchy.

ðịnh lí 1.2.11. (Các giới hạn cơ bản )

1. Nếu 0onst, , thì lim .n n

u a c n N n n u a= = ∀ ∈ ≥ =

2. Nếu 1

* thì lim ;lim 0.k

kk N n

n∈ = +∞ =

3. Nếu |q|<1 thì lim qn=0.

4. Nếu q=1 thì lim qn=1.

5. Nếu q=-1 thì lim qn không tồn tại.

6. Nếu |q|>1 thì lim qn=+∞ .

7.1

lim 1 .n

en

+ =

ðịnh lí 1.2.12. (ðịnh lí Toeplitz )

Cho dãy số Cn,k với k,n ∈N* thỏa mãn:

+)Với mỗi k cố ñịnh, , 0n k

C → ,

+) ,1

1, *n

n kk

C n N=

= ∀ ∈∑ ,

+) ,1

| | const, *.n

n kk

C C n N=

≤ = ∀ ∈∑

Khi ñó, nếu (an) hội tụ thì (bn) với ,1

, *n

n n k kk

b C a n N=

= ∀ ∈∑ cũng hội

tụ và lim an= lim bn.

ðịnh lí 1.2.13. (ðịnh lí Stolz)

Cho hai dãy số (xn) và (yn) ñồng thời thỏa mãn các ñiều kiện

1

1

1

, *

lim thì lim .

lim

+

> ∀ ∈

= +∞ ∃ =

−∃ = −

n n

n

n

n

n n

n n

y y n Nx

y Ay

x xA

y y

Page 65: Chuyên đề hội thảo một số trường chuyên về dãy số

HỘ

HỘI CÁC TRƯỜNG THPT CHUYÊN KHU VỰC DUYÊN HẢI VÀ ðỒNG BẰNG BẮC BỘ HỘI THẢO KHOA HỌC LẦN THỨ IV

65

1.3 Một số kết quả cần thiết. 1.3.1. Hai dãy số ñặc biệt:

+) Dãy 1

1n

n

+

là dãy tăng và bị chặn trên.

+) Dãy 1

11

n

n

+

+

là dãy giảm và bị chặn dưới.

1.3.2. Các bất ñẳng thức quan trọng. 2

2

1

11

1) cos 1 , .2

2) ln(1 ) , 0.2

1 13) 1 1 .

14) (1 ) 1 .

+

==

≥ − ∀ ∈

− < + < ∀ >

+ < < +

+ ≤ +

∑∏

n n

nn n

k nkk

xx x R

xx x x x

en n

c cn

(Bất ñẳng thức giá trị tương ñương) 5) (1 ) 1 , 0, 0+ ≤ + > >a a a

α α α

(Bất ñẳng thức Bernoully). CHƯƠNG II. CÁC BÀI TOÁN VỀ GIỚI HẠN CỦA DÃY SỐ 2.1 Các phương pháp tìm giới hạn của dãy số. 2.1.1 Phương pháp sử dụng ñịnh lý kẹp. Ở ñây, trình bày các ví dụ có sử dụng ñịnh kẹp ñối với trường hợp ñặc biệt:

Nếu , 0 0| | , * lim 0 lim 0.n n n n

u v n n n N và v thì u≤ ∀ ≥ ∈ = =

Hơn nữa, ta cũng có thể dùng kết quả sau lim xn=a ⇔ lim(xn-a)=0.

Ví dụ 2.1.1. Cho dãy số xn xác ñịnh: 2

1 1(1;2), 1 .2

n

n n

xx x x

+∈ = + −

Chứng minh rằng dãy (xn) có giới hạn hữu hạn. Tìm giới hạn ñó.

Hướng dẫn giải. Giả sử xn có giới hạn là a thì 2

1 22

aa a a= + − ⇒ = .

Ta sẽ chứng minh lim 2n

x = bằng cách chứng minh ( )lim 2 0.n

x − =

Thật vậy, ta có 2

1

2 1| 2 | |1 2 | | 2 | .

2 2n n

n n n

x xx x x

+

+ −− = + − − = −

Page 66: Chuyên đề hội thảo một số trường chuyên về dãy số

HỘ

HỘI CÁC TRƯỜNG THPT CHUYÊN KHU VỰC DUYÊN HẢI VÀ ðỒNG BẰNG BẮC BỘ HỘI THẢO KHOA HỌC LẦN THỨ IV

66

Mà 3

1 , 2,3,...,2n

x n< < = từ ñó suy ra

1

2

122 1 2 1, 2.

2 2

12

2| 2 | | 2 | 02

lim( 2) 0.

n

n

n

n

xn

x x

x

++ −

< < ∀ ≥

+

⇒ − < − < →

⇒ − =

Vậy lim 2n

x = .

Nhận xét. ðể tìm giới hạn a của dãy số, ta thường tìm a bằng cách giải phương trình thu ñược sau khi chuyển qua giới hạn của hệ thức truy hồi xác ñịnh dãy số ñó (a là nghiệm của phương trình ñó). Sau ñó ta chứng minh a chính là giới hạn cần tìm bằng cách chứng minh lim (xn-a)=0.

Ví dụ 2.1.2. Cho dãy số xn xác ñịnh: 2

1 1

1 1, , 1,2,...

2 2n

n

xx x n

+

−= − = ∀ =

Tìm giới hạn của dãy số (xn).

Hướng dẫn giải. Trước hết ta chứng minh 1 1

, 1(*)2 8n

x n− ≤ ≤ ∀ ≥ theo quy nạp.

Thật vậy, ta có mệnh ñề (*) hiển nhiên ñúng với n=1.

Với n = 2, x2 =1 1

8 2− suy ra (*) ñúng.

2

1

2 1| 2 | |1 2 | | 2 | .

2 2n n

n n n

x xx x x

+

+ −− = + − − = −

Giả sử (*) ñúng với n = k, nghĩa là 2

1

1 1 1 1 1 1

2 8 64 4 2 8k k kx x x

+− ≤ ≤ ⇒ ≤ ≤ ⇒ − ≤ ≤ .

Vậy (*) ñược chứng minh.

Gọi x là nghiệm của phương trình 2 1 1 1

1 2 do - .2 2 8

xx x x

−= ⇒ = − ≤ ≤

Ta có

2

1

2 2

2 1 1 1 12

;21 1 1

( ) ( )( ) ( );2 2 2

− =

− = − = − + = +

xx x

x x x x x x x x x x x

................................................................

Page 67: Chuyên đề hội thảo một số trường chuyên về dãy số

HỘ

HỘI CÁC TRƯỜNG THPT CHUYÊN KHU VỰC DUYÊN HẢI VÀ ðỒNG BẰNG BẮC BỘ HỘI THẢO KHOA HỌC LẦN THỨ IV

67

2

1 2 1

1( )( )...( ).

2 −

− = + + +

n

n nx x x x x x x x x

Mà: 1

| | | | 2 1 1, 1.2

+ ≤ + < − + < ∀ ≥n n

x x x x n

1| | 0

2

lim 1 2.

⇒ + ≤ →

⇒ = −

n

n

n

x x

x

Vậy lim 2n

x = .

Ví dụ 2.1.3. Cho dãy số( xn ) xác ñịnh: 0 1 2

12011, , 0,1,2,...

n n

n

x x x nx

+= = + ∀ =

Tìm giới hạn của dãy số3

nx

n

.

Hướng dẫn giải. Dễ dàng nhận thấy (xn) là dãy số dương. Theo công thức truy hồi xác ñịnh dãy số, ta có

3 3 3

1 6 3

3 3

1

3 3

0

3 3 3

1 2 2

0 0

1 13 3

1 21 1

1 33 3

3

3

1 1 1 13 3 , 1

3 ( 3 ) 3 9

1 13( 1) , 1.

3 9

n n n

n n

n n

n

n n n

n n

nk k

x x xx x

x x

x x n

x x x nx n x n n n

x x n nk k

+

+

− −

= =

= + + + > +

⇒ > +

⇒ > +

⇒ < + + + < + + + ∀ ≥+ +

⇒ < + − + + ∀ >∑ ∑

Mặt khác, ta lại có

1

21

1 1

21 1

1 1 1 11 ...

1.2 2.3 ( 2).( 1)

1 1 1 1 1 =1+ 1 ... 2

2 2 3 ( 2) 1

1 11 2 .

n

k

n n

k k

k n n

n n

n nk k

=

− −

= =

< + + + +− −

− + − + + − <

− −

⇒ ≤ − <

∑ ∑

Bất ñẳng thức trên có ñược là theo bất ñẳng thức Bunhiacopxki. Từ các kết quả trên ta có

Page 68: Chuyên đề hội thảo một số trường chuyên về dãy số

HỘ

HỘI CÁC TRƯỜNG THPT CHUYÊN KHU VỰC DUYÊN HẢI VÀ ðỒNG BẰNG BẮC BỘ HỘI THẢO KHOA HỌC LẦN THỨ IV

68

3 3 3

0 1

3 3 3

0 1

3 3 2 2, 2

2 23 3 , 2.

n

n

n x x x n n n

x x xn

n n n n n

+ < < + + + ∀ ≥

⇒ + < < + + + ∀ ≥

Vậy lim3

nx

n

=0.

2.1.2 Phương pháp sử dụng ñịnh lí Weierstrass. Với phương pháp này, ta cần chứng minh hai tính chất của dãy số: thứ nhất dãy ñơn ñiệu, thứ hai dãy phải bị chặn. Khi ñó theo ñịnh lí Weierstrass ta ñược dãy hội tụ. Giới hạn của dãy số chính là nghiệm của phương trình thu ñược sau khi chuyển qua giới hạn trong công thức truy hồi.

Ví dụ 2.1.4. Cho dãy số( xn ) xác ñịnh: 0 12, 2 , 0,1,2,...nx

nx x n

+= = ∀ =

Chứng minh rằng dãy số hội tụ và tìm giới hạn ñó.

Hướng dẫn giải.

ðặt ( ) 2xf x = , dãy số có dạng 0 12, ( ).

n nx x f x

+= =

Rõ ràng ( )f x là hàm số tăng. Mặt khác 2

1 02x x= > nên ( )n

x là dãy số tăng. Ta sẽ chứng minh ( )

nx bị chặn trên bởi số 2 bằng quy nạp.

Thật vậy, ta có 0 2 2x = < . Giả sử ta ñã có 2k

x < , khi ñó

2

1 2 2 2.kx

kx

+= < =

ðiều ñó chứng tỏ 2,n

x n N< ∀ ∈ . Như vậy, ( )

nx là dãy tăng và bị chặn trên nên nó có giới hạn hữu hạn. Gọi

giới hạn ñó là a.

Ta có 2 , 2.aa a= ≤ Xét phương trình

ln2 ln 2.x x

xx

= ⇔ =

Khảo sát hàm số , ta thấy phương trình trên chỉ có một nghiệm nhỏ hơn e và một nghiệm lớn hơn e. Vì 2 là một nghiệm của phương trình nên phương trình chỉ có nghiệm duy nhất là 2. Vậy a = 2. Ví dụ 2.1.5.(VMO 2011) Cho dãy số( xn ) xác ñịnh

1

0 21

21, , 2,3,4...

( 1)

n

n ii

nx x x n

n

=

= = ∀ =−

Với mỗi số nguyên dương n, ñặt 1n n ny x x

+= − . Chứng minh rằng dãy số ( )

ny hội

tụ.

Page 69: Chuyên đề hội thảo một số trường chuyên về dãy số

HỘ

HỘI CÁC TRƯỜNG THPT CHUYÊN KHU VỰC DUYÊN HẢI VÀ ðỒNG BẰNG BẮC BỘ HỘI THẢO KHOA HỌC LẦN THỨ IV

69

Hướng dẫn giải. Với 1n∀ ≥ , ta có 2 2

1 2 2 31

2( 1) 2( 1) ( 1) ( 1)( 1)1 .

2

n

n i n ni

n n n n nx x x x

n n n n+

=

+ + − + + = = + =

Suy ra 1

2

11 . , 1.

1n n

x xn

n n n

+ = + ∀ ≥

+

Do ñó với 2n∀ ≥ ta có 2

1 n n3

2 1

2 2 21

(n+1)(n +1)x = -1 x

n

1 1 1 . 1 1

n n

nn

k

y x

n n x n

n n n k

+

=

= −

+ + + = = + +

Từ ñó, với lưu ý 1 2 1 3y x x= − = và 2 3 2

7

2y x x= − = , ta có

1 20, 1,n

y n y y> ∀ ≥ < và 3n∀ ≥ , thì 2 2

2 2 2 4 3 2

1

1 ( 1) 1 2 . 1 1 1.

( 1) ( 1)n

n

y n n n

y n n n n n n n−

+ + −= = + = + >

− + − − +

Suy ra (yn) là dãy số tăng. Lại có 22, 1n n n∀ ≥ + < và

1 11 1 1

2 2 21 11

1 1 1 1 11 1 2 1 .

1 1

n nn n n

nk kk

yk n k n k

− −− − −

= ==

+ ≤ + ⇒ < +

− − ∑ ∑∏

Mặt khác 1 1 1

21 2 2

1 1 1 1 11 1 2 2, 3.

( 1) 1 1

n n n

k k k

nk k k k k n

− − −

= = =

< + = + − = − < ∀ ≥

− − − ∑ ∑ ∑

Từ ñó, suy ra 1

222 1 2e , 2.

1

n

ny n

n

< + < ∀ ≥

Do ñó (yn) là dãy số bị chặn trên, cùng với (yn) tăng nên (yn) hội tụ. Ví dụ 2.1.6. Cho dãy số( xn ) xác ñịnh

2

1 11, , 1,2,3,...2011

n

n n

uu u u n

+= = + ∀ =

ðặt 1 2

2 3 1

... .n

n

n

u u uS

u u u+

= + + + Tìm giới hạn của Sn.

Hướng dẫn giải. Ta có 2

1

1 1 1 1

2011( ) 1 12011+

+ + + +

−= = = −

n n n n

n n n n n n n

u u u u

u u u u u u u

Page 70: Chuyên đề hội thảo một số trường chuyên về dãy số

HỘ

HỘI CÁC TRƯỜNG THPT CHUYÊN KHU VỰC DUYÊN HẢI VÀ ðỒNG BẰNG BẮC BỘ HỘI THẢO KHOA HỌC LẦN THỨ IV

70

1 1 1

1 1 12011 2011 1 .

+ +

⇒ = − = −

n

n n

Su u u

Hơn nữa từ công thức truy hồi của (un) ta có 1 1, .n n

u u n N+

> ≥ ∀ ∈

Hay (un) là dãy tăng. Giả sử (un) bị chặn trên, khi ñó nó sẽ hội tụ về a hữu hạn. Thế thì

2

0.2011

aa a a= + ⇒ =

ðiều ñó vô lí vì 1, .n

u n N≥ ∀ ∈

Chứng tỏ giả sử sai, hay (un) bị chặn trên, do ñó

1

1lim lim2011 1 2011.

n

n

uu

+

= +∞⇒ − =

Vậy lim Sn = 2011.

Nhận xét. Với ví dụ trên, ta có thể thay 2011 bằng một số bất kì nào cũng ñược, chẳng hạn ta thay bằng số A thì giới hạn cần tìm chính là A. Bài này ta cần tìm giới hạn dãy số mà mỗi số hạng là một tổng, và ta ñã dùng phương pháp sai phân ñể làm gọn tổng, chính là số hạng của dãy số cần tìm giới hạn. ðây là một phương pháp rất hay ñược dùng cho bài toán liên quan ñến dãy số. 2.1.3 Phương pháp sử dụng ñịnh lí Toeplitz.

Ví dụ 2.1.7. Chứng minh rằng nếu 1 2 ...lim thì lim .n

n

a a aa a a

n

+ + += =

Hướng dẫn giải. Sử dụng ñịnh lí Toeplitz với ,

1; 1,2,..., .

n kC k n

n= =

Khi ñó 1 2,

1

...lim lim .

nn

n k kk

a a aC a a

n =

+ + += =∑

Ví dụ 2.1.8. Chứng minh rằng nếu (an) là dãy dương và

1 2lim thì lim ... .nn n

a a a a a a= =

Hướng dẫn giải. Ta có lim ln an = ln a. Theo ví dụ trên ta có

1 2

1 2

1 2

ln ln ... lnlim ln ln ... ln .

Hay lim ... .

n nn

nn

a a aa a a a a

n

a a a a

+ + += ⇒ →

=

Ví dụ 2.1.9. Cho dãy số( an ). Chứng minh rằng nếu 1lim thì lim .n nn

n

aa a a

a

+ = =

Page 71: Chuyên đề hội thảo một số trường chuyên về dãy số

HỘ

HỘI CÁC TRƯỜNG THPT CHUYÊN KHU VỰC DUYÊN HẢI VÀ ðỒNG BẰNG BẮC BỘ HỘI THẢO KHOA HỌC LẦN THỨ IV

71

Hướng dẫn giải. ðặt 1

, 2 lim .n

n n

n

ab n b a

a−

= ∀ ≥ ⇒ =

Theo ví dụ trên, ta có 1 2lim ... .nn

bb b a= Hay lim .nn

a a=

Ví dụ 2.1.10. Chứng minh rằng nếu lim =n

a a thì:

1 2( 1) ... 1. lim .

2

+ − + +=n

na n a a a

n

Hướng dẫn giải. Sử dụng ñịnh lí Toeplitz với , 2

2( 1); 1,2,..., .

n k

n kC k n

n

− += =

Ta có

, ,

, 2 21 1

0 ,lim 0,

1 ( 1 1 1) 12 2 1 1.

2

n k n k

n n

n kk k

C C k N

n k n n n nC

n n n= =

< = ∀ ∈

− + − + + − += = = + →∑ ∑

Vì thế ,1

lim .n

n k kk

C a a=

=∑

Vậy 1 2( 1) ... 1. lim .

2n

na n a a a

n

+ − + +=

Ví dụ 2.1.11. Cho (an), (bn) là hai dãy số thỏa mãn

1 2

0, ;

lim( ... ) ;

lim .

n

n

n

b n N

b b b

a a

> ∀ ∈

+ + + = +∞ =

Chứng minh rằng 1 1 2 2

1 2

... lim .

...n n

n

a b a b a ba

b b b

+ + +=

+ + +

Hướng dẫn giải.

ðặt ,

1 2

.; 1,2,..., ....

k

n k

n

bC a k n

b b b= = =

+ + +

Ta có

, ,

,1

0 ,lim 0, ,

1.

n k n k

n

n kk

C C k N

C=

< = ∀ ∈

=∑

Theo ñịnh lí Toeplitz ta có ,1

lim .n

n k kk

C a a=

=∑

Vậy 1 1 2 2

1 2

... lim .

...n n

n

a b a b a ba

b b b

+ + +=

+ + +

2.1.4 Sử dụng ñịnh lí Stolz. ðịnh lí 2.1.1. (ðịnh lí Stolz) Cho hai dãy số (xn) và (yn) ñồng thời thỏa mãn các ñiều kiện

Page 72: Chuyên đề hội thảo một số trường chuyên về dãy số

HỘ

HỘI CÁC TRƯỜNG THPT CHUYÊN KHU VỰC DUYÊN HẢI VÀ ðỒNG BẰNG BẮC BỘ HỘI THẢO KHOA HỌC LẦN THỨ IV

72

1

1

1

, *

lim thì lim .

lim

n n

n

n

n

n n

n n

y y n Nx

y Ay

x xA

y y

+

> ∀ ∈

= +∞ ∃ =

−∃ = −

Chứng minh.

ðặt 11

1

; , 2.n n

n n n n

n n

x xa b y y n

y y

−= = − ∀ ≥

− Ta có

2 3 1

0, *;

lim( ... ) lim( ) ;

lim .

n

n n

n

b n N

b b b y y

a A

> ∀ ∈

+ + + = − = +∞ =

Theo kết quả của Ví dụ 2.1.11 ta có

1 1 2 2 1

1 2 1

1

1

... lim lim

...

lim lim .1

n n n

n n

n

nn n

n

n

a b a b a b x xA

b b b y y

x x

xy yA A

y y

y

+ + + −= =

+ + + −

⇒ = ⇒ =

Vậy lim .n

n

xA

y=

Ví dụ 2.1.12. Tìm 1 1 1

lim 1 ... .2n n

+ + +

Hướng dẫn giải.

ðặt 1 1

1 ... , , 1.2

n nx y n n

n= + + + = ∀ ≥ Khi ñó

1

1

1

, *

lim

11

lim lim lim 2.1

n n

n

n n

n n

y y n N

y

x x n nn

y y n n n

+

> ∀ ∈

= +∞

− + − = = = − − −

Do ñó theo ñịnh lí Stolz, ta ñược lim 2.n

n

x

y=

Page 73: Chuyên đề hội thảo một số trường chuyên về dãy số

HỘ

HỘI CÁC TRƯỜNG THPT CHUYÊN KHU VỰC DUYÊN HẢI VÀ ðỒNG BẰNG BẮC BỘ HỘI THẢO KHOA HỌC LẦN THỨ IV

73

Vậy 1 1 1

lim 1 ... 2.2n n

+ + + =

Ví dụ 2.1.13. Chứng minh rằng nếu 1lim( ) thì lim .n

n n

aa a a a

n+

− = =

Hướng dẫn giải. ðặt , , 1.

n n nx a y n n= = ∀ ≥ Khi ñó

1

11

1

, *;

lim ;

lim lim( ) .

n n

n

n n

n n

n n

y y n N

y

x xa a a

y y

+

> ∀ ∈

= +∞

− = − = −

Do ñó theo ñịnh lí Stolz, ta ñược lim .n

n

xa

y=

Vậy lim .na

an

=

Ví dụ 2.1.14. Cho a>1. Tính 2

1lim ... .

2

n

n

n a aa

a n+

+ + +

Hướng dẫn giải.

ðặt 2 1

... , , 1.2

n n

n n

a a ax a y n

n n

+

= + + + = ∀ ≥ Khi ñó

11

1 1, 1, ì 1, 1.

1 1 1n

n n

n

y nay y n v n

a y n a

+

+

> ∀ ≥ + = > ∀ ≥ + − + −

2 2 2(1 ( 1)) ( 1) ( 1)( 1)lim vì y ;

2

n n

n

n n

a a C a n ay

n n n

+ − − − −= +∞ > = > = → +∞

11

1

1lim lim .

11

n

n n

n n

n n

a

x x na ay y a

n n

−+

−= =

− − −−

Do ñó theo ñịnh lí Stolz, ta ñược 1

lim .1

n

n

x

y a=

Vậy 2

1lim ... .

2

n

n

n a aa

a n+

+ + +

2.2 ðiều kiện hội tụ của dãy số Trong bài toán dạng này, dãy số ñược cho có cả tham số. Sự hội tụ cùa dãy số phụ thuộc vào giá trị của tham số. Câu hỏi ñặt ra ở ñây là, tìm ñiều kiện của tham số ñể dãy hội tụ.

Page 74: Chuyên đề hội thảo một số trường chuyên về dãy số

HỘ

HỘI CÁC TRƯỜNG THPT CHUYÊN KHU VỰC DUYÊN HẢI VÀ ðỒNG BẰNG BẮC BỘ HỘI THẢO KHOA HỌC LẦN THỨ IV

74

Ví dụ 2.1.14. Cho dãy số xn xác ñịnh

0 1, , 0,1,2,...n n

x x a a x n+

= − + ∀ =

Tìm tất cả các giá trị của a ñể với mọi 0 (0; ),( )n

x a x∈ xác ñịnh và có giới hạn

hữu hạn.

Hướng dẫn giải. Ta có x1 xác ñịnh khi

0 0 0 0, (0; ) hay ( -1) , (0; ) 2. a a x x a a a x x a a− + ∀ ∈ ≥ ∀ ∈ ⇒ ≥

Với 12 thì 0 .a x a≥ < <

Ta sẽ chứng minh n

x xác ñịnh và 0 , 1.n

x a n< < ∀ ≥

Thật vậy, nếu 0n

x a< < thì 2 0n

a a x a a− + > − ≥ .

Suy ra xn+1 xác ñịnh và ta cũng có 10n

x a+

< < .

Bây giờ ta chỉ ra dãy số ñã cho hội tụ với 2a ≥ .

ðặt 1

( ) '( ) .4 ( )( )

f x a a x f xa x a a x

= − + ⇒ = −+ − +

Với x (0; )a∀ ∈ , ta có

( ) 1( )( ) 2(2 2 2 )

4a x a a x a a a a+ − + > − + > − + >

(Do hàm số ( )g a a a a= − + ñồng biến trên (0; )+∞ ).

Từ ñó suy ra '( ) 1, (0; ),f x q x a< < ∀ ∈ tức là f(x) là hàm số co trên (0; )a .

Suy ra dãy ñã cho hội tụ. Vậy tất cả các giá trị của a phải tìm là 2a ≥ . Ví dụ 2.1.15 (Oliypic sinh viên Moska, 1982) Cho dãy số( xn) xác ñịnh

0 1

12011, , 0,1,2,...

4 3n

n

x x nx

+= = ∀ =

Tìm giới hạn của (xn).

Hướng dẫn giải.

Nhận thấy 2 2 3

10 , .

3x x x< < < ðặt

1( )

4 3f x

x=

−, ta có xn+1=f(xn).

Khảo sát hàm số f(x) ta ñược kết quả f(x) là một hàm số tăng,

([0;1]) [0;1]f ⊂ và 1 1

( 0; ) 0;3 3

f

⊂ .

Từ ñây ta ñược, dãy số (xn), n>1 là một dãy tăng và bị chặn trên bởi 1

3 do

ñó nó có giới hạn hữu hạn. Giả sử giới hạn là a thì a là nghiệm của phương trình f(a)=a.

Page 75: Chuyên đề hội thảo một số trường chuyên về dãy số

HỘ

HỘI CÁC TRƯỜNG THPT CHUYÊN KHU VỰC DUYÊN HẢI VÀ ðỒNG BẰNG BẮC BỘ HỘI THẢO KHOA HỌC LẦN THỨ IV

75

Giải phương trình ta ñược nghiệm là 1 và 1

3, do

1

3nx ≤ nên a =

1

3.

Chú ý. Khi phân tích bài toán ñể tìm lời giải ta nên lập bảng biến thiên cho hàm số f(x) và giải phương trình f(x)=x ñể tìm giới hạn nếu có của dãy số. Câu hỏi ñặt ra ở ñây là Nếu không cho x0 một giá trị cụ thể, với giá trị nào của x0 dãy xác ñịnh với mọi n và có giới hạn hữu hạn? Khi nào thì giới hạn ñó là 1,

khi nào thì giới hạn ñó là 1

3?

Sau ñây ta sẽ trả lời câu hỏi trên. +) Trước hết ta tìm ñiều kiện của x0 ñể dãy xác ñịnh.

Rõ ràng, xn xác ñịnh 4 4

, ( ) , .3 3n n

n x n f x n∀ ⇔ ≠ ∀ ⇔ ≠ ∀

Từ ñó,ta ñược ñiều kiện của x0 là 0 nx y≠ với (yn) xác ñịnh:

0

4

3y = và yn+1 là nghiệm của phương trình f(x) = yn với mọi n.

(yn xác ñịnh duy nhất vì f(x) là hàm ñồng biến trên từng khoảng xác ñịnh). +) Tiếp theo ta tìm ñiều kiện của x0 ñể dãy hội tụ.

Ta sẽ chứng minh với ñiều kiện của x0 như trên dãy số ñã cho luôn hội tụ. Thật vậy, từ sự biến thiên của f(x), ta thấy

Nếu tồn tại số hạng xk<1 thì xk+1 =f(xk) (0;1)∈ ,

từ ñó suy ra dãy (xn) với 1n k≥ + là dãy ñơn ñiệu bị chặn nên nó hội tụ. Ta ñược (xn) hội tụ.

Nếu tồn tại số hạng 4

3kx > thì xk+1 =f(xk)<0, xk+2 =f(xk+1)

1(0; )

3∈ , từ ñó

suy ra dãy số dãy (xn) với 2n k≥ + là dãy ñơn ñiệu bị chặn nên nó hội tụ.

Ta ñược (xn) hội tụ (về 1

3).

Nếu tồn tại số hạng 1k

x = thì các số hạng từ thứ k trở ñi ñều bằng 1 nên

dãy ñó có giới hạn bằng 1. Bây giờ ta sẽ chứng minh dãy số ñã cho luôn tồn tại số hạng thỏa mãn

một trong các khả năng trên. Thật vậy, giả sử 4

1; ,3n

x n

∈ ∀

. Nếu dãy số ñó có

giới hạn thì giới hạn ñó phải là 1.

Ta xét hàm số ( 1)(3x 1)

( ) .4 3x

xf x x

− −− =

Với 4 4

1; thì ( ) - 0 ( ) , 1; .3 3

x f x x f x x x

∀ ∈ > ⇒ > ∀ ∈

Suy ra (xn) là dãy tăng, bị chặn trên nên nó có giới hạn. Khi ñó giới hạn ñó

Page 76: Chuyên đề hội thảo một số trường chuyên về dãy số

HỘ

HỘI CÁC TRƯỜNG THPT CHUYÊN KHU VỰC DUYÊN HẢI VÀ ðỒNG BẰNG BẮC BỘ HỘI THẢO KHOA HỌC LẦN THỨ IV

76

phải lớn hơn 1. ðiều này vô lí. Như vậy giả sử sai, nói cách khác với x0 thỏa mãn ñiều kiện như trên, dãy

số luôn hội tụ.

+) Cuối cùng khi nào dãy số có giới hạn 1, khi nào dãy số có giới hạn 1

3?

Theo trên, ta có

Nếu 0

4

3x > thì giới hạn là

1

3.

Nếu 0

1 1 thì ,

3 3nx x n≤ ≤ ∀ , giới hạn là

1

3.

Nếu 0 0 1

1<1 thì

3x x x< > , suy ra dãy giảm và bị chặn dưới bởi

1

3 nên dãy

số có giới hạn 1

3.

Nếu x0=1 thì mọi số hạng của dãy ñều bằng 1 nên dãy có giới hạn là 1.

Nếu 0

41 thì , 1

3 nx n x< ≤ ∀ ≠ , nên dãy số có giới hạn

1

3.

Như vậy với ñiều kiện x0 như trên và 0 1x ≠ thì dãy số có giới hạn là 1

3,

Nếu x0 = 1 thì dãy số có giới hạn bằng 1. Ví dụ 2.1.16. Cho dãy số xn xác ñịnh

0 1, 1 , 0,1,2,...n n

x a x bx n+

= = + ∀ =

Tìm tất cả các giá trị của a,b ñể ( )n

x có giới hạn hữu hạn.

Hướng dẫn giải. Theo qui nạp ta có

2 1 1 1 11 ...

1 1 1

n

n n n n

n

bx b b b b a b a b a

b b b

− − = + + + + + = + = + −

− − − với 1b ≠ .

Từ ñó ta có kết quả như sau.

+) Nếu 1b ≠ và a(1-b)=1 hoặc |b|<1 thì dãy số có giới hạn 1

1 b−.

+) Các trường hợp còn lại dãy không hội tụ. Ví dụ 2.1.17. Cho dãy số (un) xác ñịnh

2 2

1 1, (1 2 ) , 1,2,...n n n

u b u u a u a n+

= = + − + ∀ =

Tìm tất cả các giá trị của a,b ñể ( )n

u có giới hạn hữu hạn. Tìm giới hạn ñó.

Hướng dẫn giải. Ta có 2

1 ( ) , 1,2,...n n n n

u u u a u n+

= + − ≥ ∀ = suy ra ( )n

u là dãy tăng.

Nếu dãy có giới hạn thì giới hạn ñó là nghiệm của phương trình x=x + (x-

a)2 hay giới hạn là a. Khi ñó ta có ,

nu a n b a≤ ∀ ⇒ ≤ và

Page 77: Chuyên đề hội thảo một số trường chuyên về dãy số

HỘ

HỘI CÁC TRƯỜNG THPT CHUYÊN KHU VỰC DUYÊN HẢI VÀ ðỒNG BẰNG BẮC BỘ HỘI THẢO KHOA HỌC LẦN THỨ IV

77

2 2

2 (1 2 ) 1 .u a b a b a a a b≤ ⇒ + − + ≤ ⇒ − ≤

Như vậy, ta có ñiều kiện cần là 1 .a b a− ≤ ≤ ðiều kiện ñủ.Với 1a b a− ≤ ≤ , ta có

1 1 1 1.n n n

u u b a a u+

≥ ≥ = ≥ − ⇒ − ≤ Giả sử

nu a≤ , ta sẽ chứng minh 1n

u a+

≤ . Thật vậy, ta có 2

1 ( ) ( )( ) .n n n n n n n n

u u u a u u a u a u u a a+

= + − = + − − < − + =

Như vậy dãy số (un) tăng và bị chặn trên bởi a do ñó nó có giới hạn và giới hạn ñó là a. Ví dụ 2.1.18. Cho dãy số (un) xác ñịnh

2

1 1,(0 1), 1, 1,2,...n n

u a a u qu n+

= < < = + ∀ =

Tìm tất cả các giá trị của q∈[0;1] ñể ( )n

u có giới hạn hữu hạn. .

Hướng dẫn giải. Giả sử dãy số ñã cho có giới hạn là A, khi ñó phương trình A=1+qA

2 có

nghiệm, suy ra 1

4q ≤ .

Với 1

04

q≤ ≤ , ta có 2 2

2 1 1 11 1 0u u qu u a qa− = + − = − + > (do 0<a<1).

Suy ra u2>u1 . Mặt khác, hàm số f(x)=1+qx2 ñồng biến trên [1;2) và f(1)=1+q; f(2)=1+4q

mà 1

04

q≤ ≤ nên f([1;2))⊆ [1;2).

Vì vậy (un) tăng và bị chặn trên bởi 2, do ñó dãy hội tụ. Vậy 1

04

q≤ ≤ .

2.3 Bài tập. Bài tập 2.3.1. Cho dãy số nguyên (xn) thỏa mãn

3

1 1 2

2, , 1.

3 1n

n

n

xx a x n

x+

= = ∀ ≥−

Tìm a ñể dãy số (xn) xác ñịnh và hội tụ.

Bài tập 2.3.2. Cho dãy số (xn) xác ñịnh 2

1 13, 9 11 3, 1.n n n

x x x x n+

= = + + ∀ ≥

Tìm a ñể dãy số (yn) với n

n n

xy

a= hội tụ.

Bài tập 2.3.3. Cho dãy số (xn) thỏa mãn 2

1 1 2

3 2 2 21, , 1.

2 2 2n

n

n n

xx a x n

x x+

+ −= ≠ = ∀ ≥

+ +

Xét tính hội tụ của dãy số và tìm giới hạn (nếu có) tùy theo trường hợp của a.

Bài tập 2.3.4. Cho dãy số (an) xác ñịnh

Page 78: Chuyên đề hội thảo một số trường chuyên về dãy số

HỘ

HỘI CÁC TRƯỜNG THPT CHUYÊN KHU VỰC DUYÊN HẢI VÀ ðỒNG BẰNG BẮC BỘ HỘI THẢO KHOA HỌC LẦN THỨ IV

78

1 1, (1 ), 1.n n n

a a a a a n+

= = − ∀ ≥

Tìm a ñể dãy số (an) hội tụ.

Bài tập 2.3.5. Cho dãy số (xn) xác ñịnh 2

1 1, 3 1, 1.n n n

x a x x x n+

= = + + ∀ ≥

Tìm a ñể dãy số (xn) hội tụ.

Bài tập 2.3.6. Cho dãy số (xn) xác ñịnh

1 1, , 1.nx

nx a x a n

+= = ∀ ≥

Chứng minh rằng 1

1 ea e< < thì dãy số (xn) hội tụ.

Bài tập 2.3.7. Cho dãy số (xn) xác ñịnh 3 2

1 1, 3 7 5 , 1.n n n n

x a x x x x n+

= = − + ∀ ≥

Tìm a ñể dãy số (xn) hội tụ.

Bài tập 2.3.8. Cho dãy số (xn) xác ñịnh

1 1, ln(3cos sin ) 2011, 1.n n n

x a x x x n+

= = + + ∀ ≥

Chứng minh rằng dãy số (xn) hội tụ.

Page 79: Chuyên đề hội thảo một số trường chuyên về dãy số

HỘ

HỘI CÁC TRƯỜNG THPT CHUYÊN KHU VỰC DUYÊN HẢI VÀ ðỒNG BẰNG BẮC BỘ HỘI THẢO KHOA HỌC LẦN THỨ IV

79

MỘT PHƯƠNG PHÁP DẠY TỰ HỌC KHÁI NIỆM GIỚI HẠN DÃY SỐ CHO HỌC SINH CHUYÊN TOÁN

Nguyễn Thế Sinh

Trường chuyên Nguyễn Trãi - Hải Dương Mở ñầu

Dạy học cho học sinh nói chung và học sinh chuyên Toán nói riêng, luôn cần quan tâm ñến các vấn ñề: trang bị kiến thức cơ bản ñể làm nền, hướng dẫn phương pháp suy nghĩ giải quyết vấn ñề và hướng dẫn phương pháp tự học. Với học sinh chuyên Toán, những học sinh ñã có sẵn sự say mê và tố chất tốt, thì việc hướng dẫn tự học cho học sinh càng cần ñược coi trọng. Khi học sinh biết cách tự học, thì không chỉ học ñược từ thầy cô mà còn học ñược từ bạn bè, biết cách khai thác tài liệu một cách hợp lý. Như thế, học sinh sẽ tận dụng ñược một nguồn tài liệu dồi dào trong thời ñại công nghệ thông tin như hiện nay. ðây là ñiều khiến cho học sinh sẽ tự mình hoàn thiện mình không chỉ khi còn học trong trường phổ thông mà còn cả khi ñã rời ghế nhà trường. Chính vì vậy, các thầy cô nên tạo ñiều kiện cho học sinh của mình ñược rèn luyện khả năng tự học càng nhiều càng tốt. Việc tự học của học sinh bao gồm các vấn ñề sau:

i) Xác ñịnh mục tiêu học tập ii) Tạo hứng thú và ñộng cơ học tập iii) Lập kế hoạch học tập iv) Tự mình biết cách suy nghĩ và nắm vững tri thức v) Tự mình biết cách trao ñổi thông tin, kiểm tra ñánh giá bản than

Trong các vấn ñề ñó, vấn ñề mấu chốt nằm ở chỗ học sinh có thể tự mình nắm vững tri thức. Muốn vậy, giáo viên phải hướng dẫn học sinh những cách suy nghĩ ñể giải quyết vấn ñề, các thao tác tư duy cơ bản: tương tự hóa, ñặc biệt hóa, khái quát hóa, lật ngược vấn ñề,phân tích, tổng hợp … có thể giúp học sinh tự mình tìm ñược hướng ñi khi gặp một bài toán.

Trong khuôn khổ một bài viết nhỏ, tôi chỉ muốn ñề cập ñến một phương pháp dạy học cho học sinh chuyên Toán, ñó là cách dạy học sinh tự học, và tôi cũng chỉ chọn một khái niệm ñể minh họa, ñó là khái niệm giới hạn dãy số. Bởi một lẽ, việc hiểu khái niệm và vận dụng ñược một khái niệm ñể giải quyết các vấn ñề khác mang lại sự khác biệt cho học sinh chuyên Toán so với học sinh các lớp thông thường.

Khi học một khái niệm, trước hết học sinh phải có một hình dung ñơn giản về khái niệm ñó, ñể có những chất liệu ban ñầu ñể nghĩ về nó, sau ñó dần dần ñược tiếp cận khái niệm ấy, ñịnh nghĩa nó và sau ñó là củng cố, vận dụng, cuối cùng là tổng hợp và sáng tạo.

Page 80: Chuyên đề hội thảo một số trường chuyên về dãy số

HỘ

HỘI CÁC TRƯỜNG THPT CHUYÊN KHU VỰC DUYÊN HẢI VÀ ðỒNG BẰNG BẮC BỘ HỘI THẢO KHOA HỌC LẦN THỨ IV

80

Trong phương pháp này, chúng tôi sử dụng con ñường quy nạp ñể dạy học khái niệm giới hạn dãy số. Hð1. Hình thành biểu tượng về khái niệm

Giáo viên cho học sinh thấy hình ảnh một dãy số có giới hạn 0 nhờ sự hỗ trợ của phần mềm Geometry Sketchpad chẳng hạn.

Hð2. Gợi ñộng cơ mở ñầu

• Giáo viên nêu ra một số vấn ñề thực tế có liên quan ñến giới hạn, yêu cầu học sinh dự ñoán kết quả. ðây là bước ñể học sinh ñưa ra quan ñiểm của riêng mình theo những gì học sinh vừa ñược cảm nhận bằng hình ảnh, ñồng thời tạo ñược hứng thú cho học sinh vì sự gần gũi thực tế của khái niệm

• Nghịch lý: Nếu chưa biết về giới hạn và bản chất của nó, sẽ có nhiều ñiều về sự vô hạn sẽ làm ta khó chịu, chẳng hạn:

Ví dụ: 1) Xét 23 3 31 ( ) ( )

2 2 2nx = + + + + +L L ( vô hạn số hạng)

Khi ñó: 23 3 3 3( ) ( )

2 2 2 2n

x = + + + +L L ( vô hạn số hạng)

Nên 31

2x x− = hay 2x = − . Nhưng rõ ràng x là tổng của các số dương nên x>0.

ðây là một nghịch lý

2) Bài toán A-sin (Achilis) ñuổi rùa Hð3. Tìm hiểu mục tiêu, yêu cầu và lập kế hoạch bài học

Lúc này, học sinh ñã sẵn sàng ñể bắt ñầu học về khái niệm một cách nghiêm túc và hăng hái, với hình dung bước ñầu về hình ảnh giới hạn dãy số. Giáo viên sẽ tổ chức cho học sinh tìm hiểu mục tiêu, yêu cầu theo cách của riêng mình, sau ñó giáo viên sẽ giúp thống nhất và hướng dẫn học sinh ñưa ra mục tiêu, yêu cầu cuối cùng, sau ñó lập ra kế hoạch tự học về khái niệm giới hạn dãy số. Giáo viên yêu cầu học sinh tự ñọc SGK bài dãy số có giới hạn 0, tóm lược những vấn ñề quan trọng cần chú ý ñể sau ñó trình bày trước lớp. Công việc này giáo viên phải hạn chế trong một thời gian ngắn ( 5 phút), ñể học sinh biết cách ñọc chỉ ñể lấy những thông tin quan trọng. Sau ñó giáo viên gọi ngẫu nhiên một ( hoặc một vài học sinh) ñể trình bày. Cuối cùng giáo viên sẽ thống nhất lại. Hð4. Giao nhiệm vụ tự học

Giáo viên cụ thể hóa nhiệm vụ tự học khi về nhà của học sinh dựa vào 2 căn cứ Căn cứ 1. Yêu cầu cần ñạt ñược, kế hoạch tự học tương ứng Căn cứ 2. Tài liệu Có 2 loại tài liệu:

i. Tài liệu chính: SGK, Tài liệu do giáo viên tự biên soạn, với cấu trúc phù hợp, từ dễ ñến khó, có dẫn dắt, gợi mở và tạo ñiều kiện cho các suy nghĩ mở

Page 81: Chuyên đề hội thảo một số trường chuyên về dãy số

HỘ

HỘI CÁC TRƯỜNG THPT CHUYÊN KHU VỰC DUYÊN HẢI VÀ ðỒNG BẰNG BẮC BỘ HỘI THẢO KHOA HỌC LẦN THỨ IV

81

rộng, tạo ñiều kiện cho học sinh ñược lật ngược vấn ñề, khái quát, tương tự, ñặc biệt hóa….

ii. Tài liệu tham khảo: Giáo viên giới thiệu them cho học sinh

Giáo viên nói trước với học sinh hình thức thảo luận trong buổi học tiếp theo và những yêu cầu về kết quả của việc tự học gồm

- Các câu trả lời cho các câu hỏi trong tài liệu thầy biên soạn - Tóm lược lại các vấn ñề quan trọng - Các câu hỏi tự minh ñặt ra trong quá trình tự học

Nội dung tài liệu tự học kiến thức cơ bản Tài liệu tự học kiến thức cơ bản

I. Khái niệm: Dãy số có giới hạn 0

ðoc và trả lời các câu hỏi sau: Tiếp cận khái niệm Câu 1. Biểu diễn các số hạng từ số hạng thứ nhất ñến số hạng thứ 20 của dãy số sau trên trục số:

a) 2nu

n=

b) ( 1)n

nun

−=

• Có nhận xét gì về sự thay ñổi khoảng cách giữa các ñiểm ñó với gốc 0 khi n tăng dần

• Có nhận xét gì về sự chuyển ñộng của các ñiểm ñó trên trục số khi n tăng dần

Câu 2. Các số hạng của dãy số ( 1)n

nun

−= , kể từ số hạng thứ mấy trở ñi, ñều có

giá trị tuyệt ñối nhỏ hơn 1

55 b) 1

500 c) 1

1000000 d) 1

m ( 10010m = ) f) 1

m, m là số rất lớn ñã cho

trước

Câu 3. Chuyển các mệnh ñề sau sang ký hiệu: Cho dãy số ( 1)( )

n

nun

−=

1) Với 1

500=ò ,( ò – ñọc là epsilon, ñây là một chữ cái Hy lạp, giống như

,α β vẫn thường dùng) bắt ñầu từ số hạng thứ 501 trở ñi, khoảng cách

giữa các số hạng của dãy số ( )nu ñến gốc 0 ñều nhỏ hơn ò

Page 82: Chuyên đề hội thảo một số trường chuyên về dãy số

HỘ

HỘI CÁC TRƯỜNG THPT CHUYÊN KHU VỰC DUYÊN HẢI VÀ ðỒNG BẰNG BẮC BỘ HỘI THẢO KHOA HỌC LẦN THỨ IV

82

2) Với 1

m=ò , chỉ có các số hạng từ số hạng thứ nhất ñến số hạng thứ m của

dãy ( )nu nằm ngoài khoảng ( , )−ò ò

3) Với một số ò dương bé tùy ý, từ một số hạng nào ñó trở ñi, thì mọi số hạng của dãy ( )nu ñều nằm trong khoảng ( , )−ò ò

4) Với một số ò dương, bé tùy ý, hoặc không có số hạng nào, hoặc chỉ có các số hạng từ số hạng thứ nhất ñến một số hạng thứ N nào ñó của dãy ( )nu ( N phụ thuộc vào ò), có khoảng cách ñến 0 lớn hơn ò

Câu 4. Ta nói: dãy số ( )nu với ( 1)n

nun

−= có giới hạn là 0 vì với mọi số dương bé

tùy ý cho trước, bắt ñầu từ một số hạng nào ñó trở ñi, mọi số hạng của dãy số ñều có giá trị tuyệt ñối nhỏ hơn số dương cho trước ñó. Hãy thử ñịnh nghĩa dãy số có giới hạn 0. Củng cố khái niệm Câu 5. Sau ñây là ñịnh nghĩa dãy số có giới hạn 0: ðịnh nghĩa: Dãy số ( )nu có giới hạn 0 ( hoặc có giới hạn là 0) nếu với mỗi số

dương nhỏ tùy ý cho trước, mọi số hạng của dãy số, kể từ một số hạng nào ñó

trở ñi, ñều có giá trị tuyệt ñối nhỏ hơn số dương ñó.

Khi ñó ta viết: lim( ) 0nu = hoặc lim 0nu = hoặc 0nu →

Hay có thể viết lim 0nn

u→+∞

= hoặc nu

n→+∞→ 0

Hãy chuyển ñịnh nghĩa trên sang ký hiệu. Hãy mô tả bằng hình học ñịnh nghĩa trên. Câu 6. Cho dãy số ( )nu có số hạng tổng quát:

0 ( 2 1)

1 ( 2 )

= +=

=n

n ku

n k, k nguyên, 0k ≥

Dãy trên có giới hạn bằng 0 là ñúng hay sai? Tại sao? Câu 7. Cho dãy số ( )nu có số hạng tổng quát:

0 ( 100)

1 ( 100)

>=

≤n

nu

n

Dãy trên có giới hạn bằng 0 là ñúng hay sai? Tại sao? Câu 8. Cho dãy số ( )nu có số hạng tổng quát:

1 ( 2010)

0 ( 2010)n

nu

n

>=

Dãy trên có giới hạn bằng 0 là ñúng hay sai? Tại sao?

Câu 9. Dãy số 1

n có tính chất: Với bất kỳ 0>ò , khoảng ( , )−ò ò hoặc chứa tất cả

các số hạng của dãy hoặc chỉ có một số hữu hạn các số hạng của dãy không thuộc khoảng trên là ñúng hay sai? Tại sao?

Page 83: Chuyên đề hội thảo một số trường chuyên về dãy số

HỘ

HỘI CÁC TRƯỜNG THPT CHUYÊN KHU VỰC DUYÊN HẢI VÀ ðỒNG BẰNG BẮC BỘ HỘI THẢO KHOA HỌC LẦN THỨ IV

83

Câu 10. Cho dãy số ( )nu với 2

1

1nun

=+

a) Chọn số tự nhiên N sao cho với mọi n>N thì khoảng cách giữa số hạng nu

và 0 nhỏ hơn 1

100

b) Chứng minh lim 0nu =

Câu 11. Cho dãy số ( )nu với 1n

nu

n=

+ và số 3

4=ò . Chứng minh rằng chỉ có hữu

hạn số hạng của dãy số ( )nu nhỏ hơn ò Câu 12. Hãy nêu mệnh ñề phủ ñịnh: “Dãy số ( )nu không có giới hạn 0”

Câu 13. Cho dãy số ( )nu với 3

2 1n

nu

n

−=

+. Chứng minh rằng dãy số trên không có

giới hạn 0 Câu 14. Cho dãy số ( )nu với ( 1) .n

nu n= − . Chứng minh rằng dãy số trên không có giới hạn 0 Câu 15. Cho 2 ví dụ về dãy số có giới hạn 0, 2 ví dụ về dãy số không có giới hạn 0 Câu 16. Cho dãy số ( )nu thỏa mãn: lim 0nu = và a thỏa mãn lim( ) 0nu a− = Chứng minh rằng không tồn tại số thực ò nào mà 2ò nằm giữa 0 và |a|, suy ra a=0

Câu 17. Cho dãy số ( )nu với (3 2 ) 1

2 1n

a n au

n

− + −=

−. Tìm a sao cho dãy số ñã cho có

giới hạn 0

Câu 18. Cho k là một số nguyên dương. Chứng minh rằng 1lim 0

kn

= ; 1lim 0

k n=

Câu 19. Chứng minh rằng lim 0nu = với 1

2n nu =

Gợi ý: Chú ý công thức khai triển Nhị thức Newton: ( 1)nx +

Câu 20. Chứng minh rằng sinlim 0

n

n=

Câu 21. Chứng minh rằng lim 03n

n=

Câu 22. Nếu a ∈� thỏa mãn: |a|>1 thì 1lim( ) 0n

a= hay nếu q ∈� và |q|<1 thì

lim 0nq =

Câu 23. Cho dãy số ( )nu thỏa mãn lim 0nu = . Xét dãy số ( )nv thỏa mãn: | |n nv u n N< ∀ ≥ . Chứng minh rằng lim 0nv =

II. Khái niệm: Dãy số có giới hạn hữu hạn

Tiếp cận khái niệm

Page 84: Chuyên đề hội thảo một số trường chuyên về dãy số

HỘ

HỘI CÁC TRƯỜNG THPT CHUYÊN KHU VỰC DUYÊN HẢI VÀ ðỒNG BẰNG BẮC BỘ HỘI THẢO KHOA HỌC LẦN THỨ IV

84

Câu 1. Theo ý em, dãy số ( )nu với ( 1)2

n

nun

−= + thì ñiểm biểu diễn cho số hạng

nu trên trục số thực sẽ tiến dần ñến vị trí nào khi n tăng dần, khi ñó giới hạn dãy số nu là bao nhiêu?

Câu 2. Cho dãy số ( )1n

nu

n=

+. Xét 1

1000=ò . Tìm N sao cho từ số hạng thứ N trở

ñi, các số hạng của dãy nu ñều cách số 1 một khoảng nhỏ hơn ò . Câu hỏi tương tự khi 0>ò bé tùy ý. Thử cho ý kiến : giới hạn của dãy số

nu là bao nhiêu? Câu 3. Hãy thử ñịnh nghĩa dãy số có giới hạn L ∈ � theo 2 cách ( dựa vào câu 1 và câu 2) Củng cố khái niệm Câu 4. ðịnh nghĩa 1: Ta nói dãy số ( )nu có giới hạn L ∈� nếu lim( ) 0nu L− = . Khi ñó ta

nói nu có giới hạn hữu hạn. Ký hiệu lim nu L= hoặc

nu L→

ðịnh nghĩa 2: Ta nói dãy số nu có giới hạn L ∈� nếu với mọi số dương ( bé tùy

ý) cho trước, kể từ một số hạng nào ñó trở ñi, mọi số hạng của nu khi biểu diễn

trên trục số ñều có khoảng cách ñến số L nhỏ hơn số dương cho trước ñó. Ký

hiệu lim nu L= hoặc nu L→

Hãy thử chuyển cả 2 cách ñịnh nghĩa sang ký hiệu , nò Câu 5. Chứng minh các kết quả sau:

a) 2 2lim

3 2 3

n

n=

+ b) 4 5

lim 22 7

n

n

+=

+ c)

12 1lim 2

2

n

n

+ +=

Câu 6. Cho dãy số ( )nu với 3 1

2 2n

nu

n

+=

+. Khoảng nào trong các khoảng sau chứa

hầu hết các số hạng của dãy

A.( 1 1, )

100 100− B. 9 11

, )10 10

C. 5 7( , )4 4

D. (0,1)

* ðến ñây, các em có thể trả lời cho các bài toán thực tế ( bài toán tuổi thọ, bài toán nhu cầu) ở phần ñầu Câu 7. Chứng minh dãy ( )nu với ( 1)n

nu = − không có giới hạn hữu hạn Câu 8. Chứng minh dãy số ( )nu với 2 1nu n= + không có giới hạn hữu hạn

Câu 9. Cho dãy số ( )nu với 4 2

2 7n

nu

n

+=

+. Xét số 1

100=ò . Chứng minh rằng từ một

số hạng thứ N nào ñó trở ñi, mọi số hạng của dãy ( )nu ñều thuộc khoảng 1 1

(2 , 2 )100 100

− + ( hay (2 , 2 )− +ò ò ). Tìm N. Nếu thay 1

10000=ò thì N bằng bao

nhiêu? Tìm giới hạn dãy số )nu Câu 10. Cho dãy số ( )nu thỏa mãn: lim nu L= .

a) Chứng minh rằng lim | | | |nu L=

Page 85: Chuyên đề hội thảo một số trường chuyên về dãy số

HỘ

HỘI CÁC TRƯỜNG THPT CHUYÊN KHU VỰC DUYÊN HẢI VÀ ðỒNG BẰNG BẮC BỘ HỘI THẢO KHOA HỌC LẦN THỨ IV

85

b) Chứng minh rằng 1| | | |

2nu L> với n>N nào ñó. Thay số 1

2 bởi số 1000

1001 có

ñược không?

Câu 11. Cho dãy số ( )nu thỏa mãn: lim nu L= . Chọn 0>ò tùy ý. ðặt 3 23

4M L= .

a) Chứng minh rằng | | .nu L M− < ò với n>N nào ñó

b) Chứng minh rằng 33limn

u L=

Câu 12. Cho dãy số ( )nu thỏa mãn: lim nu L= >0

a) Chứng minh với n>N nào ñó, 2n

Lu >

b) Cho k nguyên dương, k lẻ. Chứng minh: lim kkn

u L=

c) Nếu 0L ≤ , kết quả b) còn ñúng không? d) Nếu k chẵn, kết quả b) còn ñúng không?

Câu 13. Cho dãy số ( )nu với 2

2n

nu

n=

+ và ( )nv với

13 2

3

n

n nv

+ +=

a) Chứng minh rằng lim 2nu = và lim 3nv =

b) Cho 1

1000=ò . Tìm một số N sao cho 2 2 nu n N− < < + ∀ >ò ò . Tìm một số N’

sao cho 3 3 'nv n N− < < + ∀ >ò ò

c) Tìm một số 0N sao cho 05 2 5 2 n nu v n N− < + < + ∀ >ò ò

d) Tìm một số 1N sao cho 15 5 n nu v n N− < + < + ∀ >ò ò

e) Với một số ò bất kỳ, chứng minh tồn tại 1N ( phụ thuộc vào ò) sao cho

1| 5 | n nu v n N+ − < ∀ >ò

f) Tìm lim( )n nu v+

Câu 14. Cho lim nu L= , lim nv M= và c là một hằng số. Chứng minh rằng: a) lim( )n nu v L M+ = +

b) lim( )n nu v L M− = −

c) lim( . ) .nc u c L=

Câu 15. Cho lim 2nu = . Xét . 21n n

nv u

n= −

+

a) Với 0>ò tùy ý. Chứng minh rằng

2 1

1 21

| 2 |2n

n

u

< +

− <

ò

ò

với mọi n>N nào ñó

Page 86: Chuyên đề hội thảo một số trường chuyên về dãy số

HỘ

HỘI CÁC TRƯỜNG THPT CHUYÊN KHU VỰC DUYÊN HẢI VÀ ðỒNG BẰNG BẮC BỘ HỘI THẢO KHOA HỌC LẦN THỨ IV

86

b) Chứng minh lim 0nv =

Câu 16. a) Chứng minh mọi dãy số có giới hạn ñều bị chặn b) Cho lim , limn nu L v M= = . Hãy tìm cách chuyển | . |n nu v L M− về theo | |nu L− và | |nv M− ñể chứng minh lim .n nu v L M=

Câu 17. Nếu lim , limn nu L v M= = và 0M =/ a) Chứng minh rằng 0nv =/ với mọi n>N nào ñó

b) Xét dãy ,nn

n

ua n N

v= > và 1 2, , , Na a a… tùy ý. Chứng minh: lim n

La

M=

• Chú ý: Mặc dù dãy n

n

u

v có thể không xác ñịnh tại một vài số hạng ñầu (

chẳng hạn như: 0nv = khi n<100, 1n

nv

n=

+ khi n>100) nhưng với cách

hiểu như b), ta có thể coi lim n

n

u L

v M= và không cần quan tâm lắm ñến các

số hạng ñầu tiên ấy

Câu 18. [ Tính duy nhất của giới hạn] Cho lim nu L= và lim nu M= . Chứng minh L M= Gợi ý: Phản chứng, nếu L M=/ thì tồn tại số ò sao cho 0 2 | |L M< < −ò . Hãy suy ra mâu thuẫn từ ñây. Câu 19. Cho lim , limn nu a v b= = và n nu v n N≤ ∀ > . Chứng minh rằng a b≤ Câu 20. [Nguyên lý kẹp] Cho các dãy số ( ), ( ), ( )n n nu v w thỏa mãn: lim limn nu w L= = và

n n nu v w≤ ≤ với mọi n>N nào ñó. Chứng minh: lim nv L= Lời giải sau ñây có ñúng không? Nếu chưa ñúng, chỉ ra lỗi sai và chứng minh lại. Do n nu v n N≤ ∀ > nên lim limn nu L v= ≤ Tương tự: n nv w n N≤ ∀ > nên lim nv L≤ Từ 2 ñiều trên suy ra ñiều phải chứng minh Câu 21. [ Tổng của một cấp số nhân lùi vô hạn] Xét cấp số nhân lùi vô hạn 2

1 1 1 1, , , , ,nu u q u q u qL L có công bội q với |q|<1. Cấp số nhân như thế gọi là cấp số nhân lùi vô hạn. Xét 2

1 1 1 1n

nS u u q u q u q= + + + +L Khi ñó: lim nS S= ñược gọi là tổng của cấp số nhân ñã cho, ký hiệu là

21 1 1S u u q u q= + + +L . Tính S theo 1,u q

Áp dụng: Tính tổng của cấp số nhân có số hạng ñầu bằng 5 và công bội bằng 1

3

III. Khái niệm: Dãy số có giới hạn vô cực

Tiếp cận khái niệm

Page 87: Chuyên đề hội thảo một số trường chuyên về dãy số

HỘ

HỘI CÁC TRƯỜNG THPT CHUYÊN KHU VỰC DUYÊN HẢI VÀ ðỒNG BẰNG BẮC BỘ HỘI THẢO KHOA HỌC LẦN THỨ IV

87

Câu 1. Cho dãy số ( ) 2 1nu n= + . Xét M=100 000. Chứng minh bắt ñầu từ một số hạng thứ N nào ñó trở ñi, mọi số hạng của dãy ( )nu ñều lớn hơn M. ( hoặc mọi số hạng của dãy, kể từ số hạng thứ N, ñều thuộc khoảng ( , )M +∞ Nếu M=100 000 000 hay lớn hơn nữa, kết quả trên còn ñúng không? Nếu M lớn tùy ý thì sao? Câu 2. Ta nói dãy số ( )nu ở trên có giới hạn là dương vô cực ( +∞ ) Hãy ñịnh nghĩa dãy số có giới hạn +∞ Tương tự, hãy thử ñịnh nghĩa dãy số có giới hạn −∞ Củng cố khái niệm Câu 3. ðịnh nghĩa: Ta nói dãy số ( )nu có giới hạn là +∞ nếu với mỗi số dương

tùy ý cho trước, mọi số hạng của dãy số kể từ một số hạng nào ñó trở ñi ñều lớn

hơn số dương ñó. Khi ñó ta viết lim nu = +∞ hoặc nu → +∞

Ta nói dãy số ( )nu có giới hạn là −∞ nếu với mỗi số âm tùy ý cho trước, mọi số

hạng của dãy số kể từ một số hạng nào ñó trở ñi ñều bé hơn số âm ñó. Khi ñó ta

viết lim nu = −∞ hoặc nu → −∞

Các dãy số có giới hạn +∞ và −∞ ñược gọi chung là có giới hạn vô cực hay dần

ñến vô cực

Hãy chuyển các ñịnh nghĩa trên sang ký hiệu. Phát biểu mệnh ñề phủ ñịnh: “ Dãy số không có giới hạn vô cực” Câu 4. Cho dãy số ( )nu với 2 1nu n= + . Xét M=20 000. Khẳng ñịnh: “Chỉ có hữu hạn số hạng của dãy số nhỏ hơn 20 000” là ñúng hay sai? Câu 5. Chứng minh: a) lim n = +∞ b) $\lim (-3n^3)=-\infty$ Câu 6. Chứng minh 2lim( )n n− = +∞ Câu 7. Chứng minh dãy số ( )nu với ( 1) .n

nu n= − không có giới hạn vô cực Câu 8. Chứng minh dãy số ( )nu với 2( 1) cos( )nu n nπ= + không có giới hạn vô cực Câu 9. Chứng minh rằng nếu lim nu = +∞ và n nv u n N≥ ∀ > nào ñó thì lim nv = +∞ . Phát biểu kết quả tương tự cho trường hợp dãy số dần tới −∞ Áp dụng: Chứng minh: lim 2n = +∞

Câu 10. Chứng minh rằng nếu lim nu = +∞ hoặc lim nu = −∞ thì 1lim 0

nu= . ðiều

ngược lại có ñúng không? Khi nào thì ñiều ngược lại ñúng. Câu 11. Cho lim nu = +∞ , lim nv L= ∈� . Chứng minh rằng lim( )n nu v+ = +∞ . Hãy ñề xuất và chứng minh các kết quả tương tự với phép toán cộng, trừ, nhân, chia Câu 12. Cho lim 0, lim 0n nu L v= > = và 0nv > với n>N nào ñó. Chứng minh

lim n

n

u

v= +∞

ðề xuất các kết quả tương tự. Câu 13. Lấy ví dụ thỏa mãn yêu cầu sau:

a) lim , limn nu v= +∞ = +∞ nhưng lim( )n nu v− = +∞

Page 88: Chuyên đề hội thảo một số trường chuyên về dãy số

HỘ

HỘI CÁC TRƯỜNG THPT CHUYÊN KHU VỰC DUYÊN HẢI VÀ ðỒNG BẰNG BẮC BỘ HỘI THẢO KHOA HỌC LẦN THỨ IV

88

b) lim , limn nu v= +∞ = +∞ nhưng lim( )n nu v− = −∞

c) lim , limn nu v= +∞ = +∞ nhưng lim( ) 1n nu v− =

Các ví dụ trên cho thấy không thể tính lim( )n nu v− nếu biết ,n nu v→ +∞ → +∞ trong trường hợp tổng quát, mà chỉ tìm ñược khi ñã biết cụ thể ,n nu v . Vì thế không thể ñưa ra thành quy tắc cho trường hợp này. Người ta gọi giới hạn này là giới hạn dạng ∞ − ∞ , một trong những dạng vô ñịnh. Hãy ñề xuất các dạng vô ñịnh khác. Câu 14. Xem lại cách ñịnh nghĩa tổng của cấp số nhân lùi vô hạn. Hãy thử tự ñịnh nghĩa tổng của một dãy số vô hạn. Với ñịnh nghĩa ñó, hãy thử giải thích nguyên nhân gây ra nghịch lý số 1. IV- Tổng kết các kiến thức cơ bản về giới hạn Hãy tổng kết các cách ñịnh nghĩa các khái niệm giới hạn dãy số và các tính chất chính thu ñược nhờ ñịnh nghĩa. Hð5. Học sinh tự mình ñộng não, tham khảo tài liệu có sẵn ñể trả lời các câu

hỏi ñề ra và ñề xuất các ý kiến của riêng mình, chuẩn bị cho việc thảo luận

Hð6. Thảo luận, báo cáo, nhận xét, ñánh giá kết quả và thống nhất kiến thức

Tổ chức thảo luận

i) Giáo viên phân lớp thành 5 nhóm ( hoặc tùy thuộc vào tình hình riêng của lớp), mỗi nhóm phụ trách 1/3 số câu hỏi, có sự ñan xen các câu hỏi ở nhiều mức ñộ cho mỗi nhóm ( mỗi nhóm ñều có các câu hỏi dễ ( nhận biết) và khó ( vận dụng)), với các câu hỏi khó, có ít nhất 3 nhóm cùng phụ trách 1 câu ñể có nhiều ý kiến ñóng góp về cách trả lời

- Mỗi nhóm phân chia công việc cho người trả lời các câu hỏi, tạo ñiều kiện cho tất cả các thành viên trong nhóm có cơ hội phát biểu.

- Trong từng nhóm, người ñược giao trách nhiệm trả lời một câu hỏi có nhiệm vụ tham khảo ý kiến các thành viên trong nhóm ñể ñưa ra câu trả lời tốt nhất và thống nhất câu trả lời ñó ñể trả lời trước lớp nếu ñược yêu cầu. Tuy nhiên, giáo viên có thể chỉ ñịnh bất kỳ thành viên nào trong nhóm ñược yêu cầu trả lời về câu hỏi ñược ñưa ra.

- Với một câu hỏi, giáo viên sẽ tùy theo mức ñộ ñể hỏi vấn ñáp ( nếu dễ trình bày câu trả lời) hoặc viết lên bảng cụ thể ( nếu việc trình bày bài toán ñó có tính khái quát và không dễ ñể trình bày thông qua vấn ñáp)

- Mỗi nhóm có một muc tổng kết kiến thức chính cần ghi nhớ sau khi học xong

ii) Các nhóm phân công thành viên trình bày các câu trả lời theo yêu cầu của giáo viên, các nhóm khác có nhiệm vụ theo dõi, so sánh, ñối chiếu và phát biểu ý kiến thảo luận, ñăt câu hỏi cho nhóm trình bày trả lời

Page 89: Chuyên đề hội thảo một số trường chuyên về dãy số

HỘ

HỘI CÁC TRƯỜNG THPT CHUYÊN KHU VỰC DUYÊN HẢI VÀ ðỒNG BẰNG BẮC BỘ HỘI THẢO KHOA HỌC LẦN THỨ IV

89

iii) Với mỗi câu hỏi, giáo viên chia bảng làm 3 cột: cột 1 dành cho nhóm trả lời chính, cột 2: ghi các câu hỏi thảo luận chính ñược ñặt ra, cột 3: ghi lại vấn ñề sau khi ñã ñược giáo viên và cả lớp nhận xét, thống nhất ( thể chế hóa)

iv) Tổng kết lại các kiến thức quan trọng cần ghi nhớ. Một nhóm ñược giao phụ trách chính ( ngẫu nhiên), các nhóm khác ñóng góp, bổ sung, hoàn thiện

v) Giáo viên ñánh giá và tổng kết buổi thảo luận

Với hình thức thảo luận như thế, tất cả học sinh ñều phải tích cực nghiên cứu tài liệu ñược giao ñể hoàn thành thật tốt, ñồng thời ñều ñược tạo ñiều kiện ñể học sinh biết diễn ñạt vấn ñề một cách rõ ràng, ngắn gọn, chính xác. Phát hướng dẫn, ñáp án các câu trả lời ñã giao.

Việc này giúp học sinh một lần nữa ñược xem lại các kết quả ñã làm ñược, ñối chiếu so sánh kết quả của thầy với kết quả của bản thân và các bạn, ñồng thời giúp học sinh thống nhất ñược cách trình bày một bài toán sử dụng ñịnh nghĩa ñể giải bài toán giới hạn dãy số. ðáp án tài liệu tự học kiến thức cơ bản

Hð7. Giáo viên ñặt ra yêu cầu và giao nhiệm vụ tự học tiếp theo: Phần nâng

cao

Trong phần này, các bài tập ñưa ra không chỉ là vận dụng ñịnh nghĩa một cách ñơn giản nữa. Thay vào ñó, cần thực hiện các hoạt ñộng phức hợp mới có thể giải quyết ñược. Tuy nhiên, các bài tập có sự liên kết với nhau, trong ñó có những bài tập “ có tính dẫn dắt”, cùng với các bài tập tương tự hoặc ứng dụng nó. ( Một bài toán “có tính dẫn dắt” là bài toán chứa ñựng một số thông tin về kiến thức hoặc phương pháp có thể ñược dùng ñể giải các bài tập khác). Các bài tập cũng sẽ chứa ñựng một cách rõ ràng các thao tác tư duy quan trọng trong hoạt ñộng học. +) Giáo viên nêu rõ cho học sinh thấy các yêu cầu cần làm ñược với tài liệu:

- Trả lời ñược các câu hỏi trong tài liệu, trình bày rõ ràng, giải thích cách suy nghĩ dẫn ñến lời giải ( nếu ñược)

- Chọn lọc các bài toán “có tính dẫn dắt” và các bài toán tương tự với bài toán ñó trong tài liệu ñề xuất các bài toán tương tự không có trong tài liệu

- Tự ñề xuất các vấn ñề khác mình tham khảo ñược hoặc nghĩ ra ñược ñể tiếp tục nghiên cứu.

Tài liệu tự học nâng cao: Củng cố khái niệm giới hạn dãy sô Sau khi làm việc với tài liệu này, các em sẽ thấy ñược rằng, bản thân ñịnh nghĩa khái niệm giới hạn dãy số ñã là một công cụ rất mạnh ñể giải quyết các bài toán về giới hạn rồi. Hơn nữa, nó còn ñược dung ñể chứng minh một số ñịnh lý quan

Page 90: Chuyên đề hội thảo một số trường chuyên về dãy số

HỘ

HỘI CÁC TRƯỜNG THPT CHUYÊN KHU VỰC DUYÊN HẢI VÀ ðỒNG BẰNG BẮC BỘ HỘI THẢO KHOA HỌC LẦN THỨ IV

90

trọng và có ứng dụng lớn trong các bài toán về giới hạn dãy số như: ñịnh lý trung bình Cesaro, ñịnh lý Stolz… Ngoài ra, chú ý những câu hỏi phụ sau mỗi bài toán, các em sẽ dần có cách nhìn linh hoạt hơn cho từng bài toán, cách mở rộng, thêm bớt giả thiết ñể ñược các bài toán mới có ý nghĩa, thậm chí có những ý tưởng các em có thể tiếp tục phát triển thành các chuyên ñề nhỏ, hãy tự lựa chọn lấy ý tưởng ñó và viết chuyên ñề của riêng mình. Nội dung tài liệu Bài 1. Chứng minh rằng

a) lim 1n n = b) lim 1, 0n a a= > c) 2lim 0

!

n

n=

Bài 2. Tính các giới hạn sau:

a) 2 2 2

1 2 2lim( )

n n n

n n n

+ ++ + +L b)

2 2 2

1 1 1lim(1 )(1 ) (1 )

2 3 n− − … −

Bài 3. a) Cho dãy số { }nx , nếu 0| | . n

nx a c q n n− ≤ ∀ > , c là hằng số, 0<q<1 thì

lim nx a= .

b) Cho dãy số { }nx thỏa mãn 20 1

1 1, 1

3 2n nx x x+= = − . Tìm lim nx

Gợi ý: Nếu 21( ) 1

2f x x= − thì 1| | | ( ) ( ) | . | | 1,0 1n n nx f x f q x n qα α α+ − = − < − ∀ ≥ < <

• Nếu thay 0x c= bất kỳ thì lời giải bài toán sẽ như thế nào?

Bài 4. Cho dãy số

1

1

[0,1]

1

1n

n

u a

uu

+

= ∈

= +

Cmr nu hội tụ ( có giới hạn hữu hạn) và tìm lim nu .

• Thử thay ñổi giá trị a. Chẳng hạn nếu 1, 2a a> < − thì sao?

Bài 5.Cho ( )nu thỏa mãn 1 1

12; 2n

n

u uu

+= = + . Tìm lim nu

Bài 6. Tìm giới hạn ( nếu có) của dãy: ( )nu : 2

0 1

30, , 0

2( 1)n

n

n

uu u n

u+

+> = ≥

+

Bài 7. Cho hai dãy: 1 1 1 1( ), ( ) : 3, 2, 2 ,n n n n n n n na b a b a a b b a b+ += = = + = + . Tính giới hạn

lim n

n

a

b

Bài 8. Cho x>0 và 1 1(2 ), 2n n ny y xy n− −= − ≥ với 1 2, 0y y > Chứng minh rằng: ( )ny hội tụ và tìm lim ny

Page 91: Chuyên đề hội thảo một số trường chuyên về dãy số

HỘ

HỘI CÁC TRƯỜNG THPT CHUYÊN KHU VỰC DUYÊN HẢI VÀ ðỒNG BẰNG BẮC BỘ HỘI THẢO KHOA HỌC LẦN THỨ IV

91

Bài 9. Cho [0,1]a∈ và ( )ny thỏa mãn: 21 1

1, , 2

2 2 2n n

a ay y y n−= = − ≥ . Chứng minh

( )ny hội tụ và tìm lim ny

Bài 10. Cho c>2, dãy số ( )nx thỏa mãn: 0 1,n n

x c x c c x+= = − + nếu các biểu

thức trong căn không âm. Chứng minh rằng dãy ( )nx ñược xác ñịnh với mọi n và tồn tại lim nx

• Có thể hỏi câu hỏi khác cho bài toán này như sau:

“Cho 1 , 0n n

x c c x n+ = − + ≥ . Tìm c sao cho với mọi 0 (0, )x c∈ thì nx xác ñịnh và

hội tụ”

Bài 11. Cho dãy số ( )nx thỏa mãn 1

1lim( ) 0

2n nx x+ − = . Chứng minh rằng lim 0nx =

• Cần thay 1

2 bởi một số α thỏa mãn ñiều kiện gì ñể bài toán vẫn ñúng?

Bài toán: Tìm tất cả các số α ∈� sao cho na∀ mà 1lim( ) 0n na aα ++ = thì lim 0na = Bài 12.Giả sử phương trình 2 . 0x xα β+ + = có hai nghiệm trong ( 1;1)− . Chứng minh rằng với mọi dãy ( )nu thỏa mãn

1 1lim( ) 0n n nu u uα β+ −+ + =

thì lim 0nu = Các bài toán sau ñây nói ñến sự liên hệ giữa tính hội tụ của ( )nu và sự hội tụ

của các dãy con của nó

Bài 13. Chứng minh rằng nếu ( )nu là một dãy số thực, 2 2 1( ), ( )n nu u + hội tụ ñến cùng giới hạn a thì ( )nu hội tụ ñến a Bài 14. a) Cho dãy số 1( )n nu ≥ . Xét dãy số nguyên dương 1 2 ... ...nk k k< < < < . Khi ñó ( )

nku gọi là một dãy con của dãy ( )nu .Chứng minh nếu ( )nu hội tụ ñến a thì

mọi dãy con của nó cũng ñều hội tụ ñến a b) Chứng minh rằng nếu ( )nu là một dãy số thực, 2 2 1 3( ), ( ), ( )n n nu u u+ hội tụ thì ( )nu hội tụ.

• Nếu 2 trong 3 dãy trong câu b) hội tụ ñến cùng một giới hạn thì có suy ra sự hội tụ cũng như giới hạn của dãy số ( )nu hay không?

Bài 15. Cho hai dãy ( ), ( )n nx y thỏa mãn 0 0, 0x y > và

1 1

1 1; 0n n n n

n n

x x y y ny x

+ += + = + ∀ ≥

ðặt nM =Max { ; }n nx y . Chứng minh nM → +∞

Page 92: Chuyên đề hội thảo một số trường chuyên về dãy số

HỘ

HỘI CÁC TRƯỜNG THPT CHUYÊN KHU VỰC DUYÊN HẢI VÀ ðỒNG BẰNG BẮC BỘ HỘI THẢO KHOA HỌC LẦN THỨ IV

92

• Bài toán này có thể thay ñổi với việc tăng số lượng dãy số lên 3 dãy, hãy phát biểu bài toán như thế?

Tiếp theo là một số ñịnh lý quan trọng, có tính ứng dụng cao trong bài toán tìm

giới hạn dãy số

Bài 16. [ðịnh lý trung bình Cesaro]

Cho lim nu a= . Cmr 1 2lim nu u ua

n

+ + +=

L ( , , )a a a∈ = +∞ = −∞�

Sau ñây là một cách tổng quát hóa ñịnh lý trên

Bài 17.Cho lim na a= và lim nb b= . Chứng minh rằng

1 2 1 1lim n n na b a b a b

abn

−+ + +=

L

Bài 18. [ Bổ ñề Stolz] Cho 1 2 .., limn nv v v v< < < < = +∞L . Dãy ( )nu bất kỳ. Chứng minh rằng nếu

1

1

lim n n

n n

u ua

v v

+

+

−= ∈

−�

thì lim n

n

ua

v=

Hai bài toán sau ñây là các ví dụ ứng dụng ñịnh lý trung bình Cesaro và ñịnh lý

Stolz

Bài 19.Cho dãy số ( )nu thỏa mãn: 1lim( )n nu u a+ − = . Chứng minh rằng lim nua

n=

Bài 20. Cho 1

1 2k k k

n k

nu

n +

+ + +=

L . Tìm lim nu

Hð8. Học sinh tự mình thực hiện theo yêu cầu ñã ñề ra

Hð9. Tổ chức thảo luận và thống nhất kết quả.

Cách làm việc lúc này tương tự như trước, tuy nhiên sẽ có một vài sự khác biệt. ðó là hầu hết các bài giải ñều phải ñược trình bày chi tiết, ñặc biệt là các bài toán “có tính dẫn dắt”. Với các bài toán tương tự, học sinh nên chọn lọc thông tin mấu chốt ñể ghi lại thành từng ý tưởng, không cần ghi chi tiết các phép biến ñổi. ðiều này giúp những học sinh ở nhà chưa giải quyết ñược một bài tập nào ñó ( vì ñây là các bài tập khó) biết cách ñể ghi chép khi học trên lớp và biết cách tiếp tục hoàn thiện lời giải khi về nhà. Sự khác biệt nữa là cần dành thời gian cho việc ñề xuất các hướng mới có thể phát triển thành chuyên ñề ñể học sinh có thể tiếp tục tự học. Giáo viên phát ñáp án tự học phần nâng cao sau khi kết thúc buổi thảo luận ñể học sinh tham khảo. Hướng dẫn giải: Tài liệu tự học nâng cao: Củng cố khái niệm giới hạn dãy sô Bài 1. Chứng minh rằng

Page 93: Chuyên đề hội thảo một số trường chuyên về dãy số

HỘ

HỘI CÁC TRƯỜNG THPT CHUYÊN KHU VỰC DUYÊN HẢI VÀ ðỒNG BẰNG BẮC BỘ HỘI THẢO KHOA HỌC LẦN THỨ IV

93

a) lim 1n n = b) lim 1, 0n a a= > c) 2lim 0

!

n

n=

*********

a) Ta có 2 2 2 2. .1.1. .1 1 1nn n n n

n n nn nn n

+ + −= … ≤ = + − < + 1n∀ ≥

Hơn nữa 1 1n n n≥ ∀ ≥ nên với mọi 0>ò , xét 22[( ) 1]N = +

ò thì

:| 1 | 1n nn N n n∀ > − = − < ò . Vậy lim 1n n = b) Tương tự

+) Nếu a>1 thì 1 1

1 1n a n aa n

n n n

+ −≤ = + − ∀ ≥ nên với mọi 0>ò , xét [ 1]

aN = +

ò

thì ,| 1 | 1n nn N a a∀ > − = − < ò . Vậy lim 1n a =

+) Nếu a<1 thì ñặt 1

ba

= , ta có cách làm tương tự

+) Nếu a=1 thì hiển nhiên lim 1n a =

c) 2 2 2 2 2 2 4. . . 1

! 1 2 3 1

n

nn n n n

= … < ∀ ≥−

nên với mọi 0>ò , xét 4

[ 1]N = +ò

thì

2 2 ,| |

! !

n n

n Nn n

∀ > = < ò . Vậy 2lim 0

!

n

n=

Bài 2.

a) nu =2

2 2 2 2 2

1 2 2 . 1 2 3

2

n n n n n n n n

n n n n n

+ + + + + + ++ + + = =

LL . Chứng minh 3

lim2nu =

b) 2 2 2 2 2 2

1 1 1 1(1 )(1 ) (1 )

2 3 1.2 .3 .n

nu

n n

+= − − … − =

…<

2

1n

n

+ nên lim 0nu =

Bài 3. a) Cho dãy số { }nx , nếu 0| | . n

nx a c q n n− ≤ ∀ > , c là hằng số, 0 < q < 1 thì

lim nx a= .

b) Cho dãy số { }nx thỏa mãn 20 1

1 1, 1

3 2n nx x x+= = − . Tìm lim nx

*********

a) Xét 0>ò tùy ý, ta ñã có lim 0nq = nên tồn tại N sao cho , nn N q

c∀ > <

ò.

Như vậy 0 max{ , }n n N∀ > thì | | . n

nx a c q− ≤ < ò hay lim nx a=

b)

+) Nhận xét : 1 0 1nx n− < < ∀ ≥ . ( Chứng minh bằng quy nạp)

Page 94: Chuyên đề hội thảo một số trường chuyên về dãy số

HỘ

HỘI CÁC TRƯỜNG THPT CHUYÊN KHU VỰC DUYÊN HẢI VÀ ðỒNG BẰNG BẮC BỘ HỘI THẢO KHOA HỌC LẦN THỨ IV

94

+) ðặt 1 3α = − . Xét 2 21

1 1| | | ( ) | | || |

2 2n n n nx x x xα α α α+ − = − = − +

Mà 1 0 1nx n− < < ∀ ≥ nên | 1 3 | 3 1n

x n+ − < ∀ ≥ .

Vậy 11

3| | ( ) | | 1

2n

nx x nα α−− < − ∀ ≥ hay lim 1 3nx = −

*) Trong bài toán trên, ta thấy nếu 21( ) 1

2f x x= − thì

1| | | ( ) ( ) | . | | 1,0 1n n nx f x f q x n qα α α+ − = − < − ∀ ≥ < <

Lời giải trên xuất phát từ dự ñoán sau: +) Dự ñoán lim nx α= thì 1 3α = + hoặc 1 3α = −

+) 1 2

17 359;

18 648x x= − = − . Dự ñoán 0α < nên 1 3α = −

*Tại sao lại dự ñoán ñược như vậy?

Ngoài ra, trong bài toán trên, ñánh giá | 1 3 | 3n

x + − < là rất quan trọng và cần ñến 1 0nx− < < . *Vậy nếu thay 0x c= bất kỳ thì lời giải bài toán sẽ như thế nào?

Chẳng hạn: +) 1 3c = ± thì 1 3

nx n= ± ∀

+) | | 1 3c > + thì 1 3nx n> + ∀ và dãy { }nx tăng thực sự.

+) Chú ý khi ( 1,1 3)x ∈ − + thì 21( ) 1 ( 1;1 3)

2f x x= − ∈ − +

Bài 4. Cho dãy số

1

1

[0,1]

1

1n

n

u a

uu

+

= ∈

= +

Cmr nu hội tụ và tìm lim nu . ********

Xét 5 1

−= thì

1

α=

+

Ta có 1

1 1 1 1| | | | | | . | | . |

1 1 1 1n n

n n

u uu u

α αα α

+ − = − = −+ + + +

1| |

1 nu αα

< −+

do

1 1 1nu n+ > ∀ ≥

Do 1 21

1 5 1α= <

+ + nên tương tự bài 2, có lim nu α=

* Thử thay ñổi giá trị a. Chẳng hạn nếu 1, 2a a> < − thì sao?

Bài 5.Cho ( )nu thỏa mãn 1 1

12; 2n

n

u uu

+= = + . Tìm lim nu

********

Page 95: Chuyên đề hội thảo một số trường chuyên về dãy số

HỘ

HỘI CÁC TRƯỜNG THPT CHUYÊN KHU VỰC DUYÊN HẢI VÀ ðỒNG BẰNG BẮC BỘ HỘI THẢO KHOA HỌC LẦN THỨ IV

95

Tương tự bài 2,3, chứng minh 1

1| | | |n nu uα α

α+ − < − với 1 2 1α = + >

Bài 6. Tìm giới hạn ( nếu có) của dãy: ( )nu : 2

0 1

30, , 0

2( 1)n

n

n

uu u n

u+

+> = ≥

+

******** +) Bằng quy nạp, ta chứng minh ñược 0 0nu n> ∀ ≥

+) Nếu dãy ( )nu có giới hạn α thì 2 3

2( 1)

αα

α

+=

+ và 0α ≥ nên 1α =

+) Xét 2

1

( 1)| 1|

2( 1)n

n

n

uu

u+

−− =

+

Nếu 1 0nu − > thì 1 10

2( 1) 2n

n

u

u

−< <

+ nên 1

1| 1| | 1|

2n nu u+ − < −

Nếu 1 0nu − ≤ thì | 1 | 1nu − ≤ . Mà 2( 1) 2nu + > nên | 1| 1

2( 1) 2n

n

u

u

−≤

+. Suy ra

1

1| 1| | 1|

2n nu u+ − < −

Vậy 1

1| 1| | 1| 0

2n nu u n+ − ≤ − ∀ ≥

Tương tự các bài trên, có lim 1nu = Bài 7. Cho hai dãy: 1 1 1 1( ), ( ) : 3, 2, 2 ,n n n n n n n na b a b a a b b a b+ += = = + = + . Tính giới hạn

lim n

n

a

b

******** Nhận xét rằng 0 1nb n= ∀ ≥/

ðặt , 1nn

n

ac n

b= ≥ thì 1

22 2

11

n

n n n nn

nn n n

n

a

a b b cc

aa b c

b

+

++ +

= = =+ +

+

, 1n ≥

Chứng minh bằng quy nạp ñược 1 1nc n> ∀ ≥

Nếu nc có giới hạn, thì dự ñoán ñược giới hạn ñó là 2α =

Xét 1

2 1| 2 | | | 2 |

2( 2 1)( 1)n

n n

n

cc c

c+

−− = ≤ −

+ +

Suy ra lim 2n

c = Bài 8. Cho x>0 và 1 1(2 ), 2n n ny y xy n− −= − ≥ với 1 2, 0y y >

Chứng minh rằng: ( )ny hội tụ và 1

limn

yx

=

*******

Xét 1 1

1 1(2 )n n ny y xy

x x− −− = − − 1 1 1

1 1( ) ( )n n ny xy y

x x− − −= − + −

Page 96: Chuyên đề hội thảo một số trường chuyên về dãy số

HỘ

HỘI CÁC TRƯỜNG THPT CHUYÊN KHU VỰC DUYÊN HẢI VÀ ðỒNG BẰNG BẮC BỘ HỘI THẢO KHOA HỌC LẦN THỨ IV

96

1 1 1

1 1( ) ( )n n ny xy yx x

− − −= − − + − 21

1( )nx y

x−= − −

Suy ra 211 (1 ) 2n nxy xy n−− = − ∀ ≥ nên

1211 (1 ) , 2

n

nxy xy n−

− = − ∀ ≥

Hay 12

1

1 1| | |1 |

n

ny xyx x

− = − ( *)

Lại có 2 1 1 1 1(2 ) 0 2 0 1 1y y xy xy xy= − > → − > ↔ − > − Mà 11 1xy− < nên 1|1 | 1xy− < (**)

Từ (*) và (**) Suy ra 1

lim nyx

=

Bài 9. Cho [0,1]a∈ và ( )ny thỏa mãn: 21 1

1, , 2

2 2 2n n

a ay y y n−= = − ≥ . Chứng minh

( )ny hội tụ và tìm lim ny ********

+) Nhận xét: 0 1ny n> ∀ ≥

+) Dự ñoán: lim 1 1n

y L a= = + −

+) Xét 1

1| | | ( )( ) |

2n n ny L y L y L+ − = − +

+) Chứng minh 1 1 2| | 1 1

2 2 2ny L a n+ < + ≤ ∀ ≥

Suy ra lim ny L=

Bài 10. Cho c>2, dãy số ( )nx thỏa mãn: 0 1,n n

x c x c c x+= = − + nếu các biểu

thức trong căn không âm. Chứng minh rằng dãy ( )nx ñược xác ñịnh với mọi n và tồn tại lim nx

******** +) Chứng minh ( )nx xác ñịnh với mọi n bằng phương pháp quy nạp

+) Nếu ( )nx có giới hạn a thì a là nghiệm dương của phương trình 2 1x x c+ + =

+) Xét 2

1

|| | | | | |n

n n

n

c c x ax a c c x a

c c x a+

− + −− = − + − =

+ + +

2

2

1 ( 1) ( ) | || | | | | |

1 1n n n n

n

a c x a c x x a x a

a a c x a a a c c a c

+ − + + − + − −≤ = ≤ =

+ + + + + − +

Mặt khác 1 11

1 1c a c a c< <

− + +

Suy ra lim nx a=

• Có thể hỏi câu hỏi khác cho bài toán này như sau:

“Cho 1 , 0n n

x c c x n+ = − + ≥ . Tìm c sao cho với mọi 0 (0, )x c∈ thì nx xác ñịnh và

hội tụ”

Page 97: Chuyên đề hội thảo một số trường chuyên về dãy số

HỘ

HỘI CÁC TRƯỜNG THPT CHUYÊN KHU VỰC DUYÊN HẢI VÀ ðỒNG BẰNG BẮC BỘ HỘI THẢO KHOA HỌC LẦN THỨ IV

97

Các bài toán từ 2 ñến 9 là các bài tập chứng minh dãy số cho bởi công thức truy hồi có giới hạn hữu hạn bằng ñịnh nghĩa, có sử dụng một nguyên tắc ñơn giản, có thể chứng minh trục tiếp bằng ñịnh nghĩa: “Cho dãy số { }nx , nếu

0| | . n

nx a c q n n− ≤ ∀ > , c là hằng số, 0<q<1 thì lim nx a= .” Kết quả này cũng có thể nhìn theo phương diện khác và có thể mở rộng thêm.

Chẳng hạn:

1) Cho 1lim | |n

n

aq

a

+ = . Khi ñó

+) Nếu q<1 thì lim 0na = +) Nếu q>1 thì lim na = ∞

2) Cho lim | |nna q= . Khi ñó:

+) Nếu q<1 thì lim 0na = +) Nếu q>1 thì lim | |na = +∞

Bài 11. Cho dãy số ( )nx thỏa mãn 1

1lim( ) 0

2n nx x+ − = . Chứng minh rằng lim 0nx = *********

Có 0 1 0

10, :| |

2n nn x x n n+∀ > ∃ − < ∀ >ò ò

Suy ra 1

1| | | |

2n nx x+ < +ò.

Vậy 0 0

2 11 1 1 1| | (1 ( ) ( ) ) | | ( ) 1

2 2 2 2m m

n m nx x m−

+ < + + + + + ∀ ≥Lò

nên 0 0

1| | | | ( ) 1

1 212

m

n m nx x m+ < + ∀ ≥

ò . Do ñó lim 0nx =

* Cần thay 1

2 bởi một số α thỏa mãn ñiều kiện gì ñể bài toán vẫn ñúng?

Bài toán: Tìm tất cả các số α ∈� sao cho na∀ mà 1lim( ) 0n na aα ++ = thì lim 0na = Lời giải: ðiều kiện cần: Giả sử α thỏa mãn ñề bài. Chọn dãy ( 1)n n

na α= − Khi ñó 1 0n na aα ++ = nên suy ra lim 0na = hay lim( ) 0nα− = . Vậy | | 1α < ðiều kiện ñủ: Ngược lại, nếu | | 1α < , làm tương tự bài trên, ta ñược ñiều phải chứng minh. Bài 12.Giả sử phương trình 2 . 0x xα β+ + = có hai nghiệm trong ( 1;1)− . Chứng minh rằng với mọi dãy ( )nu thỏa mãn

1 1lim( ) 0n n nu u uα β+ −+ + =

thì lim 0nu = ********

Page 98: Chuyên đề hội thảo một số trường chuyên về dãy số

HỘ

HỘI CÁC TRƯỜNG THPT CHUYÊN KHU VỰC DUYÊN HẢI VÀ ðỒNG BẰNG BẮC BỘ HỘI THẢO KHOA HỌC LẦN THỨ IV

98

Giả sử phương trình 2 0x xα β+ + = có 2 nghiệm 1 2, ( 1,1)x x ∈ − . Khi ñó theo ñịnh lý Viet: 1 2 1 2, .x x x xα β+ = − = Ta có 1 1 2 1 2 1 1 1 2 1 1lim( ( ) ) 0 lim[( ) ( )] 0n n n n n n nu x x u x x u u x u x u x u+ − + −− + + = ↔ − − − = ðặt 1n n nv u u −= − thì ta có 1 2lim( ) 0n nv x v+ − = . Lập luận tương tự bài trên thì: lim 0nv = Lặp lại lập luận một lần nữa cho ( )nu , ta ñược lim 0nu = . Ta có ñiều phải chứng minh

Bài 13. Chứng minh rằng nếu ( )nu là một dãy số thực, 2 2 1( ), ( )n nu u + hội tụ ñến cùng giới hạn a thì ( )nu hội tụ ñến a

********* Ta cần chứng minh

0, / ,| |nN n N u a∀ > ∃ ∈ ∀ > − <�ò ò

trong khi ñã có

1 1 1 2 1 0, / ,| |nN n N u a∀ > ∃ ∈ ∀ > − <�ò ò

1 2 2 2 1 2 0, / , | |nN n N u a+∀ > ∃ ∈ ∀ > − <�ò ò

ñến ñây vấn ñề ñã rất ñơn giản, chỉ cần chọn 1 2= =ò ò ò và 1 2max{ , }N N N= , ta có ñiều phải chứng minh Bài 14. a) Cho dãy số 1( )n nu ≥ . Xét dãy số nguyên dương 1 2 ... ...nk k k< < < < . Khi ñó ( )

nku gọi là một dãy con của dãy ( )nu .Chứng minh nếu ( )nu hội tụ ñến a thì

mọi dãy con của nó cũng ñều hội tụ ñến a b) Chứng minh rằng nếu ( )nu là một dãy số thực, 2 2 1 3( ), ( ), ( )n n nu u u+ hội tụ thì ( )nu hội tụ.

******** a) Hiển nhiên b) Giả sử 2lim nu a= , 2 1 3lim , limn nu b u c+ = = .

Do 6( )nu là dãy con dãy 2( )nu và 3( )nu nên 6lim nu a= và 6lim nu c= . Vậy a=c. Tương tự, xét dãy 6 3nu + , ta ñược b=c. Suy ra a=b Theo bài 5, ta ñược lim nu a=

Bài 15. Cho hai dãy ( ), ( )n nx y thỏa mãn 0 0, 0x y > và

1 1

1 1; 0n n n n

n n

x x y y ny x

+ += + = + ∀ ≥

ðặt nM =Max { ; }n nx y . Chứng minh nM → +∞ ********

Page 99: Chuyên đề hội thảo một số trường chuyên về dãy số

HỘ

HỘI CÁC TRƯỜNG THPT CHUYÊN KHU VỰC DUYÊN HẢI VÀ ðỒNG BẰNG BẮC BỘ HỘI THẢO KHOA HỌC LẦN THỨ IV

99

Ta có 2 2 2 2 2 21 1 2 2

1 12 2 4n n

n n n n n n

n n n n

x yx y x y x y

x y y x+ ++ = + + + + + > + + 0n ≥

Áp dụng liên tiếp bất ñẳng thức trên, ta ñược 2 2 2 21 1 0 0 ( 1).4 4

n nx y x y n n+ ++ > + + + >

Suy ra 1 2n

M n+ >

Với M>0 bất kỳ, xét 2[ ] 12

MN = + thì n N∀ > , nM M>

Vậy lim nM = +∞ Bài toán này có thể thay ñổi với việc tăng số lượng dãy số lên 3 dãy, cụ thể: Bài toán: Cho ba dãy ( ), ( )n nx y ( )nz thỏa mãn 0 0 0, , 0x y z > và

1 1 1

1 1 1; , 0n n n n n n

n n n

x x y y z z ny z x

+ + += + = + = + ∀ ≥

ðặt nM =Max { ; , }n n nx y z . Chứng minh nM → +∞ Bài 16. [ðịnh lý trung bình Cesaro]

Cho lim nu a= . Cmr 1 2lim nu u ua

n

+ + +=

L ( , , )a a a∈ = +∞ = −∞�

******** Ta chứng minh cho trường hợp a ∈�

Với 0>ò tùy ý. Do lim nu a= nên tồn tại N sao cho 1

,| |2nn N u a∀ > − < ò

Xét 1 2 1 2( ) ( ) ( )| | | |n nu u u u a u a u a

S an n

+ + + − + − + + −= − =

L L

1 2| | | | | | ( )

2nu a u a u a M n N

n n n

− + − + + − −≤ < +

L ò 1

2

P

n= +ò

Với 1 2

1| | | | | |, .

2NM u a u a u a P M N= − + − + + − = −L ò

Mặt khác, tồn tại 1N N> sao cho 1

1

2N P<

ò .

Suy ra 1

1 1 ,

2 2n N S∀ > < + =ò ò ò

Ta có ñiều phải chứng minh Trường hợp lim nu = +∞ hoặc lim nu = −∞ , chứng minh tương tự Bài 17.Cho lim na a= và lim nb b= . Chứng minh rằng

1 2 1 1lim n n na b a b a b

abn

−+ + +=

L

******** Nhận xét: ðây là kết quả tổng quát của ñịnh lý trung bình Cesaro ở trên. Thật vậy, cho dãy 1 nb n= ∀ , bài toán này trở thành bài toán trên Ta chứng minh kết quả tổng quát Ta ñã có kết quả: “Mọi dãy số có giới hạn ñều bị chặn” nên tồn tại A dương sao cho | | 1na A n< ∀ ≥

Page 100: Chuyên đề hội thảo một số trường chuyên về dãy số

HỘ

HỘI CÁC TRƯỜNG THPT CHUYÊN KHU VỰC DUYÊN HẢI VÀ ðỒNG BẰNG BẮC BỘ HỘI THẢO KHOA HỌC LẦN THỨ IV

100

Với mọi 0>ò tùy ý. Xét 1 2 1 1.

| |n n na b a b a bS ab

n

−+ + += −

L

1 2 1 1( ) ( ) ( )| n n na b b a b b a b b

n

−− + − + + −=

L+ 1 2( ) ( ) ( )

|nb a a b a a b a a

n

− + − + + −L

< 1 1 1 2| | | | | | | | | | | || | | |n n n

b b b b b b a a a a a aA b

n n

−− + − + + − − + − + + −+

L L

Mặt khác lim na a= và lim nb b= nên tồn tại N sao cho

1 1 ,| | ,| |n nn N a a b b∀ > − < − <ò ò với 1 2(| | | |)A b=

+

òò

Suy ra 1 1 1 1 2 1( ) | | | | | | | | | | | | ( )

| | | |N n Nn N b b b b b b a a a a a a n N

S A bn n

−− + − + − + + − − + − + + − + −< +

L Lò ò

1(| | | |)M

A bn

< + +ò

Với 1 1 1 1 1 1| | | . . | | . . | | | | | | | | | | | ||N n N nM A N b N b b b b b b a a a a a a− −= − − + − + − + + − + − + − + + −L Lò ò

là hằng số

Chọn 1N N> sao cho 2

1

1

N M<

ò thì 1 2 ,M

n Nn

∀ > < ò với 2 2=

òò

Vậy, 1 n N∀ > thì S < òhay 1 2 1 1lim n n na b a b a bab

n

−+ + +=

L

Bài 18. [ Bổ ñề Stolz]

Cho 1 2 .., limn nv v v v< < < < = +∞L . Dãy ( )nu bất kỳ. Chứng minh rằng nếu: 1

1

lim n n

n n

u ua

v v

+

+

−= ∈

−�

thì lim n

n

ua

v=

********

Do 1

1

lim n n

n n

u ua

v v

+

+

−= ∈

−� nên 0, N∀ > ∃ ∈�ò sao cho

1

1

, n n

n n

u un N a

v v

+

+

−∀ > − < − <

−ò ò 1

1

n n

n n

u ua a

v v

+

+

−↔ − < < +

−ò ò(1)

Vì 1 0n nv v+ − > nên (1) 1 1 1( )( ) ( )( )n n n n n nv v a u u v v a+ + +↔ − − < − < − +ò ò Do ñó:

1 1 1( )( ) ( )( )N N N N N Nv v a u u v v a+ + +− − < − < − +ò ò

2 1 2 1 2 1( )( ) ( )( )N N N N N Nv v a u u v v a+ + + + + +− − < − < − +ò ò ….

1 1 1( )( ) ( )( )n n n n n nv v a u u v v a− − −− − < − < − +ò ò Cộng theo về các bất ñẳng thức trên, ta ñược ( )( ) ( )( )n N n N n Nv v a u u v v a− − < − < − +ò ò

Page 101: Chuyên đề hội thảo một số trường chuyên về dãy số

HỘ

HỘI CÁC TRƯỜNG THPT CHUYÊN KHU VỰC DUYÊN HẢI VÀ ðỒNG BẰNG BẮC BỘ HỘI THẢO KHOA HỌC LẦN THỨ IV

101

n N

n N

u ua a

v v

−↔ − < < +

−ò ò

Suy ra lim n N

n N

u ua

v v

−=

Mặt khác:

(1 )n N N n N

n n n n N

u u v u u

v v v v v

−= + −

Và lim lim 0N N

n n

u v

v v= = vì lim nv = +∞ nên có lim n

n

xa

y=

Trong kết quả trên, nếu thay số thực a bởi +∞ hoặc −∞ , với cách làm tương tự, ta ñược kết quả tương tự. Các bài toán sau ñây là các kết quả ứng dụng ñịnh lý trung bình Cesaro và ñịnh lý Stolz

Bài 19.Cho dãy số ( )nu thỏa mãn: 1lim( )n nu u a+ − = . Chứng minh rằng lim nua

n=

******** ðặt 1, 1n n nv u u n−= − ≥ thì lim nv a= nên theo ñịnh lý trung bình Cesaro,

1 2lim nv u va

n

+ + +=

L hay 0lim nu ua

n

−=

Từ ñó suy ra lim nua

n=

Bài 20. Cho 1

1 2k k k

n k

nu

n +

+ + +=

L .

Tìm lim nu Hướng dẫn Xét 11 2 ;k k k k

n nx n y n+= + + + =L . Rõ ràng

ny là dãy tăng và lim ny = +∞ . Lại có

1

1 11

( 1) 1

( 1) 1

k

n n

k k

n n

x x n

y y n n k

+

+ ++

− += →

− + − +

nên theo bổ ñề Stolz thì 1lim

1nuk

=+

Các hướng nghiên cứu có liên quan • Tìm giới hạn dãy số cho bởi công thức truy hồi: 1 1, ( ), 1n nu u f u n+∈ = ≥� nhờ

ñịnh nghĩa • Tìm giới hạn dãy số nhờ ứng dụng ñịnh lý trung bình Cesaro, ñịnh lý

Stolz • Tìm giới hạn dãy số thông qua các dãy min, max

• Các vấn ñề về giới hạn dãy số thông qua dãy con

Kết luận:

Page 102: Chuyên đề hội thảo một số trường chuyên về dãy số

HỘ

HỘI CÁC TRƯỜNG THPT CHUYÊN KHU VỰC DUYÊN HẢI VÀ ðỒNG BẰNG BẮC BỘ HỘI THẢO KHOA HỌC LẦN THỨ IV

102

Với cách làm việc như vậy, học sinh ñược tự mình làm việc, ñược trao ñổi thông tin ñể kiểm chứng qua bạn bè, thầy cô, tài liệu, ñược trình bày vấn ñề, ñược ñánh giá và nhận xét, ñược gợi mở các hướng làm việc tiếp theo. Ở ñây, tôi mới nói ñến chỉ một khái niệm giới hạn dãy số, mà nếu xem xét kỹ lưỡng bằng thời gian trên lớp thì phải mất ñến vài tuần, nhưng với cách làm ñưa ra, học sinh có thể tận dụng tối ña thời gian ở nhà của mình ñể học, thời gian trên lớp chỉ mất 1 tuần. Hơn nữa, học sinh sẽ có hứng thú ñể tiếp tục khai thác các vấn ñề xung quanh, hướng tới việc học ñược toàn diện mảng kiến thức về dãy số và giới hạn bằng cách tự học, giáo viên chỉ là người gợi mở và nhận xét, ñánh giá. Tuy nhiên, nhược ñiểm của phương pháp này là giáo viên sẽ mệt hơn, vì phải xem xét và ñánh giá bài làm và các bài viết chuyên ñề của học sinh. Nhưng làm việc với môn Toán chuyên, không thể thiếu ñi sự nhiệt tình như thế.

Page 103: Chuyên đề hội thảo một số trường chuyên về dãy số

HỘ

HỘI CÁC TRƯỜNG THPT CHUYÊN KHU VỰC DUYÊN HẢI VÀ ðỒNG BẰNG BẮC BỘ HỘI THẢO KHOA HỌC LẦN THỨ IV

103

CHUYÊN ðỀ VỀ CẤP SỐ, CĂN NGUYÊN THỦY

THẶNG DƯ BẬC HAI Phạm Văn Ninh

Trường THPT Chuyên Hạ Long - Quảng Ninh

I. Cấp của một số ðịnh nghĩa 1.1: Cho n là 1 số nguyên dương n > 1 và a là một số nguyên thỏa

mãn (a, n) = 1. Số nguyên dương h nhỏ nhất sao cho ah≡ 1 (modn) ñược gọi là

cấp của a modun n, ký hiệu h = ordna

Ví dụ: cấp của 2 modun 3 là 2, modun 5 là 4 vì 22 ≡ 1 (mod3), 24 ≡ 1 (mod5)

Các tính chất Tính chất 1: Nếu số nguyên a có cấp là h (modn) và số nguyên b thỏa mãn

b ≡a (modn) thì b cũng có cấp là h.

bordhnab

aordhn

n=⇒

=

)(mod

Ví dụ: 10 = ord112 ⇒ 10 = ord1113

Tính chất 2: Nếu cấp của a (modn) là h thì

2.1. ak ≡ 1 (modn) ⇔ h|k (k Ν∈ )

2.2. ak ≡ la (modn) ⇔ h (k- l )

Tính chất 3: Nếu a có cấp là h (modn), k Ν∈ *, (k, h) = 1 thì ak cùng có cấp là h

(modn).

+ Tổng quát: Nếu a có cấp là h (modn), k Ν∈ * thì ak sẽ có cấp là )(mod),gcd(

nhk

h

(gcd(k,h): ước chung lớn nhất của k, h).

Tính chất 4: Nếu a có cấp h1 (modn); b có cấp h2 (modn) và (h1, h2) = 1

Khi ñó cấp của ab (modn) là h = h1.h2

Chú ý: Trường hợp (h1, h2)> 1 thì ta có r = ordn (ab) thì h1.h2 M r

Tính chất 5: Nếu a có cấp là h(modn) thì hệ A = {1, a, a2,...,ah-1} gồm h số ñôi

một không ñồng dư với nhau (modn)

Tính chất 6: Cho các số n1, n2,…., nk nguyên tố ñôi một n = n1 .n2…. nk. Giả sử

a có cấp hi (modni). Khi ñó cấp của a (modn) là h = lcd (h1, h2,…hk)

Hệ quả từ tính chất 2.1

1. Theo ñịnh lý Ơle: aφ(m) ≡ 1 (modm)

Với (a, m) = 1 ⇒ luôn tồn tại cấp số của a mod m, (a, m)=1

Page 104: Chuyên đề hội thảo một số trường chuyên về dãy số

HỘ

HỘI CÁC TRƯỜNG THPT CHUYÊN KHU VỰC DUYÊN HẢI VÀ ðỒNG BẰNG BẮC BỘ HỘI THẢO KHOA HỌC LẦN THỨ IV

104

⇒ ordma φ (m)

2. ðịnh lý Fecma: Cho p là 1 số nguyên tố, (a, p) = 1

thì ap-1 ≡ 1 (modp)

⇒ ordp a (p-1 )

Bài tập áp dụng:

Bài 1.1: Tìm tất cả các số nguyên dương n sao cho 2n – 1Mn

Lời giải: +) n = 1 thoả mãn

Xét n > 1. Gọi p là ước nguyên tố nhỏ nhất của n

+) Giả sử nếu p = 2 ⇒ n chẵn mà 2n-1 lẻ ⇒ p ≥ 3

Ta gọi h = ord2 (p) ⇒ h (p-1 ) ⇒h ≤ (p-1)

lại có 2n ≡ 1 (modp)

Theo tính chất 2 ⇒ h|n mà p là ước nguyên tố

⇒ p≤ h (2)

Từ (1) và (2) ⇒ mâu thuẫn

Vậy n=1 thỏa mãn 2n-1 Mn

Bài 1.2: Chứng minh rằng: n φφφφ (an - 1) với ∀ a, n ∈N*, a≥ 2

Lời giải: Với n = 1, ta có ñpcm

Nếu n 2≥ , ñặt m = an - 1

⇒ an ≡ 1 (mod m). ðặt h = ordm (a)

Nếu h < n, ta có 1 < ah < an - 1 = m (do a, n 2≥ )

Mà m |(ah-1) suy ra ah > m ( vô lý)

Vậy h = n. Ta lại có h φ (m) (hệ quả)

⇒ n φ (an - 1) (ñpcm)

Bài 1.3: Tìm số n nguyên dương nhỏ nhất thỏa mãn: 22005 17n - 1

Lời giải: Giả sử tìm ñược n thoả mãn

Gọi d là cấp của 17 khi chia cho 22005

⇒ 17d ≡ 1 (mod 22005)

⇒ d φ (22005) = 22004 (theo hệ quả)

⇒ d = 2k với k = {1,2,…, 2004}

⇒ 22005 217k

- 1

Page 105: Chuyên đề hội thảo một số trường chuyên về dãy số

HỘ

HỘI CÁC TRƯỜNG THPT CHUYÊN KHU VỰC DUYÊN HẢI VÀ ðỒNG BẰNG BẮC BỘ HỘI THẢO KHOA HỌC LẦN THỨ IV

105

Ta lại có: 217

k

- 1 = (17-1) (17+1) (172+1)….(1217

−k

+1)

Với i ≥ 1 thì 217i

+1 ≡ 2 (mod4)

⇒ v2 (k217 - 1) = 4 + k

(v2 (m) là số mũ của 2 trong phân tích của m)

⇒ k + 4 ≥ 2005 ⇒ k ≥ 2001. Do tính chất của d

⇒ d = 22001

Bài 1.4: Tìm tất cả các số nguyên tố p,q thỏa mãn 2

2

1 2003 1

1 2003 1

q

p

p

q

+ +

+ +

Lời giải: Giả sử tìm ñược bộ (p,q) t/m ñề bài

Không mất tính tổng quát, giả sử p ≤ q

TH1: p = 2, ta có 5 2003q + 1

có 2003 ≡ 3 (mod5) ⇒ q chẵn (do 2003q = - 1 (mod5))

⇒ q = 2

Nhận thấy bộ (2,2) thoả mãn

TH2: p> 2, gọi r là ước nguyên tố lẻ của p2 + 1

Bổ ñề: Cho 2 số nguyên dương a và b. Khi ñó ước nguyên tố của a2 + b2 bằng 2

hoặc có dạng 4k + 1 , ((a, b) = 1) , (a, b ≥ 1)

Chứng minh

Giả sử a2+b2 có ước nguyên tố khác 2. Giả sử số nguyên tố bất kỳ ñó dạng

4k + 3, ta có: a2 = - b2 (mod 4k +3)

Theo Fecma: a4k+2 = b4k+2≡ 1 (mod 4k + 3)

⇒ a2(2k+1) ≡ b2(2k+1) ≡ (-a2) (2k+1) (mod 4k + 3)

⇒ 2a4k+2 ≡ 0 (mod 4k + 3)

Có 2a4k+2 ≡2 (mod 4k + 3) ⇒ mâu thuẫn.

* Nếu a = 1, b lẻ ⇒ a2 + b2 có 1 ước là 2

b lẻ, b = 1

Trở lại bài toán. Vì p2 + 1 2003q - 1

⇒ r | 2003q + 1 ⇒ r | 20032q - 1

Gọi d = ordr (2003) ⇒ d (2q) (t/c 2)

Giả sử gcd (d,q) = 1 ⇒ d 2 ⇒ r 20032 - 1 = 23.3.7.11.13.167

Theo bổ ñề ta có r là ước của p2 + 1 nên r = 13

Page 106: Chuyên đề hội thảo một số trường chuyên về dãy số

HỘ

HỘI CÁC TRƯỜNG THPT CHUYÊN KHU VỰC DUYÊN HẢI VÀ ðỒNG BẰNG BẮC BỘ HỘI THẢO KHOA HỌC LẦN THỨ IV

106

Có 2003q + 1 ≡ 1q + 1 ≡ 2 (mod13) (vô lý).

Vậy gcd (d,q) ≠ 1 ⇒ q d, lại có theo ñịnh lý Fecma ar-1≡ 1 (modr), a = 2003

⇒ d (r-1) ⇒ (r-1) M q hay r ≡ 1 (modq) (*)

Có phân tích p2 +1 = 2. kn

n

kkppp ...2

21

1 (trong ñó pi ≥ 3, ki 1≥ )

Có ñược phân tích này là do: p2 +1 ≡ 2 (mod4)

Từ (*) (r ≡ 1 (modq)) ⇒ ∀ i ta có pi ≡ 1 (modq)

⇒ p2 +1 ≡ 2 (modq)

⇒ q (p-1)(p+1) mà q ≥ p > p - 1

⇒ q = p + 1 ⇒ p = 2 (loại do p > 2)

Vậy bộ (p, q) = (2,2) là bộ duy nhất t/m

Bài 1.5: Tìm tất cả các bộ (b,q,r) nguyên tố thỏa mãn:

p qr +1, q rp +1, r pq +1

Lời giải: Rõ ràng các nguyên tố p, q, r phải khác nhau

Ta sẽ c/m trong 3 số p, q, r phải có 1 số bằng 2

Giả sử p, q, r > 2, ta có p qr +1 ⇒ p q2r - 1

⇒ ordp (q) 2r

+) Nếu ordp (q) lẻ ⇒( ) 1

( )

p

p

ord q

ord q r

=

=

+) Nếu ordq(p)=1 suy ra p| q-1 hay qr+1 2(mod )p≡ , ( loại do qr + 1 M p)

+) Nếu ordp (q) = r ⇒ p qr -1 mà p qr +1 ⇒p 2 ⇒ p =2 (loại)

=

=

rqord

qord

p

p

2)(

2)(

+) ordp (q) = 2r ⇒ 2r p - 1 (hệ quả)

⇒ pq - 1 = 1q - 1 ≡ 0 (modr)

mà pq + 1 ≡ 0 (modr) ⇒ 2 ≡ 0 (modr) suy ra r = 2 (loại)

+) ordp (q) = 2 ⇒ p q2 - 1

Nếu p q - 1 thì p q2 - 1 mà p qr + 1 ⇒ p 2 (loại)

⇒ p q + 1 mà q + 1 chẵn, p lẻ ⇒ p2

1+q

Tương tự: q2

1+r , r2

1+p

Page 107: Chuyên đề hội thảo một số trường chuyên về dãy số

HỘ

HỘI CÁC TRƯỜNG THPT CHUYÊN KHU VỰC DUYÊN HẢI VÀ ðỒNG BẰNG BẮC BỘ HỘI THẢO KHOA HỌC LẦN THỨ IV

107

⇒ p + q + r ≤ 2

1+q +2

1+r +2

1+p

⇒ p + q + r ≤ 3 (vô lý)

Vậy phải có ít nhất một số bằng 2. Giả sử p = 2

⇒ q, r lẻ và q r2 + 1 và r 2q +1

Ta có ordr (2) 2q

Nếu ordr (2) M q ⇒ q r - 1 (do ordr (2) r-1)

⇒ q (r2 + 1) - (r2-1) = 2 ⇒ q = 2 (loại)

Vậy ordr (2) 2⇒ r 22 - 1 hay r 3 ⇒ r = 3

⇒ q 10 ⇒ q = 5

Vậy bộ (p, q, r) = (2, 5, 3) = (3, 2, 5) = (5, 3, 2) là các bộ t/m

Bài 1.6: Tìm tất cả số nguyên tố p, q thỏa mãn pq 2p + 2q

Lời giải: ðể ý rằng (p, q)= (2, 2), (2, 3), (3, 2) thỏa mãn ycbt

Ta sẽ c/m không còn bộ nào khác thỏa mãn

Giả sử p ≠ 2, q ≠ 2

ðặt p - 1 = nl2 , q - 1 = 2km , m, n lẻ, l, k ≥ 1

Vì pq 2p + 2q nên theo ñịnh lý Fecma nhỏ ta có

0 ≡ 2p + 2q ≡ 2p + 2. 2q-1 ≡ 2p + 2 (modq)

⇒ 2p-1 = -1 (modq)

ðặt x = 2n, ta có

2p-1 = n.22l

= ( )2

2nl

= l2

x

⇒l2

x ≡ - 1 (modp) ⇒ ordq (x) = 2l+1

Lại có: ordq (x) q-1⇒2l+1 2km⇒ l +1 ≤ k

Tương tự k + 1 ≤ l ⇒ l ≤ k - 1 ≤ l - 2 (vô lý)

Vậy p = 2 hoặc q = 2

+) Giả sử q = 2 ⇒ p 2p + 2q = 2p + 4

Theo ðL Fecma 0 ≡ 2p + 4 ≡ 2 + 4 = 6 (modp) ⇒

=

=

3

2

p

p

Vậy bài toán có nghiệm (p, q) là (2, 2,), (2, 3), (3, 2)

Bài 1.7

Page 108: Chuyên đề hội thảo một số trường chuyên về dãy số

HỘ

HỘI CÁC TRƯỜNG THPT CHUYÊN KHU VỰC DUYÊN HẢI VÀ ðỒNG BẰNG BẮC BỘ HỘI THẢO KHOA HỌC LẦN THỨ IV

108

Cmr: Với một số nguyên dương n ( n > 1), 3n - 2n không chia hết cho n

Lời giải: ðịnh lý: Cho a, m là các số nguyên, (a, m) = 1. Khi ñó tồn tại số nguyên x sao

cho ax ≡ 1 (modm)

Nếu x1, x2 t/m ax1 ≡ ax2 ≡ 1 (modm) thì x1 ≡ x2 (modm)

Gợi ý: (a, m) = 1 nên{a, 2a,…(m - 1)a} lập thành hệ thặng dư ñầy ñủ modun m

Trở về bài toán:

Giả sử ∃ n thỏa mãn n 3n - 2n

Gọi p là ước nguyên tố nhỏ nhất của n

Ta có n 3n - 2n ⇒ p 3n - 2n

mà 3 không là ước của 3n - 2n và 2 không là ước của 3n - 2n

⇒ p ≥ 5.

Ta có (2, p) = 1 ⇒ ∃ số nguyên dương a t/m : 2a ≡1 (modp) (theo ñịnh lý)

Có 3n ≡ 2n (modp) ⇒ (3a)n ≡ (2a)n ≡ 1 (modp)

ðặt d = ordp (3a) ⇒

nd

pd 1 mà p là ước nguyên tố nhỏ nhất của n.

⇒ d = 1 ⇒ 3a ≡ 1 (modp) mà 2a ≡ 1(modp)

⇒ a ≡ 0 (modp) (vô lý)

Vậy ta có ñpcm

Bài 1.8: Tìm tất cả các số nguyên dương m, n thỏa mãn (n > 1)

n 1 + 3 2.3++++n n

m m

Lời giải: Từ n 1 + 1)(13233 −⇒+

+nnn

mnmm

ðặt d = ordn(m) ⇒ d 3n+1 ⇒ d = 3k (k Ν∈ )

+) Nếu k ≤ n ⇒ d 3n ⇒ n 13 −n

m

⇒ 3133.2 ≡++nn

mm (modn) ⇒ n = 3 ⇒ 3 1 +33 3.23

mm +

Từ ñây dễ thấy phải chọn m ≡ 1 (mod3)

+) Nếu k ≥ n + 1 thì d = 3n+1 mà d φ (n) ⇒ d < n

lại có 3n+1 > n (vô lý)

Vậy bài toán có nghiệm (m, n) = (m, 3) trong ñó m ≡ 1 ( mod3)

Page 109: Chuyên đề hội thảo một số trường chuyên về dãy số

HỘ

HỘI CÁC TRƯỜNG THPT CHUYÊN KHU VỰC DUYÊN HẢI VÀ ðỒNG BẰNG BẮC BỘ HỘI THẢO KHOA HỌC LẦN THỨ IV

109

Bài 1.9: Gọi a, n > 2 là các số nguyên dương thỏa mãn n an - 1 - 1 và n

không chia hết cho bất kỳ ước nào của ax - 1 với x < n - 1

C/m: n là số nguyên tố

Lời giải: ðặt d = ordn

(a)

Từ n | an - 1-1 ⇒ d | n - 1

Nếu d < n - 1, ta có gcd (n, ad - 1) = n (vô lý)

⇒ d ≥ n - 1 ⇒ d = n - 1

Lại có d | φ (n) (hệ quả) ⇒ n - 1 | φ (n)

mà φ (n) ≤ n - 1 ⇒ φ (n) = n - 1

⇒ n là số nguyên tố.

Bài 1.10: Xác ñịnh tất cả các số nguyên n > 1 thỏa mãn: 2

2 1++++∈ Ν∈ Ν∈ Ν∈ Ν

n

n

Lời giải:

Giả sử ∃ n thỏa mãn ycbt

Có 2n + 1 lẻ ⇒ n lẻ

Gọi p là ước nguyên tố nhỏ nhất của n (p lẻ)

Có 2n + 1 ≡ 0 (modn2) ⇒ 2n ≡ - 1 (modp)

⇒ 22n ≡ 1 (modp)

Theo ðL Fecma: 2p-1≡ 1 (modp)

Từ tính chất 2, ta dễ dàng có bổ ñề sau

Nếu am ≡ 1 (modp)

an ≡ 1 (modp)

=⇒

)(mod1

),gcd(

pa

nmh

h

Áp dụng bổ ñề ta có: 2d ≡ 1 (modp), d = gcd (p-1, 2n)

Do p là ước số nguyên tố nhỏ nhất của n, p-1 chẵn ⇒ d = 2

⇒ 22 ≡ 1 (modp) ⇒ p = 3

ðặt n = 3km với k ≥ 1 , gcd (m,3) = 1, m lẻ

Ta có phân tích sau:

)1)...(1)(1)(1(111 33.233.223 +−+−+−+=+

−− kkk

xxxxxxxx

⇒ )122)...(122)(122)(12(121211 33.233.22.3 +−+−+−+=+=+

−−mmmmmmmmn

kkk

Ta có )1)(9)(mod1(2 .3 ≥−≡ imi

(do m lẻ)

( ) 122

3 ≡mi

(mod9)

Page 110: Chuyên đề hội thảo một số trường chuyên về dãy số

HỘ

HỘI CÁC TRƯỜNG THPT CHUYÊN KHU VỰC DUYÊN HẢI VÀ ðỒNG BẰNG BẮC BỘ HỘI THẢO KHOA HỌC LẦN THỨ IV

110

⇒ 3122 33.2 ≡+− mm ii

(mod9)

Hơn nữa: 2

2

6 1 2 2 1 4 2 1 3(mod9)

6 5 2 2 1 25 5 1 3(mod 9)

m m

m m

m t

m t

= + ⇒ − + ≡ − + ≡

= + ⇒ − + ≡ − + ≡

⇒ 3k | (22m - 2m +1)….( )122 .3.3.2 11

+−−−

mmkk

và 3k+1 χ (22m - 2m +1)….( )122 .3.3.2 11

+−−−

mmkk

mà 32k | 2n +1 (do n2 | 2n +1)

⇒ 3k | 2m +1

Lại có: 6 1 2 1 3(mod 9)

6 5 2 1 6(mod9)

m

m

m t

m t

= + ⇒ + ≡

= + ⇒ + ≡

⇒ k = 1 ⇒ n = 3m

⇒ 9m2| 23m + 1

Gọi q là ước nguyên tố nhỏ nhất của m (q ≥ 5)

Ta có 23m ≡ - 1 (modq) ⇒ 26m ≡ 1 (modq)

Theo bổ ñề ⇒ 2t ≡ 1 (modp), t = gcd (q - 1,6m)

Do t χ m (theo cách chọn q) ⇒ t ∈ {1,2,3,6}

Có 2t ≡ 1(modq) ⇒ q = {3,7} mà q ≥ 5 ⇒ q = 7 với t =6

⇒49 | 23m + 1 ⇒ 7 | 23m + 1 ⇒ 7 | 2 (vô lý)

Vậy m = 1 ⇒ n = 3

Bài 1.11: Có bao nhiêu số nguyên dương n là bội số của 1001 và biểu diễn

ñược dưới dạng n = 10j - 10i với i, j Ν∈ , 0 ≤≤≤≤ i < j ≤≤≤≤ 99

Lời giải:

Ta có n = 10j - 10i = 10i (10j-i - 1) và 1001 = 7.11.13

là số nguyên tố cùng nhau với 10i.

⇒n M 1001 ⇔ 10j-i ≡ 1 (mod 1001)

Dễ thấy ord 1001(10) = 6 ⇒ j - i M 6 hay j - i = 6m (m Ν∈ *)

Như vậy, số các số n bằng số các bộ (i,j) thỏa mãn pt

i + 6m = j với 0 ≤ i < j ≤ 99, m Ν∈ *

Dễ thấy với mỗi m ∈ {1,2,…,16} sẽ có 100-6m giá trị của i (và j). Vậy số

nghiệm của phương trình là 16

1

(100 6 ) 784m

m=

− =∑

Vậy 784 số n thỏa mãn ñề bài

Page 111: Chuyên đề hội thảo một số trường chuyên về dãy số

HỘ

HỘI CÁC TRƯỜNG THPT CHUYÊN KHU VỰC DUYÊN HẢI VÀ ðỒNG BẰNG BẮC BỘ HỘI THẢO KHOA HỌC LẦN THỨ IV

111

Bài 1.12: Tìm tất cả các số nguyên tố p,q sao cho : pq | (5p - 2p) (5q-2q)

Lời giải:

Có (5p - 2p) (5q - 2q) M 5 và M 2 (giả sử ∃ p, q thỏa mãn )

⇒ p, q khác 2 và 5

Không mất tính tổng quát, giả sử p ≤ q

+ Nếu p | 5p - 2p . Theo ðL Fecma ta có 5p - 2p ≡ 5 - 2 = 3 (modp)

p = 3

⇒ 3q | (5q-2q). 117 ⇒ q | (5q-2q). 3. 13

⇒ 3

13

| 5 2 | 3 3( )q q

q

q

q q q theoFecma−

=

= ⇒ ⇒ =

Vậy PT có nghiệm (p,q) = (3,3) và (3,13)

+) Nếu p | (5q-2q)

+) q | (5q - 2q) thì suy ra (p,q) = (3,3)

+) q | (5p - 2p)

Tương tự bài 1.7, do (2, p) = 1, (2, q) = 1

Nên ∃ a, b Ν∈ thỏa mãn 2a ≡ 5 (modp), 2b ≡ 5 (modq)

(TQ ñịnh lý 1.7: gcd (a,m) = 1, pt ax ≡ b (modm) luôn có nghiệm )

Có 5q≡2q (modp)

⇒ 5q.aq ≡ (2a)q ≡5q (modp)

⇒aq ≡ 1 (modp)

Tương tự : bp ≡ 1 (modp)

ðặt h = ordp (a) ⇒

−1|

|

ph

qh

Do p-1 < q và q nguyên tố

⇒ h = 1

⇒ a ≡ 1 (modp) ⇒ 2a ≡ 2 ≡ 5 (modp)⇒ p = 3

Tương tự q = 3

Vậy PT có nghiệm (p, q) là (3, 3); (3, 13), (13, 3)

II. Căn nguyên thủy

ðịnh nghĩa. Cho n là số nguyên dương và số nguyên a sao cho (a, n) = 1. Khi

ñó a ñược gọi là một căn nguyên thủy (mod n) nếu cấp của a (mod n) bằng φ(n)

+) a là một căn nguyên thủy (mod n)

=

=

)()(

1),gcd(

naord

na

n φ

Page 112: Chuyên đề hội thảo một số trường chuyên về dãy số

HỘ

HỘI CÁC TRƯỜNG THPT CHUYÊN KHU VỰC DUYÊN HẢI VÀ ðỒNG BẰNG BẮC BỘ HỘI THẢO KHOA HỌC LẦN THỨ IV

112

Ví dụ: Số 3 là một căn nguyên thủy của 7 vì ϕ (7) = 6 và

31 ≡ 3; 32 ≡ 2, 33 ≡ 6, 34≡4, 35

≡ 5, 36≡1 (mod7)

Các tính chất

Tính chất 1

Nếu a là căn nguyên thủy ( modn) thì mọi số cùng lớp với a theo (modn) ñều là

căn nguyên thủy (modn).

a là căn nguyên thủy (modn) )(mod nab ≡

⇒ b là căn nguyên thủy (modn)

Tính chất 2 Nếu a là căn nguyên thủy (modn) thì tập A = {1,a,a2,…,ah-1} là hệ thặng dư thu

gọn (modn) (lúc này h = φ (n))

Tính chất 3:

Nếu p là một số nguyên tố thì có ñúngφ (p - 1) căn nguyên thủy (modp)

Tính chất 4:

Nếu p là một số nguyên tố lẻ và g là một căn nguyên thủy (modp2) thì g cũng là

căn nguyên thủy (mod pn) với n ≥3.

Ví dụ: 2 là căn nguyên thủy của 9

(ñề nghị chứng minh tại sao 2 là căn nguyên thuỷ của 9)

⇒2 là căn nguyên thủy của 3n ∀ n ≥ 3

ðịnh lý về sự tồn tại căn nguyên thủy

Cho m là một số nguyên, m > 1 khi ñó m có căn nguyên thủy khi và chỉ khi m

có một trong 4 dạng sau: 2, 4, pα, 2p α (trong ñó p là 1 số nguyên tố lẻ)

Bài tập áp dụng:

Bài 2.1: Tìm các số nguyên tố phân biệt p, q sao cho:

a3pq ≡a (mod3pq) ∀ a ∈N*

Lời giải:

Giả sử chọn ñược bộ (p,q) t/m

Giả sử p > q. Cho a= 3, ta ñược.

33pq ≡ 3 (mod3pq) ⇒ 3(33pq-1-1) M 3pq ⇒ p, q ≠ 3

Chọn a = 3pq - 1 ⇒ (-1) 3pq-1≡1 (mod3pq)

⇒ (-1) 3pq-1≡1 (mod3pq)

⇒ 3pq - 1 chẵn ⇒ p,q lẻ ⇒ p, q >3

⇒ p≥ q + 2

Chọn a là căn nguyên thủy modp

Page 113: Chuyên đề hội thảo một số trường chuyên về dãy số

HỘ

HỘI CÁC TRƯỜNG THPT CHUYÊN KHU VỰC DUYÊN HẢI VÀ ðỒNG BẰNG BẮC BỘ HỘI THẢO KHOA HỌC LẦN THỨ IV

113

Ta có ap-1 ≡ 1 (modp)

Từ a3pq≡ a (modp) ⇒ p -1 | 3pq - 1 ⇒ p - 1 | 3q - 1

Tương tự: q - 1 | 3p - 1

Có p ≥ q + 2 ⇒ 3 1 3 13

1 1

q q

p q

− −≤ <

− +

+) Nếu 3q - 1 = p - 1 ⇒ p=3q (loại)

+ ) Nếu 3q - 1 = 2(p-1) ⇒ p = 2

13 +q

⇒ (q-1) | 3p -1 = 2

19 +q ⇒ q - 1 | (9q+1) - (9q-9) = 10

⇒ q = 11 ⇒ p = 17; q=3 suy ra p=5

Bằng cách thử trực tiếp ta thấy (p,q)=(5,3) không thoả mãn.

Vậy (p,q) là (11, 17) và (17, 11) là bộ t/m

Bài 2.2: CMR: Nếu g là 1 căn nguyên thủy modun p2 thì g là một căn

nguyên thủy modun p, p là số nguyên tố.

Lời giải: ðặt h = ordp (g) ⇒ h | p - 1

Do gh ≡1 (mod p) ⇒ ∃ q ∈ N* sao cho gh = pq + 1

Ta có gph = (pq+1)p = (pq)p + 1)(...)( 111 +++−− pqcppc p

pp

p

Do 1 2 21(mod ) ( )ph

pc p g p ph pϕ= ⇒ ≡ ⇒ M

⇒ ph M p(p-1) ⇒ h M (p-1) ⇒ h = p - 1

Vậy g là 1 căn nguyên thủy của modp.

Bài 2.3: Chứng minh ñịnh lý Willsơn dựa vào căn nguyên thủy

Lời giải: Vì g1, g2… gp-1 là 1 hệ thặng dư thu gọn mod p

(trong ñó, p là số nguyên tố, g là căn nguyên thủy mod p)

⇒ (p-1) ! ≡ g1. g2… gp-1 ≡ ( 1) 1

2 2 1p p p

g g

− −

≡ ≡ − (modp)

Bài tập bổ sung

Bài 1:Tìm tất cả các số nguyên dương n, n < 1000 sao cho n có dạng

n = p1p2p3 (p1, p2, p3 là các số nguyên tố) và 2n + 2 M n

Lời giải:

Giả sử tìm ñược n t/m

Không mất tính tổng quát, g/s p1 < p2< p3

Page 114: Chuyên đề hội thảo một số trường chuyên về dãy số

HỘ

HỘI CÁC TRƯỜNG THPT CHUYÊN KHU VỰC DUYÊN HẢI VÀ ðỒNG BẰNG BẮC BỘ HỘI THẢO KHOA HỌC LẦN THỨ IV

114

Nếu p1 > 2, tức là n, p1, p2, p3 lẻ

p là một ước nguyên tố bất kỳ của n

có 2n + 2 ≡ 0 (modp) ⇒ 2n-1 ≡ -1 (modp)

⇒22(n-1) ≡ 1 (modp) ðặt h = ordp (2)

⇒ h| gcd (p-1, 2(n-1) = 2k.m1 (k ≥ 1, m1 lẻ)

⇒ 12 .2 1k m ≡ (modp)

ðặt n - 1 = 2t . m2. Nếu t ≥ k ⇒ 2n-1 ≡1 (modp) (mâu thuẫn)

⇒ m2 M m1

⇒ t ≤ k - 1

Vậy pi = 2 ik .xi + 1 (xi lẻ)

n = 2tm2 + 1 (t ≤ ki - 1 , i = 1,2,3) (mi, xi lẻ)

⇒ t ≤ k , k = min {ki}

Có: n = ( 12k x1 + 1) ( 22k x2 + 1) ( 32k + 1)

≡ 1 (mod 2k) (mâu thuẫn do n = 2tm2 + 1)

Vậy p1= 2

Ta cần tìm p2, p3 sao cho 2 322 p p + 2 M 2p2p3

⇒ 2 34 p p + 2 ≡ 0 (modp p2p3)

⇒2

3

2

3

4 2 0(mod )

4 2 0(mod )

p

p

p

p

+ ≡

+ ≡

Có n = 2p2p3 < 1000 ⇒ p2p3 < 5000 ⇒ p2 ≤19

+) Thay p2 = 3 ⇒ 43 + 2 ≡ 0 (modp3)

⇒ p3 = 11 (t/m)

+) p2 = 5 ⇒ có 34 p + 2 ≡ (-1)+2 ≡ 1 (mod5) loại

+) p2 = 7 ⇒ 47 + 2 ≡ 0 (mod p3), không có giá trị t/m.

+) p2 = 11 ⇒ 411 + 2 ≡ (modp3) ⇒ p3 = 43.

Thử lại t/m

+) p2 = 13, 17, 19 không t/m

Vậy bài toán có nghiệm: n = 2 . 3 . 11 = 66

n = 2 . 11 . 43 = 946

Bài 2: Cho n nguyên dương có dạng n = 2k + 1 , k > 1

Cmr: ñiều kiện cần và ñủ ñể n nguyên tố là tồn tại số nguyên dương a > 1

sao cho na

n

M12

1

+−

Page 115: Chuyên đề hội thảo một số trường chuyên về dãy số

HỘ

HỘI CÁC TRƯỜNG THPT CHUYÊN KHU VỰC DUYÊN HẢI VÀ ðỒNG BẰNG BẮC BỘ HỘI THẢO KHOA HỌC LẦN THỨ IV

115

Lời giải:

Thuận: Nếu n là số nguyên tố ⇒ với số nguyên dương a bất kỳ (a,n)=1 ta có

an-1 ≡ 1 (modn)

⇒ Chọn a = g, g là căn nguyên thủy modn

⇒ gn-1 ≡ 1 (modn) ⇒ 12

1

−≡−n

g (modn)

ðảo: Giả sử ∃a na

n

M12

1

+−

, cần c/m n là số nguyên tố

⇒ 112 −≡

−k

a (modp) (p là ước nguyên tố cuả n)

⇒ 12 ≡k

a (modp)

ðặt h = ordn (a)

Ν∈=⇒−

⇒k

m

h

mhph

2|

)(21|

Nếu m < k ⇒ ah 2 1m

a= ≡ (modn)

⇒ 12 ≡k

a (modn) (vô lý)

⇒ m = k ⇒ p = nk =+12

Vậy n là số nguyên tố

Chứng minh tính chất 4:

g là căn nguyên thủy của p2 thì g là căn nguyên thuỷ của pk, n ≥ 3 ( p là SNT lẻ)

Lời giải:

Có g là căn nguyên thủy của p2 ⇒ g là căn nguyên thủy của p. Theo tính chất

căn nguyên thủy, ta có

vp (gp-1 -1) = 1

ta có 1)()1()1( )1()1( +=+−=− −−kpvgvgv

k

p

p

p

pp

p

k

(k≥1)

⇒pk+1 || 1)( 1

−+k

pgφ

ðây chính là ñiều kiện cần và ñủ ñể g là căn nguyên thủy của pk+1.

Thật vậy:

Với k = 3, ñặt h = 3or ( )p

d g ⇒ gh ≡1 (modp3)

⇒h|p2 (p-1)

ðặt mh .α , (p,m) = 1, m | p-1

Nếu 11 ≡⇒≤ hgα (modp3) ⇒gh ≡ 1 (modp2)

⇒ p[(p-1)| h ⇒ = p (p-1) (do g là căn nguyên thuỷ)

⇒ gp(p-1) ≡ 1 (modp3)

Hay vp (gp(p-1) - 1) ≥ 3 mà vp(g

p(p-1) - 1) = vp(g(p-1) - 1) + 1 = 2

⇒ mâu thuẫn

Page 116: Chuyên đề hội thảo một số trường chuyên về dãy số

HỘ

HỘI CÁC TRƯỜNG THPT CHUYÊN KHU VỰC DUYÊN HẢI VÀ ðỒNG BẰNG BẮC BỘ HỘI THẢO KHOA HỌC LẦN THỨ IV

116

Vậy α = 2

Có gh ≡1 (modp2) ⇒ h M p(p-1) ⇒ p2.m M p(p-1)

⇒ m M (p-1) ⇒ m = p-1 (do m | (p-1))

Vậy g là căn nguyên thủy p3

Giả sử ñúng với k ≥ 3 nào ñó, tức là g là căn nguyên thuỷ của pk

Với k + 1, ñặt h = 1or kpd + (g), ta có:

gh ≡ 1 (modpk+1) ⇒ h|pk (p-1)

ðặt h = pα.m , (p, m) = 1 , m | p - 1 , (α≤ k)

Có gh ≡ 1 (mod pk) ⇒ h M pk-1(p-1) ⇒α≥ k-1

+) Nếu α = k - 1, ta có: h = pk-1(p-1).

Và 1 .( 1) 1 1( 1) ( 1) ( )

kp p p k

p p pv g v g v p k− − − −− = − + =

mà 1 .( 1) 1 1

kp p hg g

− − − ≡ − (modpk+1).

⇒ 1)1( )1.(1

+≥−−−

kgvpp

p

k

(vô lý)

Vậy kk h p mα = ⇒ =

có k

p mg ≡ 1 (modpk)

⇒ pk.m M pk-1(p-1) ⇒ m M (p-1) ⇒m = p -1

Vậy h = pk.(p-1) hay g là CNT của pk+1

Vậy tính chất ñược c/m.

III.Thặng dư bậc hai

ðịnh nghĩa 1: Số nguyên dương a thỏa mãn (a,m) = 1 ñược gọi là một thặng dư

bậc hai (modm) nếu phương trình

x2 ≡ a (mod m) có nghiệm

+) Nếu pt x2 ≡ a(mod m) không có nghiệm nguyên thì a không là thặng dư bậc

hai (mod m)

Ví dụ: 1,4 là thặng dư bậc hai (mod 5); 2,3 không là thặng dư bậc hai (mod5)

ðịnh nghĩa 2: Cho a là một số nguyên , p là một số nguyên tố lẻ

Ta ñịnh nghĩa ký hiệu lengendre như sau:

−=

0

1

1

p

a Nếu a là một thặng dư bậc hai (mod p)

Nếu a không là một thặng dư bậc hai (mod p)

Nếu a M p

Các ñịnh lý:

ðịnh lý 1: Cho p là số nguyên tố lẻ và (a,p) = (b,q) = 1

Page 117: Chuyên đề hội thảo một số trường chuyên về dãy số

HỘ

HỘI CÁC TRƯỜNG THPT CHUYÊN KHU VỰC DUYÊN HẢI VÀ ðỒNG BẰNG BẮC BỘ HỘI THẢO KHOA HỌC LẦN THỨ IV

117

1) 12

=

p

a

2) Nếu a ≡ b (mod p) thì

=

p

b

p

a

3) 2

1−

=

p

ap

a (mod p)

4) ab a b

p p p

=

C/m 1.3

Nếu 1=

p

a thì pt x2 ≡ a (mod p) có nghiệm. Gọi xo là nghiệm của PT

có 110

2

1

≡≡ −−

p

p

xa (mod p) ⇒

=

p

aa

p

2

1

Nếu 1−=

p

a thì pt x2 ≡ a (mod p) vô nghiệm.

Ν∈∀x , (x, p) = 1 ta có: xp-1 ≡ 1 (mod p) ⇒

−≡

≡−

)(mod1

)(mod1

2

1

2

1

pa

pa

p

p

+) Nếu )(mod12

1

pa

p

=−

. Gọi g là căn nguyên thủy của (modp).

⇒ ∃ t : 0 ≤ t ≤ p - 1 thỏa mãn gt ≡ a (mod p)

(do {gi} là hệ thặng dư thu gọn)

⇒ 1 1

2 2 1p p

t

a g

− −

= ≡ (mod p)

⇒ t. 2)1(2

1MM tp

p⇒−

− , ðặt t = 2.k

⇒ ( ) agk ≡

2 (mod p) hay pt x2 ≡ a (mod p) có nghiệm, x0 = gk.

(mâu thuẫn)

Vậy

≡−≡

p

aa

p

12

1

(mod p)

Vậy 2

1−

=

p

ap

a (mod p)

C/m ñịnh lý 1.4

Ta có ( )

===

−−−

p

ababba

p

b

p

a ppp

2

12

1

2

1

.

Page 118: Chuyên đề hội thảo một số trường chuyên về dãy số

HỘ

HỘI CÁC TRƯỜNG THPT CHUYÊN KHU VỰC DUYÊN HẢI VÀ ðỒNG BẰNG BẮC BỘ HỘI THẢO KHOA HỌC LẦN THỨ IV

118

ðịnh lý 2: (Bổ ñề Gauss)

Cho a là một số nguyên, p là một số nguyên tố lẻ

Xét các số nguyên a, 2a, 3a,…, 2

)1( ap −

Là các lớp thặng dư dương nhỏ nhất của chúng. Gọi n là các số lớp thặng dư

vượt quá 2

p

Khi ñó n

p

a)1(−=

(n xác ñịnh bởi CT: )][2/)1(

1∑−

=

=p

j p

jan

ðịnh lý 3:

Cho p là một số nguyên tố lẻ. Khi ñó 2 1

82( 1)

p

p

− = −

Tức là

−=

1

12

p

Nếu p ≡ 1 hoặc 7 (mod 8)

Nếu p ≡ 3 hoặc 5 (mod 8)

Chú ý: (a,p) = 1, luôn có 12

=

=

p

a

p

a

p

a

Thường dùng ñể nhân thêm nhằm triệt tiêu

p

a ở một vế

ðịnh lý 4 (Luật thuận nghịch bình phương Gauss)

Cho p, q là hai số nguyên tố lẻ khác nhau

Khi ñó 1 1.

2 2( 1)p q

p q

q p

− − = −

Bài tập:

Bài 1: Cho a, b là các số nguyên khác không, (a, b) = 1

Cmr: ước nguyên tố lẻ của a2+b2 phải có dạng 4k+1 Lời giải: Gọi p là 1 ước nguyên tố lẻ của a2+b2 ⇒ a2+b2

M p ⇒ a2≡ - b2 (modp) (dễ dàng ⇒ (a,p) = (b,p) = 1 do (a,b) = 1)

⇒ 12

=

p

b

Mặt khác, theo ñịnh lý 1,3

Page 119: Chuyên đề hội thảo một số trường chuyên về dãy số

HỘ

HỘI CÁC TRƯỜNG THPT CHUYÊN KHU VỰC DUYÊN HẢI VÀ ðỒNG BẰNG BẮC BỘ HỘI THẢO KHOA HỌC LẦN THỨ IV

119

1 1 122 12 2 2( ) ( 1) . ( 1)

p p p

pbb b

p

− − −− −

≡ − ≡ − ≡ −

(mod p)

Nếu p = 4k + 3 ⇒ 12

−≡

p

b (mod p)

Mà 12

p

b (mod p) ⇒ p = 2 (vô lý)

Vậy p có dạng 4k + 1

Bài 2: Cmr: nếu phương trình x2 ≡ -3 (modp) có nghiệm thì p ≡1(mod3) với

(p là số nguyên tố) và p > 3.

Lời giải:

G/s pt x2 ≡ - 3 (modp) có nghiệm

⇒ 13

=

p ta lại có

−=

ppp

313

Mặt khác 2

1

2

13.

2

1

)1()1(3

3 −−−

−=−=

ppp

p (luật tương hỗ Gauss)

−=

−=

−=

−−

31

3)1(

33

)1(33

3 2

1

2

1p

p

p

p

ppp

p

pp

=

−=

−=

33

11313 pp

ppppp

⇒ 13

=

p mặt khác

−=

1

1

3

p Nếu p = 3k + 1

Nếu p = 3k + 2 ⇒ p ≡ 1 (mod 3) (dpcm)

Hệ quả: Các ước nguyên tố lớn hơn 3 của x2 + 3 chỉ có dạng 3k + 1

Bài 3: Cmr: nếu p= 2n + 1, n ≥≥≥≥ 2 là số nguyên tố thì 3 (p-1)/2 + 1 M p

Lời giải: Nếu n = 2k + 1 ⇒ 2n+1 ≡ 22k+1 + 1 ≡ 0 (mod3)

⇒ p = 3 (loại do p≥ 5) ⇒ n = 2k (k ≥ 1)

Theo ñịnh lý 1.3, ta có: 2

1

33 −

=

p

p (mod p)

Có p = 4k + 1 ≡ 2 (mod3) ⇒ 13

p = −

p

3 1 4

2 2( 1) ( 1) 13

kpp

= − = − =

(luật tương hỗ Gauss)

Page 120: Chuyên đề hội thảo một số trường chuyên về dãy số

HỘ

HỘI CÁC TRƯỜNG THPT CHUYÊN KHU VỰC DUYÊN HẢI VÀ ðỒNG BẰNG BẮC BỘ HỘI THẢO KHOA HỌC LẦN THỨ IV

120

p

3 = - 1 ⇒ 13 2

1

−≡−p

(mod p)

⇒ 013 2

1

≡+−p

(modp) (ñpcm)

Bài 4: Cho 3 số a, b, c nguyên dương, (a, b) = 1 và thỏa mãn

a2 - ab + b2 = c2

Cmr: Mọi ước nguyên tố của c ñều có dạng 6k + 1

Lời giải: Dễ dàng c/m ñược a, b, c nguyên tố cùng nhau ñôi một

Gọi p là 1 ước nguyên tố bất kỳ của c

Do (a,b) = 1 ⇒ c lẻ ⇒ p ≥ 3

Ta có a2 - ab + b2 ≡0 (mod p) ⇒ (a-b)2 ≡ - ab (mod p) , có (-ab, p) = 1

⇒ 1=

p

ab , mặt khác,

=

pp

ab

p

ab 1

−=

pp

ab 1

Lại có a2 - ab + b2 ≡ (a+b)2 - 3ab ⇒ (a+b)2 ≡ 3ab (mod p)

+) nếu p = 3 ⇒ (a+b)2 M3 ⇒a + b M3

Có (a+b)2 - 3ab = c2 M 9

⇒ 3ab M 9 ⇒ ab M 3

3

3

M

M

b

a (vô lý do a, b, c ñôi 1 nguyên tố cùng nhau)

⇒ (p, 3) = 1

⇒ 13

=

p

ab , mặt khác

=

p

ab

pp

ab 33

=

ppp

ab 133

Có theo luật tương hỗ Gauss : 2

1

)1(3

3 −

−=

pp

p

−=

3

13 p

pp

=

−=

3

1

3

13 p

p

p

pp

ab

Vậy 13

=

p ⇒ 12

13

≡−

p (mod 3)⇒ p ≡ 1 (mod 3)

Page 121: Chuyên đề hội thảo một số trường chuyên về dãy số

HỘ

HỘI CÁC TRƯỜNG THPT CHUYÊN KHU VỰC DUYÊN HẢI VÀ ðỒNG BẰNG BẮC BỘ HỘI THẢO KHOA HỌC LẦN THỨ IV

121

⇒ p có dạng 6k + 1 hoặc 6k + 4 (loại)

Vậy p ≡ 1 (mod 6) (ñpcm)

Bài 5: Cho m, n là số nguyên dương thỏa mãn

A = m

mn

3

1)3( ++ là số nguyên

Cmr: A lẻ

Lời giải: Nếu m là lẻ thì (m + 3)n + 1 là lẻ ⇒ A lẻ

Giả sử m chẵn. Do A là số nguyên ⇒ 0 ≡ (m + 3)n + 1 ≡ mn + 1 (mod 3)

Do ñó n = 2k + 1 và m ≡ - 1 (mod3 ) (k ≥ 1)

Ta xét các trường hợp sau:

a) m = 8m’, m’ là 1 số nguyên dương. Ta có

(m + 3)n + 1 ≡ 32k+1 + 1 ≡ 4 (mod 8)

Và 3m ≡ 0 (mod 8) ⇒ A không phải là số nguyên

b) m = 2m’, m’ là số nguyên dương lẻ ⇒ (m + 3)n + 1 ≡ (m+3)+1 ≡ (2+3)+1 ≡ 2 (mod 4)

Và 3m ≡ 2 (mod 4) ⇒ A lẻ

c) m = 4m’, m’ là số nguyên dương lẻ

Vì m ≡ - 1 (mod 3) nên tồn tại 1 ước nguyên tố lẻ p của m sao cho

p ≡ -1 (mod 3). (các số dạng 3k + 2 luôn có ít nhất 1 ước nguyên tố dạng

3k+2 ,c/m bằng phản chứng).

Do A nguyên ⇒ 0 ≡ (m+3)n +1 ≡ 32k+1 + 1 (mod m) ⇒ 32k+1 ≡ - 1 (mod p)

Gọi a là căn nguyên thủy của (mod p) ⇒ ∃ b nguyên dương thỏa mãn 3 ≡ ab (mod p) ⇒ a(2k+1)b ≡ -1 (mod p)

Ta xét các trường hợp sau:

(i) p ≡ 1 (mod4) ⇒ 1)1(1 2

1

=−=

−−p

p

⇒ ∃ xo:

và ∃ c: )(mod

)(mod1

0

20

pax

px

c≡

−≡

)(mod

)(mod1)12(

2

pa

pa

bk

c

+≡

−≡⇒

⇒ (p-1) | (2k + 1) b - 2c mà (p-1) chẵn ⇒ b chẵn, lại có ab ≡ 3 (modp)

Page 122: Chuyên đề hội thảo một số trường chuyên về dãy số

HỘ

HỘI CÁC TRƯỜNG THPT CHUYÊN KHU VỰC DUYÊN HẢI VÀ ðỒNG BẰNG BẮC BỘ HỘI THẢO KHOA HỌC LẦN THỨ IV

122

⇒ 13

=

p

Áp dụng luật tương hỗ Gauss ta có:

1)1(3

3 2

13.

2

1

=−=

−−pp

p Mặt khác: p ≡ - 1 (mod3) ⇒ )3(mod1

3−≡≡

p

p

13

31

3−=

⇒−=

p

p

p Mẫu thuẫn

(ii) p ≡ 3 (mod4) ⇒ 11

−=

p

⇒ c∃ : a2c+1 ≡ - 1 ≡ a(2k+1)b (modp) (c *N∈ ) ⇒ (p-1) | (2k+1)b - (2c+1) ⇒ b lẻ

Có ab ≡ 3 (modp) ⇒ 13

−=

p (do {ai} là HTDTG modp)

Ta có:

1)1(3

31

.2

13.

2

1

−=−=

=

−−p

p

p (mâu thuẫn)

Vậy m = 4m’, m’ lẻ thì A không phải là số nguyên

Kết luận: Nếu A là số nguyên thì A lẻ.

Bài 6: Cmr 2n + 1 không có ước nguyên tố dạng 8k +7, n *N∈

Lời giải:

Giả sử tồn tại số nguyên tố p thỏa mãn

p|2n + 1 và p ≡ -1 (mod 8)

Nếu n chẵn, n = 2k theo ñó: ( ) 122

−≡k (modp)

⇒ 11

=

p , mặt khác

1

21

( 1) 1p

p

− −= − = −

(mẫu thuẫn)

Nếu n lẻ, n = 2k + 1 ⇒ 22k+1 1−≡ (modp)

⇒ ( )212 2k + ≡ − (modp)

−==

ppp

211

2

Mặt khác, theo ñịnh lý 3 ta có: 2 1

82( 1)

p

p

− = −

Page 123: Chuyên đề hội thảo một số trường chuyên về dãy số

HỘ

HỘI CÁC TRƯỜNG THPT CHUYÊN KHU VỰC DUYÊN HẢI VÀ ðỒNG BẰNG BẮC BỘ HỘI THẢO KHOA HỌC LẦN THỨ IV

123

⇒ 1)1()1.()1(2 2

1

8

1

2

1 2

−=−=−−=

−−−− ppp

p (mẫu thuẫn)

Vậy ta có ñpcm

Bài 7: Tìm số nguyên dương n thỏa mãn 2n - 1 | 3n - 1

Lời giải:

+) n = 1 ⇒21 - 1 | 31 - 1

+) Ta sẽ c/m không ∃n ≥ 2 t/m 2n - 1 | 3n - 1

Giả sử ∃n ≥ 2 t/m 2n - 1 | 3n - 1

Nếu n chẵn ⇒ 2n - 1 ≡ 0 (mod3) ⇒ 3n - 1 ≡ 0 (mod3) (vô lý) ⇒ n lẻ hay n = 2k + 1 (k ≥ 1)

Ta ñi c/m 2n-1 có dạng 12m + 7 (m ≥ 0)

⇔ 2n-1 ≡ 7 (mod 12) ⇔ 2n- 23 ≡ 0 (mod 12)

Có 2n- 23 ≡ 23 [22(k-1) - 1] = 8. [ 4k-1 - 1]

Có 4k-1 - 1 M 3 1≥∀k ⇒ 2n- 23 M 12 (ñpcm)

Gọi p là ước nguyên tố bất kỳ của 2n- 1, (p lẻ, p ≠ 3)

Ta có: 3n - 1 ≡ 0 (modp) ⇒ 32k +1 ≡ 1 (modp)

⇒ 32(k+1) ≡ 3 (modp) ⇒ 13

=

p

Lại có theo luật tương hỗ Gauss

2

1

)1(3

3 −

−=

pp

p

⇒ 2

1

)1(3

−=

p

p mà

−≡≡

1

1

3p

p Nếu p ≡ -1 (mod 6)

Nếu p ≡ 1 (mod 6)

+) p ≡ 1 (mod 6), ñặt p = 6h+ 1

⇒ 1)1( 2

1

≡−−p

(mod6) ⇒ (-1)3h ≡ 1 ⇒ h chẵn hay p = 12h' + 1

+) p ≡ - 1 (mod 6), ñặt p = 6h+ 5

⇒ 1)1( 2

1

−≡−−p

(mod6) ⇒ h lẻ hay p = 12h' + 11

Vậy ước nguyên tố của 2n-1 có dạng 12h + 1 hoặc 12h + 11

Hay pi = ± 1 (mod 12)

PT: 2n-1 = i

ipα∏ ≡ ± 1 (mod 12)

mà 2n - 1 có dạng 12k + 7 ⇒ mâu thuẫn

Page 124: Chuyên đề hội thảo một số trường chuyên về dãy số

HỘ

HỘI CÁC TRƯỜNG THPT CHUYÊN KHU VỰC DUYÊN HẢI VÀ ðỒNG BẰNG BẮC BỘ HỘI THẢO KHOA HỌC LẦN THỨ IV

124

Vậy không ∃ n ≥ 2 t/m 2n - 1 | 3n - 1

Bài 8: Cmr 2≥∀n , ước nguyên tố của 22 1n

nf = + (số Fecma)

có dạng p = s. 2n+2 + 1, s nguyên dương

Lời giải:

Gọi số nguyên tố p là ước của fn

⇒ 122 −≡n

(modp) ⇒ 1212 ≡

+n

(modp)

ðặt h = ordp (2) ⇒ 2h ≡ 1(modp) ⇒ knhh 22| 1 =⇒+ (k )N∈

Nếu k ≤ n ⇒ 22 1k

≡ (modp) kéo theo 122 ≡n

(modp)

Mâu thuẫn do 122 −≡n

(modp)

Vậy k = n + 1. Theo ñịnh lý Fecma ta có: 2p-1 ≡ 1 (modp) ⇒ 2n+1 | (p-1) ⇒ p = 12 1n

m+ + , với m nguyên dương

Do n ≥ 2⇒ p có dạng 8t+1

⇒ 2 1

280 0

2( 1) 1 : 2

p

x xp

− = − = ⇒ ∃ ∈ Ν ≡

(modp)

⇒ 12 10

2

1

≡= −−

p

p

x (modp) ⇒ 1 12 |

2n p+ −

⇒ 12.2

1 +=− n

sp hay p = 2.2 1n

s+ + (ñpcm)

Bài 9: Tìm ước nguyên tố nhỏ nhất của 112152 +

Lời giải: Gọi p là số nguyên tố phải tìm

Ta có 112152 −≡ (modp) hay 112

162 ≡ (modp)

ðặt h = ordp (12) ⇒ h| 216. ⇒ h = 2k ( k N∈ )

Nếu k < 16 ⇒ 212 1k

≡ (modp) kéo theo 112152 ≡ (modp) (vô lý)

Vậy h = 216. Theo ñl Fecma nhỏ, 12p-1 ≡ 1 (modp) ⇒ 12)1(|2 1616 +≥⇒− pp

Dễ nhận thấy 162 + 1 là số nguyên tố (dễ dàng kiểm chứng bằng cách phân tích)

Ta c/m: q = 162 + 1 là số cần tìm, hay c/m q | 112152 +

Ta có: 112152 + = 13.213.2 2

1122 1516

+=+−

−q

q

⇒ 112152 + ≡ 13. 2

1

+−q

(modp)

Mặt khác, áp dụng luật tương hỗ Gauss ta có:

1)1(3

3 2

1

=−=

−pp

p

Page 125: Chuyên đề hội thảo một số trường chuyên về dãy số

HỘ

HỘI CÁC TRƯỜNG THPT CHUYÊN KHU VỰC DUYÊN HẢI VÀ ðỒNG BẰNG BẮC BỘ HỘI THẢO KHOA HỌC LẦN THỨ IV

125

16(mod 3) 2 1 1(mod3)3

pp

≡ ≡ + ≡ −

⇒ 31 1

3

p

p

= − ⇒ = −

Mặt khác 2

1

33 −

p

p (modp)

⇒ 013 2

1

≡+−p

(modp)

Vậy p = 216 + 1 là số cần tìm

Bài tập thêm về áp dụng căn nguyên thủy

Bài 1: Cho p là 1 số nguyên tố lẻ

Cmr: 1i + 2i +…+ (p-1)i ≡ 0 (modp)

∀∀∀∀ i = 0, ( 2)p −

Lời giải: Gọi g là căn nguyên thủy (modp)

Theo tính chất 2, {gi} là HTDTG modp , i = 0, ( 2)p −

⇒ 1i + 2i +…+ (p-1)i ≡ ( ) ( ) ( )ipii

ggg 210 ... −+++ (modp)

( )1

11

−=

i

pi

g

g (modp)

Có ( ) 11

≡−pig (modp) (ñl Fecma nhỏ)

gi - 1 ≡ 0 (modp) (do g là căn nguyên thủy)

(Vì nếu gi ≡ 1 (modp) ⇒ i M (p-1) mà i <p-1 )

Vậy ta có ñpcm.

Bài 2: Cho p là số nguyên tố lẻ

Cmr: nếu pt x4 ≡ -1 (modp) có nghiệm ⇔ p ≡ 1 (mod 8).

Lời giải:

Thuận: pt x4 ≡ -1 (modp) có nghiệm ⇒∃x0∈ N: 140 −≡x (modp)

⇒ 80 1x ≡ (modp)

ðặt h = ordp (x0) ⇒ h| 8 = 23 . ðặt h = 2k (k∈ N)

Nếu k ≤ 2 ⇒ 120 ≡

k

x (modp) kéo theo 140 ≡x (modp) (vô lý)

Vậy h = 23. Theo ñịnh lý Fecma có: 110 ≡−p

x (modp)

⇒ h | (p-1) ⇒ p ≡ 1 (mod 8)

ðảo: p có dạng 8 k + 1

Page 126: Chuyên đề hội thảo một số trường chuyên về dãy số

HỘ

HỘI CÁC TRƯỜNG THPT CHUYÊN KHU VỰC DUYÊN HẢI VÀ ðỒNG BẰNG BẮC BỘ HỘI THẢO KHOA HỌC LẦN THỨ IV

126

Gọi g là căn nguyên thủy của (mod p) ⇒ gp-1 ≡ 1 (modp) ⇒ g8k ≡ 1 (modp) ⇒ (g4k - 1) (g4k + 1) ≡ 0 (modp) ⇒ g4k ≡ - 1 (modp)

(nếu g4k ≡ 1 (modp) thì mâu thuẫn với g là căn nguyên thủy ⇒ 4k M 8k (vô lý) ) ⇒ pt x4 ≡ - 1 (modp) có nghiệm gk (ñpcm).

Tài liệu tham khảo

1. Tạp trí toán học tuổi trẻ

2. Trang wed mathscope. Org

3. Các bài toán luỹ thừa của Phạm Văn Quốc THPT Chuyên KHTN Hà Nội

4. Các bài về cấp số của Lê Xuân ðại Chuyên Vĩnh Phúc

5. Các bài toán thặng dư bậc hai của Trần Xuân ðáng Chuyên Nam ðịnh

6. Cuốn 104:Number Theory của Ti tu

7. Number Theory: Structures, Examples, and Problems By Titu Andreescu

Page 127: Chuyên đề hội thảo một số trường chuyên về dãy số

HỘ

HỘI CÁC TRƯỜNG THPT CHUYÊN KHU VỰC DUYÊN HẢI VÀ ðỒNG BẰNG BẮC BỘ HỘI THẢO KHOA HỌC LẦN THỨ IV

127

SỬ DỤNG HỆ THẶNG DƯ ðẦY ðỦ HỆ THẶNG DƯ THU GỌN ðỂ GIẢI TOÁN

Nguyễn Duy Liên

Trường THPT Chuyên Vĩnh Phúc

LỜI NÓI ðẦU

Ngạn ngữ Pháp có câu: "Le Mathématique est le Roi des Sciences mais L’Arithmétique est la Reine",dịch nghĩa:"Toán học là vua của các khoa học nhưng Số học là Nữ hoàng". ðiều này nói lên tầm quan trọng của Số học trong ñời sống và khoa học. Số học giúp con người ta có cái nhìn tổng quát, sâu rộng hơn, suy luận chặt chẽ và tư duy sáng tạo.

Trong các kì thi chọn học sinh giỏi các cấp THCS, THPT cấp tỉnh, cấp Quốc gia,cấp khu vực, cấp quốc tế, các bài toán về Số học thường ñóng vai trò quan trọng. Chúng ta có thể làm quen nhiều dạng bài toán Số học, biết nhiều phương pháp giải, nhưng cũng có bài chỉ có một cách giải duy nhất. Mỗi khi gặp một bài toán mới chúng ta lại phải suy nghĩ tìm cách giải mới. Sự phong phú ña dạng của các bài toán Số học luôn là sự hấp dẫn ñối với mỗi giáoviên, học sinh giỏi yêu toán. Xuất phát từ những ý nghĩ ñó tôi ñã sưu tầm và hệ thống lại một số bài toán ñể viết lên chuyên ñề "Sử dụng hệ thặng dư ñầy ñủ, hệ thặng dư thu gọn ñể giải toán". Chuyên ñề gồm các phần : - Phần I: Kiến thức cơ bản. - Phần II:Ứng dụng hệ thặng dư ñể giải toán

• Ứng dụng 1: Sử dụng hệ thặng dư ñể tính tổng • Ứng dụng 2: Sử dụng hệ thặng dư trong các bài toán ña thức, dãy số

nguyên. • Ứng dụng 3: Sử dụng hệ thặng dư trong tập con tập số nguyên dương, bài

toán số học chia hết • Ứng dụng 4: Sử dụng hệ thặng dư trong phương trình ði Ô Phăng bậc

nhất. - Phần III: Bài tập tương tự.

Mục tiêu ở ñây là một số bài mẫu, một số bài khác biệt căn bản ñã nói lên ñược phần chính yếu của chuyên ñề. Tuy vậy, những thiếu sót nhầm lẫn cũng không thể tránh khỏi ñược tất cả , về phương diện chuyên môn cũng như phương diện sư phạm. Lối trình bày bài giải của tôi không phải là một lối duy nhất. Tôi ñã cố gắng áp dụng cách giải cho phù hợp với chuyên ñề, học sinh có thể theo mà không lạc hướng. Ngoài ra lúc viết tôi luôn luôn chú ý ñến các bạn vì nhiều lí do phải tự học, vì vậy giản dị và ñầyñủ là phương châm của tôi khi viết chuyên ñề này. PHẦN I. KIẾN THỨC CƠ BẢN

Page 128: Chuyên đề hội thảo một số trường chuyên về dãy số

HỘ

HỘI CÁC TRƯỜNG THPT CHUYÊN KHU VỰC DUYÊN HẢI VÀ ðỒNG BẰNG BẮC BỘ HỘI THẢO KHOA HỌC LẦN THỨ IV

128

1.Hệ thặng dư ñầy ñủ

Cho tập { }1 2 nA a ,a ,...,a .= Giả sử i ir ,0 r n 1≤ ≤ − là số dư khi chia ia cho ir .

Nếu tập số dư { }1 2 nr , r ,..., r trùng với tập { }0,1,...,n 1− thì ta nói A là một hệ thặng

dư ñầy ñủ (gọi tắt là Hðð) modun n.

Dễ thấy: Tập A lập thành một Hðð(modun n) nếu và chỉ nếu: ( )i ji j a a mod n≠ ⇒ ≠ .

Nếu { }1 2 nA a ,a ,...,a= là Hðð mod n thì từ ñịnh nghĩa dễ suy ra

• Với mọi m ∈Z tồn tại duy nhất ia A∈ sao cho ( )ia m mod n≡ .

• Với mọi a ∈Z tập { }1 2 na A a a ,a a ,...,a a+ = + + + là Hðð mod n

• Với mọi c∈Z ,( )c,n 1= , tập { }1 2 ncA ca ,ca ,...,ca= là Hðð mod n.

Tập { }*A 0,1,2,...,n 1= − là Hðð mod n không âm nhỏ nhất.

Số phần tử của tập A bằng A n=

2. Hệ thặng dư thu gọn.

Cho tập { }1 2 kB b ,b ,...,b= là một tập hợp k số nguyên và ( )ib ,n 1= với mọi

i 1,2,...,k= .Giả sử i i i ib q n r ,1 r n= + ≤ < .Khi ñó dễ thấy ( )ir ,n 1= . Nếu tập

{ }1 2 kr , r ,..., r bằng tập K gồm tất cả các số nguyên dương bé hơn n và nguyên tố

cùng nhau với n thì B ñược gọi là hệ thặng dư thu gọn mod n , gọi tắt là HTG

mod n.

Dễ thấy tập { }1 2 kB b ,b ,...,b= gồm k số nguyên lập thành một HTG mod n

khi và chỉ khi: 1) ( )ib ,n 1= ;

2) ( )i jb b mod n≠ với 1 i j k≤ ≠ ≤ ;

3) Số phần tử của B là ( )nϕ ( trong ñó ( )nϕ là hàm Ơle của n.

ðiều kiện 3) tương ñương với '3 )Với mọi x ∈� , ( )x,n 1= tồn tại duy nhất ib B∈ sao cho x ≡ ( )ib mod n .

Từ ñịnh nghĩa ta suy ra nếu tập { }1 2 kB b ,b ,...,b= là HTG mod n

với c ,∈� ( )c,n 1= thì tập { }1 2 kB cb ,cb ,...,cb= cũng là HTG mod n.

Ngày xưa người ta ñã sử dụng Hðð,HTG ñể chứng minh các ñịnh lí Ơle,Féc ma ….trong chuyên ñề ta không nói lại nữa. PHẦN II. ỨNG DỤNG HỆ THẶNG DƯ ðỂ GIẢI TOÁN

• Ứng dụng 1: Sử dụng hệ thặng dư ñể tính tổng

Ví dụ 1.1.

Page 129: Chuyên đề hội thảo một số trường chuyên về dãy số

HỘ

HỘI CÁC TRƯỜNG THPT CHUYÊN KHU VỰC DUYÊN HẢI VÀ ðỒNG BẰNG BẮC BỘ HỘI THẢO KHOA HỌC LẦN THỨ IV

129

Với mỗi cặp số nguyên dương nguyên tố cùng nhau ( )p,q ñặt

( )p 1 qq 2qS

p p p

− = + + +

L ,trong ñó [ ]x là số nguyên lớn nhất không vượt

quá x. Hãy xác ñịnh các giá trị của p, q ñể S là một số nguyên tố. Lời giải : Với mỗi a ∈� ñặt { } [ ]a a a= − .Khi ñó với k ∈� ta có

kkq r

p p

=

ở ñây kr là số dư trong phép chia kq cho p do vậy : k0 r p 1≤ ≤ − ,

( ) p 11 2rp 1 qq 2q r r

Sp p p p p p

−− = + + + − + + +

L L

vì ( )p,q 1= kr 0 k 1,2,....,p 1⇒ ≠ ∀ = − từ ñó ta thấy tập { }1 2 p 1A r ,r ,..., r−

= chính là

một hoán vị của tập { }A 1,2,....,p 1= − thậy vậy ngược lại :

{ }i, j 1,2,...,p 1 ,i j∃ ∈ − < mà i jr r=( )

1 j i p 2 1 j i p 2

j i q p j i p

≤ − ≤ − ≤ − ≤ − ⇒ ⇔

− − M Mvô lí

Từ ñó p 11 2rr r 1 2 p 1 p 1

p p p p 2− + + + − −

+ + + = =L

L( )( )p 1 q 1

S2

− −⇒ = ( )1

Từ ( )1 ñể S là số nguyên tố cần có p 1,q 1≠ ≠ và ít nhất 1 trong 2 số p,q lẻ

Trường hợp 1: p,q cùng là số lẻ p,q 3,p q⇒ ≥ ≠ do ( )p,q 1= ,kết hợp (1)

S⇒ là số chẵn lớn hơn 2 S⇒ không là số nguyên tố.

Trường hợp 2: p là số chẵn q là số lẻ.

( )

( )

( )

( )( )

p,q 1

p 1 1p 2

q 1q 2h 1 h2

Sq 3

p,q 1 p t 1 t , t 2 mod3p 1

q 11

2

=

− == − ∈℘ = + ∈℘∈℘⇔ ⇔ = = = + ∈℘ ≡/

− ∈℘ − =

(2)

ở ñây kí hiệu ℘ là tập số nguyên tố

Page 130: Chuyên đề hội thảo một số trường chuyên về dãy số

HỘ

HỘI CÁC TRƯỜNG THPT CHUYÊN KHU VỰC DUYÊN HẢI VÀ ðỒNG BẰNG BẮC BỘ HỘI THẢO KHOA HỌC LẦN THỨ IV

130

Trường hợp 3: q là số chẵn p là số lẻ do tính ñối của p,q của biểu thức xác ñịnh

S theo trường hợp 2 :

( )

( )( )

p 2m 1 m

q 2

p 3

q n 1 n ,n 2 mod3

= + ∈℘

= = = + ∈℘ ≡/

(3)

Vậy tóm lại tất cả các giá trị p,q cần tìm là các cặp xác ñịnh ở (2) và (3).

Ví dụ 1.2. Tính tổng :k2006

k 4

17S

11=

=

∑ .

Lời giải:

Nhận xét 1: nếu ( )a r mod b≡ ,với a,b, r ,0 r b 1∈ ≤ ≤ −Z thì a a r

b b

− =

Nhận xét 2: Vì ( )1017 1 mod11≡ nên tập { }10t 10 t 1 10t 9B 17 ,17 ,....,17+ += là HTG mod

11 nó là một hoán vị của tập { }1,2,....,10 .

Nhận xét 3: mỗi [ ]i 0;9∈ ∩Z gọi in là số phần tử của tập hợp

( ){ }iD k 4 k 2006,k i mod10= ∈ ≤ ≤ ≡Z kiểm tra ta dễ thấy :

4 5 6n n n 201,= = = và 0 1 2 3 7 8 9n n n n n n n 200= = = = = = = . Từ các nhận xét suy ra:

( )

2006k

k2006 0 1 2 3 4 5 6 7 8 9k 4

k 1

17 n 6n 3n 7n 9n 10n 5n 8n 4n 2n17

11 11=

=

− + + + + + + + + +

=

∑∑

( )2003 10

4

j 1

17 117 . 9 10 15 200 j

17 111

=

−− + + −

−=∑ 200717 259905

176

−= .

Vậy 200717 259905

S176

−=

Ví dụ 1.3. Cho m,n là 2 số nguyên dương nguyên tố cùng nhau với m chẵn và n

lẻ .Chứng minh rằng :Tổng ( )mkn 1n

k 1

mkS 1 .

n

=

= −

1

2n+ không phụ thuộc vào

m,n. Lời giải:

Ta chứng minh rằng ( )mkn 1

* n

k 1

mk 1 1S 1 .

n 2 2n

=

= − = −

Ta có: k

mkmk n r

n

= + trong ñó { }k0 r n 1 k 1,2,...,n 1≤ ≤ − ∀ ∈ − .

Ta thấy rằng :

Page 131: Chuyên đề hội thảo một số trường chuyên về dãy số

HỘ

HỘI CÁC TRƯỜNG THPT CHUYÊN KHU VỰC DUYÊN HẢI VÀ ðỒNG BẰNG BẮC BỘ HỘI THẢO KHOA HỌC LẦN THỨ IV

131

a) Do ( ) { }km,n 1 r 0 k 1,2,...,n 1= ⇒ ≠ ∀ ∈ − từ ñó tập

{ }1 2 n 1B r ,r ,..., r−

= là một hoán vị của tập: { }1,2,...,n 1− .

b) kmk r

n n

=

với k 1,2,...,n 1= − .

c) Với m chẵn n lẻ ( )k

mkr mod 2

n

≡ .

Từ ñó ( ) ( ) knk

rm1 1 k 1,2,3,...,n 1

− = − ∀ = −

( ) ( ) ( )kn 1

r*

kk 1

1S 1 r 2 4 n 1 1,3, ,n 2

n

=

= − = + + + − − − ∑ L L1 1

2 2n= − (ðpcm)

Ví dụ 1.4. Cho p là số nguyên tố ( ) ( )p 3 mod8 p 5 mod8≡ ∨ ≡ ,và p=2q+1 ( )q ∈℘

Tính tổng p 12 4 2S

= ω + ω + + ωL (với 1ω ≠ là nghiệm của pt: p 1ω = ) Lời giải:

( )p 3 mod8≡ ± ⇒2 không là số chính phương mod p 2

1p

⇔ = −

( ) ( )( )

( )p 1

*q 2q22 1 mod p 2 1 mod p 2 1 mod p−

⇔ ≡ − ⇔ ≡ − ⇒ ≡ .Gọi h là cấp của 2

theo mod p ( )h2 1 mod p⇒ ≡ .Vậy ta có h p 1 2q− =

do q ∈℘ h 1 h 2 h q h 2q⇒ = ∨ = ∨ = ∨ = .

• h 1= ( ) ( )2 1 mod p 1 0 mod p⇒ ≡ ⇔ ≡ (loại)

• h 2= ( )22 1 mod p p 3 q 1⇒ ≡ ⇒ = ⇒ = không là số nguyên tố (loại)

• h q= ( )( )**q2 1 mod p⇒ ≡ từ (*),(**) ( )2 0 mod p⇒ ≡ p 2⇒ = (loại)

• h 2q p 1= = − ⇒2 là căn nguyên thuỷ mod p ⇒ { }1 2 p 1A 2 ,2 ,...,2 −= là

HTG mod p nó là một hoán vị của tập { }1,2,3,...,p 1− từ ñó ta có tổng

( )p 1

p 1

2 4 2 2 p 11

S1

−ω ω −

= ω + ω + + ω = ω + ω + + ω =ω −

L Lp 1

11 1

ω − ω − ω= = = −

ω − ω −

Ví dụ 1.5 . Cho p là số nguyên tố lẻ với mỗi i 1,2,...,p 1= − .Kí

hiệu: ( )p

ii r mod p≡ ( ir phần dư của pi khi chia cho p).Tính tổng :

1 2 p 1S r r r−

= + + +L .

Lời giải:

( ) ( ) ( )1 p 1 2 p 2 p 1 12S r r r r r r− − −

= + + + + + +L ta có

( )pp

i p ir r i p i i 1,2,...,p 1−

+ = + − ∀ = − mà

( )pp p 1 p 1 2 p 2 2 p 11 p 1

p p pi p i p C p i C p i C pi− − − −+ − = − + + +L

Page 132: Chuyên đề hội thảo một số trường chuyên về dãy số

HỘ

HỘI CÁC TRƯỜNG THPT CHUYÊN KHU VỰC DUYÊN HẢI VÀ ðỒNG BẰNG BẮC BỘ HỘI THẢO KHOA HỌC LẦN THỨ IV

132

do ( )i

pp C 0 mod p i 1,2,...,p 1∈℘⇒ ≡ ∀ = − ( )2

i p ir r 0 mod p−

⇒ + ≡

mà 2 2 2

i p i i p i0 r p ,0 r p r r p i 1,2,...,p 1− −

< < < < ⇒ + = ∀ = − .Từ ñó ta thu ñược

( ) 2 3 2p 1 p p pS

2 2

− −= = .

• Ứng dụng 2: Sử dụng hệ thặng dư trong các bài toán ña thức ,dãy số

nguyên… Ví dụ 2.1 .Cho số nguyên dương n và số nguyên tố p lớn hơn n+1.Chứng minh

rằng ña thức ( )2 px x x

P x 1n 1 2n 1 pn 1

= + + + ++ + +

L không có nghiệm nguyên.

Lời giải: ( ) p p 1 2

p p 1 2 1 0P x 0 a x a x a x a x a 0−

−= ⇔ + + + + + =L (*) trong ñó

( )( ) ( )( )i

n 1 2n 1 ... pn 1a i 0,1,2,...,p

in 1

+ + += ∈ =

+Z .Từ giả thiết ( )p,n 1= ⇒

Tập { }A 1.n 1,2.n 1,...,p.n 1= + + + là Hðð mod p k⇒ ∃ duy nhất { }k 1,2,...,p∈

sao cho ( )kn 1 0 mod p+ ≡ hơn nữa 2k 1,0 kn 1 p kn 1≠ < + < ⇒ + M 2p .

Vì vậy với số k ñó có ka M p ñồng thời p hệ số còn lại trong ñó có 0 1a ,a

của pt (*) ñều chia hết cho p nhưng không chia hết cho 2p (**).

Giả sử pt (*) có nghiệm nguyên x c= . Khi ñó: p p 1 2

p p 1 2 1 0a c a c a c a c a 0−

−⇒ + + + + + =L ,

theo (**) thì { }ia p i 0,1,2,...,0 ,i k,k 0;1∀ ∈ ≠ ≠M .Từ ñó suy ra k

ka c pM

( do p )∈℘ { }( )k i 2 2

i 1c p c p a c p i 2,3,..p & a c p⇒ ⇒ ⇒ ∀ ∈M M M M vì 1a p,c pM M .Từ ñó

suy ra 2

0a pM mâu thuẫn với (**) vậy ñiều giả sử là sai ,tức là pt (*) không có

nghiệm nguyên ( )P x⇔ không có nghiêm nguyên (ñpcm).

Ví dụ 2.2 .Cho ña thức ( ) 3 2P x x 11x 87x m= − − + trong ñó m ∈Z .Chứng minh

rằng với mọi m tồn tại số nguyên n sao cho ( )P n 191M .

Lời giải: Bổ ñề:Cho ( ) ( ) ( )3 3p ,p 2 mod3 , x, y ,x y mod p x y mod p∈℘ ≡ ∀ ∈ ≡ ⇒ ≡Z

Thật vậy : • Nếu ( ) ( ) ( ) ( )3x 0 mod p y 0 mod p y 0 mod p x y mod p≡ ⇒ ≡ ⇒ ≡ ⇔ ≡

• Nếu x p y p/ /⇒M M ,do ( ) ( )p 2 mod3 p 3k 2 k≡ ⇒ = + ∈� ,theo ñịnh lí

Fécma: ( ) ( )p 1 3k 1 p 1 3k 1x x 1 mod p , y y 1 mod p− + − += ≡ = ≡

Page 133: Chuyên đề hội thảo một số trường chuyên về dãy số

HỘ

HỘI CÁC TRƯỜNG THPT CHUYÊN KHU VỰC DUYÊN HẢI VÀ ðỒNG BẰNG BẮC BỘ HỘI THẢO KHOA HỌC LẦN THỨ IV

133

( )3k 1 3k 1x y mod p+ +⇒ ≡ theo giả thiết ( ) ( )3 3 3k 3kx y mod p x y mod p≡ ⇒ ≡

Vậy ( ) ( ) ( )( )3k 1 3k 3ky x.x x.y mod p x y mod p do y,p 1+ ≡ ≡ ⇔ ≡ =

Trở lại bài toán : ( ) 3 2P n n 11n 87n m= − − +

Ta chứng minh ( ) ( )( )1 2P n P n mod191≡ với 1 2n ,n ∈Z thì

( )1 2n n mod191≡ .Thật vậy do

( ) ( )( ) ( ) ( )( )1 2 1 2P n P n mod191 27P n 27P n mod191≡ ⇔ ≡ ⇔

( ) ( ) ( )3 33 3

1 1 2 23n 11 18.191n 11 27m 3n 11 18.191n 11 27m mod191− − + + ≡ − − + +

( ) ( ) ( )3 3

1 23n 11 3n 11 mod191⇔ − ≡ − theo bổ ñề ta có :

( ) ( )1 2 1 23n 11 3n 11 mod191 n n mod191− ≡ − ⇔ ≡ (do (27,191)=(3,191)=1)

{ }1 2n ,n A 1,2,...,191∀ ∈ = A là Hðð mod191 thoả mãn 1 2n n≠ thì

( ) ( )( )1 2P n P n mod191≡/ . ( ) ( ) ( ){ }*A P 1 ,P 2 ,...,P 191⇔ = là Hðð mod191

Từ ñó suy ra { }n A 1,2,...,191∃ ∈ = sao cho ( ) ( )P n 191 mod191≡

( )P n 191⇔ M .Vậy với mọi m tồn tại số nguyên n sao cho ( )P n 191M .

Ví dụ 2.3. Cho dãy số { }n n 1

a+∞

= ñược xác ñịnh bởi 1 n n 1 n

2

a 1,a a a−

= = + với n 2,3,....∀ =

Chứng minh rằng dãy { }n n 1a

+∞

= chứa vô số số nguyên chia hết cho 7.

Lời giải: Phản chứng. Giả sử chỉ có hữu hạn số trong dãy chia hết cho 7 và

ka là số cuối cùng của dãy chia hết cho 7, khi ñó từ công thức xác ñịnh của dãy ta có:

2k 2k 1 k 2k 1 2k ka a a ;a a a− +

= + = + nên

( )2k 1 2k 2k 1a a a b mod7− +

≡ ≡ ≡ , ( )b 0 mod7≡/ ( )b∈Z

Mặt khác : 4k 3 4k 3a a 0.b− −

= +

( )4k 2 4k 3 2k 1 4k 3a a a a 1.b mod7− − − −

= + ≡ +

( )4k 1 4k 2 2k 1 4k 3a a a a 2.b mod7− − − −

= + ≡ +

( )4k 4k 1 2k 4k 3a a a a 3.b mod7− −

= + ≡ +

( )4k 1 4k 2k 4k 3a a a a 4.b mod7+ −

= + ≡ +

( )4k 2 4k 1 2k 1 4k 3a a a a 5.b mod7+ + + −

= + ≡ +

( )4k 3 4k 2 2k 1 4k 3a a a a 6.b mod7+ + + −

= + ≡ +

do ( )b,7 1= ⇒ tập { }6

4k 3 i 0A a ib

− == + là Hðð mod7

nên trong bảy số : 4k 3 4k 2 4k 1 4k 4k 1 4k 2 4k 3a ;a ;a ;a ;a ;a ;a− − − + + +

tồn tai một số chia hết

cho 7 mà số này lại lớn hơn ka mâu thuẫn với ka là số cuối cùng của dãy chia hết cho 7.ðiều giả sử sai nên ta có ñpcm. Ví dụ 2.4.

Page 134: Chuyên đề hội thảo một số trường chuyên về dãy số

HỘ

HỘI CÁC TRƯỜNG THPT CHUYÊN KHU VỰC DUYÊN HẢI VÀ ðỒNG BẰNG BẮC BỘ HỘI THẢO KHOA HỌC LẦN THỨ IV

134

Cho dãy số { }n n 1u

+∞

= ñược xác ñịnh bởi

3 2

1 n n 1u 2,u 3u 2n 9n 9n 3−

= = + − + − với n 2,3,...= .Chứng minh rằng với mỗi số

nguyên tố p thì p 1

ii 1

u−

=

∑ chia hết cho p.

Lời giải:

Từ giả thiết ta có ( )33

n n 1u n 3 u n 1−

+ = + −

Từ ñó ⇒ ( )33

n n 1u n 3 u n 1−

+ = + − ( ) ( )32 n 1 3 n

n 2 13 u n 2 3 u 1 3−

− = + − = = + = L

n 3

nu 3 n n 1,2,....⇔ = − ∀ =

• 1p 2 u 2 2= ⇒ = M

• p ,p 3∈℘ ≥ thì ( )p 1

32 p 1 3 3

ii 1

u 3 3 3 1 2 p 1−

=

= + + + − + + + − ∑ L L

Ta thấy ( ){ }33 3 3A 1 ,2 ,3 ,..., p 1= − là HTG mod p.Với mọi i 1,2,...,p 1= −

ta có : ( ) ( )33i p i 0 mod p+ − ≡ ( )( ) ( )

p 1 p 133 3

i 1 i 1

1i i p i 0 mod p

2

− −

= =

⇒ = + − ≡∑ ∑

trong khi ñó ( )p 1

2 p 1 p3 1 13 3 3 3. . 3 3 p

3 1 2

− −+ + + = = −

−L M .

Vậy với mỗi số nguyên tố lẻ p thì p 1

ii 1

u p−

=

∑ M

Có thể nói ( )

( )p 1 p 1

3

i 1 i 1

p p 1i i 0 mod p

2

− −

= =

−≡ ≡ ≡∑ ∑ do ( ){ }33 3 3A 1 ,2 ,3 ,..., p 1= − là HTG

mod p. Ví dụ 2.5.

Cho p 3≥ là một số nguyên tố và 1 2 p 2a ,a ,...,a−

là một dãy các số nguyên

dương sao cho p không là ước số của ka và k

ka 1− với k 1,2,...,p 2∀ = − .Chứng minh rằng tồn tại một số phần tử trong dãy 1 2 p 2a ,a ,...,a

− có tích ñồng dư với 2

modulo p. Lời giải:

Ta chứng minh bằng quy nạp mệnh ñề sau: Với mỗi số nguyên k 1,2,...,p 1= − tồn tại một tập các số nguyên

{ }k ,1 k ,2 k,kb ,b ,...,b thoả mãn

1. Mỗi k ,ib hoặc bằng 1 hoặc bằng tích của một số phần tử trong dãy

1 2 p 2a ,a ,...,a−

.

2. ( )k ,i k , jb b mod p≡/ với 1 i j k≤ ≠ ≤

Thật vậy: Với k 2= chọn ( )21 22 1b 1,b a 1 mod p= = ≡/ (do 1

1a 1 p/− M )

Page 135: Chuyên đề hội thảo một số trường chuyên về dãy số

HỘ

HỘI CÁC TRƯỜNG THPT CHUYÊN KHU VỰC DUYÊN HẢI VÀ ðỒNG BẰNG BẮC BỘ HỘI THẢO KHOA HỌC LẦN THỨ IV

135

Giả sử với 2 k p 2≤ ≤ − ta ñã chọn ñược tập { }k ,1 k ,2 k,kb ,b ,...,b thoả mãn 2 t/c

trên . Ta có:

• Vì ka p/M nên hai phần tử khác nhau bất kì trong tập

{ }k k,1 k k,2 k k,ka b ,a b ,...,a b là phân biệt theo mod p

• Vì ( ) ( )( ) ( )k

k k k ,1 k k,2 k k,k k ,1 k,2 k ,ka 1 mod p a b a b ... a b b b ..b≡ ⇒ ≡/ / ( )mod p

Từ hai ñiều trên suy ra tồn tại chỉ số j,1 j k≤ ≤ sao cho { }k k, j k ,1 k ,2 k,ka b b ,b ,...,b∈/ .

Xét tập { }k,1 k ,2 k ,k k k , jb ,b ,...,b ,a b ,sau khi ñánh số lại các phần tử ta ñược tập:

{ }k 1,1 k 1,2 k 1,k k 1,k 1b ,b ,...,b ,b+ + + + +

ta thấy tập hợp này có k 1+ phần tử thoả mãn hai

tính chất trên,theo nguyên lí quy nạp mệnh ñề ñược chứng minh.

Áp dụng xét tập { }p 1,1 p 1,2 p 1,p 1b ,b ,...,b− − − −

ta thấy tập hợp này là một HTG mod p

nên nó chứa ñúng một phần tử ñồng dư với 2 mod p.Vì phần tử này khác 1 nên nó phải ñồng dư với tích của một số ka ,suy ra ñpcm . Ứng dụng 3: Sử dụng hệ thặng dư trong tập con tập số nguyên dương, bài toán số học chia hết

Ví dụ 3.1.Cho n,k là các số nguyên dương thoả mãn các ñiều kiện sau: 1. n 3/M . 2. k n≥ .

Chứng minh rằng tồn tại số nguyên dương T sao cho ( )T n & S T k=M (trong ñó

( )S x là tổng các chữ số của số nguyên dương x trong biểu diễn thập phân )

Lời giải: ðặt 1n 2 .5 .nα β= với ( )*

1 1n , n ,10 1& ,∈ = α β∈� � .

Do ( )1 1n 3 n 3 n ,9 1/ /⇒ ⇔ =M M .Tập ( ){ }1A k,k 9,k 9.2,...,k 9. n 1= + + + − là Hðð

mod 1n

{ }1t 0,1,2,...,n 1⇒ ∃ ∈ − sao cho ( )1k 9t 0 mod n+ ≡ .

Vì ( ) *

1n ,10 1 m= ⇒ ∃ ∈� sao cho ( )m m

1 1 1

m

111...11 n 10 1 n 10 1 mod n⇔ − ⇔ ≡M M14243 .

Xét số ( ) ( ) ( ) ( )

( )

k 1 m 1 k 2 m 1 k t m 1 k t 1 m m

1

tso k t so

T 10 10 10 10 10 1− + − + − + − −

= + + + + + + +L L144444424444443 144424443

Ta có ( )im *

110 1 mod n i≡ ∀ ∈� ( )1 1T 10t k t 9t k mod n⇒ ≡ + − ≡ + và

( )1S T t k t k= + − = từ ñó số 1T 10 .Tα+β= thoả mãn yêu cầu bài toán.

Ví dụ 3.2 Cho n là số nguyên dương lẻ lớn hơn 1.Các tập hợp số nguyên dương

{ }*A k / kn 1 cp= ∈ + ∈� và { }*B j / jn cp= ∈ ∈� ( với kí hiệu cp∈ là số chính

phương).

Page 136: Chuyên đề hội thảo một số trường chuyên về dãy số

HỘ

HỘI CÁC TRƯỜNG THPT CHUYÊN KHU VỰC DUYÊN HẢI VÀ ðỒNG BẰNG BẮC BỘ HỘI THẢO KHOA HỌC LẦN THỨ IV

136

Chứng minh rằng: k n 2 k A

nj n j B

≥ − ∀ ∈∈℘⇔

≥ ∀ ∈

Lời giải:

⇒k n 2 k A

nj n j B

≥ − ∀ ∈∈℘⇒

≥ ∀ ∈

• ( ) ( )( )2 *x 1 n

kn 1 x x x 1 x 1 knx 1 n

++ = ∈ ⇔ + − = ⇔ −

M�

M ( do n ∈℘)

x n 1⇒ ≥ − vậy ( )( ) ( )kn x 1 x 1 n n 2 k n 2= + − ≥ − ⇔ ≥ − .

• 2 2jn y y n y n= ⇒ ⇒M M ( do n ∈℘) y n⇒ ≥ vậy 2 2jn y n j n= ≥ ⇔ ≥

⇐k n 2 k A

nj n j B

≥ − ∀ ∈⇒ ∈℘

≥ ∀ ∈

Phản chứng .Giả sử n là hợp số n 4⇒ ≥ Trường hợp 1: n pα= với *p , , 1∈℘ α ∈ α >�

• Nếu α chẵn chọn j 1 n= < mà jn p cpα= ∈ (vô lí)

• Nếu α lẻ ( )*2t 1 tα = + ∈� chọn j p n= < mà ( )2t 1jn p cp+= ∈ (vô lí)

Trường hợp 2: n pq= với { } ( )*p,q \ 1 , p,q 1∈ =� .

Ta chứng minh:∃1 k n 2≤ < − ( )k ∈�

mà kn 1 cp+ ∈

Xét { }p 1

x 0D xq 2

== + là Hðð mod p { }x 0,1,2,...,p 1⇒ ∃ ∈ − sao cho

( )xq 2 0 mod p xq 2 mp+ ≡ ⇒ + = với *m∈� .

Chọn ( ) ( )k mx m p 1 xq p 1 q n 2= ≤ − < ≤ − < −

Ta có: ( ) ( )2

kn 1 mx.n 1 mpxq 1 xq xq 2 1 xq 1 cp+ = + = + = + + = + ∈ ( vô lí)

Từ hai trường hợp ta thấy ñiều giả sử là sai .Vậy n ∈℘ . Ví dụ 3.3 Cho ( ) ( ) ( )p 2 p 3 2p ,p 3,f x p 1 x p 2 x 3x 2x 1− −∈℘ ≥ = − + − + + + +L .

Tập { }1 2 pA a ,a ,...,a= là Hðð mod p .

Chứng minh rằng tập ( ) ( ) ( ){ }1 2 pB f a ,f a ,...,f a= cũng là Hðð mod p .

Lời giải: ( ) ( ) ( )p 2 p 3 2f x p 1 x p 2 x 3x 2x 1− −= − + − + + + +L

Nếu ( ) ( ) ( )( )p p 1

x 1 f 1 p 1 p 2 2 1 p2

−= ⇒ = − + − + + + =L M

Page 137: Chuyên đề hội thảo một số trường chuyên về dãy số

HỘ

HỘI CÁC TRƯỜNG THPT CHUYÊN KHU VỰC DUYÊN HẢI VÀ ðỒNG BẰNG BẮC BỘ HỘI THẢO KHOA HỌC LẦN THỨ IV

137

Nếu: x 1≠

( ) ( )( ) ( )( ) ( ) ( )p 1 p 2 p 2 p 3 3 2 21f x p 1 x x p 2 x x 3 x x 2 x x x 1

x 1− − − − = − − + − − + + − + − + − −

L

( ) ( ) ( ) ( )p 1 p 2 p 3 2x 1 f x p 1 x x x x x 1− − −− = − − + + + + +L

( ) ( ) ( )( ) ( ) ( )2 p 1 p 1x 1 f x p 1 x 1 x x 1 1 x mod p− −− = − − − − ≡ −

Giả sử B không là Hðð mod p ( )i ja ,a A 1 i j p⇒ ∃ ∈ ≤ < ≤ sao cho

( ) ( ) ( )i jf a f a m mod p≡ ≡ khi ñó ta có

:( ) ( )

( ) ( )

2

i i

2

j j

m a 1 m 1 a mod p

m a 1 m 1 a mod p

− ≡ −

− ≡ −

[ ]( )m 0;p 1∈ −

• Nếu m 0= từ hệ ( )i ja a mod p⇒ ≡ (vô lí)

• Nếu m 0≠ do ( ) i jf 1 p a 1,a 1⇒ ≠ ≠M từ hệ suy ra:

( ) ( ) ( ) ( )( )22

i j j im a 1 1 a m a 1 1 a mod p− − ≡ − −

( ) ( )i j i j1 a 1 a mod p a a mod p⇔ − ≡ − ⇔ ≡ (vô lí).

Vậy ñiều giả sử là sai . Vậy tập B cũng là Hðð mod p . Ví dụ 3.4 . Cho số nguyên tố lẻ p mk 2= + trong ñó *m,k ,m 2.∈ >� Tìm số dư

của phép chia của ( )p

m 1 m 2

t 1

S t t t 1− −

=

= + + + +∏ L cho p .

Lời giải: Bổ ñề : nếu { }1 2 pA a ,a ,...,a= là Hðð mod p .

Thì { }m m m

1 2 pB a ,a ,...,a= cũng là Hðð mod p .Thật vậy

Giả sử: m m

i ja ,a B∈ sao cho ( )m m

i ja a mod p≡ ( )( )km km

i ja a mod p *⇒ ≡

• Nếu ( ) ( )i ja 0 mod p a 0 mod p≡ ⇒ ≡ ( )i ja a mod p i j⇔ ≡ ⇒ =

• Nếu ( ) ( )i ja ,p 1 a ,p 1= ⇒ = theo ñịnh lí Fécma

( ) ( )p 1 km 1 p 1 km 1

i i j ja a 1 mod p ,a a 1 mod p− + − += ≡ = ≡ ( )( )km 1 km 1

i ja a mod p **+ +⇔ ≡

Từ ( ) ( )* , ** ( )i j i ja a mod p a a⇒ ≡ ⇒ = nên B là Hðð mod p .

Áp dụng : ( )p

m 1 m 2

t 1

S t t t 1− −

=

= + + + +∏ L ( )p

m 1 m 2

t 2

m t t t 1− −

=

= + + + +∏ L

( ) ( )p p

m

t 2 t 2

S. t 1 t 1= =

− = −∏ ∏ ( )S. p 1⇒ − ! ( )p

m

t 2

m t 1=

= −∏ (1)

Do { }A 1,2,...,p= là Hðð mod p⇒ { }m m mB 1 ,2 ,...,p= là Hðð mod p

Page 138: Chuyên đề hội thảo một số trường chuyên về dãy số

HỘ

HỘI CÁC TRƯỜNG THPT CHUYÊN KHU VỰC DUYÊN HẢI VÀ ðỒNG BẰNG BẮC BỘ HỘI THẢO KHOA HỌC LẦN THỨ IV

138

{ }* m m mB 1 1,2 1,...,p 1⇔ = − − − là Hðð mod

p { }** m m mB 2 1,3 1,...,p 1⇔ = − − − là HTG mod p.Từ (1)

( )S. p 1⇒ − ! ( )p

m

t 2

m t 1=

= −∏ ( )m p 1≡ − ! ( ) ( )mod p S m mod p⇔ ≡

Vậy S chia cho p ñược số dư là m. Ví dụ 3.5 .Cho các số nguyên không âm : 1 2 101a a a 5050< < < <L .Chứng minh rằng tồn tại bốn số nguyên phân biệt k l m na ,a ,a ,a thoả mãn

( )k l m na a a a 5050+ − − M

Lời giải: Xét các tổng ( )i ja a 1 i j 101+ ≤ < ≤ có tất cả 2

101C 5050= tổng như vậy.

• Nếu tồn tại 2 tổng ( )k l m na a a a mod5050+ ≡ + với

{ } { }k l,m n, k,l m,n< < ≠ thì k,l,m,n phải là 4 số phân biệt .Khi ñó bộ

4 số k l m na ,a ,a ,a thoả mãn ñiều kiện

( )k l m na a a a 5050+ − − M .

• Nếu không tồn tại 4 số k,l,m,n với { } { }k l,m n, k,l m,n< < ≠ sao cho

( )k l m na a a a mod5050+ ≡ + { }i jS a a 1 i j 101⇒ = + ≤ < ≤ là Hðð mod

5050.

Từ ñó ( ) ( )i j1 i j 101

a a 0 1 2 5049 25255049 mod5050≤ < ≤

+ ≡ + + + + ≡∑ L ⇒

( )i j1 i j 101

a a≤ < ≤

+∑ là một số lẻ (1)

Mặt khác ( )i j1 i j 101

a a≤ < ≤

+∑ ( )1 2 100100 a a a= + + +L là một số chẵn (2) .

Ta thấy (1) và (2) mâu thuẫn .Vậy ñiều giả sử là sai và ta có ñpcm . Ví dụ 3.6 .Chứng minh rằng với mọi số nguyên dương n, tồn tại số tự nhiên gồm n chữ số ñều lẻ và nó chia hết cho n5 . Lời giải:

Xét số n

1 2 na a ...a 5 .a= thoả mãn (với ia +∈Z lẻ, i 1,2,...,n∀ = và a +∈Z ) Ta chứng minh bài toán bằng quy nạp toán học . Với 1

1n 1 a 5 5= ⇒ ∃ = M vậy mệnh ñề ñúng với n 1= .

Giả sử mệnh ñề ñúng với n ⇔ n

1 2 na a ...a 5 .a= ,cần chứng minh mệnh ñề ñúng với n 1+ . Xét 5 số sau ñây :

• ( )n n

1 2 n1a a ...a 5 1.2 a= +

• ( )n n

1 2 n3a a ...a 5 3.2 a= +

• ( )n n

1 2 n5a a ...a 5 5.2 a= +

Page 139: Chuyên đề hội thảo một số trường chuyên về dãy số

HỘ

HỘI CÁC TRƯỜNG THPT CHUYÊN KHU VỰC DUYÊN HẢI VÀ ðỒNG BẰNG BẮC BỘ HỘI THẢO KHOA HỌC LẦN THỨ IV

139

• ( )n n

1 2 n7a a ...a 5 7.2 a= +

• ( )n n

1 2 n9a a ...a 5 9.2 a= +

Do { }B 1,3,5,7,9= là Hðð mod 5

{ }* n n n n nB 1.2 a,3.2 a,5.2 a,7.2 a,9.2 a⇒ = + + + + + cũng là Hðð mod 5 nên tồn

tại một số duy nhất trong *B chia hết cho 5 ⇒ trong 5 số

1 2 n1a a ...a , 1 2 n3a a ...a , 1 2 n5a a ...a , 1 2 n7a a ...a , 1 2 n9a a ...a ,có một số duy nhất chia hết

cho n 15 + mà số này gồm n 1+ chữ số lẻ vậy mệnh ñề ñúng với n 1+ .Theo nguyên lí quy nạp mệnh ñề ñúng với mọi n.Vậy với mọi số nguyên dương n, tồn tại số tự nhiên gồm n chữ số ñều lẻ và nó chia hết cho n5 . Ứng dụng 4: Sử dụng hệ thặng dư trong phương trình ðiôPhăng bậc nhất.

Ví dụ 4.1:Cho ( )a,b , a,b 1.+∈ =Z Số nguyên dương n ñược gọi là ñẹp nếu tồn

tại x, y +∈Z , sao cho : n ax by= + .

1. Chứng minh rằng : n ab= là số xấu lớn nhất. 2. Chứng minh rằng nếu [ ]n I a b;ab∈ = + ,n là ñẹp ⇔ số ab a b n+ + − là

xấu 3. Tìm số lượng các số xấu .

Lời giải: 1/Ta chứng minh n ab= là số xấu lớn nhất.

• Giả sử n ab= ñẹp ⇔ phương trình ab ax by= + (*) có nghiệm nguyên dương

( )( ) ( )1

1 1

1

x bxax b x bdo a,b 1 x , y

by a y a y ay+

= ⇒ = ⇒ ⇒ ∈

=

M M

M MZ khi ñó phương trình (*)

( )1 1 1 1ab x y ab x y 1⇔ + = ⇔ + = Vô lí .Vậy ñiều giả sử sai tức là n ab= là số

xấu. • Ta chứng minh mọi n ab> thì thì phương trình n ax by= + có

nghiệm nguyên dương .

Do ( ) { }b

i 1a,b 1 A ai

== ⇒ = là Hðð mod b { }x 1,2,...,b⇒ ∃ ∈ sao cho

( ) ( )ax n mod b n ax by y≡ ⇔ − = ∈Z ax by n⇔ + =

do1 x b n ax n ab 0 by 0 y +≤ ≤ ⇒ − ≥ − > ⇒ > ⇒ ∈Z vậytồntại

x, y : ax by n+∈ + =Z n ab⇔ ∀ > ñều là số ñẹp. Từ hai ñiều trên ta thấy n ab= là số xấu lớn nhất.

2/ ⇒ n là ñẹp x, y : ax by n+∃ ∈ + =Z .

Khi ñó ( ) ( ) ( )ab a b n ab a b ax by ab x 1 a y 1 b+ + − = + + − + = − − − − (**)

Page 140: Chuyên đề hội thảo một số trường chuyên về dãy số

HỘ

HỘI CÁC TRƯỜNG THPT CHUYÊN KHU VỰC DUYÊN HẢI VÀ ðỒNG BẰNG BẮC BỘ HỘI THẢO KHOA HỌC LẦN THỨ IV

140

Giả sử ab a b n+ + − là số ñẹp 1 1 1 1x , y : ab a b n ax by+∃ ∈ + + − = +Z (***) từ

(**) và (***) ta có ( ) ( )1 1ab x x 1 a y y 1 b= + − + + − mà

1 1x x 1 , y y 1 n ab+ ++ − ∈ + − ∈ ⇒ =Z Z ñẹp vô lí.

⇐ ñặt k ab a b n= + + − .Do ( ) { }b

i 1a,b 1 A ai

== ⇒ = làHððmodb

{ }x 1,2,...,b⇒ ∃ ∈ saocho

( ) ( )ax k mod b k ax 0 mod b k ax by≡ ⇔ − ≡ ⇔ − = ax by k⇔ + = ( )y∈Z .Theo

giả thiết k là số xấu k ab⇒ ≤ y 0⇒ ≤ .Khi ñó ta có:

( ) ( ) ( )n ab a b k ab a b ax by a b 1 x b 1 y= + + − = + + − + = + − + − ñẹpdo

b 1 x ,++ − ∈Z 1 y +− ∈Z . 3/ Theo phần (1) mọi n ab> ñều là số ñẹp. • [ ]n 1;a b 1∀ ∈ + − ñều là số xấu ,trong ñoạn này có tất cả a b 1+ − số xấu

• [ ]n a b;ab∀ ∈ + thì [ ]ab a b n a b;ab+ + − ∈ + theo phần (2) ta thấy nếu

lấy một số ñẹp thuộc ñoạn [ ]a b;ab+ thì có một số xấu cũng thuộc ñoạn

[ ]a b;ab+ ,từ ñó số số xấu trong ñoạn [ ]a b;ab+ là: ab a b 1

2

− − +.

Vậy tổng cộng có tất cả : ab a b 1 ab a b 1

a b 12 2

− − + + + −+ − + = số số xấu.

Ví dụ 4.2: Cho ( ) ( ) ( )a,b,c , a,b b,c c,a 1.+∈ = = =Z Số nguyên dương n ñược

gọi là ñẹp nếu tồn tại x, y,z +∈Z , sao cho : n bcx cay abz= + + . 1.Chứng minh rằng : n 2abc= là số xấu lớn nhất.

2.Chứng minh rằng nếu [ ]n I ab bc ca;2abc∈ = + + ,n là xấu ⇔ số

k 2abc ab bc ca n= + + + − là ñẹp . 3.Tìm số lượng các số xấu .

Lời giải: 1/Ta chứng minh n 2abc= là số xấu lớn nhất.

• Giả sử n 2abc= ñẹp ⇔ phương trình 2abc bcx cay abz= + + (*) có nghiệm nguyên dương

( ) ( ) ( )( ) ( )1

1 1 1 1

1

x axbcx a x a

cay b do a,b b,c c,a 1 y b y by x , y ,z

abz c z c z cz

+

=

⇒ = = = ⇒ ⇒ = ∈

=

M M

M M

M M

Z

khi ñó phương trình (*) ( )1 1 1 1 1 1abc x y z 2abc x y z 2⇔ + + = ⇔ + + = Vô lí .Vậy ñiều giả sử là sai tức là

n 2abc= là số xấu. • Ta chứng minh mọi n 2abc> thì thì phương trình

n bcx cay abz= + + có nghiệm nguyên dương .

Page 141: Chuyên đề hội thảo một số trường chuyên về dãy số

HỘ

HỘI CÁC TRƯỜNG THPT CHUYÊN KHU VỰC DUYÊN HẢI VÀ ðỒNG BẰNG BẮC BỘ HỘI THẢO KHOA HỌC LẦN THỨ IV

141

Do ( ) ( ) ( ) { }c

i 1a,b b,c c,a 1 A n abi

== = = ⇒ = − là Hðð mod c

{ }z 1,2,...,c⇒ ∃ ∈ sao cho ( ) ( )n abz 0 modc n abz tc t− ≡ ⇔ − = ∈Z ,mà

n 2abc>n abc 2abc abc

t abc c

− −⇒ = > = tức là t , t ab+∈ >Z vậy tồn tại

x, y : bx ay t+∈ + =Z , từ ñó ta có :

( )n abz bx ay c n bcx cay abz− = + ⇔ = + + n 2abc⇔ ∀ > ñều là số ñẹp.

Từ hai ñiều trên ta thấy n 2abc= là số xấu lớn nhất 2/Tương tự phần 2 của ví dụ 4.1 3/ Tương tự phần 3 của ví dụ 4.1.Ta tính ñược số lượng số xấu là:

2abc bc ca ab 1

2

+ + + −.

Ví dụ 4.3: Cho a,b,c là các số nguyên dương thoả mãn: ( )a b c, a,b,c 1≤ ≤ = .Chứng minh rằng nếu: n ac b> + thì phương trình:

n ax by cz= + + có nghiệm nguyên dương. Lời giải:

Gọi ( ) ( ) { }d

i 1a,c d b,d 1 A bi

== ⇒ = ⇒ = là Hðð mod d

{ } ( )y 1,2,...,d : by n modd n by d⇒ ∃ ∈ ≡ ⇔ − M .

Do ( ) ( )( )1 1 1 1 1 1a,c d a a d,c c d a ,c , a ,c 1+= ⇒ = = ∈ =Z { } 1c

1 j 1B a j

=⇒ = là Hððmod

c 1 .

{ }1x 1,2,...,c⇒ ∃ ∈ : ( )1 1

n bya x modc

d

−≡ 1 1

n byz : a x c z

d

−⇒ ∃ ∈ = +Z . Mặt khác

ta có:

( ) 11 1 1 1 1

n by ac b by ca bd 1 a c a c a x

d d d

− + − −> = − + ≥ ≥ z +⇒ ∈Z .

Từ ñây n by ax cz n ax by cz⇒ − = + ⇔ = + + ta có ñpcm

Ví dụ 4.4 :Cho các số nguyên dương a,b thoả mãn ( )a,b 1= .Chứng minh rằng

phương trình: ax by 1+ = có vô số nghiệm nguyên ( )x, y và ( ) ( )x,a y,b 1= = .

Lời giải: Do ( )a,b 1= { }

b

i 1A ai

=⇒ = là Hðð mod b

{ } ( )x 1,2,..,b : ax 1 mod b y : ax by 1⇒ ∃ ∈ ≡ ⇔ ∈ + =Z

( )a,b 1= { }a

j 1B x jb

=⇒ = + là Hðð mod a

{ } ( ) ( )k 1,2,..,a : x kb 1 moda x kb,a 1⇒ ∃ ∈ + ≡ ⇒ + =

( )x kb mab,a 1 m +⇒ + + = ∀ ∈Z .Từ ñó ta có:

( ) ( ) ( )2 21 a x kb mab b y ka ma b t ma− + + = − − = − với

t y ka= − ∈Z ( )2

0 0m : t m a 1 mod b+⇒ ∃ ∈ − ≡Z

Page 142: Chuyên đề hội thảo một số trường chuyên về dãy số

HỘ

HỘI CÁC TRƯỜNG THPT CHUYÊN KHU VỰC DUYÊN HẢI VÀ ðỒNG BẰNG BẮC BỘ HỘI THẢO KHOA HỌC LẦN THỨ IV

142

( ) ( )2 2

0 0t m a ,b 1 t m a nab,b 1− = ⇔ − − = vớimọi n +∈Z .Cácbộ

( ) ( )2 2 2

0 0x,y x kb m ab nab ,y ka m a na b= + + + − − − 2∈Z thoả mãn ñề bài với

mọi n +∈Z .

PHẦN III. BÀI TẬP TƯƠNG TỰ

Bài 1: Cho ( )n ,p ,p 7 mod8∈ ∈℘ ≡� .Chứng minh rằng: n2p 1

k 1

k 1 p 1

p 2 2

=

−+ =

Bài 2: Cho p ,p 3∈℘ > .Tính tổng:

1.

p 12 22

k 1

2k kS 2

p p

=

= −

∑ nếu ( )p 1 mod 4≡

2.

p 122

k 1

kQ

p

=

=

∑ nếu ( )p 1 mod8≡

Bài 3: Cho p ,p 3∈℘ ≥ .Chứng minh rằng: ( )pp 1

k 1

k k p 1mod p

p 2

=

− +≡∑

Bài 4: Cho p ,p 3∈℘ ≥ .Tính tổng 3p 1

k 1

kT

p

=

=

Bài 5: Cho p ,p 3∈℘ ≥ , q +∈Z , q p/M

Chứng minh rằng: ( )( )( )2p 1

k 4

k 1

q p 1 p p 1q p 11 k

p 2 2

=

− − − −− = −

Bài 6: Cho ( )*p ,p 3k 2 k∈℘ = + ∈� .Hỏi ( )p

2

k 1

S k k 1=

= + +∑ chia cho p dư bao

nhiêu? Bài7: Cho ( )*p ,p 3k 2 k∈℘ = + ∈� , ( )4 2 2 4 *A x x y y x, y= + + ∈� .

Chứng minh rằng nếu A pM thì 4A pM

Bài8: Tìm tất cả *n ∈� sao cho tồn tại tập { }1 2 nA a ,a ,...,a= là một hoán vị của

tập { }B 1,2,...,n= thoả mãn { }1 1 2 1 2 nC a ,a a ,...,a a ...a= là Hðð mod n

Bài 9: Liệu có thể ñặt 12 số 1,2,…,12 theo ñường tròn sao cho với 3 số liền nhau a,b,c bất kì số 2b ac− chia hết cho 13.

Bài 10: Cho ( ) ( ){ }2p ,p 1 mod 4 ,L 1 r p r x mod p ,x +∈℘ ≡ = ≤ < ≡ ∈Z . Tính tổng

r L

S r∈

=∑

Bài 11: Cho ( )m,n , m,n 1+∈ ≠Z . { }m

i i 1A a

== là Hðð mod m và { }

n

j j 1B b

== là

Hðð mod n . Hỏi { }i j i 1,m, j 1,nC a b

= == có là Hðð mod mn hay không?

Page 143: Chuyên đề hội thảo một số trường chuyên về dãy số

HỘ

HỘI CÁC TRƯỜNG THPT CHUYÊN KHU VỰC DUYÊN HẢI VÀ ðỒNG BẰNG BẮC BỘ HỘI THẢO KHOA HỌC LẦN THỨ IV

143

Bài 12: Chứng minh rằng với mỗi số nguyên dương n ñều tồn tại số nguyên dương k, sao cho: n

m m 1 2 1k.5 a a ....a a−

= viết trong hệ thập phân thoả mãn i và ia

có cùng tính chẵn lẻ với i 1,n∀ = . Bài 13: Cho ( )p ,p 2,p 2 mod3∈℘ > ≡ .

Chứng minh rằng : { }2 3A y x 1 x, y ,x p, y p= − − ∈ < <� có nhiều nhất p 1− số

chi hết cho p. Bài 14: Cho số nguyên dương n thoả mãn :

( ) ( )n 1n 1 n 11 1 2 n 1 0 mod n

−− −+ + + + − ≡L .Chứng minh rằng n không có ước chính

phương khác 1 . Bài 15: Cho { }A 1,2,...,17= ,với mỗi hàm: f : A A→ .

kí hiệu ( ) ( ) ( ) ( )( )1 n 1 nf x f x ,f x f f x n+

= = ∀ ∈� ,Tìm số tự nhiên M lớn nhất sao

cho tồn tại song ánh f : A A→ thoả mãn các ñiều kiện sau: 1. Nếu m M< và 1 i 17≤ ≤ thì ( ) ( ) ( )m mf i 1 f i 1 mod17+ − ≡ ±/ .

2. Với 1 i 17≤ ≤ Thì ( ) ( ) ( )M Mf i 1 f i 1 mod17+ − ≡ ± .(quy ước ( ) ( )m mf 18 f 1= )

Bài 16: Tồn tại bao nhiêu số tự nhiên nhỏ hơn 10000, sao cho n 22 n 7− M Bài 17: Số nguyên dương n ñược gọi là ñẹp nếu phương trình : 2x 4y 5z n+ + = có nghiệm nguyên dương.Tìm số xấu lớn nhất. Bài 18: Số nguyên dương n ñược gọi là ñẹp nếu phương trình :6x 10y 15z n+ + = có nghiệm nguyên dương.Tìm số xấu lớn nhất.

Bài 19: Cho ( )1 2 k 1 2 ka ,a ,...,a : a ,a ,...,a 1+∈ =Z .Chứng minh rằng:Nếu

( ) 1 2 kn k 1 a a ...a> − thì phương trình k

i ii 1

n a x=

=∑ có nghiệm *

ix ∈� với

i 1,2,...,k= . Bài 20: Cho m +∈Z . Chứng minh rằng tồn tại các số nguyên a,b,k sao cho a,b cùng lẻ, k không âm và 19 5 20112m a b 2 k= + + . Bài 21:Tìm tất cả những số nguyên dương n ñể tồn tại các hệ thặng dư ñầy ñủ modn : { }1 2 nA a ,a ,...,a= và { }1 2 nB b ,b ,...,b= cũng là hệ thặng dư ñầy ñủ modn.

Bài 22: Cho dãy số nhận các giá trị trong các số 1,2,...,k 1− ñược ñịnh nghĩa như sau: ( )1 n n 1a 1,a a n mod k

−= ≡ + .Với giá trị nào của k thì dãy này nhận tất cả k

giá trị 1,2,...,k 1.− Bài 23: Chứng minh rằng trong 1900 số tự nhiên liên tiếp có một số có tổng các chữ số chia hết cho 27. Bài 24: Cho p là số nguyên tố lớn hơn 5.Chứng minh rằng tồn tại số nguyên dương a không ñổi : 2 a p 2≤ ≤ − ,sao cho tập hợp các số có dạng

:nc/s1

111...1114243 ( )a n 1,2,...+ = không có số nào chia hết cho p.

Page 144: Chuyên đề hội thảo một số trường chuyên về dãy số

HỘ

HỘI CÁC TRƯỜNG THPT CHUYÊN KHU VỰC DUYÊN HẢI VÀ ðỒNG BẰNG BẮC BỘ HỘI THẢO KHOA HỌC LẦN THỨ IV

144

THẶNG DƯ TOÀN PHƯƠNG Trường THPT chuyên Trần Phú - Hải Phòng

1. MỞ ðẦU Nghiên cứu về thặng dư toàn phương không chỉ là sự tiếp nối về tư duy

sau khi tìm hiểu về lý thuyết ñồng dư mà còn là chìa khoá giải quyết vấn ñề bảo

mật dữ liệu trong ngành công nghệ thông tin. Một ñặc ñiểm quan trọng và ñáng

mong muốn của bất kỳ lược ñồ mã hoá trong bảo mật dữ liệu là nó phải ñược

chứng minh việc phá khoá khó tương ñương với việc giải một bài toán nào ñó.

Chính vì vậy bằng lý thuyết thặng dư toàn phương người ta ñã mã hoá thông tin

thành những bài toán số học khó, ñáp ứng yêu cầu an toàn bảo mật khiến cho

người tấn công thụ ñộng gặp phải khi muốn giải mã dữ liệu.

Với những ý nghĩa trong thực tiễn và trong lý thuyết toán học ñó thặng

dư toàn phương ñang ngày càng ñược xuất hiện trong các ñề thi học sinh giỏi

của các quốc gia trên thế giới. Vì vậy bài viết này xin trình bày một cách hệ

thống các kiến thức về thặng dư toàn phương trong ñó ñưa ra cách chứng minh

“ñịnh lý vàng- luật tương hỗ” của Eisíentein. Với chứng minh này ñã ñem lại

nhiều kết quả ñẹp ñể ứng dụng cho ngành lý thuyết số. Bên cạnh ñó còn trình

bày hướng khai thác các bài toán số học bằng lý thuyết thặng dư toàn phương

qua các ñề thi học sinh giỏi toán quốc tế. 2. THẶNG DƯ TOÀN PHƯƠNG 2.1 ðịnh nghĩa Cho n

+∈� . Số a+∈� ñược gọi là thặng dư toàn phương (mod m) nếu tồn

tại số nguyên x sao cho 2 ( )x a mod m≡ .

Số a không là thặng dư toàn phương gọi là phi thặng dư toàn phương. 2.2 Thặng dư toàn phương mod p 2.2.1 Kiến thức tổng quát ðịnh lý 1 Cho p ∈P , p lẻ ( , ) 1a p = . Khi ñó a là thặng dư toàn phương mod p khi và chỉ

khi 1

2 1( )p

a mod p−

≡ .

Chứng minh Giả sử a là thặng dư toàn phương mod p. Khi ñó tồn tại số nguyên x mà ( , ) 1x p = sao cho 2 ( )x a mod p≡ . Theo ñịnh lý Fecma có: 1 1( )p

x mod p− ≡

nên 1 1( )pa mod p

− ≡ .

Page 145: Chuyên đề hội thảo một số trường chuyên về dãy số

HỘ

HỘI CÁC TRƯỜNG THPT CHUYÊN KHU VỰC DUYÊN HẢI VÀ ðỒNG BẰNG BẮC BỘ HỘI THẢO KHOA HỌC LẦN THỨ IV

145

Giả sử 1

2 1( )p

a mod p−

Phản chứng không tồn tại số nguyên x, ( , ) 1x p = sao cho 2 1( )x mod p≡ . Xét tập A=1,2,..., 1p − là hệ thặng dư thu gọn mod p. Với k bất kỳ thuộc A ta có { ,2 ,..., ( 1) }k k p k− là hệ thặng dư thu gọn mod p. Suy ra tồn tại duy nhất

* *: . (mod )k A k k a p∈ ≡ với (k *k=/ )thoả mãn *. (mod )k k a p≡ .

Nên ( 1)/2 ( 1)!( )pa p mod p

− ≡ −

Theo ñịnh lý Wilson ( 1)! 1( )p mod p− ≡ − . Vậy 1 1( )pa mod p

− ≡ − (vô lý) �

Ký hiệu Legendre Do (a,p)=1 suy ra ( 1) ( 1)/2 ( 1)/21( ) ( 1)( 1) 0( )p p p

a mod p a a mod p− − −≡ ⇔ − + ≡

( 1)/2 1( )pa mod p

−⇔ ≡ hoặc ( 1)/2 1( )pa mod p

− ≡ −

Nên ta có ký hiệu Legendre như sau: p∈P p lẻ, ( )a,p 1=

1a

p

=

nếu a là thặng dư toàn phương mod p.

1a

p

= −

nếu a là phi thặng dư toàn phương mod p.

Tính chất 2

a) ( 1)/2 ( )pa

a mod pp

− ≡

b) a b

p p

= ab

p

c) Nếu ( )a b mod p≡ thì a

p

=b

p

d) 2

1a

p

=

.

e) 2 ba b

p p

=

VD: Có tồn tại a∈� hay không ñể 2 2030a + chia hết cho 19. Trả lời: Theo tính chất có 2 2030 0( 19)a mod+ ≡ 2 2011( 19)a mod⇒ ≡

Do: 2011

19

= 16

19

= 2 4

19

=1

Nên tồn tại a thoả mãn.

Page 146: Chuyên đề hội thảo một số trường chuyên về dãy số

HỘ

HỘI CÁC TRƯỜNG THPT CHUYÊN KHU VỰC DUYÊN HẢI VÀ ðỒNG BẰNG BẮC BỘ HỘI THẢO KHOA HỌC LẦN THỨ IV

146

Câu hỏi ñặt ra là cho a là số nguyên vậy với những số nguyên tố p nào ñể tồn

tại x sao cho 2 ( )x a mod p≡ . Câu trả lời ñó chính là ñịnh lý luật tương hỗ thặng

dư toàn phương do Euler tìm ra và sau này ñuợc Gauss là người ñầu tiên chứng

minh hoàn chỉnh. Vài năm sau ñó ñã có hàng trăm cách chứng minh ñịnh lý luật

tương hỗ thặng dư toàn phương ñược công bố nhưng ở ñây chỉ xin trình bày một

cách chứng minh mà từ cách chứng minh này ñã ñem lại nhiều kết quả ñể ứng

dụng cho ngành lý thuyết số.

ðịnh lý 3( luật tương hỗ thặng dư toàn phương ) Cho p, q là các số nguyên tố lẻ. Ta có:

(a) 1

21

( 1)

p

p

−− = −

(b) 2 1

82

( 1)p

p

− = −

(c)q p

p q

( 1)( 1)

2 2( 1)p q− −

= −

Chứng minh Số phức ζ ñược gọi là nghiệm cấp n của 1 nếu 1nζ = . Nếu n là số nguyên nhỏ nhất thoả mãn tính chất này thí ζ ñuợc gọi là căn nguyên thuỷ cấp n của 1.

Nghiệm cấp n của 1 là 2 2

.( 1){1, ,..., }

i in

n ne e

π π−

. Trong số các nghiệm này căn

nguyên thuỷ cấp n của 1 là 2 i

kne

π

với ( , ) 1k n = .

Bổ ñề 1 Cho n > 0 là số lẻ thì ta có:

1

0

( )n

n n k k

k

x y x yζ ζ=

=

− = −∏

với 2 /i ne

πζ = .

Bổ ñề 2 Nếu n là số nguyên dương lẻ và 2 2( ) iz iz

f z e eπ π−= − thì

( 1)/2

1

( )( ) ( )

( )

n

k

f nz k kf z f z

f z n n

=

= + −∏

Bổ ñề 3 Nếu p là số nguyên lẻ , a ∈� và |p a/ thì

( 1)/2 ( 1)/2

1 1

p p

l l

la a lf f

p p p

− −

= −

=

∏ ∏

Page 147: Chuyên đề hội thảo một số trường chuyên về dãy số

HỘ

HỘI CÁC TRƯỜNG THPT CHUYÊN KHU VỰC DUYÊN HẢI VÀ ðỒNG BẰNG BẮC BỘ HỘI THẢO KHOA HỌC LẦN THỨ IV

147

Bây giờ ta chứng minh luật tương hỗ thặng dư toàn phương: Theo bổ ñề 3

( 1)/2 ( 1)/2

1 1

p p

l l

lq q lf f

p p p

− −

= −

=

∏ ∏

và bổ ñề 1

( 1)/2

1

( / / ) ( / / )q

m

qlf

pf l p m q f l p m q

lf

p

=

= + −

Thế hai phương trình vào cho nhau ta có:

( 1)/2 ( 1)/2

1 1

( / / ) ( / / )q p

m l

qf l p m q f m q l p

p

− −

= =

= + −

∏ ∏

Tương tự ta có:

( 1)/2 1

1 ( 1)/2

( / / ) ( / / )q l

m q

pf l p m q f m q l p

q

− =

= −

= + −

∏ ∏

Vì ( / / ) ( / / )f m q l p f l p m q− = − − nên ta có:

( 1)/2.( 1)/2( 1) p qq p

p q

− −

− =

* CM b)

Cho p là số nguyên tố lẻ và µ là số các phần tử của tập ( )p 1

2.1, 2.1, ..., 2.2

.

m là số thoả mãn hai ñiều kiện ( 1)

22

pm

−≤ và 2(m+1) > (p - 1)/2 thì

(( 1) / 2)p mµ = − − . Nếu p = 8k + 1 thì (p - 1)/2 = 4k, m = 2k. Vì vậy 4 2 2k k kµ = − = là chẵn và

2

p

=1

Nếu p = 8k + 7 thì (p - 1)/2 = 4k + 3 , m = 2k+1 và µ = 2k + 2 là chẵn và 2

p

=1

Nếu p = 8k + 3 thì (p - 1)/2 = 4k + 1, ( , ) 1u v

a a = = 2k+1 là lẻ nên 2

p

= -1

Page 148: Chuyên đề hội thảo một số trường chuyên về dãy số

HỘ

HỘI CÁC TRƯỜNG THPT CHUYÊN KHU VỰC DUYÊN HẢI VÀ ðỒNG BẰNG BẮC BỘ HỘI THẢO KHOA HỌC LẦN THỨ IV

148

Nếu p = 8k + 5 thì µ = 2k + 1 là lẻ nên 2

p

= -1

Vì vậy ta có ñiều phải chứng minh. � Nhận xét

2 là thặng dư toàn phương mod p khi và chỉ khi 1( 8)p mod≡ hoặc

7( 8)p mod≡ .

3 là thặng dư toàn phương mod p khi và chỉ khi 1( 12)p mod≡ hoặc

11( 12)p mod≡ .

Hệ quả 4:

Cho p là số nguyên tố lẻ và q là số nguyên tố mà q > p. Số r ñược xác ñịnh như sau: Nếu 1( 4)p mod≡ thì r q kp= − , 0 < r < p Nếu 3( 4)p mod≡ thì q = 4kp ± r với 0 < r < 4p và 1( 4)r mod≡ Khi ñó

p r

q p

=

Hệ quả 5 Cho *

a ∈� , p, q là các số nguyên tố lẻ phân biệt. Khi ñó các khẳng ñịnh sau ñây là tương ñương

a) ( 1)( 1)

2 2. ( 1)p qq p

p q

− − = −

b) nếu ( 4 )p q mod a≡ ± thì a a

p q

=

Từ hệ quả 2.2.1.5 ta nhận thấy nếu ( )r,4a 1= và a là thặng dư toàn phương thì

tất cả các số nguyên tố dạng ( r 4at+ )và ( 4 ).r at− + với a ∈� cũng là thặng dư

toàn phương. 2.2.2 Bài tập áp dụng

Bài toán 1.Cho h 1= và 2km = . Khi ñó 22 1k

kn F= = + . Số kF ñược gọi là

Fermat và những số Fermat là số nguyên tố gọi là số nguyên tố Fermat. Fermat ñã từng dự ñoán tất cả các số Fermat ñều là số nguyên tố nhưng Euler ñã chỉ ra rằng 5641 | F .

Chứng minh Trước hết ta có nhận xét sau: “Cho m 2≥ và h < 2m , giả sử .2 1m

n h= + là phi thặng dư toàn phương mod p với p là số nguyên tố lẻ . Khi ñó n là một số nguyên tố khi và chỉ khi

1

2 1( )n

p mod n−

≡ − (*)”

Page 149: Chuyên đề hội thảo một số trường chuyên về dãy số

HỘ

HỘI CÁC TRƯỜNG THPT CHUYÊN KHU VỰC DUYÊN HẢI VÀ ðỒNG BẰNG BẮC BỘ HỘI THẢO KHOA HỌC LẦN THỨ IV

149

Thật vậy: Giả sử n là số nguyên tố. Từ giả thiết suy ra 1( 4)n mod≡ ta có

1p n

n p

= = −

suy ra ñẳng thức (*) cần chứng minh. Giả sử có (*). Gọi q là ước nguyên tố bất kỳ của n và d là cấp của p (mod q). Khi ñó:

( 1)/2 1 11 , 1 ; 1( )n n qp p p mod q

− − −≡ − ≡ ≡

Vậy 1

| , | ( 1); | 12

nd d n d q

−− −/

Nên 1| 2 ; | (2 ); | 1m md h d h d q

− −/

Suy ra 2 1mq x= + . Mà 1 ( 2 )m

n q mod≡ ≡ nên ta có 1 (mod 2 )mn

q≡ . Do vậy

n=(2 1)(2 1)m mx y+ + với 1, 0x y≥ ≥

Mà 2 2 2m m mxy xy x y h< + + = <

Vậy y 0= và n q= là số nguyên tố.

Áp dụng nhận xét trên ta có: 25 32 28 4 282 2 16.2 (641 5 )2= = = −

7 4 4641 (5.2 ) 641 (641 1) 641 - - 1m m n= = − = −

với m = 282 và n = 6700417 Ta có 2( 3)

kF mod≡ . Nên

kF là phi thặng dư toàn phương của 3. Theo nhận

xét trên ta có 1

2| (3 1)kF

kF−

+ . �

Bài toán 2. Cho p là số nguyên tố và

ia , i 1,2,..,p 1= − là ký hiệu Lagendre

của i theo mod p. Cho ña thức: 2

1 2 1... p

pf a a X a X −

−= + + +

a) CMR 1 là nghiệm ñơn của f khi và chỉ khi 1( 4)p mod≡ b) CMR nếu 5( 8)p mod≡ thì 1 là nghiệm bậc hai của f

( Romani-2004) Chứng minh

a) Ta có:

ðặt {i| a 1, 1,( 1)}iA i p= = = −

{i| a 1, 1,( 1)}iB i p= = − = −

Giả sử f(1) = 0 thì ( )' 1 0f = . Mà

Page 150: Chuyên đề hội thảo một số trường chuyên về dãy số

HỘ

HỘI CÁC TRƯỜNG THPT CHUYÊN KHU VỰC DUYÊN HẢI VÀ ðỒNG BẰNG BẮC BỘ HỘI THẢO KHOA HỌC LẦN THỨ IV

150

1

'

1 1

(1) ( 1)p p

i i

i i i A i B

f i a ia i i−

= = ∈ ∈

= − = = −∑ ∑ ∑ ∑

Suy ra '(1) 0f = khi và chỉ khi tổng các thặng dư toàn phương mod p bằng với

tổng các phi thặng dư toàn phương mod p. Do ( 1)

2i A i B

p pi i

∈ ∈

−+ =∑ ∑ nên suy ra

1( 4)p mod≡

Khi (-1) A∈ thì i A p i A∈ ⇒ − ∈ . Vậy 1

2

p − phần tử thuộc A nhóm thành

1

4

p − cặp, mỗi cặp có tổng bằng p. Do ñó:

( 1)

4i A

p pi

−=∑

Vậy 1( 4)p mod≡

Mặt khác 2 2 (mod p) ( ) ( )x y x y x y p≡ ⇔ = ∧ + = nên |A|= |B|= 1

2

p −và f (1)= 0.

Luôn có 1 là nghiệm ñơn của f. b) Ta cần chứng minh rằng 2( 1)X − là ước của f.

Nhận thấy 1

' 2

1

(1)p

i

i

f i a−

=

=∑

ðặt tập

{ | }2

pA r A r< = ∈ <

{ | }2

pA r A r> = ∈ >

{ | }2

pB r A r< = ∈ <

{ | }2

pB r A r> = ∈ <

Ta thấy cả A< và A> ñều có 1

4

p − phần tử và:

2

2 2 2( 1(

)) 2

4r A r A r A r A

p pr r r rp p

> > < <∈ ∈ ∈ ∈

−= −− = +∑ ∑ ∑ ∑

Như vậy

2

2 2( 1)2 2

4 r A r A

r A

p pr p r r

< <∈ ∈∈

−= − +∑ ∑∑ (1)

tương tự

Page 151: Chuyên đề hội thảo một số trường chuyên về dãy số

HỘ

HỘI CÁC TRƯỜNG THPT CHUYÊN KHU VỰC DUYÊN HẢI VÀ ðỒNG BẰNG BẮC BỘ HỘI THẢO KHOA HỌC LẦN THỨ IV

151

2

2 2( 1)2 2

4 r B r B

r B

p pr p r r

< <∈ ∈∈

−= − +∑ ∑∑ (2)

Thay r bởi 2r (tương ứng p-2r) ta xác ñịnh ánh xạ 1-1 từ A< vào 2kB ,(tương ứng

2 1kN + ) Vì vậy:

2

2 2( 1)4 8

4 r B r B

r B

p pr p r r

< <∈ ∈∈

−= − +∑ ∑∑

(3) Tương tự:

2

2 2( 1)4 8

4 r A r A

r A

p pr p r r

< <∈ ∈∈

−= − +∑ ∑∑

(4) Giả sử '(1) 0f = nên:

2 2

r A r B

r r∈ ∈

=∑ ∑ (5)

Kết hợp với (1) và (2) ta có:

2

2 ( 1) 4

4 3 r A

r A

p pr p r

<∈∈

−= − ∑∑

2

2 ( 1) 4

4 3 r B

r B

p pr p r

<∈∈

−= − ∑∑

Sử dụng (1) và (4) suy ra

r A r B

r r< <∈ ∈=∑ ∑

Mặt khác:

2

/2

1

8r A r B

r p

pr r

< <∈ ∈<

−+ =∑ ∑ ∑

là số lẻ (mâu thuẫn). Vậy ñiều giả sử ở (3) là sai � Bài toán 3 Tìm k ∈� sao cho 2: 2 1| 9k

m m∃ − + (*)

Chứng minh Ta có ñpcm 2 9( 2 1)k

m mod⇔ ≡ − −

Giả sử p| 2 1k −

*Nếu 3p ≠ có 9

1 p

− =

nên p 4n 1= +

Khi p= 4n+1 thì 2 1 1(mod 4)k − ≡ nên k=1

*Nếu p=3 ðặt 2tk s= , s lẻ. 2 1s⇒ − không chia hết cho 3. Mà 22 1| 9k

m− + nên 22 1| 9 1s

m s− + ⇒ = theo trường hợp trên.

Page 152: Chuyên đề hội thảo một số trường chuyên về dãy số

HỘ

HỘI CÁC TRƯỜNG THPT CHUYÊN KHU VỰC DUYÊN HẢI VÀ ðỒNG BẰNG BẮC BỘ HỘI THẢO KHOA HỌC LẦN THỨ IV

152

Ta sẽ chứng minh với mọi 2tk = thoả mãn (*) bằng cách áp dụng ñịnh lý Trung

hoa cho bộ số 1 2( , ,.., )t

a a a với 122 1

i

ia−

= + Thật vậy: Ta thấy ( , ) 1u va a = 1 u v t∀ ≤ ≠ ≤ vì giả sử tồn tại p mà ( , )u va a q=

Giả sử 1 122 2 .u vk

− −= 1 12 2 22 1 (2 ) 1

u v k− −

− = − chia hết 12 2(2 ) 1

v−

Mà 1 1 12 2 2 2(2 ) 1 (2 1)(2 1)

v v v− − −

− = − + nên 2

va − chia hết cho

ua

2 chia hết cho q suy ra q =1 (mâu thuẫn) Theo ñịnh lý về phần dư trung hoa tồn tại m sao cho

10( )m mod a≡ 02

23.2 ( )m mod a≡ 223.2 ( )

t

tm mod a−

suy ra 12 29.2 ( )

t

tm mod a−

Vậy 2 2 11 29 9(2 1)( ... )

t

tm mod a a a−+ ≡ +

ñpcm. �

Bài toán 4 Cho n= 4km-1 với k, n, m *∈� . Chứng minh a∃/ sao cho 2 ( - )a m mod n≡

Chứng minh Gọi 1 2 ... kq q q< < < là tất cả các ước nguyên tố với số mũ lẻ của m. TH1: Nếu 1 2q > . Với p là số nguyên tố p|n

11... 1

km q q

p p p p

− − = =

Theo luật thặng dư toàn phương

1

11(1 )

2 2

1

( 1)

kiqp

k

i i

p

q

−−+

=

∑ = −

Suy ra:

1

|

( 1)( 1)1

2 2

1

( 1)

ki

ip n

qp

k

i i

n

q

α

α=

−−

+

=

∑ ∑

= −

(4)

Mà luôn có: 1

1

2

1

( 1)

kiq

k

i i

n

q

=

∑ = −

∏ (5)

Page 153: Chuyên đề hội thảo một số trường chuyên về dãy số

HỘ

HỘI CÁC TRƯỜNG THPT CHUYÊN KHU VỰC DUYÊN HẢI VÀ ðỒNG BẰNG BẮC BỘ HỘI THẢO KHOA HỌC LẦN THỨ IV

153

(do n 1( q )i

mod≡ − )

Vì n 3( 4)mod≡ nên |

11(mod 2)

2p n

p

α

α−

≡∑ từ (4) (5) suy ra vô lý.

TH2: 1 2q =

Giả sử phản chứng:

21

1

(( 1)/2)( ( 1)/2 1) ( 1)/8

1

( 1)

k

i

i

k p q p

i i

p

q=

− − + + −

=

∑ = −

Do |

11(mod 2)

2p n

p

α

α−

≡∑ suy ra

2

|

1

81 ( 1) p n

p

α

α−

∑− ≡ − . Vậy 3( 8)n mod≡ ± mâu

thuẫn với giả thiết. � Bài toán 5 Cho a, b là các số nguyên sao cho 15 16a b+ và 16 15a b− là bình phương của hai số nguyên dương. Tìm giá trị nhỏ nhất của số nhỏ nhất trong hai số bình phương ñó. (IMO 1996)

Chứng minh ðặt 215 16a b m+ = , 216 15a b n− = với m và n là các số nguyên dương. Biến ñổi ta có:

2 2 2 2 2 2 2 2

2 2 2 2

15 16 15 16 16 15 16 15,

15 16 13.37 15 16 13.37

m n m n m n m na b

+ + − −= = = =

+ +

Suy ra 2 215 16 0( 13.37)m n mod+ ≡ và 2 216 15 0( 13.37)m n mod− ≡

Vậy:

2 2

2 2

15 16 0( 13)

16 15 0( 13)

m n mod

m n mod

+ ≡ − ≡

và:

2 2

2 2

15 16 0( 37)

16 15 0( 37)

m n mod

m n mod

+ ≡ − ≡

Xét hệ thứ nhất: Ta có : 2 2 2 215 16 0( 13) (4 ) 15 ( 13)m n mod n m mod+ ≡ ⇔ ≡ − (*). Giả sử (13, ) 1m = tồn tai 1m sao cho 1. 1( 13)m m mod≡ . Từ (*) có:

21(4 ) 15 1( 13)nm mod≡ − ≡ −

Tức (-2) là thặng dư toàn phương mod 13

Page 154: Chuyên đề hội thảo một số trường chuyên về dãy số

HỘ

HỘI CÁC TRƯỜNG THPT CHUYÊN KHU VỰC DUYÊN HẢI VÀ ðỒNG BẰNG BẮC BỘ HỘI THẢO KHOA HỌC LẦN THỨ IV

154

Nhưng 2

113

− = −

nên vô lý. Vậy 0( 13)m mod≡ . Tương tự có

0( 13)n mod≡ .(**) Xét tương tự với hệ hai. Ta có 0( 37)m mod≡ và 0( 37)n mod≡ (***)

Từ (**) và (***) có 0( 481)

0( 481)

m mod

n mod

≡.

Vậy 2 2 2min{min ,{ 8}} 4 1m n = với 31, 1a b= = �

2.2.3 Bài tập ñề nghị 1) Cho (RR) là số các cặp ( , 1)n n + trong tập 1,2,3...,( 1)p − sao cho n và n+1

ñều là thặng dư toàn phương mod p. Cho (NR) là số các cặp (n,n+1) trong tập 1,2,3...,( 1)p − sao cho n là phi thặng dư toàn phương và n+1 là thặng dư

toàn phương. Tương tự ñịnh nghĩa (RN) và (NN). Chứng minh

å) (RR)+(NN)-(RN)-(NR)=1

1

( 1)p

n

n n

p

=

+∑ . ðánh giá tổng này và từ ñó suy ra tổng

này bằng -1.

b) Chứng minh (RR)= 1

4(p - 4- ε ) với ( 1)/2( 1) pε −= −

2) Cho p là số nguyên lẻ. Chứng minh

a) ( 1)/2

1

2 22cos

p

j

j

p p

π−

=

=

b) ( 1)/2

2

1

3 2(3 4sin )

p

j

j

p p

π−

=

= −

3) Cho vô số số nguyên tố dạng 8k+3, 8k+5, 5k+1. Cho p ∈ P có tồn tại a ñể 2\ 15p a +

Cho (a,b)= 1. Chứng minh rằng mọi ước số lẻ của 2 22a b+ có dạng 8k +1 hoặc 8k+2

Cho ,a b∈� , p là số nguyên tố.

Chứng minh rằng nếu 3 2p k= + thì 2 2\ 3p a b+ khi và chỉ khi ,\ a \ bp p /0 * 2 2 *| 2 , },{A n N n a b a b= ∈ = + ∈� . Tìm ñiều kiện cần và ñủ ñể n A∈ .

Cho 4 1np = + ( 1n ≥ ). Chứng minh rằng p ∈P

1

23 1 0( )p

mod p−

⇔ + ≡

Cho p , 1( 4)p mod∈ ≡P . Chứng minh rằng tồn tại q số p< sao cho ( 1)/2 1( )p

q mod p− ≡ .

Page 155: Chuyên đề hội thảo một số trường chuyên về dãy số

HỘ

HỘI CÁC TRƯỜNG THPT CHUYÊN KHU VỰC DUYÊN HẢI VÀ ðỒNG BẰNG BẮC BỘ HỘI THẢO KHOA HỌC LẦN THỨ IV

155

Cho 4 3p k= + là số nguyên tố, m là số các thặng dư phi toàn phương1

2

p −≤ .

Chứng minh rằng ( )1.3.5.....( 2) ( 1) ( )m kp mod p

+− ≡ − .

Cho , 3p p∈ >P . Tìm ñiều kiện của p ñể: 2 2 23 .5 ...( 1) 1( )p mod p− ≡

Page 156: Chuyên đề hội thảo một số trường chuyên về dãy số

HỘ

HỘI CÁC TRƯỜNG THPT CHUYÊN KHU VỰC DUYÊN HẢI VÀ ðỒNG BẰNG BẮC BỘ HỘI THẢO KHOA HỌC LẦN THỨ IV

156

ðỊNH LÝ PHECMA VÀ MỘT VÀI ỨNG DỤNG

Trường THPT Chuyên Lê Hồng Phong – Nam ðịnh ðịnh lý: Cho p là một số nguyên tố và a là một số nguyên không chia hết cho p.

Khi ñó: ( )1 1 modpa p

− ≡

Chứng minh: Ta có: với (a;p)=1 thì ( ){ };2 ;3 ;...; 1a a a p a− là một hệ thặng dư thu gọn modp

Suy ra: ( ) ( )( ) ( )1.2 .... 1 1.2.... 1 mod 1 modpa a p a p p a p

−− ≡ − ⇒ ≡

Hệ quả: Cho p là một số nguyên tố và a là một số nguyên bất kì. Khi ñó:

( )modpa a p≡

Một bài toán ñơn giản áp dụng ñịnh lý Phecma: Tìm số dư của 2013 khi chia cho 11. Áp dụng ñịnh lý Phecma với 11 là số nguyên tố ta có:

( ) ( ) ( )10 10.20 201 2003 1 mod11 3 1 mod11 3 3 .3 3 mod11≡ ⇒ ≡ ⇒ = ≡

ðịnh lý Phecma chủ yếu ứng dụng trong các bài toán chứng minh sự chia hết, sau ñây chúng ta cùng ñến với một số bài toán. Dạng 1: Chứng minh chia hết

Bài toán 1: Cho p là số nguyên tố. Chứng minh rằng p là ước của p pab ba−

với a, b là các số nguyên bất kì. Giải Ta có: ( )1 1p p p p

ab ba ab b a− −− = − .

Nếu |p ab thì bài toán ñược giải quyết Nếu p không là ước của ab hay p không là ước của cả a và b, theo Phecma ta

có: ( ) ( )1 1 1 11 mod 0 modp p p pa b p a b p

− − − −≡ ≡ ⇒ − ≡ . Bài toán ñược chứng minh.

Bài toán 2: Cho p là số nguyên tố lẻ, q và r là các số nguyên tố thỏa mãn: | 1r

p q + . Chứng minh rằng: 2 | 1r p − hoặc 2| 1p q − .

Chứng minh ðặt ( )ord 1 modd

pd q q p= ⇒ ≡

Theo giả thiết: ( ) ( )21 mod 1 modr rq p q p≡ − ⇒ ≡

Suy ra: | 2

|

d r

d

2

2

d

d rr

=⇔ =

Nếu 2d = thì 2 1q p− M

Nếu 2d r= thì 1 2p r− M (do ( )1 1 modpq p

− ≡

Page 157: Chuyên đề hội thảo một số trường chuyên về dãy số

HỘ

HỘI CÁC TRƯỜNG THPT CHUYÊN KHU VỰC DUYÊN HẢI VÀ ðỒNG BẰNG BẮC BỘ HỘI THẢO KHOA HỌC LẦN THỨ IV

157

Bài toán 3: Cho số nguyên 1a > và số nguyên dương n. Nếu p là ước nguyên tố

lẻ của 2 1n

a + thì 11 2np

+− M

Giải

Do p là ước của 2 1n

a + nên a không chia hết cho p, áp dụng Phecma ta có: ( )1 1 modp

a p− ≡ .

Theo giả thiết bài toán:

( ) ( ) ( ) ( )12

2 2 21 mod 1 mod 1 modn n n

a p a p a p+

≡ − ⇒ ≡ ⇒ ≡

Ta cần chứng minh ( )12 ordn

p a+ = , ñiều này hiển nhiên do 2n

a không ñồng dư 1

modp

Bài toán 4: Cho 1n > là số nguyên lẻ. Chứng minh rằng: |n 3 1n + .

Giải Giả sử tồn tại số nguyên lẻ 1n > mà: | 3 1n

n + Gọi p là số nguyên tố nhỏ nhất là ước của n suy ra 3p > .

Ta có: ( ) ( )23 1 mod 3 1 modn np p≡ − ⇒ ≡

Theo Phecma ta có: ( ) ( ) ( )1,213 1 mod 3 1 modp npp p

−− ≡ ⇒ ≡

Do p-1 chẵn nên ( )1,2 2p n− = (do n lẻ).

Từ ñó suy ra: ( )23 1 mod 8p p≡ ⇔ = , ñiều này vô lí �

Dạng 2: Tìm số nguyên thỏa mãn ñiều kiện cho trước Bài toán 5: Tìm tất cả các số tự nhiên n thỏa mãn: ( )3 | .2 1n

n + .

Giải Nhận thấy: ( )22 1 mod3k ≡ nên 2n chia 3 dư 1 hoặc 2 tùy theo n là số chẵn hay lẻ

tương ứng. Do ñó tất cả các số tự nhiên n cần tìm có dạng: 6 1n m= + hoặc 6 2n m= + . Dạng 3: Chứng minh tồn tại vô số các số nguyên dương thỏa mãn ñiều kiện chia hết. Ta xét hai bài toán sau: Bài toán 7 (Putnam 1972): Chứng minh rằng: với mỗi số nguyên tố lẻ p , tồn

tại vô số số nguyên dương n sao cho: | .2 1np n + . Bài toán 8 (Canada 1983): Chứng minh rằng: với mỗi số nguyên tố lẻ p , tồn

tại vô số số nguyên dương n sao cho: | 2np n− .

Page 158: Chuyên đề hội thảo một số trường chuyên về dãy số

HỘ

HỘI CÁC TRƯỜNG THPT CHUYÊN KHU VỰC DUYÊN HẢI VÀ ðỒNG BẰNG BẮC BỘ HỘI THẢO KHOA HỌC LẦN THỨ IV

158

Cả hai bài toán ñều liên quan ñến tính chia hết và số nguyên tố, ñiều này giúp ta nghĩ ñến các ñịnh lý ñồng dư liên quan ñến số nguyên tố, một trong những ñịnh lý hay sử dụng là ñịnh lý Phecma. Ta xét bài toán 1: Theo ñịnh lý Phecma: ( )12 1 modp

p− ≡ nên nếu ( )1n p −M thì ( )2 1 modn

p≡ , vì

vậy nếu ta chọn ( )1n p −M thì .2 1 1(mod )+ ≡ +nn n p ở ñây n có thể là lũy thừa

của 1p − , hoặc có thể bằng 1p − nhân với một hệ số nào ñó.

Mặt khác: ( ) ( ).2 1 mod 1 modnn p n p≡ − ⇒ ≡ − .

• Nếu chọn n là lũy thừa của 1p − thì số mũ phải lẻ thỏa mãn hai ñiều kiện trên:

( )2 1

1 ,k

n p k+

= − ∈�

Khi k chạy trên tập số tự nhiên cho ta vô số số n thỏa mãn ñiều kiện bài toán. • Nếu n bằng tích của 1p − với một hệ số thì hệ số ñó phải chia p dư 1, ta chọn

( )( )1 1= − +n p kp , với k là một số tự nhiên bất kì.

Khi k chạy trên tập số tự nhiên sẽ cho ta vô số các số n thỏa mãn ñiều kiện. Với các ý tưởng như trên thì bài toán 7 hoàn toàn ñược giải quyết. � ðể xây dựng ñược bài toán này, một trong những cách có lẽ ta ñi từ kết luận của bài toán, có nghĩa chúng ta lấy dạng của số n và ñưa ra ñiều kiện về chia hết.

• Nếu xét n là một lũy thừa của 1p − .

Có thể ñưa ra 1 ví dụ: Lấy ( ) ( )2 1 *1 , 1 mod

kn p k n p

+= − ∈ ⇒ ≡ −� thì có thể

ñưa ra ñiều kiện bài toán: 2 1np− M hoặc | 2n

p n+ .

Lấy ( )2 *1 ,

kn p k= − ∈� ta có: ( )2 1 modn

p≡ và ( )1 modn p≡ thì ñiều kiện bài

toán có thể ñưa ra là: | .2 1np n − hoặc ( )2 mod 2n n

n p n p≡ ⇔ − M

Và ñây chính là bài toán 8. • Nếu lấy n bằng tích của 1p − với một hệ số. Với dạng này bài toán ñưa ra ñiều

kiện ña dạng hơn, có thể chọn hệ số ñồng dư với k (mod p), ( )( )1 .= − +n p s p k , ta ñược ( )modn k p≡ − .

Khi ñó ta ñược bài toán:” Với mỗi số tự nhiên k cho trước, tồn tại vô hạn số tự

nhiên n ñể .2nn k p+ M (p là số nguyên tố lẻ ñã cho)”.

Trên ñây là 1 suy nghĩ nhỏ về ứng dụng của ñịnh lý Phecma trong bài toán chứng minh tồn tại vô số số nguyên dương thỏa mãn ñiều kiện chia hết cho một số nguyên tố, ứng dụng này không chỉ giúp chúng ta giải quyết bài toán 7 và bài toán 8 ở trên mà còn giúp ta xây dựng ñược một số bài toán mới cùng dạng. �

Page 159: Chuyên đề hội thảo một số trường chuyên về dãy số

HỘ

HỘI CÁC TRƯỜNG THPT CHUYÊN KHU VỰC DUYÊN HẢI VÀ ðỒNG BẰNG BẮC BỘ HỘI THẢO KHOA HỌC LẦN THỨ IV

159

Một số bài tập ñề nghị Bài toán 1: Chứng minh rằng với mọi số nguyên m, n, ta có: ( )60 60

mn m n− là

bội của 56786730. Bài toán 2: Chứng minh rằng: 142 | 22.....2 (có 140 chữ số 2).

Bài toán 3: Cho p là số nguyên tố lẻ và ( )| p pp m n+ . Chứng minh rằng:

( )2 | p pp m n+

Bài toán 4: Cho p, q là hai số nguyên tố phân biệt. Chứng minh rằng:

( )| pq p qpq a a a a− − +

Bài toán 5: Chứng minh rằng không tồn tại số nguyên 1n > thỏa mãn:

( )| 2 1nn − .

Bài toán 6: Tìm số nguyên tố p thỏa mãn: | 2 1pp + .

Bài toán 7 (Bulgaria 1995): Tìm các số nguyên tố p, q sao cho:

( )( )| 5 2 5 2p p q qpq − −

Bài toán 8 (USAMO 2005): Chứng minh rằng hệ:

6 3 3 157

3 3 2 9 147

147

157

x x x y y

x x y y y z

+ + + =

+ + + + =

Không có nghiệm nguyên ( ), ,x y z

Bài toán 9 (Turkey 1997): Chứng minh rằng với số nguyên tố 7p > , tồn tại số nguyên n và các số nguyên 1 2; ;...;

nx x x ; 1 2; ;...;

ny y y không chia hết cho p thỏa

mãn:

( )

( )

( )

2 2 21 1 2

2 2 22 2 3

2 2 21

mod

mod

...

modn n

x y x p

x y x p

x y x p

+ ≡

+ ≡ + ≡

Bài toán 10 (Zhenfu Cao): Cho p là số nguyên tố và dãy ( )ka các số nguyên

ñược xác ñịnh: 0 1 2 10; 1; 2 , 0,1,...k k ka a a a pa k+ += = = − ∀ = .

Giả sử số 1− thuộc dãy trên, tìm p.

Page 160: Chuyên đề hội thảo một số trường chuyên về dãy số

HỘ

HỘI CÁC TRƯỜNG THPT CHUYÊN KHU VỰC DUYÊN HẢI VÀ ðỒNG BẰNG BẮC BỘ HỘI THẢO KHOA HỌC LẦN THỨ IV

160

Tài liệu tham khảo 1. David A.Santos, Number Theory for Mathematical Contests, August 13,

2005 Revision. 2. Titu Andreescu, Dorin Andrica, Zuming Feng, 104 Number Theory

Problem. 3. Arthur Engel, Problem – Solving Strategies. 4. Kenneth H.Rosen, Elementary Number Theory 5. Titu Andreescu, Zuming Feng, National Contests: Problems anh

Solutions. 6. Dusan Djukic, Vladimir Jankovic, Ivan Matic, Nikola Petrovic, The IMO

Compendium

Page 161: Chuyên đề hội thảo một số trường chuyên về dãy số

HỘ

HỘI CÁC TRƯỜNG THPT CHUYÊN KHU VỰC DUYÊN HẢI VÀ ðỒNG BẰNG BẮC BỘ HỘI THẢO KHOA HỌC LẦN THỨ IV

161

MỘT SỐ ỨNG DỤNG CỦA ðỊNH LÝ FERMAT NHỎ VÀ ðỊNH LÝ CƠ BẢN VỀ SỐ NGUYÊN TỐ

Nguyễn Văn Thảo Trường trung học phổ thông Chuyên Bắc Giang

A. Mở ñầu Số học là phần rất quan trọng trong chương trình Toán phổ thông. Trong hầu hết các ñề thi học sinh giỏi các cấp, bài toán số học luôn xuất hiện và thường là một thử thách thực sự ñối với các thí sinh. Trong bài viết này, tôi muốn trình bày một số ứng dụng của hai ñịnh lí quan trọng bậc nhất của số học hiện nay: ðịnh lí cơ bản về số nguyên tố và ñịnh lí Fermat nhỏ B. Nội dung I. Lý thuyết 1. ðịnh lí 1 (ðịnh lí cơ bản của số học hay còn gọi là ñịnh lí cơ bản về số nguyên tố) Cho số nguyên dương n (n > 1). Khi ñó n luôn có thể biểu diễn duy nhất

(không kể thứ tự) dưới dạng sau: n = 1 21 2 ... mkk k

mp p p . (1)

trong ñó p1, p2, …, pm là các số nguyên tố phân biệt. k1, k2, …, km là các số nguyên dương. (Dạng phân tích trên ñược gọi là khai triển chính tắc của số nguyên dương n) 2. Nhận xét a) Cho n là số nguyên bất kì, p là một số nguyên tố. Khi ñó luôn tồn tại k nguyên không âm sao cho: n = pk

N, N ∈ Z, (p, N) = 1. Khi ñó k còn ñược gọi là số mũ ñúng của p trong n. kí hiệu k = vp(n). b) Bổ ñề về số mũ ñúng: b1. Cho x, y ∈ Z, n nguyên dương, p là số nguyên tố lẻ và x – y M p. Khi ñó vp(x

n – yn) = vp(x – y) + vp(n). b2. Cho x, y là các số nguyên lẻ, n nguyên dương chẵn. Khi ñó v2(x

n – yn) = v2(x2 – y2) +v2(n) – 1.

c) Trong khai triển (1), n là số chính phương khi và chỉ khi ki chẵn với mọi i = 1, 2, …, m. d) ðể chứng minh A M B, ta có thể gọi p là ước nguyên tố bất kì của B và khai triển A =p

kA1; B = pl

B1 sau ñó chứng minh k ≥ l. Việc ñó giống như phép chiếu trong hình học, bởi ta hạn chế ñược trên một “phương” p. e) Công thức Polignac Trong phân tích chính tắc của n!: n! = kn

k

nnppp .... 21

21

Page 162: Chuyên đề hội thảo một số trường chuyên về dãy số

HỘ

HỘI CÁC TRƯỜNG THPT CHUYÊN KHU VỰC DUYÊN HẢI VÀ ðỒNG BẰNG BẮC BỘ HỘI THẢO KHOA HỌC LẦN THỨ IV

162

số mũ ni của pi ñược tính theo công thức: ni = ...32

+

+

+

iii p

n

p

n

p

n

3. ðịnh lí Fermat nhỏ Cho p là một số nguyên tố, a là một số nguyên bất kì không chia hết cho p. Khi ñó a

p-1 ≡ 1 (mod p). Với mọi a nguyên thì a

p – a ≡ 0 (mod p) II. Áp dụng 1. Một số bài tập áp dụng ñịnh lí cơ bản về số nguyên tố Ví dụ 1. Cho a, b, c là các số nguyên thỏa mãn a3 chia hết cho b, b3 chia hết cho c và c3 chia hết cho a. Chứng minh rằng (a + b + c)13 M abc.

(Olimpic Balan) Lời giải Với những bài chia hết chứa tích các số hạng như trên thì việc vận dụng

việc xét theo “một phương” nguyên tố là cách làm hợp lí nhất!

Goi p là một ước nguyên tố bất kì của abc. Khi ñó tồn tại α, β, γ nguyên không âm sao cho: a =p

α A, b = p

β B, c = p

γ C và (p, A) = (p, B) = (p, C) = 1.

do a3 M b ⇒ p3α A

3 M pβ B mà (p, A) = 1 nên 3α ≥ β.

Tương tự ta thấy 3β ≥ γ và 3γ ≥ α. Ta có abc = pα + β + γ

ABC và (p, ABC) = 1 Như vậy ta chỉ cần chứng minh số mũ ñúng của p trong (a + b + c)13 không nhỏ hơn α + β + γ Giả sử α = min {α, β, γ } Khi ñó, số mũ ñúng của (a + b + c)13 là 13α Mà α + β + γ ≤ α + 3α + 9γ = 13α Suy ra (a + b + c)13 M pα + β + γ Từ ñó có ñiều phải chứng minh. Ví dụ 2. Tìm bộ ba số nguyên dương (x;y;z) thỏa mãn ñồng thời hai ñiều kiện: i) z và 2012 nguyên tố cùng nhau. ii) 2 2 2012( )x y+ =(xy)

z .

(Chọn ñội tuyển Bắc Giang năm 2011) Lời giải

Page 163: Chuyên đề hội thảo một số trường chuyên về dãy số

HỘ

HỘI CÁC TRƯỜNG THPT CHUYÊN KHU VỰC DUYÊN HẢI VÀ ðỒNG BẰNG BẮC BỘ HỘI THẢO KHOA HỌC LẦN THỨ IV

163

(Từ ñiều kiện ii) ta có thể nghĩ tới việc ñồng nhất các số mũ hai vế, việc

ñó cũng giống như ñồng nhất hệ số của hai ña thức bằng nhau!)

Ta có (xy)z = (x2 + y2)2012 ≥ (2xy)2012 z 2012⇒ > . Với một số nguyên tố p bất kỳ, gọi a, b là các lũy thừa của p trong phân tích chính tắc của x, y tương ứng thì lũy thừa của p trong phân tích chính tắc của

z(xy) là (a + b)z.

Nếu a < b thì lũy thừa của p trong phân tích chính tắc của ( )20122 2x y+ là

2a.2012. Mà 2a.2012 < (a + b).2012 < (a + b)z, mâu thuẫn với ñiều ii).

Nếu a > b thì lũy thừa của p trong phân tích chính tắc của ( )20122 2x y+ là

2b.2012. Mà 2b.2012 < (a + b).2012 < (a + b)z, mâu thuẫn với ñiều ii). Do ñó a = b, mà p là số nguyên tố bất kỳ nên x = y. Khi ñó ii) trở thành

t2 2z 2012 2(z 2012)2012 x 2

(2x ) x 2 x zt 1006(2t 1)2t(z 2012) 2012

− == ⇒ = ⇒ ⇒ = +

− =.

Mặt khác (t;2t + 1) = 1, (1006;z) = 1 (do ñiều 1))

Suy ra t 1006 t 1006

z 2t 1 z 2013

= = ⇒

= + = .

Vậy ( )1006 1006(x; y;z) 2 ;2 ;2013= .

Ví dụ 3. Cho x, a, b là các số nguyên dương thỏa mãn x

a + b = a

bb.

(3). Chứng minh rằng a = x và b =ax.

(Olimpic Iran 1998) Lời giải (từ (3), ta có thể nghĩ tới việc ñồng nhất các số mũ hai vế) x = 1, bài toán hiển nhiên ñúng. Gọi p là một ước nguyên tố bất kì của một trong ba số x, a, b, Khi ñó x = pα

X; a = pβ A, b = pγ

B; (p, X) = (p, A) = (p, B) = 1 với α, β, γ không âm và không ñồng thời bằng 0. Thay vào (3) ta ñược: (pα

X)a + b = (pβ A)b. pγ

B ðồng nhất số mũ của p ở hai vế ta ñược α(a + b) = γ + bβ. Dễ thấy γ > 0 vì nếu γ = 0 thì α(a + b) = bβ ⇔ (β - α )b = αa ⇔ (β - α )b = α.p

βA

Page 164: Chuyên đề hội thảo một số trường chuyên về dãy số

HỘ

HỘI CÁC TRƯỜNG THPT CHUYÊN KHU VỰC DUYÊN HẢI VÀ ðỒNG BẰNG BẮC BỘ HỘI THẢO KHOA HỌC LẦN THỨ IV

164

Mà (b, p) = 1 (do γ = 0) nên β - α M pβ ⇒ β > pβ (vô lí)

Vậy γ > 0 Ta có α(a + b) = γ + bβ ⇔ α(a

+ p

γ B) = γ + p

γ Bβ ⇔ αa - γ =p

γ (Bβ -αB)

Do γ không chia hết cho pγ nên a không chia hết cho pγ Hay β < γ ⇒ b M a Mặt khác αa - γ = b(β - α) M a nên γ M a Tư ñó suy ra tồn tại một số nguyên dương c sao cho b = ca Thay vào (3) ta dễ dàng có ñược ñiều phải chứng minh. * Trong các bài toán chứng minh a M b, ngoài việc chứng minh trực tiếp, ta có

thể chọn phương án chứng minh a không chia hết cho một ước nguyên tố nào ñó

của a. Ví dụ 4. Cho x, y, p là các số nguyên, p > 1 sao cho x2011 M p và y2012 M p. Chứng minh rằng 1 + x + y không chia hết cho p. Lời giải Do p > 1 nên p phải có ít nhất một ước nguyên tố q. Mà x2011 M p nên x2011 M q mà q nguyên tố nên x M q. Tương tự ta có y M q. Từ ñó suy ra 1 + x + y ≡ 1 (mod q) Hay 1 + x + y không chia hết cho q nên cũng không chia hết cho p. * Tuy nhiên, trong một số trường hợp cụ thể thì việc lựa chọn ước nguyên tố như

thế nào cũng rất quan trọng, thường là ta chọn ước nguyên tố nhỏ nhấ hoặc lớn

nhât! Ví dụ 5. Tìm tất cả các cặp số nguyên dương (n, p) thỏa mãn ñồng thời ba ñiều kiện sau:

i) p là số nguyên tố ii) n ≤ 2p iii) (p – 1)n + 1 M np – 1.

(Olimpic Quốc tế lần 40) Lời giải Trước hết ta thấy mọi cặp số (1, p) ñều thỏa mãn. Nếu p = 2 suy ra n = 2 cũng thỏa mãn ñầu bài. Xét p > 2: Khi ñó (p – 1)n + 1 là số lẻ mà (p – 1)n + 1 M np – 1 nên n là số nguyên dương lẻ Suy ra n < 2p. Gọi q là ước nguyên tố nhỏ nhất của n.

Page 165: Chuyên đề hội thảo một số trường chuyên về dãy số

HỘ

HỘI CÁC TRƯỜNG THPT CHUYÊN KHU VỰC DUYÊN HẢI VÀ ðỒNG BẰNG BẮC BỘ HỘI THẢO KHOA HỌC LẦN THỨ IV

165

Vì n lẻ nên q lẻ. Do (p – 1)n + 1 M np – 1 nên (p – 1)n + 1 M q ⇒ (p - 1)n ≡ -1 (mod q). Mà q là ước nhỏ nhất của n nên (n, q – 1) = 1 do ñó tồn tại hai số nguyên a, b

sao cho an + b(q – 1) = 1. q – 1 chẵn nên a phải là số nguyên lẻ. Ta có p – 1 ≡ (p – 1)an + b(q – 1) ≡ [(p – 1)n]a[(p – 1)q -1]b ≡ (-1)a1b ≡ -1 (mod q) (Do a lẻ) Do ñó p ≡ 0 (mod q) ⇒ p = q (vì p, q là các số nguyên tố) Mà n < 2p, có ước nguyên tố nhỏ nhất là p nên n = p. Thay vào iii) ta ñược p

p – 1 | (p – 1)p + 1 = 1 1 2 2 1... 1 1p p p p

p p pp C p C p C p

− − −− + − + − +

= 1 1 2 2. ... .p p p

p pp C p C p p p− −− + − +

= 2 2 1 3 2 2 2( ... 1)p p p p

p p pp p C p C p C p

− − − −− + − + +

Do 2 1 3 2 2 2( , ... 1) 1p p p p

p p pp p C p C p C p− − − −− + − + + =

Nên pp -1 | p2 ⇒ p – 1 = 2 ⇔ p = 3 ⇒ n = 3 Thử lại thấy cặp (3; 3) cũng thỏa mãn. Tóm lại các cặp (n; p) cần tìm là (1; p) (2; 2) và (3; 3). Ví dụ 6. Chứng minh rằng: ∀ m, n ∈ N ta có (2m)!(2n)! M m!n!(m+n)!

(Olimpic Quốc tế - 1972) Lời giải Gọi p là một số nguyên tố bất kì Số mũ của p trong phân tích chính tắc của m!n!(m+n)! là

α =∑∞

=

++

+

1

)(k

kkkp

nm

p

n

p

m

Số mũ của p trong phân tích chính tắc của (2m)!(2n)! là:

β =∑∞

=

+

1

)22

(k

kkp

n

p

m

Ta dễ dàng chứng minh ñược

++

+

+

kkkkk

p

nm

p

n

p

m

p

n

p

m 22

⇒ β ≥ α Từ ñó có ñiều phải chứng minh. Ví dụ 7. Chứng minh rằng ∀ n ∈ N thì 2n không chia hết ñược n!. Lời giải Ta có n! = 2 m .q Trong ñó (2, q) = 1

Page 166: Chuyên đề hội thảo một số trường chuyên về dãy số

HỘ

HỘI CÁC TRƯỜNG THPT CHUYÊN KHU VỰC DUYÊN HẢI VÀ ðỒNG BẰNG BẮC BỘ HỘI THẢO KHOA HỌC LẦN THỨ IV

166

Ta phải chứng minh n > m. Theo công thức Polignac ta có:

m =

++

+

p

nnn

2...

22 2 với 2p ≤ n < 2p+1

Mặt khác

++

+

p

nnn

2...

22 2 ≤

+++

p

nnn

2...

22 2

=

p

nn

2 < n

Từ ñó suy ra ñiều phải chứng minh. Ví dụ 8. Tìm tất cả các cặp số nguyên dương (x, y) sao cho 3x = 2x

y + 1 (ðề chọn ñội tuyển Romanian)

Lời giải

Ta viết lại (8) thành 3 1

2

x

xy

−= . Như vậy cần tìm x sao cho v2(3

x – 1) ≥ x.

Ta có 3x – 1 ≡ (-1)x – 1 (mod 4) Do ñó nếu x lẻ thì v2(3

x – 1) = 1 ⇒ x = 1 ⇒ y = 1. Nếu x chẵn thì v2(3

x – 1) = v2(8) + v2(x) – 1 = 2 + v2(x) ðặt v2(x) = α ≥ 1. Nên ta cần tìm x sao cho 2 + α ≥ x = 2α A ≥ 2α , (2,A) = 1, A ≥ 1. (*) ⇒ 2 + α ≥ 2α ⇒ α = 1 hoặc α = 2. +) Với α = 1, từ (*) suy ra 3 ≥ 2A ⇒ A = 1 ⇒ x = 2 ⇒ y = 2. +) Với α = 2, từ (*) suy ra A = 1 ⇒ x = 4 ⇒ x = 5. Vậy các cặp (x, y) cần tìm là: (1, 1), (2, 2), (4, 5). Ví dụ 9. Cho p là một số nguyên tố và a, n là hai số nguyên dương. Chứng minh rằng nếu 2p + 3p = an thì n = 1.

(Irish 1996) Lời giải Do 2p + 3p ≡ 0 (mod 5) suy ra a M 5 ⇒ v5(a

n) ≥ n ⇒ v5(2p + 3p) ≤ n

Ta có v5(2p + 3p) = 1 + v5(p)

Nếu p ≠ 5 thì v5(p) = 0 nên n = 1 Nếu p = 5 thì 25 + 35 = 275 không thể là lũy thừa khác 1 của một số nguyên

Page 167: Chuyên đề hội thảo một số trường chuyên về dãy số

HỘ

HỘI CÁC TRƯỜNG THPT CHUYÊN KHU VỰC DUYÊN HẢI VÀ ðỒNG BẰNG BẮC BỘ HỘI THẢO KHOA HỌC LẦN THỨ IV

167

Từ ñó có ñiều phải chứng minh. 2. Ứng dụng của ñịnh lí Fermat nhỏ. Ví dụ 1. Cho p là một số nguyên tố lớn hơn 7. Chứng minh rằng (3p – 2p - 1)M 42p. Lời giải Ta có 3p – 2p – 1 = (3p - 3) – (2p - 2) ≡ 0 (mod p). (1) Mặt khác 3p – 2p – 1 = (3p - 1) – 2p M 2 (2) Mà p > 7 ⇒ p lẻ Do ñó 3p – 2p – 1 ≡ - (-1)p – 1 ≡ 0 (mod 3) (3) Bây giờ ta cần chứng minh 3p – 2p – 1 M 7 Ta có 3p – 2p - 1 = 3.3p-1 – 2p – 1

=1 1

2 23.9 2 1 3.2 2 1p p

p p

− −

− − ≡ − − (mod 7)

= 1 1

2 22 2 2 1p p

p

+ −

+ − − Do (p, 3) = 1 nên p = 3k + 1 hoặc p = 3k + 2 Nếu p = 3k + 1 thì

1 1

2 22 2 2 1p p

p

+ −

+ − − = 1 1

2 28 1 2 2 2 2 (mod 7)p p

k p p

+ +

− + − ≡ − 1 1 1

2 2 22 (2 1) 2 (8 1) 0(mod7)p p p

k

+ − +

= − = − ≡

Nếu p = 3k + 2 ta chứng minh tương tự. Từ ñó suy ra ñpcm. Ví dụ 2. Cho a, b là hai số nguyên thỏa mãn 24a

2 + 1 = b2. Chứng minh rằng có một và chỉ một trong hai số ñó chia hết cho 5. Lời giải Ta có 24a

2 – b2 = 1 không chia hết cho 5 nên a và b không thể cùng chia hết cho 5. Giả sử a và b cùng không chia hết cho 5. Theo ñịnh lí Fermat ta có a

4 – 1 ≡ 0 (mod 5) b

4 – 1 ≡ 0 (mod 5) Do ñó (a2 – b2)(a2 + b2) = a4 – b4 ≡ 0 (mod 5) Nếu a2 + b2 ≡ 0 (mod 5) thì 25a

2 + 1 = a2 + b2 ≡ 0 (mod 5) (vô lí) Vậy a2 – b2 ≡ 0 (mod 5) ⇒ 23a

2 + 1 = b2 – a2 ≡ 0 (mod 5) ⇒ 23a2 + 1 ≡ 0 (mod 5)

Vì (a, 5) = 1 nên a ≡ ± 1 (mod 5) hoặc a ≡ ± 2 (mod 5) Nếu a ≡ ± 1 (mod 5) thì

Page 168: Chuyên đề hội thảo một số trường chuyên về dãy số

HỘ

HỘI CÁC TRƯỜNG THPT CHUYÊN KHU VỰC DUYÊN HẢI VÀ ðỒNG BẰNG BẮC BỘ HỘI THẢO KHOA HỌC LẦN THỨ IV

168

0 ≡ 23a2 + 1 ≡ 23( ± 1)2 + 1 ≡ -1 (mod 5) (vô lí)

Nếu a ≡ ± 2 (mod 5) thì 0 ≡ 23a

2 + 1 ≡ 23( ± 2)2 + 1 ≡ 3 (mod 5) (vô lí). Vậy ñiều giả sử là sai. Từ ñó ta có ñpcm. Ví dụ 3. Cho a là một số nguyên. Chứng minh rằng a

2 + 1 không có ước nguyên tố dạng 4k + 3. Từ ñó suy ra các phương trình sau không có nghiệm nguyên dương.

a) 4xy – x – y = z2 b) x

2 – y3 = 7. Lời giải Giả sử a2 + 1 có ước nguyên tố p = 4k + 3 ⇒ (a, p) = 1. Khi ñó ap-1 + 1 = a4k+ 2 + 1 = (a2)2k + 1 + 1 M a2 + 1 (1) Mặt khác, theo ñịnh lí Fermat, ta có ap – 1 – 1 M p (2) Từ (1) và (2) suy ra 2 M p ⇒ p = 2 (vô lí vì 2 không có dạng 4k + 3) Vậy a2 + 1 không có ước nguyên tố dạng 4k + 3. Áp dụng a) Ta có 4xy – x – y = z2 ⇔ (4x - 1)(4y - 1) = 4z

2 + 1 ⇔ (4x - 1)(4y - 1) = (2z)2 + 1 Do 4x – 1 ≥ 3 với mọi x nguyên dương và có dạng 4k + 3 nên nó có ít nhất một ước nguyên tố dạng 4k + 3 Mà (2z)2 + 1 không có ước nguyên tố dạng 4k + 3 nên phương trình trên vô nghiệm. b) Ta có x2 – y3 = 7 ⇔ x2 + 1 = y3 + 8 ⇔ x2 + 1 = (y + 2)(y2 – 2y + 4) Nếu y chẵn thì (y + 2)(y2 – 2y + 4) M 4 ⇒ x2 + 1 M 4 ⇒ x2 ≡ -1 (mod 4) vô lí Do ñó y lẻ ⇒ y = 4k + 1 hoặc y = 4k + 3 Nếu y = 4k + 1 ⇒ y + 2 = 4k + 3 nên có có ít nhất một ước nguyên tố dạng 4k + 3. Mà x2 + 1 không có ước nguyên tố dạng 4k + 3 ⇒ phương trình trên không có nghiệm trong trường hợp này. Nếu y = 4k + 3 ⇒ y2 – 2y + 4 ≡ (-1)2 – 2(-1) (mod 4) ≡ 3 (mod 4) nên phương trình trên cũng vô nghiệm trong trường hợp này. Từ ñó ta có ñpcm.

Page 169: Chuyên đề hội thảo một số trường chuyên về dãy số

HỘ

HỘI CÁC TRƯỜNG THPT CHUYÊN KHU VỰC DUYÊN HẢI VÀ ðỒNG BẰNG BẮC BỘ HỘI THẢO KHOA HỌC LẦN THỨ IV

169

Ví dụ 4. Cho a, b là các số nguyên. p là một số nguyên tố có dạng 4k + 3. Chứng minh rằng nếu x2 + y2 M p thì x M p và y M p. Từ ñó suy ra phương trình sau vô nghiệm nguyên: x2 + 2y + 4y

2 = 37 Lời giải Giả sử p = 4k + 3 là số nguyên tố thỏa mãn x2 + y2 M p nhưng x không chia hết cho p ⇒ x2 không chia hết cho p ⇒ y không chia hết cho p. Theo ñịnh lí Fermat ta có xp – 1 – 1 M p ⇒ (x2)2k + 1 – 1 M p Tương tự, ta cũng có (y2)2k + 1 – 1 M p suy ra (x2)2k + 1 + (y2)2k + 1 ≡ 2 (mod p) Mà (x2)2k + 1 + (y2)2k + 1 M x2 + y2 M p ⇒ 2 M p ⇒ p = 2 (vô lí) Vậy x M p và y M p. Áp dụng ta có x

2 + 2x + 4y2 = 37 ⇔ (x +1)2 + (2y)2 = 38 M 19 = 4.4 + 3

Do ñó x + 1 M 19 và 2y M 19. Vì x + 1 và 2y không thể cùng bằng 0 nên |x + 1| ≥ 19 hoặc |2y| ≥ 19 Khi ñó (x + 1)2 + (2y)2 ≥ 192 > 38 nên phương trình trên vô nghiệm. Ví dụ 5. Cho p ≥ 7, p nguyên tố. Chứng minh rằng số S = 11…1 (p – 1 chữ số 1) chia hết cho p. Lời giải

Ta có S = 110 1

9

p− −

Vì p ≥ 7 nên (10, p) = 1. Do ñó 10p – 1 – 1 M p và 10p – 1 – 1 M (10 - ) = 9 Mà (p, 9) = 1 nên 10p – 1 – 1 M 9p. Từ ñó suy ra S M p. ñpcm. Ví dụ 6. Cho dãy số (an) xác ñịnh như sau an

= 2n + 3n + 6n – 1 với n = 1, 2, … Tìm số tự nhiên nguyên tố cùng nhau với mọi số hạng của dãy trên.

(Olimpic QT – 2005) Lời giải Ta sẽ chứng minh rằng với mọi số nguyên tố p ñều tồn tại một số hạng an chia hết cho p. Thật vậy, ta có a2 = 22 + 32 + 62 – 1 = 48 chia hết cho 2 và 3. Xét p ≥ 5 Ta có (2, p) = 1; (3, p) = 1; (6, p) = 1 Do ñó, từ ñịnh lí Fermat suy ra 2p – 1 ≡ 3p – 1 ≡ 6p – 1 ≡ 1 (mod p) Từ ñó dễ dàng chứng minh ñược 6ap – 2 M p Mà (p, 6) = 1 nên ap – 2 M p. Do ñó chỉ có số 1 là số tự nhiên duy nhất nguyên tố cùng nhau với mọi số hạng của dãy (an).

Page 170: Chuyên đề hội thảo một số trường chuyên về dãy số

HỘ

HỘI CÁC TRƯỜNG THPT CHUYÊN KHU VỰC DUYÊN HẢI VÀ ðỒNG BẰNG BẮC BỘ HỘI THẢO KHOA HỌC LẦN THỨ IV

170

Ví dụ 7. Tìm số nguyên dương k nhỏ nhất sao cho tồn tại các số x1, x2, …, xk sao cho: x1

3 + x23 + … + xk

3 = 20022002. (Olimpic QT – 2003)

Lời giải Ta có 2002 ≡ 4 (mod 9) ⇒ 20023 ≡ 43 ≡ 1 (mod 9) 20022002 = (20023)667.2002 ≡ 2002 (mod 9) ≡ 4 (mod 9) Mặt khác, với mọi số nguyên a ta có a

3 ≡ ± 1 (mod 9) hoặc a3 ≡ 0 (mod 9) Do ñó x1

3 ; x13 + x2

3; x13 + x2

3 + x33 không thể ñồng dư với 4 modulo 9 ñược.

Tức là với k ≤ 3 thì phương trình trên không có nghiệm nguyên. Ta sẽ chứng minh k = 4 là giá trị cần tìm. Thật vậy, ta có 2002 = 103 + 103 + 13 + 13 Mà 2002 = 3. 667 + 1 ⇒ 20022002 = 2002. (2002667)3 = (103 + 103 + 13 + 13 ) (2002667)3 = (10.2002667)3 + (10.2002667)3 + (2002667)3 + (2002667)3 Vậy với k = 4 thì phương trình trên có nghiệm. KL: k = 4 là giá trị cần tìm. Ví dụ 8. Tìm số các số tự nhiên n > 1 sao cho với mọi số nguyên a thì a25- a

chia hết cho n. (Olimpic Bulgarian 1995)

Lời giải Dễ thấy với mọi p nguyên tố thì p25 – p không chia hết cho p. Do ñó n không thể chia hết cho p2 với p nguyên tố (Số có tính chất ñó ñược gọi là số Squarfree) Mặt khác 225 – 2 = 2.325.7.13.17.241 (1) Ta có 325 ≡ 14 (mod 17) và 335 ≡ 32 (mod 241) Do ñó, n không có ước nguyên tố là 3 và 241 (2) Từ (1) và (2) suy ra n chỉ có thể có các ước nguyên tố là 2, 3, 5, 7, 13. Theo ñịnh lí Fermat nhỏ ta có: a

2 ≡ a (mod 2) ⇒ a25 ≡ a (mod 2) a

25 ≡ (a2)12. a ≡ a (mod 3) a4 ≡ 1 (mod 5) ⇒ a24 ≡ 1 (mod 5) ⇒ a25 ≡ a (mod 5) a

25 ≡ (a6)4.a ≡ a (mod 7) a

12 ≡ 1 (mod 13) ⇒ a25 ≡ a (mod 13) Từ ñó suy ra nếu n chỉ có các ước nguyên tố là 2, 3, 5, 7, 13 và là số squarfree thì a25 – a M n. Do ñó có 25 – 1 = 31 số như vậy.

Page 171: Chuyên đề hội thảo một số trường chuyên về dãy số

HỘ

HỘI CÁC TRƯỜNG THPT CHUYÊN KHU VỰC DUYÊN HẢI VÀ ðỒNG BẰNG BẮC BỘ HỘI THẢO KHOA HỌC LẦN THỨ IV

171

Ví dụ 9. Cho 3n – 2n là lũy thừa của một số nguyên tố nào ñó. Chứng minh rằng n là số nguyên tố. Lời giải Ta có 3n – 2n = pα với p là một số nguyên tố nào ñó và α > 1. Giả sử q là một ước nguyên tố của n và q ≠ n. Thế thì n = kq, k > 1. 3n – 2n = (3k)q – (2k)q M 3k – 2k. Do ñó, 3k – 2k = pm , m ≥ 1.hay 3k = pm + 2k Suy ra

pα = 3n – 2n = ( 2k + pm) q – 2kq = q.pm.2k(q – 1) + ( 1)

2

q q + .p2m.2k(q-2)+… +pqm

Do α > m nên số mũ Ví dụ 10. Với mọi số nguyên tố p, tồn tại vô số các số có dạng 2n – n (n ∈ N) chia hết cho p.

(Olimpic Canada – 1983) Lời giải Với p = 2 thì mọi số dạng 22k – (2k) ñều chia hết cho p. Với p > 2, theo ñịnh lí Fermat nhỏ, ta có 2p – 1 ≡ 1 (mod p) ⇒ 2m(p – 1) ≡ 1 (mod p) ⇒ 2m(p – 1) – m(p – 1) ≡ 1 – m(p – 1) ≡ m + 1 (mod p) Ta chỉ cần chọn m = kp – 1 với mọi k nguyên dương thì 2m(p – 1) – m(p – 1) ≡ 0 (mod p). Từ ñó có ñiều phải chứng minh. Ví dụ 11. Cho p, q là hai số nguyên tố phân biệt. Chứng minh rằng p

q – 1 + qp – 1 – 1 M pq. Lời giải Do p, q nguyên tố nên (p, q) = 1, vì vậy ta chỉ cần chứng minh p

q – 1 + qp – 1 – 1 M p và pq – 1 + qp – 1 – 1 M q Thật vây pq – 1 + (qp – 1 – 1) ≡ qp – 1 – 1 ≡ 0 (mod p) qp – 1 + (pq – 1– 1) ≡ 0 (mod q) Từ ñó có ñiều phải chứng minh. Ví dụ 12. Tìm các số nguyên tố p và q sao cho p3 – q5 = (p + q)2 (1)

(Olimpic Nga 2001) Lời giải Nếu q = 3 thì tìm ñược p = 7. Giả sử q ≠ 3

Page 172: Chuyên đề hội thảo một số trường chuyên về dãy số

HỘ

HỘI CÁC TRƯỜNG THPT CHUYÊN KHU VỰC DUYÊN HẢI VÀ ðỒNG BẰNG BẮC BỘ HỘI THẢO KHOA HỌC LẦN THỨ IV

172

Từ (1) suy ra q > 3 Theo ñịnh lí Fermat nhỏ ta có p

3 – q5 ≡ p – q (mod 3) Nếu p ≡ q (mod 3) thì p + q không chia hết cho 3 Khi ñó, theo ñịnh lí Fermat nhỏ thì (p + q)2 ≡ 1 (mod 3) mà p3 – q5 ≡ 0 (mod 3) Do ñó không thỏa mãn (1) Nếu p và q không ñồng dư (mod 3), p và q không chia hết cho 3 nên p + q M 3 Khi ñó vế trái (1) không chia hết cho 3 và vế phải (1) chia hết cho 3, nên cũng không thỏa mãn (1). Vậy q = 3, p = 7. Bài tập Bài 1. Cho k > 1, p = 6k + 1 là một số nguyên tố và m = 2p – 1. Chứng minh

rằng 12 1

127

m

m

− − là một số nguyên

(Thổ Nhĩ Kì – 2007) Bài 2. Tìm các số nguyên dương x, y, z sao cho 3x + 11y = z2.

(Thổ nhĩ Kì – 2001) Bài 3. Cho m, n là hai số nguyên dương sao cho mn | (m2010 + n2010 + n). Chứng minh rằng có một số nguyên dương k sao cho n = k2010.

(Indonesia – 2010) Bài 4. Tìm số nguyên tố p sao cho 2p + p2 cũng là số nguyên tố.

(Chọn ñội tuyển Anbania – 2011) Bài 5. Cho các số nguyên dương a, b, p, n, k thỏa mãn an + bn = pk. Chứng minh rằng nếu n > 1, n lẻ và p là số nguyên tố lẻ thì n = pα .

(Olimpic Nga - 1996) Bài 6. Cho p là một số nguyên tố. Chứng minh rằng tồn tại số nguyên tố q sao cho với mọi số nguyên dương n thì np – p không chia hết cho q.

(ðề nghị IMO - 2003) Bài 7. Cho a, b, c, n nguyên dương và thỏa mãn an M b, bn M c và cn M a. Chứng minh rằng (a + b + c)

2 1n n+ + M abc. C. Kết luận Trên ñây, tôi ñã chọn, trình bày một số ứng dụng của hai ñịnh lí cơ bản nhất của số học. Trong ñó, một ñịnh lí thường xuyên ñược nhắc tới trong các kì thi Olimpic, ñó là ñịnh lí Fermat nhỏ, ñịnh lí còn lại ít ñược nhắc ñến hơn, mặc dù nó rất quen thuộc ñối với học sinh. Những ví dụ trên phần nào cho thấy: Mọi

Page 173: Chuyên đề hội thảo một số trường chuyên về dãy số

HỘ

HỘI CÁC TRƯỜNG THPT CHUYÊN KHU VỰC DUYÊN HẢI VÀ ðỒNG BẰNG BẮC BỘ HỘI THẢO KHOA HỌC LẦN THỨ IV

173

ñịnh lí, tính chất ñều ẩn chứa trong nó một nguồn năng lượng vô cùng lớn. Nếu bạn biết khai thác và sử dụng hợp lí thì nguồn năng lượng ñó sẽ giúp bạn vượt qua mọi trở ngại khó khăn khi gặp phải. TÀI LIỆU THAM KHẢO [1] ðặng Hùng Thắng, Nguyễn Văn Ngọc, Vũ Kim Thuỷ. Bài giảng số học. NXBGD, 1997. [2] Nguyễn Vũ Lương, Nguyễn Ngọc Thắng, Nguyễn Lưu Sơn, Phạm Văn Hùng . Các bài giảng số học. NXB ðại Học Quốc Gia Hà Nội, 2006. [3] Phan Huy Khải. Các chuyên ñề Số học. NXBGD, 2005. [4] Nguyễn Văn Mậu, Trần nam Dũng, ðặng Hùng Thắng, ðặng Huy Ruận. Các vấn ñề chọn lọc của số học. NXBGD, 2008. [5] Nguyễn Sinh Nguyên, Nguyễn Văn Nho, Lê Hoành Phò.Tuyển tập các bài dự thi Olimpiad Toán học Quốc tế 1991 – 2001. NXBGD, 2001. [6] Titu Andreescu, Dorin Andrica, Zuming Feng. 104 Number theory problems from the training of the USA IMO team. NXB Birkhauser, 2006. [5] Tạp chí Toán học và tuổi trẻ. [6] Các ñề thi vô ñịch các nước. [7] Các tài liệu trên mạng Internet. [8] Chuyên ñề về các bài toán lũy thừa của TS Phạm Văn Quốc ðH KHTN.

Page 174: Chuyên đề hội thảo một số trường chuyên về dãy số

HỘ

HỘI CÁC TRƯỜNG THPT CHUYÊN KHU VỰC DUYÊN HẢI VÀ ðỒNG BẰNG BẮC BỘ HỘI THẢO KHOA HỌC LẦN THỨ IV

174

SỬ DỤNG PHÉP ðỒNG DƯ TRONG BÀI TOÁN CHIA HẾT

Trường THPT chuyên Hưng Yên

I. Một số vấn ñề cơ bản về ñồng dư.

1. ðịnh nghĩa: Cho a, b, m là các số nguyên, m ≠ 0. Nếu a – b chia hết cho m

thì a ñược gọi là ñồng dư với b modulo m, ký hiệu a ≡ b (mod m).

2. Tính chất: Cho a, b, c, d là các số nguyên

2.1 . Nếu a ≡ b (mod m) thì b ≡ a (mod m)

2.2 . Nếu a ≡ b (mod m) và b ≡ c (mod m) thì a ≡ c (mod m)

2.3 . Nếu a ≡ b (mod m) và c ≡ d (mod m) thì a + c ≡ b + d (mod m)

2.4 . Nếu a ≡ b (mod m) và c ≡ d (mod m) thì ac ≡ bd (mod m)

2.5 . Nếu a ≡ b (mod m), k nguyên dương thì ak ≡ bk (mod m)

2.6 . Nếu a ≡ b (mod m) và d|m thì a ≡ b (mod d)

2.7 . Nếu a ≡ b (mod m) thì ac ≡ bc (mod cm), với mọi c≠ 0.

2.8 . Nếu ab ≡ ac (mod m) và (a,m) = 1 thì b ≡ c (mod m)

2.9 . Nếu a ≡ b (mod mi) ( i =1,2,…,n) ⇔ a ≡ b (mod [m1,m2,…,mn])

3. ðịnh lý Fermat nhỏ: Giả sử p nguyên tố, (a, p) = 1. Khi ñó ap–1 ≡ 1 (mod p)

Chứng minh.

Xét p – 1 số: a, 2a, 3a, …, (p – 1)a. Ta chứng minh rằng không tồn tại 2 số ñồng

dư trong phép chi a cho p.

Giả sử ka ≡ la (mod p) với k, l ∈{1,2,…,p – 1} và k ≠ l

⇒ a(k – l) M p ⇒ k – l M p ⇒ k = l (mâu thuẩn)

Page 175: Chuyên đề hội thảo một số trường chuyên về dãy số

HỘ

HỘI CÁC TRƯỜNG THPT CHUYÊN KHU VỰC DUYÊN HẢI VÀ ðỒNG BẰNG BẮC BỘ HỘI THẢO KHOA HỌC LẦN THỨ IV

175

Vậy khi chia p – 1 số trên cho p ta nhận ñược p – 1 số dư khác nhau từ 1, 2, …,

p – 1

Suy ra a.2a.…(p – 1)a ≡ 1.2….(p – 1) (mod p)

⇔ (p – 1)!. ap–1 ≡ (p – 1)! (mod p)

Vì ((p – 1)!,p) = 1 nên ap–1 ≡ 1 (mod p).

Hệ quả: Nếu p nguyên tố, (a, p) =1 thì ap ≡ a (mod p)

4. ðịnh lý Euler. m là số nguyên dương và (a, m) = 1. Khi ñó (m)a 1ϕ ≡ (mod m).

Chứng minh.

Giả sử r1, r2, …, (m)rϕ là hệ thặng dư thu gọn gồm các số nguyên dương không

vượt quá m và nguyên tố cùng nhau với m. Ta có ar1, ar2, …, (m)arϕ là một hệ

thặng dư thu gọn modulo m. Như vậy các ñồng dư dương bé nhất của ar1, ar2,..,

(m)arϕ là các số r1, r2, …, (m)rϕ xếp theo một thứ tự nào ñó. Vì thế ta có

1 2 (m) 1 2 (m)ar .ar ....ar r r ...r (mod m)ϕ ϕ≡ hay (m)1 2 (m) 1 2 (m)a r r ...r r r ...r (mod m)ϕ

ϕ ϕ≡

Vì 1 2 (m)(r r ...r ,m) 1ϕ = nên (m)a 1(mod m)ϕ ≡

5. ðịnh lý Wilson: Với số nguyên tố p, ta có (p – 1)! ≡ – 1 (mod p)

Chứng minh.

Khi p = 2, ta có (p – 1)! = 1 ≡ –1 mod 2

Giả sử p là số nguyên tố lớn hơn 2, khi ñó mỗi số nguyên a với 1 ≤ a ≤ p – 1 tồn

tại nghịch ñảo a’ với 1 ≤ a’ ≤ p – 1 sao cho aa’ ≡ 1 (mod p). Theo mệnh ñề trên

chỉ có 2 số 1 và p – 1 là nghịch ñảo modulo p của chính nó. Như vậy, ta có thể

nhóm các số 2, 3,…, p – 2 thành (p – 3)/2 cặp mà tích của chúng ñồng dư 1

modulo p. ⇒ 2.3. …(p – 3)(p – 2) ≡ 1 mod p

⇒ (p – 1)! ≡ 1(p – 1) ≡ –1 mod p.

Page 176: Chuyên đề hội thảo một số trường chuyên về dãy số

HỘ

HỘI CÁC TRƯỜNG THPT CHUYÊN KHU VỰC DUYÊN HẢI VÀ ðỒNG BẰNG BẮC BỘ HỘI THẢO KHOA HỌC LẦN THỨ IV

176

II. Bài tập vận dụng.

Bài 1: Chứng mình rằng với mọi số tự nhiên n ≥ 1 thì n85 23+ chia hết cho 24.

Lời giải:

Ta có: n n 1 n 18 8.8 4.85 5 25

− −

= =

Mà 25 ≡1(mod 24) ⇒ ( )n 14.825 1 mod 24

⇒ ( )n85 23 0 mod 24+ ≡ hay ( )n85 23 24+ M (ñpcm)

Bài 2: Cho a, b, c là các số nguyên dương. Chứng minh rằng nếu a2 + b2 + c2

chia hết cho 9 thì ít nhất một trong các số: a2 – b2; b2 – c2; c2 – a2 chia hết cho 9.

Lời giải:

Bài toán tương ñương: chứng minh tồn tại ít nhất 2 trong 3 số a2, b2, c2 có cùng

số dư khi chia cho 9.

NX: khi chia n2 cho 9 số dư có thể là: 0, 1, 4, 7

giả sử: ( )21a r mod9≡ ; ( )2

2b r mod9≡ ; ( )23c r mod9≡

⇒ a2 + b2 + c2 ≡ r1 + r2 + r3 (mod 9) ⇒ r1 + r2 + r3 ≡ 0 (mod 9)

Do r1, r2, r3 ∈ { }0;1;4;7 ⇒ tồn tại ít nhất hai số bằng nhau.

Vậy ít nhất một trong các số: a2 – b2; b2 – c2; c2 – a2 chia hết cho 9. (ñpcm)

Bài 3: Cho p là số nguyên tố lớn hơn 7. Chứng minh rằng: (3p – 2p – 1)M42p

Lời giải:

Ta chứng minh: 3p – 2p – 1 chia hết cho 2; 3; 7 và p

Thật vậy: 3p – 2p – 1 = (3p – 3) – (2p – 2) ≡ 0 (mod p)

3p – 2p – 1 = (3p – 1) – 2p ≡ 0 (mod 2)

Do p nguyên tố và p > 7 nên p lẻ ⇒ 3p – 2p – 1 ≡ -(-1)p – 1 ≡ 0 (mod 3)

Page 177: Chuyên đề hội thảo một số trường chuyên về dãy số

HỘ

HỘI CÁC TRƯỜNG THPT CHUYÊN KHU VỰC DUYÊN HẢI VÀ ðỒNG BẰNG BẮC BỘ HỘI THẢO KHOA HỌC LẦN THỨ IV

177

Ta có: 3p – 2p – 1 = 3.3p – 1 – 2p – 1 = 3.p 1

29−

- 2p – 1 ≡ 3.p 1

22−

- 2p – 1 (mod 7)

≡ ( )p 1 p 1

p2 22 2 2 1 mod7+ −

+ − −

Do (p, 3) = 1 nên p = 3k + 1 hoặc p = 3k + 2 với k ∈ �

Nếu p = 3k + 1 thì ( )p 1 p 1 p 1 p 1

p k p p2 2 2 22 2 2 1 8 1 2 2 2 2 mod7+ − + +

+ − − = − + − ≡ −

= ( ) ( )p 1 p 1 p 1

k2 2 22 2 1 2 8 1 0 mod7+ − +

− = − ≡

Nều p = 3k + 2, chứng minh tương tự.

Vậy (3p – 2p – 1) chia hết cho 42p (ñpcm).

Bài 4: Chứng minh rằng với mọi số nguyên tố p, tồn tại vô số nguyên dương n

thoả mãn: (2n – n)Mp

Lời giải:

Nếu p = 2 thì mọi n chẵn ñều thoả mãn ñiều kiện ñề bài.

Nều p > 2, khi ñó theo ñịnh lý Fermat, ta có: ( ) ( )m p 12 1 mod p−≡ với m ∈ *�

Lấy n = m(p – 1) với m ≡ -1 (mod p)

Khi ñó: n = m(p – 1) ≡ 1 (mod p) và 2n – n ≡ 2n – 1 ≡ 0 (mod p).

Do có vô số số nguyên dương m sao cho m ≡ -1 (mod p) nên tồn tại vô số số

nguyên dương n thoả mãn (2n – n)Mp (ñpcm).

Bài 5: Chứng minh rằng với mọi số nguyên m ñều tồn tại một số nguyên n sao

cho: n3 – 11n2 – 87n + m chia hết cho 191

Lời giải

ðặt P(x) = x3 – 11x2 – 87x + m.

Ta chứng minh tồn tại a, b nguyên ñể P(x) ≡ (x + a)3 + b (mod 191)

Page 178: Chuyên đề hội thảo một số trường chuyên về dãy số

HỘ

HỘI CÁC TRƯỜNG THPT CHUYÊN KHU VỰC DUYÊN HẢI VÀ ðỒNG BẰNG BẮC BỘ HỘI THẢO KHOA HỌC LẦN THỨ IV

178

⇔ x3 + 3ax2 + 3a2x + a3 + b ≡ x3 – 11x2 – 87x + m (mod 191)

Chọn a nguyên sao cho: 3a ≡ -11 (mod 191) ⇔ a ≡ 60 (mod 191).

⇒ 3a2 ≡ -87 (mod 191)

⇒ với mọi m chỉ cần chọn b ≡ m – a3 (mod 191).

Khi ñó, với i, j nguyên mà P(i) ≡ P(j) (mod 191)

⇔ (i + a)3 ≡ (j + a)3 (mod 191)

⇔ (i + a)3.63(j + a)2 ≡ (j + a)191 (mod 191)

≡ j + a (mod 191)

⇒ (j + a)2 ≡ (i + a)189(j + a)3 (mod 191) ≡ (i + a)192 (mod 191)

≡ (i + a)2 (mod 191)

⇒ (i + a)3.63(j + a)2 ≡ (i + a)189(i + a)2 (mod 191) ≡ (i + a) (mod 191)

⇒ P(i) ≡ P(j) (mod 191) ⇔ i ≡ j (mod 191)

Từ ñó tập {P(1), P(2), …, P(191)} có 191 số dư khác nhau khi chia cho 191

Do ño tồn tại n ∈ {1, 2, …, n} sao cho P(n) chia hết cho 191 (ñpcm)

Bài 6: Cho hai số nguyên a, b thoả mãn: 24a2 + 1 = b2. Chứng minh rằng có một

và chỉ một trong hai số a, b chia hết cho 5.

Lời giải:

Do b2 – 24a2 = 1 không chia hết cho 5 nên hai số a và b không thể cùng chia hết

cho 5

Giải sử a và b cùng không chia hết cho 5.

Theo ñịnh lý Fermat, ta có: a4 ( )1 mod5≡ ; b4 ( )1 mod5≡

Do ñó: ( )( ) ( )2 2 2 2 4 4a b a b a b 0 mod5+ − = − ≡

Page 179: Chuyên đề hội thảo một số trường chuyên về dãy số

HỘ

HỘI CÁC TRƯỜNG THPT CHUYÊN KHU VỰC DUYÊN HẢI VÀ ðỒNG BẰNG BẮC BỘ HỘI THẢO KHOA HỌC LẦN THỨ IV

179

Nếu ( )2 2a b 0 mod5+ ≡ thì 25a2 + 1 = a2 = b2 ≡ 0 (mod 5) (vô lí)

⇒ a2 – b2 ≡ 0 (mod 5) ⇔ 23a2 + 1 = b2 – a2 ≡ 0 (mod 5)

Vì (a, 5) = 1 nên ( )a 1 mod5≡ ± hoặc ( )a 2 mod5≡ ±

Nếu ( )a 1 mod5≡ ± thì ( ) ( )220 23a 1 23 1 1 1 mod5≡ + ≡ ± + ≡ − (vô lí)

Nếu ( )a 2 mod5≡ ± thì ( ) ( )220 23a 1 23 2 1 3 mod5≡ + ≡ ± + ≡ (vô lí)

⇒ ñiều giả sử sai. Vậy có ñúng 1 trong hai số a và b chia hết cho 5.

Bài 7: Tìm tât cả các số nguyên tố p, q sao cho (5p – 2p)(5q – 2q) chia hết cho pq.

Lời giải:

NX: p và q khác 2 và 5.

Không mất tính tổng quát, giả sử p ≥ q

Do p, q nguyên tố nên (5p – 2p)(5q – 2q) ( )

( )

p p

q q

5 2 ppq

5 2 p

−⇔ −

MM

M

Nếu (5p – 2p)Mp. Theo ñịnh lý Fermat, ta có: ( )p5 5 mod p≡ và ( )p2 2 mod p≡

⇒ 0 ≡ 5p – 2p ≡ 3 (mod p) ⇒ p = 3

Nếu (5p – 2p)Mp

Do (p – 1, q) = 1 nên tồn tại u, v ∈ *� : -(p – 1)u + qv = 1

Ta có: ( ) ( )q q vq vq5 2 0 mod p 5 2 0 mod p− ≡ ⇔ − ≡

⇔ ( ) ( ) ( )1 p 1 u 1 p 1 u5 2 0 mod p+ − + −− ≡

⇔ 3 ≡ 0 (mod p) ⇔ p = 3.

Với p = 3, ta có: (53 – 23)(5q – 2q) 3qM ⇒ 39(5q – 2q) qM ⇒ 39 qM ⇒ q = 3 hoặc

q = 13.

Page 180: Chuyên đề hội thảo một số trường chuyên về dãy số

HỘ

HỘI CÁC TRƯỜNG THPT CHUYÊN KHU VỰC DUYÊN HẢI VÀ ðỒNG BẰNG BẮC BỘ HỘI THẢO KHOA HỌC LẦN THỨ IV

180

Vậy các cặp số (p, q) cần tìm là: (3, 3); (3, 13); (13, 3).

Bài 8: Cho dãy số (an) ñược xác ñịnh:

a1 = 1; an = an – 1 + n

2

a

với mọi n ≥ 2.

Chứng minh rằng dãy số (an) có vô số số hạng chia hết cho 7.

Lời giải.

Giả sử trong dãy số trên có hữu hạn số hạng chia hết cho 7.

Gọi ak là số hạng cuối cùng của dãy trên chia hết cho 7.

Ta có: a2k + 1 = a2k + ak; a2k = a2k – 1 + a2k

Do ñó: a2k + 1 ≡ a2k ≡ a2k – 1 ≡ x (mod 7) với x không chia hết cho 7.

Mặt khác: a4k – 2 = a4k – 3 + a2k – 1 ≡ a4k – 3 + x (mod 7)

a4k – 1 = a4k – 2 + a2k – 1 ≡ a4k – 2 + x (mod 7) ≡ a4k – 3 + 2x (mod 7)

a4k = a4k – 1 + a2k ≡ a4k – 3 + 3x (mod 7)

a4k + 1 = a4k + a2k ≡ a4k – 3 + 4x (mod 7)

a4k + 2 = a4k + 1 + a2k + 1 ≡ a4k – 3 + 5x (mod 7)

a4k + 3 = a4k + 2 + a2k + 1 ≡ a4k – 3 + 6x (mod 7)

Ta có: a4k – 3 và x không chia hết cho 7 nên các số a4k – 3 + ix (i = 1, 2, .., 6) phải

có số chia hết cho 7 nên trong các số a4k – 3 + i (i = 1, 2, …, 6) phải chia hết cho 7.

⇒ mâu thuẫn với ak là số cuối cùng chia hết cho 7.

Vậy có vô số số trong dãy trên chia hết cho 7.

Bài tập:

Bài 1: (Dự tuyển IMO 2006) Chứng minh rằng với mọi số nguyên dương n luôn

tồn tại số nguyên m sao cho 2m + m chia hết cho n

Page 181: Chuyên đề hội thảo một số trường chuyên về dãy số

HỘ

HỘI CÁC TRƯỜNG THPT CHUYÊN KHU VỰC DUYÊN HẢI VÀ ðỒNG BẰNG BẮC BỘ HỘI THẢO KHOA HỌC LẦN THỨ IV

181

Bài 2: (Tạp chí Animath của Pháp năm 2006) Chứng minh rằng tồn tại vô số số

nguyên dương n sao cho ước nguyên tố lớn nhất của n2 + 1 lớn hơn 2n

Bài 3: Cho p là một số nguyên tố. Chứng minh rằng tồn tại một số nguyên tố q

sao cho với mọi số nguyên n, số np – p không chia hết cho q.

Bài 4: (VMO 2001) Cho n là số nguyên dương và a, b là các số nguyên tố cùng

nhau lớn hơn 1. Giả sử p, q là hai ước lẻ lớn hơn 1 của n n6 6a b+ . Tìm số dư

trong phép chia của n n6 6a b+ cho 6.12n.

Bài 5: Cho n2k 2 1= + với n là số nguyên dương. Chứng minh rằng k là số

nguyên tố khi và chỉ khi k 1

23 1−

+ chia hết cho k.

Bài 6: Cho số nguyên n ≥ 5. Chứng minh rằng: ( )

( )n 1 !

n 1n

− −

M

Tài liệu tham khảo

1. Các bài giảng về số học - Nhà xuất bản ðHQG Hà Nội

2. Một số vấn ñề số học chọn lọc – Nhà xuất bản giáo dục

3. Tạp chí toán học và tuổi trẻ

4. Trang web: mathscope.org; math.vn…

Page 182: Chuyên đề hội thảo một số trường chuyên về dãy số

HỘ

HỘI CÁC TRƯỜNG THPT CHUYÊN KHU VỰC DUYÊN HẢI VÀ ðỒNG BẰNG BẮC BỘ HỘI THẢO KHOA HỌC LẦN THỨ IV

182

MỘT VÀI BIẾN ðỔI

VÀ ỨNG DỤNG ðƠN GIẢN TRONG SỐ HỌC Dương Châu Dinh

Trường THPT chuyên Lê Quý ðôn - Quảng Trị

Số học là một trong những bài toán khó trong các kỳ thi học sinh giỏi.

Học sinh ít ñược tiếp cận với dạng toán này và thường lúng túng trong quá trình

tìm hướng giải quyết.

Vì thế, tôi xin trình bày với bạn ñọc một vài vấn ñề sơ lược sau ñây.

1. ðịnh lý (Ferma - nhỏ)

Giả sử a Z∈ , P là số nguyên tố thoả mãn (a ; P) = 1. Khi ñó p 1a 1− ≡ (modp) Chứng minh: Xét các số: a, 2a, …, (P -1)a

Giả sử tồn tại ( ){ } ( )i, J 1; 2;... p 1 , i J∈ − ≠ ñể ia Ja≡ (modp)

( )i J a 0⇒ − ≡ (modp)

i J 0⇒ − ≡ (modp), vì (a; P) = 1 0 i J P≤ − < i J 0⇒ − = ⇒ i J= (vô lý)

Vì thế, khi chia các số a, 2a, …, (P -1)a cho P có các dư số ñôi một khác nhau và khác 0.

( ) ( )a.2a... P 1 a P 1 !⇒ − ≡ − (modp)

( ) ( )P 1P 1 !a P 1 !−⇒ − ≡ − (modp) P 1a 1−⇒ ≡ (modp), vì: ( )( )P; P 1 ! 1− =

2. Bổ ñề Cho ia Z, i 0,n 1∈ = − ( )n 2≥

Giả sử phương trình: n n 1n 1 1 0x a x ... a x a 0−

−+ + + + = (1) có nghiệm hữu tỉ

0x . Khi ñó 0x Z∈ .

Lời giải:

(1) có nghiệm 0

Px

q= , ( )( )p,q Z,q 0, p,q 1∈ > =

n n 1

n 1 1 0n n 1

nn 1 n 2 n 1

n 1 1 0

p p pa ... a a 0

q q q

pa p ... a p.q a q 0

q

− −

− − −

⇒ + + + + =

⇒ + + + + =

Page 183: Chuyên đề hội thảo một số trường chuyên về dãy số

HỘ

HỘI CÁC TRƯỜNG THPT CHUYÊN KHU VỰC DUYÊN HẢI VÀ ðỒNG BẰNG BẮC BỘ HỘI THẢO KHOA HỌC LẦN THỨ IV

183

npZ

q⇒ ∈

Giả sử trái lại rằng q 1> , thì tồn tại số nguyên tố a

Sao cho ( )* nq ka k N a / p= ∈ ⇒

a: nguyên tố

vô lý; vì a là số nguyên tố.

Vì vậy: 0

0 q 1q 1 x p Z

q Z

< ≤ ⇒ = ⇒ = ∈

∈ (ñpcm).

3. Chú ý: a,b Z ; b 0∈ >

*

a 0

a 0a b aN

a bb

=

=⇒ ∈ ⇒ ≥

M

4. Ứng dụng: Ví dụ 1. Cho a Z∈ .

CMR 2a 1+ không có ước nguyên tố dạng 4k 3+ , ( )k N∈

Giải: P 4k 3= + . Số nguyên tố ( )k N∈

Giả sử trái lại rằng: 2a 1 0+ ≡ (modp) 2 P 1a P a P a 1−⇒ ⇒ ⇒ ≡M M (modp) , (Ferma – nhỏ)

Mặt khác: 2a 1≡ − (modp) 4k 2a 1+⇒ ≡ − (modp) P 1a 1−⇒ ≡ − (modp) 2 0⇒ ≡ (modp) , mâu thuẩn P 3≥ ⇒ (ñpcm)

Ví dụ 2. Cho n Z, n 1∈ > thoả mãn: n3 1 n− M CMR: n là số chẵn

Giải:

n Z

n 2n 1

∈⇒ ≥

>. Gọi P là ước nguyên tố bé nhất của n. Do n3 1 P P 3− ⇒ ≠M

( ) P 1P,3 1 3 1−⇒ = ⇒ ≡ (modp) (Ferma - nhỏ)

Gọi h là số nguyên dương bé nhất sao cho: 13 ≡h (modp) Giả sử: n kh r= + (0 r h)≤ < ; Ta có n kh r r3 1 3 1 3 1+− = − ≡ − (modp)

r3 1 0⇒ − ≡ (modp) ; do cách chọn h nên r = 0 n h⇒ M ; lập luận tương tự ( )P 1 h− M

Nếu h > 1 thì h có ước nguyên tố q và ( )P 1 q− M p q⇒ >

⇒a / p

a / q

a / (p,q) 1⇒ =

Page 184: Chuyên đề hội thảo một số trường chuyên về dãy số

HỘ

HỘI CÁC TRƯỜNG THPT CHUYÊN KHU VỰC DUYÊN HẢI VÀ ðỒNG BẰNG BẮC BỘ HỘI THẢO KHOA HỌC LẦN THỨ IV

184

q là ước nguyên tố của n, ñiều này mâu thuẩn với cách chọn p, vậy h = 1. 3 1⇒ ≡ (modp) 2 0⇒ ≡ (modp)

p số nguyên tố P 2⇒ = Vì p = 2 là ước của n, nên n là số chẵn (ñpcm)

Ví dụ 3. Giải phương trình nghiệm nguyên

24(a x)(x b) b a y− − + − = (*) Trong ñó: a,b Z∈ và a b>

Giải: Giả sử phương trình (*) có nghiệm nguyên (x, y) , khi ñó: (*) [ ][ ] ( )

24(a x) 1 4(x b) 1 2y 1⇔ − − − − = +

( )4 a x 1 4(x b) 1 4(a b) 2 0− − + − − = − − > , do:

a b 0a b 1

a,b Z

− > ⇒ − ≥

4(a x) 1⇒ − − có dạng ( )4m 3 m N+ ∈

4(a x) 1⇒ − − có ước nguyen tố dạng 4k 3+ ( )k N∈

Mà ( )2

2y 1+ không có ước dạng này (vô lý); (Theo VD 1)

Vậy, phương trình (*) không có nghiệm nguyên.

Ví dụ 4. Tìm nghiệm nguyên của hệ: 3 2

2

4x y 16 (1)

z 2yz 3 (2)

+ =

+ =

Giải: Giả sử hệ có nghiệm nguyên (x, y,z) , Từ (1) ⇒ y chẵn 1y 2y⇒ = , khi

ñó: ( )3 2

21 312

1

x y 4x y z 7

z 2y z 3

+ =⇒ + + =

+ =

( ) ( ) ( )2 2

1y z 1 2 x x 1 3 ⇒ + + = − + +

x chẵn – không xảy ra.

⇒ x lẻ ( )2

x 1 3⇒ + + có ước nguyên tố dạng 4k 3+ ( )k N∈

mà ( )2

1y z 1+ + không có ước dạng này (vô lý). (Theo ví dụ 1)

Vậy, hệ ñã cho không có nghiệm nguyên.

Ví dụ 5. Cho { }a,b Z \ 1∈ − sao cho 3 3a 1 b 1

Zb 1 a 1

+ ++ ∈

+ +. CMR: ( ) ( )2010a 1 b 1− +M

Giải:

Page 185: Chuyên đề hội thảo một số trường chuyên về dãy số

HỘ

HỘI CÁC TRƯỜNG THPT CHUYÊN KHU VỰC DUYÊN HẢI VÀ ðỒNG BẰNG BẮC BỘ HỘI THẢO KHOA HỌC LẦN THỨ IV

185

3 3

3 3

a 1 b 1m Z

b 1 a 1

a 1 b 1n Z

b 1 a 1

+ ++ = ∈

+ +

+ += ∈

+ +

3a 1

b 1

+⇒

+ và

3b 1

a 1

+

+ là các nghiệm của phương trình

2x mx n 0− + = (*)

Theo bổ ñề, nếu (*) có nghiệm hửu tỉ 0x thì 0x Z∈

( ) ( )3

3a 1Z a 1 b 1

b 1

+⇒ ∈ ⇒ + +

+M

( )3352010 6 6

6 3

a 1 a 1 a 1

a 1 a 1

− = − − ⇒

− +

M

M( ) ( )2010a 1 b 1− +M (ñpcm)

Ví dụ 6. Cho x, y là các số nguyên, x 1≠ − và y 1≠ −

Sao cho: 4 4x 1 y 1

y 1 x 1

− −+

+ + là số nguyên. Chứng minh rằng 4 44x y 1− chia hết

cho x 1+ Giải:

Lập luận tương tự trên, ta có: 4y 1

Zx 1

−∈

+ 4y 1 (x 1)⇒ − +M

( ) ( )4 44 4 44 4x y 1 x y 1 x 1 x 1⇒ − = − + − +M

(do ( )44 4 44y 1 y 1) y 1 x 1 ,− − ⇒ − +M M và 4x 1 x 1)− +M

Ví dụ 7. Tìm các số x, y,z nguyên dương sao cho ( )( )2xy 1 z x 1 y 1− = − − (1)

Giải: x, y,z N∈ , 2xy 1− là số lẽ ( )( )z x 1 y 1⇒ − − là số lẻ

x, y⇒ là số chẵn, còn z là số lẻ Không mất tính tổng quát, coi rằng: x y≥

ðặt: a x 1= − ; b y 1= − (a ,b lẻ và )a b≥

PT (1) trở thành:

( )( ) *2 2 12 a 1 b 1 1 zab N

a b ab+ + − = ⇒ + + ∈ (2)

Nếu a b= thì *2

4a 1N

a

+∈

2*

2

4a a 14 N

a a

+⇒ = + ∈

*1N a 1

a⇒ ∈ ⇒ = ( )b 1=

Tìm ñược ( ) ( )x, y,z 2;2;7=

Page 186: Chuyên đề hội thảo một số trường chuyên về dãy số

HỘ

HỘI CÁC TRƯỜNG THPT CHUYÊN KHU VỰC DUYÊN HẢI VÀ ðỒNG BẰNG BẮC BỘ HỘI THẢO KHOA HỌC LẦN THỨ IV

186

Nếu a ≥ b ≥ 1

* Xét: 2 2 1 2 2 1 19

b 4 a 5 0a b ab 5 4 20 20

≥ ⇒ ≥ ⇒ ≤ + + ≤ + + =

*2 2 1N

a b ab⇒ + + ∉

Vậy, b 4< , b lẻ nên { }b 1;3∈

* Xét *3b 1 N

a= ⇒ ∈ (do: (2))

a 1> do ñó: a 3= Tìm ñược ( ) ( )x, y,z 2;4;5=

* Xét b 3= , từ (2) *7 2N

3a 3⇒ + ∈ ; a 3 a 7> ⇒ =

Tìm ñược: ( ) ( )x;y;z 4,8,3=

Do vai trò x, y như nhau nên pt (1) có 5 nghiệm

( ) ( ) ( ) ( ) ( ) ( ){ }x;y;z 2;2;7 ; 2;4;5 ; 4;2;5 ; 4;8;3 ; 8;4;3∈

Ví dụ 8. Tìm tất cả các bộ ba số nguyên lớn hơn 1 có tính chất: Khi chia tích 2 số cho số thứ ba ta ñược dư là 1.

Giải: Giả sử tìm ñược bộ ba số ( )a,b,c thoả mãn ñiều kiện bài toán, không mất

tính tổng quát, coi rằng: 2 a b c≤ ≤ ≤ (1) ( )ab 1 c− M ,( )bc 1 a− M . ( )ca 1 b− M

( ) *1 1 1 1ab bc ca 1 abc N

a b c abc⇒ + + − ⇒ + + − ∈M

Ta có: *3 1 1 1 1 1 1 1

N2 a b c a b c abc1 1 1 1

1 (1)a b c abc

≥ + + > + + − ∈

+ + − =

* 1 1 1

a 3 1a b c

≥ ⇒ + + ≤ (loại)

a 2= , từ ñó ta suy ra: 1 1 1 1

b c 2bc 2+ − =

⇒ 2b 1 3

c 2b 2 b 2

−= = +

− −

⇒ (b 2) 3 b 3,c 5

b 5 (loai)c b 2

− = =⇒ => ≥

Page 187: Chuyên đề hội thảo một số trường chuyên về dãy số

HỘ

HỘI CÁC TRƯỜNG THPT CHUYÊN KHU VỰC DUYÊN HẢI VÀ ðỒNG BẰNG BẮC BỘ HỘI THẢO KHOA HỌC LẦN THỨ IV

187

Tìm ñược (a;b;c) (2; 3; 5)= và các hoán vị của nó

Ví dụ 9. Tìm tất cả các cặp số tự nhiên ),( ba sao cho 2

2

a b

b a

+

− và

2

2

b a

a b

+

− là các

số nguyên. Giải: Do vai trò (a,b) như nhau nên coi rằng: a b≤

* a b

(a 0)

=

≠ ⇒

a 1Z

a 1

+∈

− ⇒ { }

2Z

a 2;3a 1a N

⇒ ∈− ∈

Tìm ñược: { }(a,b) (2;2),(3;3)∈

*Xét 0 a b≤ < ⇒ (1)

a 1 b+ ≤ , 2a b<

⇒2

2

a b

b a

+

*N∈ ⇒ 2 2a b b a+ ≥ −

⇒ (a b)(a b 1) 0 a 1 b+ − + ≥ ⇒ + ≥

Từ (1) suy ra: 2

(2)

2

(a 1) ab a 1 Z

a a 1

+ += + ⇒ ∈

− −

⇒ 2

4a 2Z

a a 1

+∈

− −

2

2a 1Z

a a 1

+⇒ ∈

− − ; ( )2a a 1; 2 1− − =

Xét: a = 0 ⇒ b = 1 (thoả mãn) a = 1 ⇒ b = 2 ( thoả mãn)

Xét: *2

2a 1a 2 N

a a 1

+≥ ⇒ ∈

− − 22a 1 a a 1⇒ + ≥ − −

2a 3a 2 0⇒ − − ≤ 2 a 4⇒ ≤ < ⇒ { }a 2;3∈ , (do : a N)∈

* a = 3 (loại) * a = 2 ⇒ b = 3 (thoả mãn) Thử lại, ñi ñến kết quả:

( ) ( ) ( ) ( ) ( ) ( ){ }a;b 2;2 , 3;3 , 0;1 , 1;2 , 2;3∈ và các hoán vị của nó.

Ví dụ 10. Tìm tất cả các bộ ba số nguyên ( )a,b,c sao cho: 1 a b c< < <

và số ( )( )( )a 1 b 1 c 1− − − là ước của abc 1− .

Giải: ðặt : a 1 m, b 1 n, c 1 p− = − = − = (0 m n p)< < <

*1 1 1 1 1 1A N

m n p mn mp np= + + + + + ∈ (1)

Page 188: Chuyên đề hội thảo một số trường chuyên về dãy số

HỘ

HỘI CÁC TRƯỜNG THPT CHUYÊN KHU VỰC DUYÊN HẢI VÀ ðỒNG BẰNG BẮC BỘ HỘI THẢO KHOA HỌC LẦN THỨ IV

188

x y z m

xy yz zx n

xyz p

+ + =

+ + = =

Xét: m 3 n 4; p 5 A 1≥ ⇒ ≥ ≥ ⇒ < , nên *A N∉

{ }m 1;2⇒ ∈

Xét: ( )(1)

*2 2 1m 1 n 2;p 3 B N

n p np= ≥ ≥ ⇒ = + + ∈ (2)

(2)*5

n 2 N2p

= ⇒ ∈ (không xảy ra)

*2 7n 3 N 4 p 7

3 3p= ⇒ + ∈ ⇒ ≤ ≤

Thử trực tiếp, chỉ có: p = 7 (thoả mãn) Tìm ñược ( ) ( )a;b;c 2;4;8=

* n 4≥ p 5⇒ ≥ 2 2 1

1n p np

⇒ + + < *B N⇒ ∉

Xét m 2= , (n 3,p 4)≥ ≥ (1)

⇒ *1 3 3 1C N

2 2n 2p np= + + + ∈

n 3= (3)

⇒ *11N

6p∈ (không xảy ra)

(3)*7 7

n 4 p 5 N 5 p 148 4p

= ⇒ ≥ ⇒ + ∈ ⇒ ≤ ≤

Thử trực tiếp, chỉ có 14=p thoả mãn Tìm ñược (a;b;c) (3;5;15)=

* (3)

*8p 17n 5 N

10

+= ⇒ ∈ (không xảy ra)

* 81

n 6 p 7 C 184

≥ ⇒ ≥ ⇒ ≤ <

*C N⇒ ∉ Thử lại, ñi ñến kết luận: ( ) ( ) ( ){ }a;b;c 2;4;8 , 3;5;15∈

Ví dụ 11 Xác ñịnh các bộ số hữu tỉ dương ( )x;y;z sao cho

x y z,+ + 1 1 1

x y z+ + và xyz là các số tự nhiên.

Giải: Từ ñiều kiện của bài, ta có:

*m,n,p N⇒ ∈

x, y⇒ và z là các nghiệm hữu tỉ của phương trình: 3 2t mt nt p 0− + − = *x, y,z N⇒ ∈ (Theo bổ ñề)

Page 189: Chuyên đề hội thảo một số trường chuyên về dãy số

HỘ

HỘI CÁC TRƯỜNG THPT CHUYÊN KHU VỰC DUYÊN HẢI VÀ ðỒNG BẰNG BẮC BỘ HỘI THẢO KHOA HỌC LẦN THỨ IV

189

Không giảm tổng quát, coi rằng: zyx ≤≤≤1

*1 1 13 N

x y z⇒ ≥ + + ∈ { }

1 1 11;2;3

x y z⇒ + + ∈

Xét: 1 1 1

3 x y z 1x y z

+ + = ⇒ = = =

Xét: 1 1 1 3

2x y z x

+ + = ≤ ⇒ *

31 x

2

x N

≤ ≤

⇒∈

1 1

x 1 1y z

= ⇒ + =

1z 1 y z 2

y 1⇒ = + ⇒ = =

Xét: 1 1 1 3

1 1 x 3x y z x

+ + = ≤ ⇒ ≤ ≤

x 3 y z 3

1 1 1x 2

y z 2

= ⇒ = =

= ⇒ + =

4z 2

y 2⇒ = +

( ){ }

y 2 / 4y 3;4;6

2 y z

− ⇒ ∈

≤ ≤

36 =⇒= zy (loại)

( ) ( ) ( ) ( ) ( ) ( ){ }x;y;z 1;1;1 , 3;3;3 , 1;2;2 , 2;4;4 , 2;3;6= và các hoán vị của nó.

Ví dụ 12. Một tứ giác lồi có 4 cạnh là 4 số tự nhiên, sao cho tổng 3 số bất kỳ trong chúng chia hết cho số còn lại. CMR tứ giác ñó có 2 cạnh bằng nhau.

Giải: Gọi ñộ dài các cạnh của tứ giác là *a,b,c,d N∈

Giả sử trái lại rằng: 1 a b c d≤ < < < theo ñiều kiện của bài. Suy ra: (1)a b c d ma nb pc qd+ + + = = = =

(Với: *m,n,p,q N ,q p n m)∈ < < < a b c d+ + > qd 2d⇒ > q 3,p 4,n 5,m 6⇒ ≥ ≥ ≥ ≥

1 1 1 1 191

m n p q 20⇒ + + + ≤ < ⇒

Từ (1), suy ra: 1 1 1 1

1m n p q

⇒ + + + =

Vậy, ñiều khẳng ñịnh bài toán ñược chứng minh. 5. Một số bài tập

mâu thuẫn

Page 190: Chuyên đề hội thảo một số trường chuyên về dãy số

HỘ

HỘI CÁC TRƯỜNG THPT CHUYÊN KHU VỰC DUYÊN HẢI VÀ ðỒNG BẰNG BẮC BỘ HỘI THẢO KHOA HỌC LẦN THỨ IV

190

Bài 1. Xác ñịnh các cặp số tự nhiên (x;y) sao cho: x 1

y

+ và

y 1

x

+ là các

số tự nhiên. Bài 2. Giải các phương trình nghiệm nguyên dương a. 2 3x y 7− =

b. 24xy x y z− − =

Bài 3. Cho a ∈ Z . CMR a2 + a + 1 không có ước nguyên tố dạng 3k + 1 Bài 4. Tìm các số nguyên tố p sao hệ sau có nghiệm nguyên dương

2

2 2

2x p 1

2y p 1

= +

= +

Bài 5. Tìm nghiệm nguyên dương của phương trình: x y z xyz+ + =

Bài 6. Xác ñịnh các bộ ba số nguyên dương ( )x;y;z sao cho: 1 1 1

x y z+ +

là số tự nhiên

Bài 7. Xác ñịnh các cặp số nguyên dương ( )x;y thoả mãn 3x x

xy 1

+

− là số tự

nhiên. Bài 8. Ví dụ 3: Cho p = 4k + 3 (k N)∈ là số nguyên tố. CMR

x, y Z∀ ∈ thoả mãn: 2 2x y p+ M thì x pM và y pM

Vậy, ñiều khẳng ñịnh bài toán ñược chứng minh. Bài 9. Tìm tất cả các bộ số nguyên dương ( )x;y;z;a;b;c thoả mãn hệ

Phương trình: a b c xyz

x y z abc

+ + =

+ + =

Bài 10. Giả sử *a,b N∈ sao cho ( )

b

*a 3 1N

3a

+ +∈ . CMR

( )b

a 3 1

3a

+ + là

một số lẻ. Bài 11. Cho dãy số ( )na ñược xác ñịnh bởi: 1 2 3a 0;a 38,a 90= = = − và

n 1 n 1 n 2a 19a 30a , n 3+ − −= − ∀ ≥ . CMR 2011a chia hết cho 2011.

Bài 12. Chứng minh rằng nếu p là ước nguyên tố của n22 1+ ( )n 2≥ thì

p 1− chia hết cho n 22 + .

TÀI LIỆU THAM KHẢO - Tạp chí toán học & Tuổi trẻ - Tài liệu tập huấn (Phát triển chuyên môn GV trường THPT Chuyên).

PHƯƠNG PHÁP GEN

Page 191: Chuyên đề hội thảo một số trường chuyên về dãy số

HỘ

HỘI CÁC TRƯỜNG THPT CHUYÊN KHU VỰC DUYÊN HẢI VÀ ðỒNG BẰNG BẮC BỘ HỘI THẢO KHOA HỌC LẦN THỨ IV

191

Phạm Quang Thăng Trường THPT Chuyên Thái Bình

Nếu từ một nghiệm của phương trình ta tìm ra quy tắc ñể xây dựng một

nghiệm mới thì quy tắc ñó gọi là quy tắc Gen. Phương pháp Gen là phương pháp dựa trên quy tắc Gen ñể tìm các nghiệm của phương trình ñã cho từ các nghiệm cơ sở. ðể minh hoạ cho phương pháp này ta xét hai phương trình ñó là phương trình Pell và phương trình Markov

I. Phương trình Pell Là phương trình có dạng 2 2 1x Dy− = trong ñó D là số nguyên dương

không chính phương. ðặt { }2 2( ; ) / 1S x y N N x Dy

+ += ∈ × − = là tập hợp tất cả các nghiệm nguyên

dương của phương trình Pell thì ta có ñịnh lý sau: ðịnh lý: Nếu D là số nguyên dương không chính phương thì S φ≠ , tức là phương

trình Pell có nghiệm không tầm thường Vì S φ≠ gọi (x; y), (x’; y’) là nghiệm thuộc S.

Nếu x > x’ thì y > y’ do ñó ta có thể sắp xếp S như là một tập hợp sắp thứ tự tốt, do ñó S có phần tử nhỏ nhất mà ta kí hiệu là (a; b). Ta gọi (a; b) là nghiệm cơ sở của phương trình Pell. Sử dụng phương pháp Gen ñể chứng minh ñịnh lý sau ñây: ðịnh lý: Nếu (a; b) là nghiệm cơ sở của phương trình Pell và (x; y) là nghiệm tuỳ ý

thuộc S thì tồn tại số nguyên dương n sao cho ( )+ = +n

x y D a b D và

từ ñó suy ra mọi nghiệm của phương trình Pell có dạng:

( ) ( )

( ) ( )2

2

+ + − = + − − =

n n

n n

a b D a b Dx

a b D a b Dy

D

Chứng minh: Sử dụng phản chứng: giả sử kết luận của bài toán là sai do ñó tồn tại cặp

số (x; y) thuộc S mà +x y D không có dạng ( )+n

a b D . Vì (x; y) thuộc

S nên ta có x > a, y > b. ðặt :

Page 192: Chuyên đề hội thảo một số trường chuyên về dãy số

HỘ

HỘI CÁC TRƯỜNG THPT CHUYÊN KHU VỰC DUYÊN HẢI VÀ ðỒNG BẰNG BẮC BỘ HỘI THẢO KHOA HỌC LẦN THỨ IV

192

1 1 1

1 1 1

= − = + ⇔

= − = +

x ax Dby x ax Dby

y ay bx y bx ay

Ta có :

( ) ( )2 22 2

1 1

2 2 2 2( )( )

1

− = − − −

= − −

=

x Dy ax Dby D ay bx

a Db x Dy

Do ñó ( )1 1;x y cũng là nghiệm của phương trình 2 2 1− =x Dy .

Mặt khác : 2 2

1

2 22 2

1 2 2

1 1 1

1 1 1

1 1 0

1 1 01 1

( 1) 0

( 1) 0

= − = + + − >

−= − = + − + = >

+ + +

− = − + >

− = − + >

x ax Dby Db Dy Dby

y by ay bx y Db b Dy

y Db b Dy

x x a x Dby

x y a y bx

Vậy ta có :

11 1

1

0( ; )

0

> > ⇒ ∈

> >

x xx y S

y y

Do ñó tồn tại n sao cho : ( )1 1+ = +n

x y D a b D . Vậy:

( ) ( )( ) ( )1

1 1 1 1 1 1

+

+ = + + + = + + = +n

x y D ax Dby bx ay D x D y a b D a b D

(vô lý) Vậy giả sử là sai, do ñó tồn tại n sao cho:

( )

( ) ( )

( )

( ) ( )

( ) ( )

2 2 2 2( ) ( )

2

2

+ = +

⇒ − − = − −

⇒ − = −

+ + − =

⇒ + − − =

n

nn

n

n n

n n

x y D a b D

x Dy a b D a Db x y D

x y D a b D

a b D a b Dx

a b D a b Dy

D

II. Phương trình Markov Là phương trình có dạng:

2 2 21 2 1 2... ( )+ + + = ∗L n nx x x kx x x

Page 193: Chuyên đề hội thảo một số trường chuyên về dãy số

HỘ

HỘI CÁC TRƯỜNG THPT CHUYÊN KHU VỰC DUYÊN HẢI VÀ ðỒNG BẰNG BẮC BỘ HỘI THẢO KHOA HỌC LẦN THỨ IV

193

Với k, n nguyên dương. Xét phương trình bậc hai:

2 2 2 21 2 1 1 2 1( ) ... 0 ( )− −= − + + + + = ∗∗Ln nf x x kx x x x x x x

Giả sử phương trình (*) có nghiệm là 1 2( ; ; ; )n

x x xK thì n

x là nghiệm của

phương trình (**) do ñó (**) còn một nghiệm khác là 'nu x=

Theo ñịnh lý Viet ta có: 2 2 21 2 1

1 2 1... ( )−−

+ + += − = ∗∗

L nn n

n

x x xu kx x x x

x

Nên u nguyên dương, từ ñó ta thu ñược nghiệm '1 2( ; ; ; )K nx x x của phương

trình (*). III. Bài tập minh họa Bài 1: Tìm tất cả các số nguyên a sao cho phương trình 2 2 1+ + =x axy y có vô

số nghiệm nguyên phân biệt (x; y). Lời giải Ta thấy (1; -a) là nghiệm của phương trình 2 2 1+ + =x axy y (*)

Giả sử (x ; y) là nghiệm của phương trình (*), do ñó x là nghiệm của phương trình bậc hai ẩn X :

2 2 1 0+ + − =X ayX y

Theo ñịnh lý Vi-et phương trình này còn có nghiệm nguyên là –ay – x do ñó phương trình (*) có nghiệm (–ay – x; y). Tương tự phương trình (*) cũng có nghiệm (x; –ax – y)

• Nếu |a| > 2 thì với x, y khác không ta có: � Nếu ≥x y thì:

2 min( ; )− − ≥ − > − = ≥ax y a x y x x x x y

� Nếu ≤x y thì

2 min( ; )− − ≥ − > − = ≥ay x a y x y y y x y

Vậy ta có: min( ; ) min( ; )

min( ; ) min( ; )

− − >

− − >

ay x y x y

x ax y x y

ðặt 0 01;x y a= = − ta xây dựng ñược dãy nghiệm ( );n n

x y mà:

1 1min( ; ) min( ; )+ +

>n n n n

x y x y

Phương trình có vô số nghiệm • Nếu a = 2 ta có phương trình: 2( ) 1+ =x y phương trình này có vô số

nghiệm nguyên.

Page 194: Chuyên đề hội thảo một số trường chuyên về dãy số

HỘ

HỘI CÁC TRƯỜNG THPT CHUYÊN KHU VỰC DUYÊN HẢI VÀ ðỒNG BẰNG BẮC BỘ HỘI THẢO KHOA HỌC LẦN THỨ IV

194

• Nếu a = -2 ta có phương trình: 2( ) 1− =x y phương trình này có vô

số nghiệm nguyên. • Nếu a = 1 ta có phương trình :

2 2 21 ( ) 1 1 1+ = − ⇒ + = + ⇒ − < <x y xy x y xy xy suy ra 0, 1= = ±x y hoặc 0, 1= = ±y x (loại)

Vậy với 2≥a phương trình ñã cho có vô số nghiệm.

Bài 2: Tìm các cặp sắp thứ tự các số nguyên dương (m; n) sao cho mn – 1 chia hết 2 2+m n

Lời giải Ta có: 2 2 1+ −Mm n mn khi và chỉ khi tồn tại số k nguyên sao cho:

2 2

2 2

( 1)

0

+ = −

⇔ − + + =

m n k mn

m knm n k

Giả sử (m; n) là cặp số cần tìm với m + n nhỏ nhất, do vai trò của m, n là như nhau do ñó ta có thể giả sử ≥m n Xét phương trình bậc hai: 2 2( ) 0= − + + =f x x knx n k (*)

Phương trình (*) có nghiệm x = m do ñó nó có nghiệm nữa là x = u, theo

ñịnh lý Vi-et ta có: 2

*+= − = ⇒ ∈�

n ku kn m u

m

Vậy (u; n) cũng là một cặp nghiệm cần tìm. Do ñó + ≥ + ⇒ ≥ ≥u n m n u m n

Theo ñịnh lý về dấu tam thức bậc hai ta có 2 2 2 2 2( ) 0 ( 1) 2= − + + ≥ ⇒ − ≤f n n kn n k n k n

Nếu n > 1 ta có 2

2 2

2

23 2 ( 1) 2

1≤ < ⇒ ≤ + = − < ≤

nk mn m n k mn kmn mn

n

(vô lý)

Nếu n = 1 ta có { }2 1 2

1 2,31 1

+= = + + ∈ ⇔ ∈

− −�

mk m m

m m

Kết luận : Vậy (1 ; 2), (1 ; 3), (2 ; 1), (3 ; 1) là các cặp cần tìm.

Bài 3 : Chứng minh rằng nếu 2 2

1

+=

+

a bn

ab là số nguyên dương với a, b là các số

nguyên dương thì n là số chính phương. Lời giải Giả sử (a, b) là cặp số nguyên dương sao cho n là số nguyên, vì vai trò của a, b là như nhau do ñó ta có thể giả sử a nhỏ nhất và ≤a b . Xét phương trình bậc hai : 2 2( ) 0= − + + =f x x kax a k

Page 195: Chuyên đề hội thảo một số trường chuyên về dãy số

HỘ

HỘI CÁC TRƯỜNG THPT CHUYÊN KHU VỰC DUYÊN HẢI VÀ ðỒNG BẰNG BẮC BỘ HỘI THẢO KHOA HỌC LẦN THỨ IV

195

Phương trình này có nghiệm x = b nên có nghiệm nữa là 2 −

= − =a k

u na bb

Nếu na – b > 0 thì (na – b; b) cũng là cặp số thỏa mãn do ñó − ≥na b a Suy ra: 2 2≤ − = − ≤ −ab nab b a n ab n (vô lý) Vậy 0− ≤na b .

• Nếu b = na thì ta có n = a2 • Nếu b > na ta có :

2 2 2 21 ( )≥ + ≥ ⇒ = + − = + − ≥ +b na n n a b nab a b b na a n vô lý

Bài 4: Tìm tất cả các số nguyên dương n sao cho phương trình: (x + y + z)2 = nxyz có nghiệm nguyên dương.

Lời giải Giả sử (x ; y ; z) là nghiệm nguyên dương của phương trình, vì vai trò của x, y, z như nhau do ñó ta có thể giả sử : x + y + z nhỏ nhất và ≥ ≥x y z

Xét phương trình bậc hai ( )2 2 2( ) ( ) ( ) 0= + + − + + =f X X y z nyz X y z

Phương trình này có nghiệm X = x nên có nghiệm nữa là 2

2 ( )( )

+= + − − =

y zu y z nyz x

x

Suy ra u nguyên dương, do ñó (u; y; z) cũng là nghiệm của phương trình ñã cho Ta có + + ≥ + + ⇒ ≥ ≥u y z x y z u x y

Theo ñịnh lý về dấu tam thức bậc hai ta có: ( )2 2 2( ) ( ) ( ) 0= + + − + + ≥f y y y z nyz y y z

2 2

2

(2 ) 0

4 4 1

9

⇔ + − ≥

⇔ ≤ + +

⇒ ≤

y z ny z

nz y y

n

Nếu n = 9 ta có nghiệm x = y = z = 1 Nếu n = 8 ta có nghiệm x = y = z = 2 Nếu n = 6 ta có nghiệm x=1, y = 2, z = 3 Nếu n = 4 ta có nghiệm là x= y = 1, z = 2 Nếu n = 1 ta có nghiệm là x = y = z = 9 Nếu n = 7 ta có 2 2 29 (2 ) 7 0 9 / 7 1⇔ ≥ + − ≥ ⇒ ≤ ⇒ =y y z y z z z

Thay vào ta ñược bpt 23 4 1 0 1⇔ − − ≤ ⇒ =y y y

Thay vào phương trình ñầu ta ñược (x + 2)2 = 7x phương trình này vô nghiệm Vậy n = 7 loại

Page 196: Chuyên đề hội thảo một số trường chuyên về dãy số

HỘ

HỘI CÁC TRƯỜNG THPT CHUYÊN KHU VỰC DUYÊN HẢI VÀ ðỒNG BẰNG BẮC BỘ HỘI THẢO KHOA HỌC LẦN THỨ IV

196

Tương tự cho n = 5, n = 3, n = 2. Bài 5: Tồn tại hay không nghiệm của phương trình:

2 2 2 2 2 65+ + + + = −x y z u v xyzuv

Trong tập các số nguyên lớn hơn 2011? Lời giải Dễ thấy (1; 2; 3; 4; 5) là một nghiệm của phương trình Gọi (x; y; z; u; v) là nghiệm dương bất kì của phương trình, vì vai trò của x, y, z, u, v là như nhau nên ta có thể giả sử ≤ ≤ ≤ ≤x y z u v . Xét phương

trình bậc hai ẩn X: 2 2 2 2 2( ) 65 0= − + + + + − =f X X yzuvX y z u v

Phương trình này có một nghiệm bằng x, do ñó còn nghiệm nữa là: 2 2 2 2

1

+ + += − =

y z u vx yzuv x

x

Suy ra 1x nguyên dương; 1 >x x Vậy 1( ; ; ; ; )x y z u v là một nghiệm của phương trình.

Lại sắp xếp bộ nghiệm này và làm tương tự như vậy ta ñược một nghiệm khác mà giá trị của x; y; z; u; v ñều tăng lên. Như vậy phương trình ñã cho luôn có nghiệm trong tập hợp các số nguyên lớn hơn 2011.

Bài 6: Chứng minh rằng tồn tại số nguyên 2011≥m và m số nguyên dương phân biệt 1 2; ;K ma a a sao cho:

2 2

11

4==

− ∑∏m m

i i

ii

a a là số chính phương.

Lời giải Ta sẽ chỉ ra tồn tại 2011≥m và m số nguyên dương phân biệt 1 2; ;K

ma a a

là nghiệm của phương trình: 2

2 2

11 1

4 2== =

− = −

∑∏ ∏

m mm

i i i

ii i

a a a có nghiệm.

2

2 2

11 1

2

1 1

4 2

1 (*)

== =

= =

− = −

⇔ = −

∑∏ ∏

∑ ∏

m mm

i i i

ii i

mm

i i

i i

a a a

a a

Chọn 1 2 1 22; 1+= = = = = = = =K Kk k m

a a a a a a , thay vào phương trình ta

có: 4 lim2 1→∞

+ − = −k

xk m k

2 3 1⇔ = − −km k Chọn k ñủ lớn cố ñịnh sao cho 2011≥m .

Page 197: Chuyên đề hội thảo một số trường chuyên về dãy số

HỘ

HỘI CÁC TRƯỜNG THPT CHUYÊN KHU VỰC DUYÊN HẢI VÀ ðỒNG BẰNG BẮC BỘ HỘI THẢO KHOA HỌC LẦN THỨ IV

197

Giả sử 1 2; ;Km

a a a là nghiệm nguyên dương của phương trình (*). Ta có thể giả sử 1 2≤ ≤ ≤K ma a a và dấu bằng không ñồng thời xảy ra

Xét phương trình bậc hai ẩn x:

2 2

22

( ) 1 0==

= − + + =∑∏m m

i i

ii

f x x x a a

Phương trình này có nghiệm là 1x a= do ñó có nghiệm nữa là:

2

21

2 1

1=

=

+

= − =∑

m

im

ii

i

a

u a aa

Suy ra 1

12

=

= − > ∏m

i m m

i

u a a a a và u nguyên dương. Như vậy ta thu ñược

nghiệm của phương trình là ( )2; ;Km

u a a , tiếp tục sắp xếp và lặp lại quá

trình như trên ta thu ñược bộ m số phân biệt thỏa mãn yêu cầu của bài toán.

Bài 7: Cho A là tập hợp hữu hạn các số nguyên dương. Chứng minh rằng tồn tại tập hợp hữu hạn các số nguyên dương B sao cho ⊂A B và 2

∈∈

=∑∏x Bx B

x x .

Lời giải: ðặt

= ∏x A

a x ; 2

=∑x A

b x

Xét phương trình dạng Markov: 2

1 1

0 (*)= =

− + =∑ ∏nn

i i

i i

x a x b

Chọn 1 2 12; 1+= = = = = = =K Lk k nx x x x x ta có:

4 .2 0

.2 3

+ − − + =

⇔ = − −

k

k

k n k a b

n a k b

Chọn k ñủ lớn ñể có n nguyên dương; n lớn hơn số phần tử của tập A. Với k ñủ lớn ñã chọn, gọi 1 2; ; ;K

nx x x là nghiệm của phương trình (*). Vì

vai trò của các ẩn là như nhau nên ta có thể giả sử 1 2≤ ≤ ≤K nx x x và dấu

bằng không ñồng thời xảy ra.

Xét phương trình bậc hai ẩn x: 2 2

22

( ) +b=0 (**)==

= − +∑∏n n

i i

ii

f x x xa x x

Phương trình này có nghiệm 1=x x nên nó còn có nghiệm nữa mà ta kí

hiệu là u. Theo ñịnh lý Vi – et ta có:

2

21

2 1

+b =

=

= − =∑

n

in

ii

i

x

u a x xx

Page 198: Chuyên đề hội thảo một số trường chuyên về dãy số

HỘ

HỘI CÁC TRƯỜNG THPT CHUYÊN KHU VỰC DUYÊN HẢI VÀ ðỒNG BẰNG BẮC BỘ HỘI THẢO KHOA HỌC LẦN THỨ IV

198

Do ñó u nguyên hơn nữa: 1

1 12

> 2 > −

=

= − −

∏n

i n n n

i

u a x x x x x x

Như vậy ta ñược 2; ; ;K nu x x cũng là nghiệm của phương trình (*). Tiếp

tục sắp xếp và lặp lại quá trình như trên ta thu ñược n giá trị phân biệt

1 2; ; ;Kn

x x x là nghiệm của phương trình (*).

ðặt { }1 2; ; ;= ∪ Kn

B A x x x thì B là tập hợp cần tìm.

IV. Bài tập Bài 1: Tìm tất cả các số nguyên dương n sao cho phương trình sau:

+ + + =x y z t n xyzt

luôn có nghiệm nguyên dương. Bài 2: Với giá trị nguyên dương nào của p thì phương trình: 2 2 1+ + =x y pxy

có nghiệm nguyên dương? Bài 3: Tìm tất cả các số nguyên dương k sao cho phương trình:

2 2 2( 4) 24− − = −x k y

có nghiệm nguyên. Bài 4: Chứng minh rằng phương trình 2 2 2( 4) 1− − =k x y không có nghiệm

nguyên với mọi k > 3. Bài 5: Tìm tất cả các ña thức p(x) với hệ số nguyên sao cho ña thức:

2 2( ) ( 6 10)( ( )) 1= + + −q x x x p x

là bình phương của một ña thức với hệ số nguyên. Bài 6: Tìm các số nguyên dương n nhỏ nhất sao cho 19n + 1 và 95n + 1 ñều là các số chính phương.

Page 199: Chuyên đề hội thảo một số trường chuyên về dãy số

HỘ

HỘI CÁC TRƯỜNG THPT CHUYÊN KHU VỰC DUYÊN HẢI VÀ ðỒNG BẰNG BẮC BỘ HỘI THẢO KHOA HỌC LẦN THỨ IV

199

p� �� �� �� �

Nguyễn Trung Tuân Trường THPT Chuyên Hạ Long - Quảng Ninh

Cho số nguyên tố p. Với mỗi số nguyên a ta ký hiệu a là tập hợp

( ){ }b b a mod p∈ ≡� và p� � là tập { }a a ∈� . Như thế nếu a và b là các số

nguyên thoả mãn ( )a b mod p≡ thì a b= (ngược lại cũng ñúng) và p p=� � ,

cụ thể là { }p 0;1;...;p 1= −� � , hơn nữa ta còn thấy rằng p� � là một phân

hoạch của � . Tổng của hai tập a và b , ký hiệu là a b+ sẽ là tập a b+ , và tích của hai tập ñó, ký hiệu là a.b sẽ là tập ab . Bởi các tính chất cơ bản của ñồng dư ta thấy việc ñịnh nghĩa như trên là thoả ñáng, và với các phép toán ñó p� � là một trường. Ta có thể ñịnh nghĩa n� � với n không phải là số nguyên tố bằng

cách tương tự như trên, tất cả các tính chất vẫn còn ñúng trừ tính tồn tại phần tử nghịch ñảo của phép nhân.

Bài 1. Chứng minh rằng phương trình 2 2 2x y 3z+ = có nghiệm nguyên duy

nhất là (0; 0; 0). Lời giải và bình luận. Kiểm tra thấy (0; 0; 0) là một nghiệm nguyên của

phương trình. Gỉa sử (x; y; z) là nghiệm nguyên của phương trình mà x; y; z không ñồng thời bằng 0, do hai vế là ñẳng cấp cùng bậc nên không giảm tổng

quát ta có thể giả sử gcd(x, y, z) = 1. Trong 4� � ta có ( ) ( ) ( )2 2 2

x y 3 z+ = ,

nhưng { }2

a 0;1 a 4∈ ∀ ∈� � nên ta phải có ( ) ( ) ( )2 2 2

x y z 0= = = , suy ra cả ba

số x; y; z phải là bội của 2, ñiều này là không thể và dó ñó bài toán ñược giải Rõ ràng là việc chuyển một phương trình trên � (là tập vô hạn) sang một

phương trình trên n� � (một tập hữu hạn) theo một nghĩa nào ñó là ñơn giản

hơn. Không may, có những phương trình có nghiệm không tầm thường trên n� � với mỗi số nguyên dương nhưng lại không có nghiệm không tầm thường

trên � , ví dụ 3 3 33x 4y 5z 0+ + = . Chứng minh kết quả này không dễ và nó vượt

quá khuôn khổ của bài giảng này. Bài 2. Cho p là một số nguyên tố lẻ. Chứng minh rằng khi viết số hữu tỉ

1 1 1...

1 2 p 1+ + +

−dưới dạng phân số tối giản thì tử số của nó chia hết cho p. Lời

Page 200: Chuyên đề hội thảo một số trường chuyên về dãy số

HỘ

HỘI CÁC TRƯỜNG THPT CHUYÊN KHU VỰC DUYÊN HẢI VÀ ðỒNG BẰNG BẮC BỘ HỘI THẢO KHOA HỌC LẦN THỨ IV

200

giải và bình luận. Trong p� � ta có: ( )p 1 p

02

− =

. Do ñó ta có ñiều phải

chứng minh. Tính toán thật là thuận lợi! Bài 3(ðịnh lý Lagrange). Cho p là một số nguyên tố và f(x) = anx

n + an – 1 xn – 1 + …. + a1x + a0 là một

ña thức với hệ số nguyên có bậc n > 0. Chứng minh rằng nếu an không chia hết cho p thì ñồng dư ( ) ( )f x 0 mod p≡ có nhiều nhất n nghiệm phân biệt modulo p.

Lời giải và bình luận. Trong dạng khác, bài toán trên có dạng: Cho ( ) [ ]f x p x∈� � có bậc n > 0 thì f(x) có nhiều nhất n nghiệm trong p� � . Rõ

ràng ñây là kết quả tương tự như trong [ ] [ ]x , x� � và [ ]x� mà các bạn ñã biết.

Bởi vậy tôi ñể lại chứng minh cho các bạn.

Bài 4. Cho p là một số nguyên tố và ( )n

ii

i 0

f x a x=

=∑ là một ña thức với hệ số

nguyên. Nếu ñồng dư ( ) ( )f x 0 mod p≡ có nhiều hơn n nghiệm phân biệt modulo

p thì tất cả các hệ số của f(x) phải chia hết cho p. Lời giải và bình luận. Bài toán này ñược suy ra ngay lập tức từ bài toán

trước. Kết quả tương tự trong [ ] [ ]x , x� � và [ ]x� ñược áp dụng rộng rãi trong

ðại số nên ñương nhiên kết quả này sẽ ñược áp dụng rộng rãi trong Lý thuyết số theo cái cách mà nó ñã làm với ðại số. Bài toán sau ñây là một ví dụ, các ví dụ khác tôi cho vào trong phần Bài tập ở cuối bài giảng.

Bài 5(ðịnh lý Wilson). Chứng minh rằng nếu p là một số nguyên tố thì

( ) ( )p 1 ! 1 0 mod p− + ≡ .

Lời giải và bình luận. Quan tâm ñến ña thức ( ) ( )( ) ( ) p 1f x 1 x 2 x ... p 1 x 1 x −= − − − − + − .

Bài 6. Cho p là một số nguyên tố lớn hơn 3. Chứng minh rằng [ ]2p 3

rp

r 1

C=

∑ chia

hết cho p2. Lời giải và bình luận. Vì i

pC là bội của p với mỗi { }i 1;2;...;p 1∈ − nên ta chỉ

cần chứng minh rkp

r 1

C

p=

∑ chia hết cho p là ñủ, ở ñây [ ]k 2p 3= . (Như vậy từ

modulo p2 chuyển về modulo p) Trong p� � , nhưng bỏ ñi dấu gạch ngang trên

Page 201: Chuyên đề hội thảo một số trường chuyên về dãy số

HỘ

HỘI CÁC TRƯỜNG THPT CHUYÊN KHU VỰC DUYÊN HẢI VÀ ðỒNG BẰNG BẮC BỘ HỘI THẢO KHOA HỌC LẦN THỨ IV

201

ñầu cho gọn, ta có ( )( ) ( )

( )r

r 1pC p r 1 p r 2 ... p 1 11

p r! r−− + − + −

= = − với mỗi r = 1;

2;…; k. Giờ ta xét hai trường hợp Nếu p = 3h + 1 với h là số nguyên dương lớn hơn 1 thì k = 2h, là một số chẵn,

và tổng ta ñang quan tâm bằng 2h h

i 1 j 1

1 1 1 1 1...

1 2 2h i j= =

− + − = −∑ ∑

Nếu p=3h + 2 với h là số nguyên dương thì chúng ta xét tương tự như trên. Bài toán ñược chứng minh ñầy ñủ. Bài tập

Bài 7. Viết cụ thể chứng minh của ðịnh lý Lagrange ñã nói ñến trong bài giảng.

Bài 8. Cho p là một số nguyên tố lẻ khác 3. Chứng minh rằng khi viết số hữu

tỉ 1 1 1

...1 2 p 1

+ + +−

dưới dạng phân số tối giản thì tử số của nó chia hết cho p2.

Bài 9. Hai dãy (an) và (bn) xác ñịnh bởi a0 = 1, b0 = 4 và 2001 2001

n 1 n n n 1 n na a b ,b b a+ += + = + với mỗi n ≥ 0. Chứng minh rằng n ≥ 0 không là

ước số của bất cứ số hạng nào trong cả hai dãy này. Bài 10. Cho p > 5 là một số nguyên tố. Với mỗi số nguyên x, ñịnh nghĩa

( )( )

p 1

p 2k 1

1f x

px k

=

=+

∑ . Chứng minh rằng với mỗi cặp số nguyên dương x, y khi

viết phân số fp(x) – fp(y) dưới dạng tối giản thì tử số của nó chia hết cho p3.

Bài 11. Với mỗi số nguyên tố p, tìm dư khi chia ( )p

2

i 0

1 i=

+∏ cho p.

Bài 12. Cho p > 3 là một số nguyên tố. Chứng minh rằng 3 p 12p 1p C 1−

− − .

Bài 13. Cho p > 3là một số nguyên tố. Chứng minh rằng 2p 1

2 k2k

k 1

p C−

=

∑ .

Bài 14. Chứng minh rằng nếu p là một số nguyên tố và 0 ≤ m < n < p thì

( ) ( )m n 1mp n 2np mC 1 p mod p

+ ++

+ ≡ − .

Bài 15. Cho là một số nguyên tố lẻ và ( )

1 1 1 a...

1 2 p 1 p 1 !− + − ≡

− −. Chứng minh

rằng ( )p2 2

a mod pp

−≡

Page 202: Chuyên đề hội thảo một số trường chuyên về dãy số

HỘ

HỘI CÁC TRƯỜNG THPT CHUYÊN KHU VỰC DUYÊN HẢI VÀ ðỒNG BẰNG BẮC BỘ HỘI THẢO KHOA HỌC LẦN THỨ IV

202

Tài liệu tham khảo 1. David S. Dummit and Richard M. Foote, Abstract Algebra, third edition. 2. Gareth A. Jones and J. Mary Jones, Elementary number theory, Springer. 3. Hà Huy Khoái, Số học, NXB Giáo dục.

Page 203: Chuyên đề hội thảo một số trường chuyên về dãy số

HỘ

HỘI CÁC TRƯỜNG THPT CHUYÊN KHU VỰC DUYÊN HẢI VÀ ðỒNG BẰNG BẮC BỘ HỘI THẢO KHOA HỌC LẦN THỨ IV

203

SỬ DỤNG TÍNH CHẤT NGHIỆM CỦA PHƯƠNG TRÌNH BẬC BA TÌM HỆ THỨC LƯỢNG MỚI TRONG TAM GIÁC

Trường THPT Chuyên Hưng Yên

Hệ thức lượng trong tam giác và bất ñẳng thức tam giác luôn gây ñược hứng thú ñối với học sinh, ñặc biệt là ñối với học sinh giỏi. Trong chương trình THPT, hệ thức lượng trong tam giác ñược giảng dạy ở lớp 10, bao gồm một số ñịnh lí cơ bản : ñịnh lí cosin, ñịnh lí sin trong tam giác, công thức ñường trung tuyến, các công thức tính diện tích trong tam giác,...Các hệ thức này ñược xây dựng dựa trên hệ thức lượng trong tam giác vuông và các phép toán vectơ. Trong bài viết này, tôi sử dụng tính chất nghiệm của phương trình ñại số bậc ba ñể xây dựng các hệ thức lượng giác, và từ các hệ thức lượng giác này tìm một vài bất ñẳng thức ñơn giản. Ở ñây, tôi chỉ khai thác trên cơ sở một phương trình bậc ba nhận ñộ dài ba cạnh của tam giác làm nghiệm. Phần này, tôi ñã thực hiện giảng dạy cho ñội tuyển học sinh giỏi lớp 10, nhằm khơi dậy sự ñam mê khám phá, tìm tòi của học sinh. A. Một số tính chất nghiệm của phương trình bậc ba Cho phương trình bậc ba: 3 2 0x ax bx c+ + + = có 3 nghiệm x1; x2; x3 ( kể cả nghiệm bội, nghiệm phức) . Khi ñó:

1. (ðịnh lí viet): 1 2 3

1 2 2 3 3 1

1 2 3

x x x a

x x x x x x b

x x x c

ì + + = -ïïïï + + =íïï = -ïïî

2. 2 2 2 21 2 3 2x x x a b+ + = -

3. 2

1 2 2 3 3 1

1 1 1 a b

x x x x x x c ab

++ + =

+ + + -

B. Tìm và chứng minh các hệ thức lượng trong tam giác Từ phần này, A, B, C, a, b, c, p, r, R lần lượt là các góc, các cạnh, nửa chu vi , bán kính ñường tròn nội, ngoại tiếp tam giác. Trước hết, ta chứng minh kết quả sau: Chứng minh rằng: ðộ dài 3 cạnh của tam giác ABC là nghiệm của phương trình bậc ba: 3 2 2 22 ( 4 ) 4 0x px p r Rr x Rrp- + + + - = (1)

Ta có thể chứng minh ñiều này bằng nhiều cách, ở ñây, tôi xin trình bày một cách chứng minh như sau: Ta có:

Page 204: Chuyên đề hội thảo một số trường chuyên về dãy số

HỘ

HỘI CÁC TRƯỜNG THPT CHUYÊN KHU VỰC DUYÊN HẢI VÀ ðỒNG BẰNG BẮC BỘ HỘI THẢO KHOA HỌC LẦN THỨ IV

204

2 sin 4 sin cos2 2

cos2.cot .

2 sin2

A Aa R A R

AA

p a r rA

= =

- = =

Suy ta: 2 24( ) 4 cos ; sin

2 2

A a R Aa p a Rr

p a r- = =

-

Do ñó: 2 2( )cos ;sin

2 4 2 4 ( )

A a p a A ar

Rr R p a

-= =

-

Vậy: ( )

14 4 ( )

a p a ar

Rr R p a

-+ =

- hay: 3 2 2 22 ( 4 ) 4 0a pa p r Rr a Rrp- + + + - =

Vậy a là một nghiệm của phương trình (1) Tương tự b, c cũng là nghiệm của phương trình (1) Từ bài toán trên, ta có ñược một số kết quả sau: 1. Với mọi tam giác ABC, ta luôn có:

2 2 2 2 22( 4 )

4

a b c p r Rr

abc Rrp

+ + = - -

=

2. Với mọi tam giác ABC, ta luôn có:

2 2

2 2

1 1 1 1 5 4.

2 2

p r Rr

a b b c c a p p r Rr

+ ++ + =

+ + + + +

3. Cho tam giác ABC, gọi I, G theo thứ tự là tâm ñường tròn ngoại tiếp tam giác ABC và trọng tâm của tam giác. Khi ấy:

a) 2 2 2 2 219 3 2( )

3IG r p a b c= - + + +

b) (Bất ñẳng thức Gerretsen) : 2 (16 5 )p r R r³ -

Chứng minh:

a) Theo công thức Leibniz ta có: 2 2 2 2 2 2 213 ( )

3IA IB IC IG a b c+ + = + + +

Mà 2 2 2 2 2 2 2 2 2( ) ; ( ) ; ( )IA r p a IB r p b IC r p c= - - = - - = - -

Suy ra: 2 2 2 2 219 3 2( )

3IG r p a b c= - + + +

b) Từ câu a, ta có: 2 2 2 2 29 3 2( ) 0r p a b c- + + + ³

Mà 2 2 2 2 22( 4 )a b c p r Rr+ + = - - nên: 2 2 2 2 2 2 29 3 4( 4 ) 0 5 16 0 (16 5 )r p p r Rr r p Rr p r R r- + - - ³ Û + - ³ Û ³ -

4. Với mọi tam giác ABC, ta luôn có:

Page 205: Chuyên đề hội thảo một số trường chuyên về dãy số

HỘ

HỘI CÁC TRƯỜNG THPT CHUYÊN KHU VỰC DUYÊN HẢI VÀ ðỒNG BẰNG BẮC BỘ HỘI THẢO KHOA HỌC LẦN THỨ IV

205

a) 2 2 2 29a b c R+ + £

b) 3 3

2p R£

Chứng minh: a) Giả sử O là tâm ñường tròn ngoại tiếp tam giác ABC , còn G là trọng tâm tam

giác. Ta có: 3OG OA OB OC= + +uuur uur uur uuur

2 29. 3 2( . . . )OG R OAOB OB OC OC OAÛ = + + +uur uur uur uuur uuur uur

Mà: 2 2 2 2 22. . 2OAOB OA OB BA R c= + - = -uur uur

,

2 2 2 22. . 2 ;2. . 2OB OC R a OC OA R b= - = -uur uuur uuur uur

Nên: 2 2 2 2 2 2 2 2 2 29. 3 (6 ) 9 ( )OG R R a b c R a b c= + - - - = - + +

2 2 2 29a b c RÞ + + £ b) Bất ñẳng thức tương ñương với 2 2 2 24 27 ( ) 27p R a b c R£ Û + + £

Nhưng theo bất ñẳng thức Bunhiacopski 2 2 2 2( ) 3( )a b c a b c+ + £ + +

Kết hợp với câu a, ta ñược ñiều phải chứng minh.

5. Chứng minh rằng với mọi tam giác ABC, ta luôn có: 3 3p r³

Chứng minh: Ta có: 3 38 ( ) 27p a b c abc= + + ³ mà 27 27.4.abc Rrp=

Nên 2 2 22 27 27p Rr p r³ Û ³ (do R 2r³ ) 3 3p rÛ ³

6. Chứng minh rằng với mọi tam giác ABC:

a) 4 3r R p+ ³

b) 2 2 4 4 3p r rR pr+ + ³

Chứng minh: Theo kết quả 1 ta có:

( )

2 2 2 2 2 2 2 2 2

222 2 2 2 2 2 2

2( 4 ) 9 2 9 2 8

273 9 2 8 9 8 4

4 4

4 3

a b c p r rR R p R r rR

Rp p R r rR R R r rR R r

R r p

+ + = - - £ Û £ + +

Û £ + + + £ + + + + = +

Û + ³

C. Tìm ra các hệ thức mới và chứng minh 1. Ví dụ 1: Chứng minh rằng với mọi tam giác ABC, ta luôn có: 2 2 2 2 2 2 6( 2 )( 2 )( 2 ) 2 ( 2 ) 64a Rr b Rr c Rr Rr p r Rr R+ + + = + + £

Giải: Xét phép biến ñổi 2 2y x Rr= +

Ta khử x từ hệ:

Page 206: Chuyên đề hội thảo một số trường chuyên về dãy số

HỘ

HỘI CÁC TRƯỜNG THPT CHUYÊN KHU VỰC DUYÊN HẢI VÀ ðỒNG BẰNG BẮC BỘ HỘI THẢO KHOA HỌC LẦN THỨ IV

206

3 2 2 2

2

2 ( 4 ) 4 0(1)

2

x px p r Rr x Rrp

y x Rr

ìï - + + + - =ïíï = +ïî

Ta có: 3

2

2(*)

2

x xy Rrx

x y Rr

ìï = -ïíï = -ïî

Thay (*) vào (1), ta ñược:

( )

2 2

2 2

2 2 ( 2 ) ( ) 4

22

xy Rrx p y Rr p r Rr x Rrp

pyx T p r Rr

y T

- - - + + + -

Þ = = + ++

Thay x vào phương trình bậc ba lần nữa và rút gọn ta ñược: 3 2 2 2 22( 2 ) ( 2 ) 2 0y T p Rr y T Rr y RrT- - - + - - = (2)

Như vậy, (2) nhận 2 2 22 ; 2 ; 2a Rr b Rr c Rr+ + + làm 3 nghiệm. Từ ñó, dễ dàng

suy ra: 2 2 2 2 2 2 2( 2 )( 2 )( 2 ) 2 2 ( 2 )a Rr b Rr c Rr RrT Rr p r Rr+ + + = = + +

Trước tiên, ta chứng minh: 2 2 22 8p r Rr R+ + £

Thật vậy:

Ta có:

223 3 1

;2 2

p r

R R

æ öæ ö ÷ç÷ç ÷£ £ç÷ç ÷ç÷ç ÷è ø çè ø

Nên: 2 2

2 8p r r

R R R

æ ö æ ö÷ ÷ç ç+ + £÷ ÷ç ç÷ ÷ç çè ø è ø

Vậy: 2 2 22 8p r Rr R+ + £

Từ ñó, suy ra ñiều phải chứng minh. Dấu “=” xảy ra khi và chỉ khi tam giác ABC ñều.

2. Bây giờ, chúng ta ñể ý rằng: 2 sin sin2

aa R A A

R= Û = . Như vậy, phép biến

ñổi 2

xy

R= của phương trình (1) sẽ thu ñược phương trình bậc ba nhận: sinA,

sinB, sinC làm 3 nghiệm.

Khử x từ hệ:

3 2 2 22 ( 4 ) 4 0

2

x px p r Rr x Rrp

xy

R

ìï - + + + - =ïïíï =ïïî

Ta ñược phương trình: 2 2

3 22 2

40

4 2

p p r Rr pry y y

R R R

+ +- + - =

Vậy phương trình bậc ba nhận: sinA, sinB, sinC làm nghiệm là: 2 2

3 22 2

40

4 2

p p r Rr prx x x

R R R

+ +- + - = (3)

Page 207: Chuyên đề hội thảo một số trường chuyên về dãy số

HỘ

HỘI CÁC TRƯỜNG THPT CHUYÊN KHU VỰC DUYÊN HẢI VÀ ðỒNG BẰNG BẮC BỘ HỘI THẢO KHOA HỌC LẦN THỨ IV

207

Từ ñây ta tìm lại một số ñiều quen thuộc sau:

3 3sin sin sin

2

pA B C

R+ + = £

2 2 2 2 2 2 2 22 2 2

2 2 2

4 4 9sin sin sin 2

4 2 4 4

p p r Rr p r Rr a b cA B C

R R R R

æ ö + + - - + +÷ç+ + = - = = £÷ç ÷çè ø

3. Ví dụ 2: Chứng minh rằng, với mọi tam giác ABC ta luôn có:

2 23 3 3 3 41 1 1 6 3 1 1

sin sin sin 2

p r Rr R R

A B C pr r p

æ öæ öæ ö æ ö+ +÷ ÷ ÷ç ç ç ÷ç÷ ÷ ÷- - - = + - -÷ç ç ç ç÷ ÷ ÷ ÷ç ç ç ç ÷÷ ÷ ÷ç ç ç è øè øè øè ø

Giải:

Nhìn vào bài toán, ta có thể liên tưởng ñến phép biến ñổi 3

1yx

= - cho

phương trình (3). Tuy nhiên, ñể cho gọn ta có thể làm thực hiện ñơn giản hơn

như sau: trong (3) thế x bởi x

1 thì nhận ñược phương trình bậc 3 nhận

1 1 1; ;

sin sin sinA B C làm 3 nghiệm là:

2 2 23 24 2

2 02

p r Rr R Rx x x

pr r pr

+ +- + - = (4)

ðến ñây, ta xét phép biến ñổi 3 1y x= - cho phương trình (4).

Khử x từ hệ:

2 2 23 24 2 2

02

3 1

p r Rr R Rx x x

pr r pr

y x

ìï + +ï - + - =ïïíïïï = -ïî

Ta ñược phương trình:

3 2 2 2 2 2

2 2

3 3 63 ( 4 ) 3 ( 4 )

2

3 41 . 6 3 1 0

2

Ry p r Rr y p r pr y

pr pr r

p r Rr R R

pr r p

é ù é ùê ú ê ú+ - + + + - + + +ê ú ê úë û ë û

æ ö+ + ÷ç+ - - - =÷ç ÷ç ÷è ø

ðây chính là phương trình bậc ba nhận 3 3 3

1; 1; 1sin sin sinA B C

- - - làm 3

nghiệm. Bởi vậy: 2 23 3 3 3 4

1 1 1 . 6 3 1 1sin sin sin 2

p r Rr R R

A B C pr r p

æ öæ öæ ö æ ö+ +÷ ÷ ÷ç ç ç ÷ç÷ ÷ ÷- - - = + - -÷ç ç ç ç÷ ÷ ÷ ÷ç ç ç ç ÷÷ ÷ ÷ç ç ç è øè øè øè ø

Page 208: Chuyên đề hội thảo một số trường chuyên về dãy số

HỘ

HỘI CÁC TRƯỜNG THPT CHUYÊN KHU VỰC DUYÊN HẢI VÀ ðỒNG BẰNG BẮC BỘ HỘI THẢO KHOA HỌC LẦN THỨ IV

208

4. Chứng minh ñẳng thức sau: sin

2

cos .cos2 2

Aa

B C p=

Thật vậy: 4 sin cos 4sin cos sin

2 2 2 2 2(sin sin sin ) 4cos cos cos cos cos

2 2 2 2 2

A A A A AR

a

A B C B Cp R A B C= = =

+ +

Như vậy, nếu ta dùng phép biến ñổi x

yp

= cho phương trình (1) sẽ nhận ñược

phương trình bậc ba có 3 nghiệm là : sin sin sin

2 2 2; ;cos .cos cos cos cos cos

2 2 2 2 2 2

A B C

B C C A A B

Cụ thể khử x từ hệ:

3 2 2 22 ( 4 ) 4 0x px p r Rr x Rrp

xy

p

ìï - + + + - =ïïïíï =ïïïî

Ta ñược : 2 2

3 22 2

4 42 0

p r Rr Rry y y

p p

+ +- + - = (5)

Từ ñó, ta thu ñược hệ thức sau: sin sin sin

2 2 2 2cos cos cos cos cos cos

2 2 2 2 2 2

A B C

B C C A A B+ + =

Sử dụng tính chất 2 ta có: 2 2 2

2 2

22 2 2 2 2 2

sin sin sin 42 2 2 4 2cos cos cos cos cos cos

2 2 2 2 2 2

A B Cp r Rr

B C C A A B p

+ ++ + = -

Hay:

2 2

2

1 cos 1 cos 1 cos42 2 2 4 2

1 cos 1 cos 1 cos 1 cos 1 cos 1 cos. . .

2 2 2 2 2 2

A B Cp r Rr

B C C A A B p

- - -+ +

+ + = -+ + + + + +

2 2

2

1 cos 1 cos 1 cos 42

(1 cos )(1 cos ) (1 cos )(1 cos ) (1 cos )(1 cos )

A B C p r Rr

B C C A A B p

- - - + +Û + + = -

+ + + + + +

Như vậy, ta có hệ thức sau trong mọi tam giác:

1 cos 1 cos 1 cos

(1 cos )(1 cos ) (1 cos )(1 cos ) (1 cos )(1 cos )

A B C

B C C A A B

- - -+ +

+ + + + + +

Page 209: Chuyên đề hội thảo một số trường chuyên về dãy số

HỘ

HỘI CÁC TRƯỜNG THPT CHUYÊN KHU VỰC DUYÊN HẢI VÀ ðỒNG BẰNG BẮC BỘ HỘI THẢO KHOA HỌC LẦN THỨ IV

209

2 2

2

42

p r Rr

p

+ += -

Nếu chưa muốn dừng lại ở ñó chúng ta có thể sử dụng các kết quả sau: 2 2

2 222 2

2

44 4.

( )

2

44.

( ) 3

p r Rr ab bc ca ab bc cap r Rr ab bc ca

p a b ca b c

ab bc ca

a b c

+ + + + + ++ + = + + Þ = =

+ +æ ö+ + ÷ç ÷ç ÷çè ø

+ +Þ £

+ +

(Do 2 2 2a b c ab bc ca+ + ³ + + )

Vậy ta có một bất ñẳng thức rất ñẹp sau: Với mọi tam giác ABC, ta luôn có: 1 cos 1 cos 1 cos 2

(1 cos )(1 cos ) (1 cos )(1 cos ) (1 cos )(1 cos ) 3

A B C

B C C A A B

- - -+ + ³

+ + + + + +

Dấu “=” xảy ra khi và chỉ khi tam giác ABC ñều. 5. Xét tam giác ABC, gọi la là phân giác trong góc A. Khi ñó, ta có:

2.sin

2 2a

A prl

p a=

-

Như vậy, phép biến ñổi 2

2

pry

p x=

- của phương trình (1) sẽ cho ta phương trình

bậc ba nhận sin ; sin ; sin2 2 2a b c

A B Cl l l là nghiệm. Cụ thể, ta khử x từ hệ:

3 2 2 22 ( 4 ) 4 0

22

x px p r Rr x Rrp

pry

p x

ìï - + + + - =ïïïíï =ï -ïïî

ðược phương trình: 2 2 2 2 3 2

3 22 2 2 2 2 2

5 4 8 40

2 2 2

p r Rr r p r py r y y

p r Rr p r Rr p r Rr

+ +- + - =

+ + + + + + (6)

Khi ấy, theo tính chất 1 ta có công thức:

i) 2 2

2 2

5 4sin sin sin

2 2 2 2a b c

A B C p r Rrl l l r

p r Rr

+ ++ + =

+ +

ii) 2 2

2 2

8sin sin sin sin sin sin

2 2 2 2 2 2 2a b b c c a

A B B C C A r pl l l l l l

p r Rr+ + = =

+ +

iii) 3 2

2 2

4sin sin sin

2 2 2 2a b c

A B C r pl l l

p r Rr=

+ +

Ta lại có theo kết quả 2:

Page 210: Chuyên đề hội thảo một số trường chuyên về dãy số

HỘ

HỘI CÁC TRƯỜNG THPT CHUYÊN KHU VỰC DUYÊN HẢI VÀ ðỒNG BẰNG BẮC BỘ HỘI THẢO KHOA HỌC LẦN THỨ IV

210

2 2

2 2

2 2

2 2

1 1 1 1 5 4.

2 2

5 4 1 1 1 92 3

2 2

p r Rr

a b b c c a p p r Rr

p r Rr a b cp

p r Rr a b b c c a b c c a a b

+ ++ + =

+ + + + +

æ ö+ + ÷çÞ = + + = + + + ³÷ç ÷çè ø+ + + + + + + +

Từ ñó, ta tu ñược một bất ñẳng thức khá hay sau: 9

sin sin sin2 2 2 2a b c

A B C rl l l+ + ³

Dấu “=” xảy ra khi và chỉ khi tam giác ABC ñều Từ ii kết hợp với bất ñẳng thức Gerretsen :

2 2 2 2 2 2(16 5 ) 16 5 14 2 4p r R r p r Rr r r Rr Rr r³ - Þ + ³ - + = + - 14 2 ( 2 ) 14Rr r R r Rr= + - ³ (Vì 2 0)R r- ³

Vậy ta thu ñược bất ñẳng thức sau: 2

sin sin sin sin sin sin2 2 2 2 2 2 4a b b c c a

A B B C C A pl l l l l l+ + £

D. Bài tập: Bài 1: Chứng minh rằng ; ;a b cr r r là 3 nghiệm của phương trình:

3 2 2 2(4 ) 0x R r x p x p r- + + - = (7)

Từ ñó, suy ra các kết quả sau:

2

2

4 9

1.

12.

a b c

a b b c c a

a b c

a b c

b c c a a b

r r r R r r

r r r r r r p

r r r p r

r r r

r r r r r r r

ì + + = + £ïïïï + + =íïïï =ïî

+ + ³

Bài 2: Chứng minh rằng ; ;a b ch h h là 3 nghiệm của phương trình: 2 2 2 2 2

3 24 22 0

2

p r Rr p r p rx x x

R R R

+ +- + - = (8)

Từ ñó, suy ra các kết quả sau: 2 2

22

4

21.2

a b c

a b b c c a

p r Rrh h h

R

p rh h h h h h p

R

ìï + +ï + + =ïïïíïï + + = £ïïïî

22( )2. 9a b c

h h h r

ab bc ca R

æ ö+ + ÷ç³ ÷ç ÷çè ø+ +

Page 211: Chuyên đề hội thảo một số trường chuyên về dãy số

HỘ

HỘI CÁC TRƯỜNG THPT CHUYÊN KHU VỰC DUYÊN HẢI VÀ ðỒNG BẰNG BẮC BỘ HỘI THẢO KHOA HỌC LẦN THỨ IV

211

Bài 3: Chứng minh rằng cosA, cosB, cosC là 3 nghiệm của phương trình: 2 2 2 2 2

3 22 2

4 (2 )0

4 4

R r p r R R r px x x

R R R

æ ö+ + - + -÷ç- + + =÷ç ÷çè ø (9)

Từ ñó, suy ra các kết quả sau:

3 3 3

31.1 cos cos cos

23 3

2.cos cos cos cos cos cos3 3 3 8 4 3 3 3

A B C

A B C A B C

£ + + £

æ ö÷ç+ + ³ + + + ÷ç ÷çè ø

3. Chứng minh rằng: tam giác ABC nhọn 2p R rÛ > + Tam giác ABC tù 2p R rÛ < + Tam giác ABC vuông 2p R rÛ = +

Bài 4: Chứng minh rằng tan ; tan ; tan2 2 2

A B C là nghiệm của phương trình:

3 240

R r rx x x

p p

+- + - = (10)

Từ ñó, ta có các kết quả sau: 4

tan tan tan 32 2 2

1. tan tan tan tan tan tan 12 2 2 2 2 2

1tan tan tan

2 2 2 3

4 8 32. tan tan tan tan tan tan

2 2 2 2 2 2 9

tan tan tan t2 2 23.tan

2

A B C R r

p

A B B C C A

A B C r

p

A B B C C A R

p

A B B

C

ì +ïï + + = ³ïïïïïï + + =íïïïïï = £ïïïî

æ öæ öæ ö÷ ÷ ÷ç ç ç+ + + = ³÷ ÷ ÷ç ç ç÷ ÷ ÷ç ç çè øè øè ø

+ ++

an tan tan 4 22 2 2 6tan tan

2 2

C C AR r

A B r

+ -+ = ³

HÀNG ðIỂM ðIỀU HÒA - CHÙM ðIỀU HÒA

Trường THPT chuyên Lê Hồng Phong Nam ðịnh

Page 212: Chuyên đề hội thảo một số trường chuyên về dãy số

HỘ

HỘI CÁC TRƯỜNG THPT CHUYÊN KHU VỰC DUYÊN HẢI VÀ ðỒNG BẰNG BẮC BỘ HỘI THẢO KHOA HỌC LẦN THỨ IV

212

A. Tỉ số kép của hàng ñiểm - Hàng ñiểm ñiều hòa I. Tỉ số kép của hàng ñiểm 1. ðịnh nghĩa:

- Bộ bốn ñiểm ñôi một khác nhau, có kể ñến thứ tự , cùng thuộc một ñường thẳng ñược gọi là hàng ñiểm.

- Tỉ số kép của hàng ñiểm , , ,A B C D là một số, kí hiệu là ( )ABCD và

ñược xác ñịnh như sau: ( ) :=CA DA

ABCDCB DB

2. Các tính chất của tỉ số kép: +) ( ) ( ) ( ) ( )ABCD CDAB BADC DCBA= = =

+) ( )( ) ( )

1 1ABCD

BADC ABDC= =

+) ( ) ( ) ( )1 1ABCD ACBD DBCA= − = −

+) Nếu ( ) ( )'ABCD ABCD= thì 'D D≡

+) ( ) 1ABCD ≠

II. Hàng ñiểm ñiều hòa. 1. ðịnh nghĩa: Nếu ( ) 1ABCD = − thì hàng ñiểm , , ,A B C D ñược gọi là hàng ñiểm ñiều

hòa.

Nói cách khác nếu CA DA

CB DB= − thì hàng ñiểm , , ,A B C D ñược gọi là

hàng ñiểm ñiều hòa. 2. Các tính chất:

+) Hệ thức Descartes: ( )2 1 1

1ABCDAB AC AD

= − ⇔ = +

+) Hệ thức Newton: ( )2

1 .ABCD IA IC ID= − ⇔ = ( I là trung ñiểm của ñoạn

AB )

+) Hệ thức Maclaurin: ( ) 1 . .ABCD AC AD AB AJ= − ⇔ = (J là trung ñiểm của ñoạn

CD ) 3. Một số hàng ñiểm ñiều hòa cơ bản: Bài toán 1: Cho tam giác ABC . Gọi ,AD AE tương ứng là ñường phân giác

trong, ñường phân giác ngoài của tam giác ABC . Khi ñó ( ) 1BCDE = − .

Bài toán 2: Cho tam giác ABC và ñiểm O không thuộc các ñường thẳng , ,BC CA AB . Các ñường thẳng , ,AO BO CO theo thứ tự cắt các ñường

Page 213: Chuyên đề hội thảo một số trường chuyên về dãy số

HỘ

HỘI CÁC TRƯỜNG THPT CHUYÊN KHU VỰC DUYÊN HẢI VÀ ðỒNG BẰNG BẮC BỘ HỘI THẢO KHOA HỌC LẦN THỨ IV

213

, ,BC CA AB tại , ,M N P . Hai ñường thẳng ,BC NP cắt nhau tại Q . Khi ñó

( ) 1BCMQ = − .

Bài toán 3: Từ ñiểm S bên ngoài ñường tròn ( )O , kẻ tới ( )O các tiếp tuyến

( )( ), ,SA SB A B O∈ . Một ñường thẳng qua S , cắt ñường tròn ( )O tại ,M N và

cắt AB tại I . Khi ñó ( ) 1SIMN = − .

B. Tỉ số kép của chùm ñường thẳng - Chùm ñiều hòa I. Chùm ñường thẳng và tỉ số kép của nó: 1. Chùm ñường thẳng: - Tập hợp các ñường thẳng trong mặt phẳng cùng ñi qua một ñiểm S ñược gọi là chùm ñầy ñủ ñường thẳng tâm S . - Bộ 4 ñường thẳng ñôi một khác nhau, có kể ñến thứ tự, cùng thuộc một chùm ñầy ñủ ñường thẳng ñược gọi là chùm ñường thẳng 2. Tỉ số kép của chùm ñường thẳng: *) ðịnh lí 1: Cho , , ,a b c d là chùm ñường thẳng tâm O . ðường thẳng ∆ không ñi qua O theo thứ tự cắt , , ,a b c d tại , , ,A B C D . ðường thẳng '∆ không ñi qua

O theo thứ tự cắt , ,a b c tại ', ', 'A B C . Khi ñó ( )' '

' / /' '

C Ad ABCD

C B∆ ⇔ = .

*) ðịnh lí 2: Cho , , ,a b c d là chùm ñường thẳng tâm O . ðường thẳng ∆ không ñi qua O theo thứ tự cắt , , ,a b c d tại , , ,A B C D . ðường thẳng '∆ không ñi qua

O theo thứ tự cắt , , ,a b c d tại ', ', ', 'A B C D . Khi ñó ( ) ( )' ' ' 'ABCD A B C D= .

Từ ñịnh lí 2, ta nhận thấy, tỉ số kép ( )ABCD không phụ thuộc vào vị trí của

ñường thẳng ∆ . Khi ñó giá trị không ñổi của tỉ số kép ( )ABCD ñược gọi là tỉ số

kép của chùm ñường thẳng , , ,a b c d kí hiệu là ( )abcd hoặc ( )O abcd với O là

tâm của chùm.

Từ ñó ta suy ra ( ) ( )( )( )

( )( )

sin , sin ,:

sin , sin ,

OA OC OB OCabcd ABCD

OA OD OB OD= =

uuur uuur uuur uuur

uuur uuur uuur uuur

3. Phép chiếu xuyên tâm: a. ðịnh nghĩa: Cho hai ñường thẳng , '∆ ∆ và ñiểm S không thuộc , '∆ ∆ . Gọi K

là ñiểm thuộc ∆ sao cho SK // '∆ . Ánh xạ { }: \ 'f K∆ → ∆ xác ñịnh bởi

( ) 'f M M= sao cho , , 'S M M thẳng hàng ñược gọi là phép chiếu xuyên tâm từ

{ }\ K∆ ñến '∆ . S ñược gọi là tâm của phép chiếu.

b. Tính chất: *) Tính chất 1: Phép chiếu xuyên tâm bào tồn tỉ số kép của hàng ñiểm.

Page 214: Chuyên đề hội thảo một số trường chuyên về dãy số

HỘ

HỘI CÁC TRƯỜNG THPT CHUYÊN KHU VỰC DUYÊN HẢI VÀ ðỒNG BẰNG BẮC BỘ HỘI THẢO KHOA HỌC LẦN THỨ IV

214

*) Tính chất 2: Cho hai ñường thẳng , '∆ ∆ cắt nhau tại O . Các ñiểm , ,A B C thuộc ∆ , các ñiểm ', ', 'A B C thuộc '∆ . Khi ñó các ñường thẳng ', ', 'AA BB CC

hoặc ñồng quy hoặc ñôi một song song khi và chỉ khi ( ) ( )'OABC O ABC=

*) Tính chất 3: Cho hai chùm ( ) ( )' à 'O ABCO v O ABCO . Khi ñó , ,A B C thẳng

hàng khi và chỉ khi ( ) ( )' 'O ABCO O ABCO=

4. Chùm ñiều hòa: a. ðịnh nghĩa: Chùm , , ,a b c d ñược gọi là chùm ñiều hòa nếu ( ) 1abcd = −

b. Tính chất: Với chùm ñiều hòa , , ,a b c d , các ñiều kiện sau là tương ñương:

(i) c d⊥ (ii) c là một phân giác của góc tạo bởi ,a b .

(iii) d là một phân giác của góc tạo bởi ,a b .

C. Một số ví dụ áp dụng: Bài viết chỉ nêu một số ví dụ áp dụng các tính chất của hàng ñiểm ñiều hòa

ñể chứng minh các ñường thẳng ñồng quy và các ñiểm thẳng hàng.

Bài 1: Cho tứ giác ABCD nội tiếp ñường tròn ( )O . ,AB CD cắt nhau tại S ;

,AD BC cắt nhau tại F ; ,AC BD cắt nhau tại E . Kẻ các tiếp tuyến ,SM SN với

ñường tròn ( )O ( )( ),M N O∈ . Chứng minh rằng 4 ñiểm , , ,E F M N thẳng hàng.

T

Y

Z

X

M

N

E

S

C

B

D

A

O

F

Chứng minh:

Gọi ,Y T tương ứng là giao ñiểm của SE với ,BC AD và ,X Z tương ứng là giao ñiểm của MN với ,AB CD .

+) Ta có ( ) ( )1SXAB SZCD= − =

Theo tính chất 2 của phép chiếu xuyên tâm ta suy ra , ,XZ AC BD ñồng

quy , ,F X Z⇒ thẳng hàng , ,F M N⇒ thẳng hàng (1)

+) Ta có: ( ) ( )1SXAB SETY= − =

Page 215: Chuyên đề hội thảo một số trường chuyên về dãy số

HỘ

HỘI CÁC TRƯỜNG THPT CHUYÊN KHU VỰC DUYÊN HẢI VÀ ðỒNG BẰNG BẮC BỘ HỘI THẢO KHOA HỌC LẦN THỨ IV

215

Theo tính chất 2 của phép chiếu xuyên tâm ta suy ra , ,XE AT BY ñồng quy , ,F X E⇒ thẳng hàng (2)

+) ( ) ( )1SZCD SEYT= − =

Theo tính chất 2 của phép chiếu xuyên tâm ta suy ra , ,ZE DT CY ñồng quy , ,F Z E⇒ thẳng hàng (3)

Từ (1), (2), (3) suy ra 4 ñiểm , , ,E F M N thẳng hàng.

Bài 2: Cho tam giác ABC ngoại tiếp ñường tròn ( )I . Gọi , ,D E F tương ứng là

tiếp ñiểm của ñường tròn ( )I với các cạnh , ,BC CA AB . AD cắt ñường tròn ( )I

tại ñiểm thứ hai X . Các ñường ,BX CX cắt ñường tròn ( )I tại các ñiểm thứ hai

,Y Z . Chứng minh rằng các ñường thẳng , ,AX BZ CY ñồng quy.

K

Z

Y

X

E

F

D

I

B C

A

Chứng minh: Kẻ tiếp tuyến tại X của ñường tròn ( )I , tiếp tuyến này cắt BC tại K .

Trong tứ giác XEDF có tiếp tuyến tại ,E F và ñường thẳng XD ñồng quy

nên tứ giác XEDF là tứ giác ñiều hòa. Mà ,KX KD là tiếp tuyến tại ,X D của ñường tròn ( )I nên , ,K E F thẳng

hàng

Do ñó ( ) ( )1 1KDBC X KDBC= − ⇒ = − ( )( )

( )( )

sin , sin ,: 1

sin , sin ,

XK XB XD XB

XK XC XD XC⇒ = −

uuur uuur uuur uuur

uuur uuur uuur uuur

�( )�

sinsin sin sin: 1 : 1 : 1 . 1

sin sin sin sin

YXDXDY XDY YXD XY YD XY DZ

XZ DZ XZ DYXDZ DZX XDZ DZX

−⇒ = − ⇒ = ⇒ = ⇒ =

(1) Mặt khác theo ñịnh lí Ceva ta có:

Page 216: Chuyên đề hội thảo một số trường chuyên về dãy số

HỘ

HỘI CÁC TRƯỜNG THPT CHUYÊN KHU VỰC DUYÊN HẢI VÀ ðỒNG BẰNG BẮC BỘ HỘI THẢO KHOA HỌC LẦN THỨ IV

216

, ,AX BZ CY ñồng quy . . 1 . . 1YB ZX DC YB ZX DC

YX ZC DBYX ZC DB⇔ = − ⇔ =

. . 1YB DC ZX

BD ZC XY⇔ =

. . 1 . 1YD XD ZX YD ZX

XD ZD XY ZD XY⇔ = ⇔ = (luôn ñúng do (1))

Vậy , ,AX BZ CY ñồng quy.

Bài 3: Cho tứ giác ABCD nội tiếp ñường tròn ( )O . Gọi ,M N lần lượt là trung

ñiểm của ,AB CD . ðường tròn ngoại tiếp tam giác ABN cắt CD tại P . ðường tròn ngoại tiếp tam giác CDM cắt AB tại Q . Chứng minh rằng , ,AC BD PQ

ñồng quy.

Q

P

S

N

MA

B

D

C

Chứng minh: *) Nếu AB // CD thì ABCD là hình thang cân do ñó , ,AC BD PQ ñồng

quy. *) Nếu àAB v CD không song song, gọi S là giao ñiểm của ,AB CD

Khi ñó do tứ giác ABCD nội tiếp nên ta có . .SA SB SC SD= (1)

Do 4 ñiểm , , ,A B N P cùng thuộc 1 ñường tròn nên . .SA SB SN SP= (2) Mặt khác 4 ñiểm , , ,C D M Q cùng thuộc 1 ñường tròn nên

. .SM SQ SC SD= (3)

Từ (1), (2), (3) ta suy ra:. .

. .

SA SB SM SQ

SC SD SN SP

=

=

Mà ,M N lần lượt là trung ñiểm của ,AB CD nên theo hệ thức Maclaurin ta

có: ( ) ( )1 à 1SQAB v SPCD= − = −

Vậy , ,AC BD PQ ñồng quy.

Page 217: Chuyên đề hội thảo một số trường chuyên về dãy số

HỘ

HỘI CÁC TRƯỜNG THPT CHUYÊN KHU VỰC DUYÊN HẢI VÀ ðỒNG BẰNG BẮC BỘ HỘI THẢO KHOA HỌC LẦN THỨ IV

217

Bài 4: Cho tam giác ABC nội tiếp ñường tròn ( )O �( )090BAC ≠ . ðiểm E chạy

trên ñường tròn ( )O sao cho AE không song song với BC ( ),E B E C≠ ≠ . AE

theo thứ tự cắt các tiếp tuyến của ñường tròn ( )O tại ,B C ở ,M N . Gọi F là giao

ñiểm của àBN v CM . Chứng minh rằng EF luôn ñi qua một ñiểm cố ñịnh.

P

F

K

L

S

N

M

O

BC

A

E

Chứng minh:

Vì � 090BAC ≠ nên tiếp tuyến với ( )O tại àB v C cắt nhau tại S . Gọi K là

giao ñiểm của àSA v BC .

Vì AE không song song với BC ; khi ñó gọi L AE BC= ∩ Lấy P trên BC sao cho NP // MB thì tam giác NCP cân tại N NC NP⇒ =

Áp dụng ñịnh lí Thales dạng ñại số ta có: 2 2 2 2

MB NP MB NP MB NC

ML NL ML NL ML NL

= ⇒ = ⇒ =

2 2

. .: :

ME MA NA NE ME NE NA MA

ML NL NL MLML NL⇒ = ⇒ =

( ) ( ) ( ) ( )MNEL NMAL MNEL CBKL⇒ = ⇒ = (xét phép chiếu xuyên tâm S )

Do ñó , ,MC BN EK ñồng quy. Suy ra EK ñi qua F

Vậy EF luôn ñi qua ñiểm cố ñịnh K . Bài 5: Cho tứ giác ABCD nội tiếp ñường tròn ( )O . , ,AB AC AD theo thứ tự cắt

các ñường , ,CB DB BC tại , ,M N P . Chứng minh rằng O là trực tâm của tam

giác MNP .

Page 218: Chuyên đề hội thảo một số trường chuyên về dãy số

HỘ

HỘI CÁC TRƯỜNG THPT CHUYÊN KHU VỰC DUYÊN HẢI VÀ ðỒNG BẰNG BẮC BỘ HỘI THẢO KHOA HỌC LẦN THỨ IV

218

L

K

F

E

N

P

MO

A

B

D

C

Chưng minh: Kẻ các tiếp tuyến ,ME MF với ñường tròn ( ) ( )( ),O E F O∈

Gọi ,K L là giao ñiểm của EF với ,AB CD ( ) ( ) 1MKAB MLDC⇒ = = −

, ,KL AD BC⇒ ñồng quy P KL⇒ ∈ (1)

Mặt khác ta có ( ) ( ) 1MKAB MLCD= = − , ,KL AC BD⇒ ñồng quy N KL⇒ ∈

(2) Từ (1) và (2) suy ra KL EF NP≡ ≡ mà OM EF⊥ nên OM NP⊥ Chứng minh tương tự ta cũng có ON MP⊥ Do ñó O là trực tâm của tam giác MNP .

Page 219: Chuyên đề hội thảo một số trường chuyên về dãy số

HỘ

HỘI CÁC TRƯỜNG THPT CHUYÊN KHU VỰC DUYÊN HẢI VÀ ðỒNG BẰNG BẮC BỘ HỘI THẢO KHOA HỌC LẦN THỨ IV

219

Một số bài tập ñề nghị: Bài 6: Cho hai ñường tròn ( ) ( )1 2;O O cắt nhau tại 2 ñiểm ,E F . ðiểm A bất kì

trên tia EF kéo dài. Kẻ 2 tiếp tuyến ,AM AN với ñường tròn ( )1O và 2 tiếp tuyến

,AP AQ với ñường tròn ( )2O ( ) ( )( )1 2, , ,M N O P Q O∈ ∈ . Chứng minh rằng các

ñường thẳng , ,MN PQ EF ñồng quy. Bài 7: Cho tam giác ABC , các ñiểm ,M N thuộc cạnh BC . Các ñiểm ,P Q theo thứ tự thuộc các cạnh ,AC AB . Gọi O là giao ñiểm của àMP v NQ , K là giao ñiểm của àBO v NP , L là giao ñiểm của àCO v MQ . Chứng minh rằng

, ,AO BL CK ñồng quy.

Bài 8: Cho hai ñường tròn ( ) ( )1 2;O O cắt nhau tại 2 ñiểm phân biệt ,A B . Tiếp

tuyến tại ,A B của ñường tròn ( )1O cắt nhau ở Q . Xét ñiểm M trên ñường tròn

( )1O . Các ñường thẳng ,AM BM cắt ñường tròn ( )2O tại ,N P theo thứ tự ñó.

Chứng minh rằng ñường thẳng MQ luôn ñi qua trung ñiểm của ñoạn NP .

Tài liệu tham khảo: - Tài liệu giáo khoa chuyên Toán Hình học 10 – ðoàn Quỳnh, Văn Như Cương, Trần Nam Dũng, Nguyễn Minh Hà, ðỗ Thanh Sơn, Lê Bá Khánh Trình - Diễn ñàn toán học, Mathscope

BÀI TOÁN VỀ ðƯỜNG THẲNG SIMSON VÀ ỨNG DỤNG

Page 220: Chuyên đề hội thảo một số trường chuyên về dãy số

HỘ

HỘI CÁC TRƯỜNG THPT CHUYÊN KHU VỰC DUYÊN HẢI VÀ ðỒNG BẰNG BẮC BỘ HỘI THẢO KHOA HỌC LẦN THỨ IV

220

Phạm ðức Tùng TrườngTHPT chuyên Lương Văn Tụy

Bài toán:

Cho tam giác ABC nội tiếp trong ñường tròn (O). Giả sử P là một ñiểm nằm trên (O) sao cho P không trùng với các ñỉnh A, B, C của tam giác.

a) Giả sử A0, B0, C0 là hình chiếu của P trên các cạnh BC, CA, AB. Khi ñó A0, B0, C0 thẳng hàng. ðường thẳng chứa A0, B0, C0 ñược gọi là ñường thẳng Simson của P ñối với tam giác ABC. b) Gọi A1, B1, C1 là các ñiểm ñối xứng của P qua BC, CA, AB. Khi ñó A1, B1, C1 thuộc một ñường thẳng ñi qua trực tâm H của tam giác ABC. ðường thẳng chứa A1, B1, C1 ñược gọi là ñường thẳng Steiner của P ñối với tam giác ABC. Chứng minh:

a) Ta có: � � 00 0 90 ,CB P CA P= = suy ra tứ giác A0B0CP nội tiếp, suy ra

� �0 0 0 .B A C B PC= Mặt khác ABPC nội tiếp nên � � �

0 0C BP ACP B CP= = suy ra tam giác

PC0B ñồng dạng với tam giác PB0C(g.g) � � �0 0 0 0 .BPC B PC B A C⇒ = = Mà

� � � �0 0 0 0 0 0 0.BPC BA C B A C BA C= ⇒ = Suy ra A0, B0, C0 thẳng hàng.

Nhận xét: ðiều ngược lại của bài toán ñường thẳng Simson cũng ñúng. Cụ thể là nếu P chạy trong mặt phẳng của tam giác ABC sao cho P không trùng với các

A

B C

PCo

Ao

Bo

Page 221: Chuyên đề hội thảo một số trường chuyên về dãy số

HỘ

HỘI CÁC TRƯỜNG THPT CHUYÊN KHU VỰC DUYÊN HẢI VÀ ðỒNG BẰNG BẮC BỘ HỘI THẢO KHOA HỌC LẦN THỨ IV

221

ñỉnh của tam giác và hình chiếu của P trên 3 cạnh tam giác nằm trên một ñường thẳng. Khi ñó P nằm trên ñường tròn ngoại tiếp của tam giác. b) Dễ thấy A1, B1, C1 thẳng hàng và các ñiểm ñối xứng của H qua AB, BC, CA thuộc ñường tròn ngoại tiếp tam giác ABC. Ta có: � �

1 bHB C H PC= (tính chất ñối xứng trục AC)=� � �bH BC HBC HAC= = ⇒ tứ giác

AHCB1 nội tiếp. Suy ra � � � � � �1 1 1c c cB HC B AC PAC PH C PH H C HH= = = = = , suy ra C1, H,

B1 thẳng hàng. Ta ñược ñiều phải chứng minh. Tiếp theo ta ñưa ra một số bài toán ứng dụng: Bài toán 1: Ba ñiểm A, B, C thuộc một ñường thẳng và P không thuộc ñường thẳng này. Chứng minh rằng tâm ñường tròn ngoại tiếp các tam giác PAB, PBC, PCA và P thuộc một ñường tròn.

A

B

C

P

Co

Ao

Bo

C1

B1

A1

Hc

Hb

E

D

H

A B C

P

D EF

Oc

Ob

Oa

Page 222: Chuyên đề hội thảo một số trường chuyên về dãy số

HỘ

HỘI CÁC TRƯỜNG THPT CHUYÊN KHU VỰC DUYÊN HẢI VÀ ðỒNG BẰNG BẮC BỘ HỘI THẢO KHOA HỌC LẦN THỨ IV

222

Lời giải: Gọi Oa, Ob, Oc tương ứng là tâm ñường tròn ngoại tiếp các tam giác PBC, PCA, PAB. D, E, F tương ứng là trung ñiểm PA, PB, PC. Ta có: ; ;c b c a a bPD O O PE O O PF O O⊥ ⊥ ⊥ nên D, E, F là hình chiếu của P

trên các cạnh của tam giác OaObOc. Mặt khác D, E, F thẳng hàng nên theo ñiều ngược lại của bài toán ñường thẳng Simson ta có P thuộc ñường tròn ngoại tiếp của tam giác OaObOc. Bài toán 2: Tam giác ABC, phân giác AD. Hạ DB’, DC’ lần lượt vuông góc với AB, AC. M thuộc B’C’ thỏa mãn DM vuông góc với BC. Chứng minh rằng M thuộc trung tuyến AA1 của tam giác ABC. Lời giải: Gọi P là giao ñiểm của AD và ñường tròn ngoại tiếp tam giác ABC. Hạ PE, PF lần lượt vuông góc với AB, AC. Theo bài toán ñường thẳng Simson ta có

E, A1, F thẳng hàng. Phép vị tự tâm A tỉ số AD

AP biến

1; '; ';P D E B F C A M→ → → → (do DM //A1P và M thuộc B’C’). Vậy A, M,

A1 thẳng hàng. Bài toán 3: Tam giác ABC nội tiếp ñường tròn (O). P, Q thuộc ñường tròn (O) sao cho CP, CQ ñối xứng nhau qua phân giác góc �BCA . Chứng minh rằng CQ vuông góc với ñường thẳng Simson của P ñối với tam giác ABC.

O

A

BC

P

D

B'

C'

E

F

M

A1

Page 223: Chuyên đề hội thảo một số trường chuyên về dãy số

HỘ

HỘI CÁC TRƯỜNG THPT CHUYÊN KHU VỰC DUYÊN HẢI VÀ ðỒNG BẰNG BẮC BỘ HỘI THẢO KHOA HỌC LẦN THỨ IV

223

Lời giải: Ta có: � � � � � � � �EDC DCQ EPC DCA ACQ EPC BCA BCP+ = + + = + + (do P, Q ñối xứng

nhau qua phân giác góc �BCA nên � �QA BP= ) = � � 090 .EPC PCA+ = Suy ra ñiều phải

chứng minh. Nhận xét:

Bài toán trên cho ta một hướng xử lí các bài toán liên quan ñến góc của ñường thẳng Simson. (Bài tập 4,5) Bài toán 4: a) Chứng minh rằng ñường thẳng Simson ứng với một ñiểm chia ñôi ñoạn thẳng nối từ ñiểm ñó ñến trực tâm của tam giác. Hơn nữa trung ñiểm của ñoạn thẳng ấy nằm trên ñường tròn Euler của tam giác. b) ðường thẳng Simson ứng với hai ñiểm ñối xứng nhau qua tâm thì cắt nhau tại một ñiểm thuộc ñường tròn Euler của tam giác.

O

A

B

C

P

F

D

E

Q

O

A

B

C

P

Q

H

E

D

Sp

Sq

J

I

R

A'

B'

Page 224: Chuyên đề hội thảo một số trường chuyên về dãy số

HỘ

HỘI CÁC TRƯỜNG THPT CHUYÊN KHU VỰC DUYÊN HẢI VÀ ðỒNG BẰNG BẮC BỘ HỘI THẢO KHOA HỌC LẦN THỨ IV

224

Lời giải: a) Tam giác ABC, các ñường cao AD, BE, CF cắt ñường tròn ngoại tiếp tam giác tại A’, B’, C’. Sp là ñường thẳng Simson của P ñối với tam giác ABC. H là trực tâm của tam giác. Do tính chất ñường thẳng Steiner ñi qua trực tâm H nên ñường thẳng Sp ñi qua trung ñiểm I của PH.

Phép vị tự tâm H tỉ số 1

2biến A’, B’, C’, P thành D, E, F, I. Mà A’, B’, C’,

P thuộc ñường tròn ngoại tiếp tam giác ABC nên D, E, F, I thuộc một ñường tròn. Mặt khác ñường tròn Euler của tam giác ABC là ñường tròn ngoại tiếp tam giác DEF nên ta có I thuộc ñường tròn Euler của ABC. b) Gọi Q là ñiểm ñối xứng với P qua O và P’, Q’ là ñiểm ñối xứng của P, Q qua phân giác góc �BCA . Theo kết quả bài toán 3 ta có: '; 'p qS CP S CQ⊥ ⊥ (Sq

là ñường thẳng Simson của Q ñối với tam giác ABC). Do P, Q ñối xứng qua O nên P’, Q’ cũng ñối xứng qua O suy ra ' 'CP CQ⊥ từ ñó

p qS S⊥ .

Gọi R là giao ñiểm của Sp và Sq. J là trung ñiểm của HQ. Ta có: I, J thuộc

ñường tròn Euler của tam giác ABC và 1

2IJ PQ= nên IJ là ñường kính của

ñường tròn Euler, mà � 090IRJ = nên R thuộc ñường tròn Euler. Bài toán 5: Tứ giác ABCD nội tiếp (O). Gọi da là ñường thẳng Simson của A ñối với tam giác BCD. Các ñường thẳng db, dc, dd ñược ñịnh nghĩa một cách tương tự. Chứng minh rằng bốn ñường này ñồng qui. Lời giải: Ta có: / / ;b a b aAH BH AH BH= (vì chúng cùng bằng 2 lần khoảng cách từ

O ñến CD) suy ra ABHaHb là hình bình hành. Do ñó AHa và BHb cắt nhau tại trung ñiểm I của mỗi ñoạn. Theo bài toán 4 ta có da, db ñi qua I.

O

A

B

CD

Hb

Ha

Page 225: Chuyên đề hội thảo một số trường chuyên về dãy số

HỘ

HỘI CÁC TRƯỜNG THPT CHUYÊN KHU VỰC DUYÊN HẢI VÀ ðỒNG BẰNG BẮC BỘ HỘI THẢO KHOA HỌC LẦN THỨ IV

225

Tương tự CHc, BHb cắt nhau tại trung ñiểm I của mỗi ñoạn. CHc, DHd cắt nhau tại trung ñiểm I của mỗi ñoạn. Suy ra dc, dd ñi qua I. Vậy da, db, dc, dd ñồng qui. Bài toán 6: Cho hai ñường tròn (O1), (O2) cắt nhau tại hai ñiểm A, B (hai ñiểm O1, O2 nằm về hai phía của AB). Một ñường thẳng d thay ñổi qua A cắt (O1), (O2) tại C, D (A nằm giữa C và D). Tiếp tuyến tại C của (O1) và tại D của (O2) cắt nhau tại T. Gọi P, Q lần lượt là hình chiếu vuông góc của B xuống hai tiếp tuyến này. Chứng minh PQ tiếp xúc với ñường tròn cố ñịnh. Lời giải: Hạ ( )BR CD R CD⊥ ∈ suy ra P, Q, R thẳng hàng (ñường thẳng Simson B

ñối với tam giác TCD). Tứ giác BPCR nội tiếp nên � � (1)PBC PRC= ; � �PBR RCT= .

TC là tiếp tuyến của (O1) nên � � �RCT ACT ABC= = . Do ñó � � (2)ABR PBC=

Từ (1) và (2) suy ra: � �ABR PRC= ⇒PQ tiếp xúc với ñường tròn ñường kính AB. Bài toán 7: Cho ñường tròn (O) và ñường thẳng d không cắt (O). ðiểm M thay ñổi trên d. Kẻ các tiếp tuyến MT, MH với (O). Gọi A là hình chiếu vuông góc của O

O1

O2

A

B

C

D

T

PQ

I

R

Page 226: Chuyên đề hội thảo một số trường chuyên về dãy số

HỘ

HỘI CÁC TRƯỜNG THPT CHUYÊN KHU VỰC DUYÊN HẢI VÀ ðỒNG BẰNG BẮC BỘ HỘI THẢO KHOA HỌC LẦN THỨ IV

226

trên d. Gọi E, F lần lượt là hình chiếu vuông góc của A lên MT, MH. Chứng minh rằng:

a) ðường thẳng TH ñi qua một ñiểm cố ñịnh. b) ðường thẳng EF ñi qua một ñiểm cố ñịnh.

Lời giải:

a) Gọi ; ; .I TH OA J EF OA G OM TH= ∩ = ∩ = ∩

TH OM MGIA⊥ ⇒ là tứ giác nội tiếp 2 2. .OI OA OG OM OT R⇒ = = = . Suy ra I cố ñịnh. Vậy ñường thẳng TH ñi qua ñiểm cố ñịnh I. b) M, T, O, H, A thuộc ñường tròn ñường kính MO. Hạ AK TH⊥ suy ra E, F, K thẳng hàng. OMAH, AFHK là các tứ giác nội tiếp � � � �; .MOA MHA FAH FKH⇒ = =

Vì � � � �0 090 ; 90MHA FAH MOA AIH+ = + = nên � �FKH AIH= ⇒ tam giác IJK cân

tại J. Mà tam giác IAK vuông tại K nên J là trung ñiểm của IA suy ra J cố ñịnh. Vậy ñường thẳng EF ñi qua ñiểm cố ñịnh J. Bài toán 8: Cho tam giác ABC nội tiếp (O;R). Gọi D là ñiểm ñối xứng với A qua BC. E là ñiểm ñối xứng với B qua AC, F là ñiểm ñối xứng với C qua BA. H là trực tâm của tam giác ABC. Chứng minh: D, E, F thẳng hàng khi và chỉ khi OH=2R.

O

M

T

H

A

K

E

F

G

I

J

Page 227: Chuyên đề hội thảo một số trường chuyên về dãy số

HỘ

HỘI CÁC TRƯỜNG THPT CHUYÊN KHU VỰC DUYÊN HẢI VÀ ðỒNG BẰNG BẮC BỘ HỘI THẢO KHOA HỌC LẦN THỨ IV

227

Lời giải:

Gọi G là trọng tâm tam giác ABC, qua A, B, C kẻ các ñường thẳng a, b, c song song với BC, CA, AB. Gọi 2 2 2; ;a b C a c B b c A∩ = ∩ = ∩ = . Suy ra A, B, C

lần lượt là trung ñiểm của B2C2, C2A2, A2B2 và HA, HB, HC lần lượt là trung trực của B2C2, C2A2, A2B2 suy ra H là tâm ñường tròn ngoại tiếp tam giác A2B2C2.

Vì G là trọng tâm tam giác ABC nên G cũng là trọng tâm tam giác

A2B2C2 và phép vị tự 1

2GV−

( A2B2C2)=(ABC) suy ra bán kính ñường tròn ngoại

tiếp tam giác A2B2C2 bằng 2R. Gọi A1 là trung ñiểm BC và D’ là trung ñiểm B2C2. Ta có:

1 1

1 1' ;

2 2A D AD GA GA= − = −uuuur uuur uuur uuur

nên 1'

2GD GD= − hay

1

2 ( ) 'GV D D−

= . Tương tự

1

2 ( ) 'GV E E−

= ; 1

2 ( ) 'GV F F−

= . Như vậy D, E, F thẳng hàng khi và chỉ khi D’, E’, F’

thẳng hàng. Mặt khác D’, E’, F’ là hình chiếu vuông góc của O trên 3 cạnh tam giác A2B2C2 nên D’, E’, F’ thẳng hàng 2 2 2( ) 2 .O A B C OH R⇔ ∈ ⇔ =

Bài toán 9: Tam giác ABC không ñều, P là chân ñường vuông góc hạ từ A xuống BC. Gọi D, E, F là trung ñiểm BC, CA, AB. Gọi la là ñường thẳng ñi qua chân hai ñường cao từ P xuống DE, DF. Tương tự cho lb, lc. CMR: la, lb, lc ñồng qui. Lời giải:

A

B C

A2

A1

G

D

D'B2C2

H

Page 228: Chuyên đề hội thảo một số trường chuyên về dãy số

HỘ

HỘI CÁC TRƯỜNG THPT CHUYÊN KHU VỰC DUYÊN HẢI VÀ ðỒNG BẰNG BẮC BỘ HỘI THẢO KHOA HỌC LẦN THỨ IV

228

Lời giải: la là ñường thẳng Simson của P ñối với tam giác DEF. Theo bài toán 3 al⇒ vuông góc với tiếp tuyến tại D của ñường tròn ngoại

tiếp tam giác DEF ( vì � �ED PF= ). Mà D là ñiểm giữa của cung � .aQR l QR⇒ ⊥

Mặt khác dễ thấy O là trực tâm của tam giác DEF nên theo bài toán 4

al⇒ ñi qua trung ñiểm P’ của OP.

Gọi Q’, R’ lần lượt là trung ñiểm của OQ, OR suy ra la là ñường cao từ P’ của tam giác P’Q’R’. Tương tự ta ñược lb, lc là ñường cao từ Q’, R’ của tam giác P’Q’R’. Do ñó la, lb, lc ñồng qui tại trực tâm của tam giác P’Q’R’. Cuối cùng xin ñưa ra một số bài tập:

1. Hai ñiểm P, C cố ñịnh trên ñường tròn. A, B di chuyển trên ñường tròn thỏa mãn �ACB α= . Chứng minh rằng ñường thẳng Simson của P ñối với tam giác ABC tiếp xúc với một ñường tròn cố ñịnh.

2. Tam giác ABC. P thuộc ñường tròn ngoại tiếp tam giác. A1, B1, C1 thuộc các ñường thẳng BC, CA, AB thỏa mãn các góc ñịnh hướng

1 1 1 1 1 1( ; ), ( ; ), ( ; )PA A A PB B C PC C A bằng nhau. Chứng minh rằng A1, B1, C1

thẳng hàng. 3. Tam giác ABC nhọn nội tiếp (O). ðiểm M thuộc cung nhỏ BC, hạ MB’,

MC’ lần lượt vuông góc với AC, BA. Tìm vị trí của M ñể B’C’ lơn nhất. 4. a) Cho tam giác ABC nội tiếp (O). PQ là ñường kính. Chứng minh rằng

ñường thẳng Simson của tam giác ABC ứng với các ñiểm P, Q vuông góc với nhau.

O

A

B C

F E

DP

N

M

T

H

Q

R

P'

Page 229: Chuyên đề hội thảo một số trường chuyên về dãy số

HỘ

HỘI CÁC TRƯỜNG THPT CHUYÊN KHU VỰC DUYÊN HẢI VÀ ðỒNG BẰNG BẮC BỘ HỘI THẢO KHOA HỌC LẦN THỨ IV

229

c) Tổng quát hơn nếu PQ là dây cung bất kỳ. Chứng minh rằng ñường thẳng Simson ứng với P, Q tạo với nhau một góc bằng nửa số ño cung nhỏ PQ.

5. Chứng minh rằng tam giác tạo bởi 3 ñường thẳng Simson dựng trên 3 ñiểm thì ñồng dạng với tam giác tạo bởi 3 ñiểm ñó.

6. Tam giác ABC nội tiếp dường tròn (O). Một ñiểm M thay ñổi trên cung BC không chứa A. Gọi P, Q là hình chiếu của A trên MB, MC. Chứng minh rằng: PQ luôn ñi qua một ñiểm cố ñịnh.

7. Tam giác ABC, M thay ñổi trên BC. Gọi D, E là ñiểm ñối xứng của M qua AB, AC. Chứng minh rằng trung ñiểm của DE thuộc một ñường thẳng cố ñịnh khi M thay ñổi trên BC.

8. Tam giác ABC nội tiếp (O) a. Chứng minh rằng OX song song với ñường thẳng Simson của X ñối với

tam giác ABC khi và chỉ khi � � �3XA CA AB= − (mod 3600) b. Chứng minh rằng có chính xác 3 ñiểm X thỏa mãn ñiều kiện câu a. c. Chứng minh rằng 3 ñường thẳng Simson ứng với 3 ñiểm trong câu b ñồng

qui. 9. Tam giác ABC nội tiếp (O), P thuộc (O). Gọi t là tiếp tuyến tại P của (O)

và s là ñường thẳng Simson của P ñối với tam giác. Chứng minh rằng: ( , ) ( , ) ( , ) ( , )s t s BC s CA s AB= + +

10. Tam giác ABC nội tiếp (O). Chứng minh rằng có 3 ñiểm trên (O) mà ñường thẳng Simson của nó tiếp xúc với ñường tròn Euler của tam giác ABC.

Page 230: Chuyên đề hội thảo một số trường chuyên về dãy số

HỘ

HỘI CÁC TRƯỜNG THPT CHUYÊN KHU VỰC DUYÊN HẢI VÀ ðỒNG BẰNG BẮC BỘ HỘI THẢO KHOA HỌC LẦN THỨ IV

230